You are on page 1of 654

1.

Cauda equina is formed by:


A) cervical, thoracic and lumbar nerves
B) thoracic, lumbar and sacral nerves
C) cervical and thoracic nerves
D) only sacral and coccygeal nerves
E) lumbar, sacral and coccygeal nerves
2. It is remnant of the cavity of the telencephalon:
A) ventriculus lateralis
B) central canal
C) ventriculus tertius
D) ventriculus quartus

E) aqueductus cerebri
3. The visual center is located:
A) frontal lobe
B) parietal lobe
C) occipital lobe

D) insula
E) temporal lobe
4. Which part of the diaphragm does superior phrenic artery supply:
A) sternal
B) costal
C) centrum tendineum
D) lumbar
E) only peritoneal covering of diaphragm

5. The lateral ventricles:


I. Through the foramen interventriculare connect with the third ventricle.
II. The floor is formed by the pons and the medulla oblongata
III. Roof is formed by nerve fibers that separate from the corpus collosum
IV. There are three openings on its posterior wall
V. Anterior horns are separated from each other by septum pellucidum

A) I;II;III
B) I;III;V
C) II;III;IV
D) I;II;V
E) II;IV;V
6. Deep fibular nerve innervates:

A) posterior muscles of leg


B) anterior muscles of leg
C) lateral muscles of leg
D) posterior muscles of thigh
E) anterior muscles of thigh
7. The weight of the heart in women (gr) :

A) 280
B) 250
C) 220
D) 180
E) 350
8. The roof of the fourth ventricle forms from the posterior-lower (back) aspect:

I. Velum medullare superior


II. Pedunculi cerebellares medii
III. Pedunculi cerebellares superiors
IV. Velum medullare inferiores
V. Tela choroidea ventriculi quarti
A) II, V
B) III, IV
C) IV, V
D) II, III
E) I, III

9. It is not the branch of a.profunda brachii:


A) a. collateralis radialis
B) a. collateralis media
C) а. nutriciae humeri
D) a. collateralis ulnaris inferior
E) r. deltoideus
10. Determine which anatomical derivatives are innervated by the parasympathetic
fibers of the n. ciliares breves?

I. Eyeball
II. M. sphincter pupillae
III. Ciliary body
IV. M. dilatator pupillae
V. Vessels of the eyeball

A) II, III
B) II, V
C) I, IV
D) I, V
E) III, IV
11. Specify sources of an innervation of the mucous membrane of the nasal cavity:

A) n. oculomotorius
B) n. vestibulocochlearis
C) n. ophthalmicus
D) n. facialis
E) n. mandibularis
12. Before entering into the mandibular canal inferior alveolar nerve innervates:

A) mylohyoid muscle
B) temporalis muscle
C) lateral pterygoid muscle
D) medial pterygoid muscle
E) posterior belly of digastrics muscle
13. The nucleus of the olfactory analyzer lies in the:

A) medial surface of the occipital lobe


B) superior temporal gyrus
C) uncus of the parahippocampal gyrus
D) middle temporal gyrus
E) superior parietal lobule
14. It doesn’t belong to the isthmus rhombencephali:
A) velum medullare superius
B) pedunculi cerebellares superiores
C) trigonum lemnisci
D) upper part of the IV ventricle.
E) fornix
15. It is the part of associative system:
A) internal capsule
B) amygdaloid body
C) claustrum
D) putamen
E) external capsule

16. They do not belong to the visceral lymph nodes of pelvis:


A) nodi lymhpatici parauterini
B) nodi lymhpatici gluteales
C) nodi lymhpatici paravaginales
D) nodi lymhpatici paravesiculares
E) nodi lymhpatici pararectales
17. It is remnant of the cavity of the mesencephalon:
A) ventriculus tertius
B) aqueductus cerebri
C) central canal
D) ventriculus lateralis
E) ventriculus quartus

18. It doesn’t develop from the diencephalon:


A) commissura posterior
B) oliva
C) corpora mamillaria
D) thalamus

E) lateral geniculate body


19. The morpho-functional unit of the nervous system is called as:
A) neurofibers
B) neuroglia

C) bundle of the nerve fibers


D) neuron
E) ganglia

20. The lingual nerve connects with the chorda tympani at the level of:

A) posterior border of temporalis muscle


B) anterior border of temporalis muscle
C) lateral surface of the buccinator muscle
D) upper edge of medial pterygoid muscle
E) lower edge of medial pterygoid muscle
21. It is not supplied by the a. vesicalis inferior:

A) prostate (in male)


B) fundus of urinary bladder
C) vagina (in female)
D) rectum
E) seminal vesicles (in male)

22. It doesn’t belong to the short branches of sacral plexus:

A) n. gluteus inferior
B) n. gluteus superior
C) n. pudendus
D) n. piriformis
E) n. cutaneus femoris posterior
23. Which from the listed below is the branch of the vagus nerve?
A) n. caroticus externus
B) n. mandibularis
C) n. laryngeus superior
D) n. buccalis
E) n. caroticus internus
24. These muscles rotate the eyeball around the frontal axis:
A) medial rectus and lateral rectus
B) superior rectus and inferior rectus
C) superior oblique and inferior oblique
D) inferior oblique and medial rectus

E) superior rectus and lateral rectus


25. The front wall of the tympanic cavity is called (paries...):

A) membranaceus
B) caroticus
C) tegmentalis
D) jugularis
E) labyrithicus
26. These nerves belong to the motor nerves:

A) I, II, VIII, IX, X


B) V, VII, IX, X, XII
C) III, IV, VI, XI, XII
D) I, III,V, VIII, XI

E) VI, IX, X, XI, XII


27. It belongs to the branches of a. lienalis (splenic artery) :

A) a. gastrica dextra
B) a. gastroduodenalis
C) a. gastroepiploica sinistra
D) a. gastrica sinistra

E) a. cystica
28. Tegmentum is belonged to:

A) telencephalon
B) myelencephalon
C) diencephalon
D) metencephalon
E) mesencephalon
29. The vena saphena magna opens into the:

A) femoral vein
B) popliteal vein
C) anterior tibial vein
D) posterior tibial vein

E) marginal veins
30. The retropharyngeal lymph nodes do not accept lymph from the:
A) middle ear
B) frontal sinuses
C) lower teeth
D) nasal part of pharynx
E) root of the tongue
31. These nerves belong to the mixed nerves:
A) I, III, V, IX
B) I, II, VIII, XII
C) VI, X, XI, XII

D) VI, VII, X, XI
E) V, VII, IX, X

32. The internal carotid artery passes forward and medialward across the
I. the anterior perforated substance
II. Oculomotor nerve
III. above the optic nerve
IV. Hypophysis
V. to the commencement of the longitudinal fissure.
VI. Posterior perforated substance

A) I; III; IV
B) I; II; V
C) II; IV; VI
D) IV; V; VI
E) II; III; V
33. The terminating nuclei of which sensory nerves are not located in the brain
stem?

A) olfactory nerve
B) cochlear nerve
C) optic nerve
D) vestibular nerve

E) sensory part of the trigeminal nerve


34. It is the continuation of “n. musculocutaneus”:
A) n.cutaneus lateralis inferior
B) n.cutaneus antebrachii posterior
C) n.cutaneus antebrachii lateralis
D) n.cutaneus antebrachii medialis
E) n.cutaneus brachii posterior
35. It passes from the foramen infrapiriformis:

A) a. uterina
B) a. rectalis media
C) a. glutea inferior
D) a. vesicalis inferior
E) a. renalis
36. Inferior laryngeal branches from the recurrent laryngeal nerve supply all
muscles of the larynx except:

A) m. thyroarythenoideus
B) m. arythenoideus posterior
C) m. cricothyroideus
D) m. arythenoideus obliquus

E) m. cricoarythenoideus posterior
37. It does not belong to the motor branches of the mandibular nerve:

A) medial pterygoid nerve


B) buccal nerve
C) masseteric nerve
D) deep temporal nerve
E) lateral pterygoid nerve
38. The branch of the mandibular part of the maxillary artery is the:

A) a. meningea media
B) a. masseterica
C) a. infraorbitalis
D) aa. alveolares anteriores
E) a. palatina descendens
39. On each side of the median sulcus of the rhomboid fossa are located:

A) hypoglossal triangle
B) vestibular area
C) eminentia medialis

D) vagal triangle
E) lateral recesses
40. Which bundles pull the ciliary body towards the center (towards the lens) during
contraction

A) circular
B) longitudinal
C) radial
D) obliqua
E) lateral

41. Which of the followings is belonged to the middle ear?

A) cochlea
B) vestibule
C) semicircular canalis
D) ossicles
E) auricle
42. Where is located the central part of the lateral ventricle?
A) insula

B) frontal lobe
C) occipital lobe
D) parietal lobe
E) temporal lobe

43. The nucleus of the general skin analyzer is situated in cerebral cortex in the:
A) temporal lobe

B) insula
C) frontal lobe
D) parietal lobe
E) occipital lobe
44. Which anatomical derivatives are connected by the brachium colliculi
superioris?

A) the superior collicle and the medical geniculate body


B) the inferior collicle and medial geniculate body

C) the superior collicle and the thalamus


D) the superior collicle and lateral geniculate body
E) the inferior collicle and the lateral geniculate body
45. Zygomatic nerve passes into the orbit through the:

A) posterior ethmoid foramen


B) anterior ethmoid foramen
C) inferior orbital fissure

D) foramen ovale
E) superior orbital fissure

46. The milky cistern found in the initial part of thr thoracic duct (in %):
A) 30
B) 90
C) 25

D) 75
E) 50

47. Which from the listed below doesn’t locate on the medial surface of cerebral
hemisphere?
A) sulcus intraparietalis
B) cingulate sulcus
C) sulcus subparietalis
D) groove of corpus callosum
E) sulcus paracentralis.
48. The superior vesical artery is the branch of the:
A) a. vesicalis inferior
B) a. uterina
C) a. glutea superior
D) a. rectalis
E) a. umbilicalis
49. The lateral ventricles is belonged to the:

A) myelencephalon
B) metencephalon
C) diencephalon
D) telencephalon
E) mesencephalon
50. To which cranial nerves does nucleus tractus solitari belong?

A) V, VII, XI
B) I, II, V
C) VII, IX, XI
D) IX, X, XI
E) VII, IX, X

51. Pyloric lymph nodes are located near the:

A) right gastric artery


B) splenic vein
C) left gastric artery
D) upper gastro-duodenal artery
E) splenic artery
52. Which cranial nerve doesn’t form the trunk?

A) VIII
B) I
C) XII
D) VI
E) II
53. The infraorbital nerve enters the orbit through the:

A) superior orbital fissure


B) posterior ethmoid foramen
C) anterior ethmoid foramen
D) inferior orbital fissure
E) foramen ovale
54. The visseral branches of the thoracic aorta are the:
A) aa. intercostales anteriores
B) aa. bronchiales
C) aa. phrenicae inferiores
D) aa. phrenicae superiores

E) a. intercostalis suprema
55. Define the cranial nerve roots that emerge between the pyramid and the olive:

A) X
B) IX
C) XII
D) VIII
E) VII

56. Fovea centralis is belonged to:


A) iris
B) cornea
C) sclera
D) ciliary body
E) retina
57. They are belonged to middle ear:
I. Auricula
II. Tuba auditiva
III. Meatus acusticus externus osseus
IV. Meatus acusticus externus cartilagineus
V. Cavitas tympani

A) I;III
B) II;V
C) II;III
D) III;IV
E) IV,I
58. The superior branch of the oculomotor nerve innervates:

A) superior rectus muscle


B) inferior rectus muscle
C) inferior oblique muscle
D) medial rectus muscle
E) superior oblique muscle
59. How is called the sinus locating near the the Turkish saddle?
A) superior sagittal
B) transverse
C) rectus

D) inferior sagittal
E) cavernosus
60. Which wall of the III ventricle is formed by the pineal body?

A) lateral
B) inferior
C) posterior
D) superior
E) anterior
61. It is the direct continuation of the femoral artery:

A) popliteal artery
B) superficial epigastric artery
C) superficial circumflex iliac artery
D) profunda femoris artery
E) external pudendal artery

62. The saphenus nerve is the branch of the:


A) n. femoralis
B) n. pudendus
C) n. obturatorius
D) n. genitofemoralis
E) n. ischiadicus
63. In formation of which wall of III ventricle does take place columna fornicis?
A) anterior
B) superior
C) inferior
D) posterior
E) lateral
64. The nuclei of eyeball are:
I. The ciliary body
II. The lens
III. The sclera
IV. The aqueous humor
V. The vitreous body
VI. The iris
A) III;IV;VI
B) I;II;III
C) II;IV;V
D) I;IV;VI
E) I;III;V
65. Which anatomical derivatives are connected by brachium colliculi inferiores?

A) the inferior collicle and medial geniculate body


B) the superior collicle and medial geniculate body
C) the inferior collicle and lateral geniculate body

D) the superior collicle and pulvinar thalami


E) the superior collicle and lateral geniculate body

66. The cranial part of the parasympathetic system forms vegetative nuclei of the
A) III, VII, IX, and X cranial nerves
B) II, VIII and XI cranial nerves
C) III,IV and XII cranial nerves
D) V,VII,VIII,X cranial nerves

E) I,VI,VII and XII cranial nerves


67. The lingual nerve descends between the:

A) m. pterygoideus medialis and m. buccinator


B) m. pterygoideus lateralis and m. buccinator
C) m. buccinator and m. temporalis
D) m. pterygoideus medialis and m. pterygoideus lateralis
E) m. masseter and m. temporalis
68. This does not belong to the subcortical nuclei:

A) nucleus ruber
B) nucleus caudatus
C) corpus amygdaloideum
D) claustrum
E) nucleus lentiformis
69. Which anatomical structure does connect the inferior cerebellar peduncles
cerebellum?

A) medulla spinalis
B) medulla oblongata
C) mesencephalon
D) diencephalon

E) pons
70. It does not belong to the branches of the ophthalmic nerve:
I. N. infaorbitalis
II. N. lacrimalis
III. Rr. pterygopalatini
IV. N. nasociliaris
V. N. frontalis
VI. N. zygomaticus

A) II; III; IV
B) I; III; V

C) III; IV; VI
D) I; III; VI
E) II; IV; V
71. The abducens nerve away of the brain from?

A) between the pons and middle cerebellar peduncle


B) on the lateral lorder of the pedunculi cerebri
C) fissura postpontinia
D) on the medial border of the pedunculi cerebri
E) foramen ethmoidale anterior

72. They are belonged to white matter of the cerebral hemispheres:


I. Commissural fibers
II. External granular plate
III. Assiciative fibers
IV. Internal granular plate
V. Claustrum

A) II;V
B) III;IV
C) I;III
D) II;IV
E) I;V
73. How does the agraphia form?

A) destruction of the motor center of the written speech


B) destruction of the auditory center of the spoken speech
C) destruction of the center of the praxia

D) destruction of the motor center of the spoken speech


E) destruction of the visual center of the written speech

74. A. thoracica superior supplies:


A) V-VI intercostal muscles
B) I-II intercostal muscles
C) III-IV intercostal muscles
D) deep muscles of the neck
E) superficial muscles of the chest

75. The superficial epigastric artery is the branch of the:


A) external iliac artery
B) internal iliac artery
C) external genital artery
D) femoral artery
E) deep femoral artery
76. 1. Trigonum lemnisci is limited anteriorly by:

A) upper cerebellar pedunculi


B) brachium colliculi inferioris
C) fornix
D) sulcus lateralis mesencephali
E) internal capsule

77. Superficial fibular nerve innervates:

A) anterior muscles of thigh


B) posterior muscles of thigh
C) lateral muscles of leg
D) anterior muscles of leg
E) posterior muscles of leg
78. Which anatomical structure does connect the superior cerebellar peduncles with
cerebellum?

A) pons
B) midbrain
C) spinal cord
D) cerebral peduncles
E) vermis

79. Where is located common carotid artery bifurcation?


A) superior border of the thyroid cartilage
B) root of the tongue
C) inferior border of the thyroid cartilage

D) posterior border of the palate


E) V cervical vertebra

80. Which from the listed below is the branch of ophthalmic nerve?
A) n. masseter
B) n. frontalis
C) n. buccalis
D) n. zygomaticus
E) n. lingualis
81. It doesn’t belong to the rhomboid fossa:

A) median sulcus
B) area vestibularis
C) medial eminence
D) substance nigra
E) triangle of vagus nerve
82. Which nerves do convey the taste impilses?

A) IX, XII
B) V, VII
C) VII, IX, X
D) IX, X, XI
E) V, VII, X
83. The anterior ethmoid nerve innervates the:
I. Posterior ethmoid cells
II. Frontal sinus
III. Sphenoid sinus
IV. Mucous membrane of the nasal cavity
V. Skin tip of the nose

A) II, IV
B) I, II
C) II, III

D) I, V
E) IV, V

84. How many parts are distinguished in the internal capsule?


A) 2
B) 4
C) 6
D) 5
E) 3

85. Trigonum lemnisci is limited laterally by:

A) sulcus lateralis mesencephali


B) fornix
C) brachium colliculi inferioris
D) basilar groove
E) upper cerebellar pedunculi
86. The parts of the maxillary artery is the:
I. Mandibular
II. Frontal
III. Pterygo-palatinal
IV. Pterygoid
V. Temporal
A) I;III;V

B) III;IV;V
C) I;II;III
D) II;III;IV
E) I ;III;IV

87. The inguinal lymph nodes are located in the

A) in the wall of the pelvic cavity


B) in the deep ring of the femoral canal
C) medially from the femoral vein
D) femoral triangle, below the inguinal ligament.
E) around the large vessels
88. To which vesicle the corpus callosum is belonged to?
A) myelencephalon
B) telencephalon
C) metencephalon
D) diencephalon
E) mesencephalon

89. The core of the visual analyzer of written speech is located in:

A) superior temporal gyrus


B) supramarginal gyrus
C) angular gyrus
D) inferior frontal gyrus
E) middle temporal gyrus
90. This is belonged to the innervation zone of the n. thoracicus longus:
A) m. levator scapulae
B) m. subclavius
C) m. serratus anterior
D) m. pectoralis major
E) m. latissimus dorsi

91. The renal artery starts at the level of:

A) L4
B) L1
C) L3
D) S1
E) L2
92. Determine the source of innervation of inferior oblique muscle:

A) n. trochlearis
B) n. oculomotorius
C) n. facialis
D) n. ophthalmicus
E) n. abducens
93. The nucleus of the auditory analyzer is situated in cerebral cortex in the:

A) occipital lobe
B) temporal lobe
C) frontal lobe
D) parietal lobe
E) insula

94. The optic nerve (nn. opticus) is the second pair of cranial nerves that forms the

A) pulvinar thalami
B) nucleus of the visual analyzer
C) axons of multipolar (ganglious) nerve cells of the retina.
D) From the lateral geniculate body
E) fibers of the optic radiation
95. The recurrent laryngeal nerve doesn’t innervate:

A) cervical part of the esophagus


B) cricothyroid muscle
C) thyroid gland
D) laryngeal mucosa below the glottis
E) transverse arytenoid muscle
96. It takes place in arterial circle of the cerebrum:

A) a. cerebri anterior
B) a. chorioidea posterior
C) a. cerebri media
D) a. cerebelli superior
E) a. choroidea anterior
97. Rotate the eyeball around the anteroposterior axis

A) superior oblique and inferior oblique


B) medial rectus and lateral rectus
C) superior rectus and inferior rectus
D) superior rectus and lateral rectus
E) inferior oblique and medial rectus
98. It does not leave the cerebral part of the skull:

A) oculomotor nerve
B) glossopharyngeal nerve
C) vagus nerve
D) vestibulocochlear nerve
E) optic nerve
99. “n.masticatorius” goes to:
A) temporalis muscle
B) lateral pterygoid muscle
C) medial pterygoid muscle
D) buccinator muscle
E) masseter muscle
100. How is called the part of the lateral ventricle that located in the frontal lobe?

A) inferior horn
B) central part
C) anterior horn
D) posterior horn
E) insula.

101. The cerebellum via the upper cerebellar pedunculi is connected with:
A) pons
B) diencephalon
C) roof of the midbrain
D) spinal cord
E) medulla oblongata
102. What does the abducens nerve innervate?

A) m. rectus inferior
B) m. rectus lateralis
C) m. obliquus inferior
D) m. rectus superior
E) m. obliquus superior
103. It doesn’t belong to the visceral lymph nodes of the pelvis:
A) nodi limhpatici parauterini
B) nodi limhpatici paravesiculares
C) nodi limhpatici paravaginales

D) nodi limhpatici pararectales


E) nodi limhpatici obturatorii

104. The weight of the spinal cord at 1-year-old is (g)


A) 10
B) 22
C) 18
D) 16

E) 17
105. They are belonged to the dura mater derivatives:
I. Falx cerebri
II. Velum medullare superior
III. Tuber cinereum
IV. Falx cerebelli
V. Diaphragm of Turkish saddle

A) I; IV; V
B) I; II; III
C) I; III; IV
D) II; III; IV
E) II; IV; V
106. The thoracic part of the sympathetic trunk
A) consist of 12 or 14 ganglia
B) consist of 14 or 15 ganglia
C) consist of 11 or 13 ganglia
D) consist of 15 or 17 ganglia
E) consists of 10 or 12 ganglia
107. At which level does the common iliac artery divide into the internal and external
iliac arteries?

A) sacroiliac joint
B) iliac fossa
C) anterior superior iliac spine
D) anterior inferior iliac spine
E) promontorium
108. Which bundles stretch the lens capsule, provide distant vision of objects?

A) radial
B) circular
C) obliqua
D) longitudinal

E) meridional
109. In the centre in which layer of the eyeball is located the pupilla ?

A) cornea
B) corpus ciliares
C) retina
D) sclera
E) iris
110. The superior branch of the oculomotor nerve innervates:

A) inferior and medial rectus muscles, inferior obliquus muscle


B) inferior and medial rectus muscles
C) inferior and medial rectus muscles, the levator palpebralis superior muscles
D) superior obliquus and the lateralis rectus muscle.

E) superior rectus and the levator palpebralis superior muscles


111. Which nerve fibers contain the oculomotor nerve?

A) motor, parasympathetic
B) sympathetic and motor
C) sensory
D) motor and sensory
E) parasympathetic and sensory

112. The medial wall of the tympanic cavity is called:


A) paries labyrinthicus
B) paries tegmentalis
C) paries jugularis
D) paries membranaceus
E) paries caroticus
113. They do not belong to the processes of cerebral dura mater:
I. Cerebellar vermis
II. Falx cerebelli
III. Tela chorioidea
IV. Tentorium cerebelli
V. Diaphragm of Turkish saddle
VI. Falx cerebri
A) IV; V
B) I; VI
C) IV; VI
D) II; V
E) I; III
114. The superficial temporal artery begins at the:

A) coronoid process of the mandibula


B) neck of the mandibula
C) head of the mandibule

D) angle of the mandibula


E) lateral to the branch of the mandibula

115. The metatalamic region includes:


I. Medial geniculate body
II. Trigonum habenulae
III. Lateral geniculate body
IV. Habenula
V. Commissura habenularum

A) III;IV
B) II;V
C) I;II
D) IV;V
E) I;III
116. The roof of the fourth ventricle forms from the frontal aspect:
I. Velum medullare superior
II. Pedunculi cerebellares medii
III. Pedunculi cerebellares superioris
IV. Velum medullare inferiores
V. Tela choroidea ventriculi quarti

A) II, V
B) I, III
C) IV, V
D) II, III

E) III, IV
117. The lateral deep lymph nodes of the neck are located around the:

A) internal jugular vein


B) external jugular vein
C) internal carotid artery
D) common carotid artery
E) anterior jugular vein
118. According to the function, the internal capsule belongs to the:
A) basal nuclei
B) commissural fibers
C) projection fibers system
D) association fibers system
E) lateral ventricles
119. The largest branch of the axillary artery is:

A) superior thoracic artery


B) thoracicoacromial artery
C) lateral thoracic artery
D) subscapular artery
E) anterior artery circumflex the humerus
120. These veins are located in the lateral groove of brain:
A) vv. cerebri inferiores

B) vv. cerebri superiores


C) vv. cerebelli superiors
D) vv. cerebelli inferiores

E) vv. cerebri mediae


121. The paired visceral branches of abdominal aorta:
A) a.suprarenalis inferior, a. gastrica sinistra, a. lienalis
B) a.renalis, a. lienalis, a.suprarenalis media
C) a.renalis, a.testicularis, a.suprarenalis media
D) a. suprarenalis superior, a. mesenterica superior, a.testicularis
E) a.testicularis, a. mesenterica superior, a.suprarenalis inferior
122. “in the superior cranial fovea region.” corresponds to the motor nucleus of:

A) trigeminal nerve
B) hypoglossal nerve
C) vagus nerve
D) facial nerve
E) accessory nerve
123. It belongs to the branches of the pterygopalatine ganglion:

A) auriculotemporal nerve
B) buccal nerve
C) deep temporal nerve
D) meningeal branch
E) orbital branches

124. Semicircular canalis of the bony labyrinth divides into:

A) middle, inferior, superior


B) anterior, superior, posterior
C) lateral, middle, anterior
D) posterior, inferior, anterior
E) anterior, posterior, lateral
125. Indicate the cranial nerve roots that emerge from the dorsolateral
(posterolateral) groove of the medulla oblongata:

A) VII, VIII, IX
B) IX, X, XI
C) VI, VII, VIII
D) X, XI, XII
E) VII, IX, XI
126. Which from the listed below is opening of auditory tube?
A) pharyngeal opening
B) sphenoid opening
C) mastoid opening
D) laryngeal opening
E) temporal opening
127. Between them there is a perivascular space, spatium perichoroidale
I. Sclera
II. Choroid
III. Lens
IV. Ciliary body
V. Retina

A) II; III
B) I; II
C) IV; V
D) I; III
E) II; IV
128. It doesn’t belong to middle ear:

A) tuba auditiva
B) incus
C) malleus
D) stapes
E) labyrinthus osseus
129. Which conducting pathways is not located in the anterior funiculus of the
spinal cord?

A) tractus corticospinalis ventralis


B) tractus tectospinalis
C) fasciculus gracilis
D) fasciculus longitudinalis dorsalis

E) tractus vestibulospinalis
130. It belongs to the motor center of speech articulation:
A) gyrus angularis
B) the posterior part of the gyrus temporalis superior
C) the posterior part of the gyrus frontalis inferior
D) the posterior part of the gyrus temporalis superior and around gyrus supramarginalis
E) the posterior part of the gyrus frontalis medius
131. Which wall of the III ventricle is formed by the lamina terminalis?

A) anterior
B) posterior
C) lateral
D) inferior
E) superior

132. It descends along the canalis cruropopliteus:


A) posterior tibial artery
B) arcuate artery
C) middle genicular artery
D) anterior tibial artery
E) tarsal artery

133. The obturator nerve divides into:


A) superficial and deep branches

B) infeior and lateral branches


C) superior and inferior branches
D) anterior and posterior branches
E) medial and lateral branches
134. What is the weight of the spinal cord in the newborns (g) ?
A) 5-6
B) 2-2.5
C) 10-12
D) 7-8
E) 6-7
135. These are the branches of pterygo-palatine part of maxillary artery:

A) a. masseterica, a. meningea media, a. alveolaris superior posterior


B) aa. alveolares superiores anteriores, a.palatina desendens, a. tympanica anterior
C) a.palatina desendens, a. tympanica anterior, a. sphenopalatina
D) a. meningea media, aa. temporales profundae, a. alveolaris superior posterior
E) a. infraorbitalis, a. sphenopalatina, a. palatina desendens
136. It doesn’t belong to the branches of the axillary artery:
A) a.thoracica lateralis

B) a.throracica superior
C) a.thoracicoacromalis
D) a.suprascapularis
E) a.subscapularis

137. It doesn’t belong to the visceral branches of thoracic aorta:

A) aa. phrenicae superiores


B) rr. pericardiaci
C) rr. esophageales
D) rr. mediastinales
E) rr. bronchiales
138. Ascending aorta is continuous with the arcus aortae
A) behind the third sternocostal joint
B) behind of the hiatus aorticus
C) behind the left second sternocostal joint
D) behind the manubrium of the sternum
E) behind the fourth sternocostal joint
139. At which level the lingual nerve is joined by the chorda tympani?

A) superior border of m. pterygoideus medialis


B) inferior border of m. pterygoideus lateralis
C) midst of the m. buccinator
D) between the m. masseter and m. temporalis
E) superior border of m. buccinators
140. It is not innervated by the buccal branches of the facial nerve:
A) m. zygomaticus minor
B) m. nasalis
C) m. zygomaticus major
D) m. orbicularis oculi
E) m. levator labii superioris
141. It passes from foramen quadrilaterum:

A) a. thoracica lateralis
B) a. circumflexa humeri anterior
C) a. thoracoacromialis
D) a. subscapularis
E) a. circumflexa humeri posterior
142. The digastric branch of the facial nerve innervates:

A) Styloglossal muscle
B) Stylohyoid muscle
C) Posterior belly of digastric muscle
D) Stylopharyngeal muscle
E) Anterior belly of digastric muscle
143. The trochlear nerve emerges from the cranial cavity through:

A) foramen rotundum
B) fissura orbitalis inferior
C) fissura orbitalis superior
D) canalis opticus

E) foramen ethmoidale anterior


144. Which sinus does internal carotid artery pass through?

A) inferior sagittal
B) superior sagittal
C) rectus
D) cavernous

E) transversus
145. The anterior spino-cerebellar tract passes by:
A) cerebral peduncle

B) capsula interna
C) middle cerebellar peduncle
D) inferior cerebellar peduncle
E) superior cerebellar peduncle
146. How does alexia form?

A) destruction of the motor center of the spoken speeth


B) destruction of the auditory center of the speech signals
C) destruction of the center of the praxia
D) destruction of the motor center of the written speech
E) destruction of the vision center of the writing speech
147. How many neurons does consist of tractus cerebellorubro-spinalis pathway?

A) 2
B) 5
C) 3

D) 1
E) 4

148. The left testicular (ovarian) vein does open into:


A) v. renalis dextra
B) v. portae
C) v. lumbalis
D) v. cava interior
E) v. renalis sinistra
149. These present in formation of the lumbar plexus:

A) Th12-L4
B) Th12-L3
C) L1-L4
D) L1-L5
E) L2-L5

150. It doesn’t belong to the midbrain:

A) tuber cinereum
B) fossa interpeduncularis
C) nucleus ruber
D) substantia nigra

E) pedunculi cerebri
151. The oculomotor nerve emerges to the base of the brain from where?

A) on the medial border of the pedinculi cerebri


B) on the lateral border of the pedinculi cerebri
C) foramen ethmoidale anterior
D) fissura orbitalis inferior
E) between the pons and middle cerebellar peduncle

152. Chorda tympani leaves the skull through:

A) sphenopetrous fissure
B) foramen rotundum
C) stylomastoid foramen
D) tympano-petrosal fissure
E) petrosquamous fissure
153. Middle cervical ganglion (ganglion cervicale medium) lies
A) in front of the transverse process of the 3th cervical vertebrae
B) in front of the transverse process of the 7th cervical vertebrae
C) in front of the transverse process of the 4th cervical vertebrae
D) in front of the transverse process of the 5th cervical vertebrae
E) in front of the transverse process of the 6 th cervical vertebrae
154. The fornix starts from the:

A) infundibulum
B) lamina terminalis
C) commissura anterior
D) commissura posterior
E) mamillary body
155. The visceral branches of the thoracic aorta are the followings:
I. Rr. bronchiales
II. Rr. mediastinales
III. Superior phrenic arteries
IV. Posterior intercostalis arteries
V. Rr. pericardiaci

A) I,II,V
B) II,III,IV
C) III,IV,V
D) II,III,V

E) I,II,IV
156. The posterior ethmoid nerve innervates the:

A) the anterior part of the nasal septum


B) lateral wall of the nasal cavity
C) posterior ethmoid cells
D) the skin of the dorsum nasi
E) the posterior part of the nasal septum
157. Which muscle of the eyeball is innervated by the trochlear nerve?

A) lateral rectus

B) superior obliquus
C) inferior rectus
D) inferior obliquus
E) superior rectus
158. Which anatomical derivative is innervated by the supraorbital nerve?

A) posterior ethmoid cells, skin of the nose


B) skin of the dorsum of nasi and superior eyelid

C) the anterior part of the septum nasi, superior eyelid


D) skin of the forehead
E) lateral wall of the nasal cavity

159. It is remnant of the cavity of the diencephalon:


A) aqueductus cerebri
B) ventriculus tertius
C) central canal
D) ventriculus quartus
E) ventriculus lateralis.
160. It leads to the loss of the ability to read text :
A) sensory aphasia
B) astereognosia
C) alexia
D) agraphia
E) motor aphasia
161. How the continuation of the musculocutaneous nerve is called in the forearm ?
A) the posterior cutaneous nerve of the forearm
B) anterior interosseal nerve
C) the lateral cutaneous nerve of the forearm

D) posterior interosseal nerve


E) the medial cutaneous nerve of the forearm

162. The inferior mesenteric artery starts at the level of:


A) L3
B) L4
C) L2
D) S1
E) L1
163. The largest branch of the lumbar plexus is:

A) n.femoralis
B) n.genitofemoralis
C) n.ilioinguinalis
D) n.iliohypogastricus
E) n.obturatorius
164. The lesser (pulmonary) circulation begins in the:
A) left ventricle
B) pulmonary trunk
C) left atrium
D) left ventricle
E) right ventricle
165. Where midlle meningeal artery enter to the skull?
A) for. ovale
B) for. lacerum
C) for. rotundum

D) carotid canal
E) for. spinosum
166. Which branch of the internal carotid artery does form the choroid plexus?

A) anterior cerebral artery.


B) anterior choroid artery
C) midlle cerebral artery

D) posterior cerebral artery


E) posterior communicated artery

167. Gyrus angularis is belonged to:

A) occipital lobe
B) frontal lobe
C) temporal lobe
D) insula
E) parietal lobe
168. The lateral wall of the tympanic cavity is called (paries...):

A) jugularis
B) caroticus
C) membranaceus
D) tegmentalis
E) labyrithicus
169. The mandibular nerve passes out of the skull through the:

A) superior orbital fissure


B) foramen ovale
C) foramen rotundum
D) foramen lacerum
E) inferior orbital fissure
170. Cerebrospinal fluid passes from third to the fourth ventricle through:
A) apertura mediana ventriculi quarti
B) cerebral aqueduct
C) apertura lateralis ventriculi quarti
D) interventricular foramen
E) central canal
171. The nuclei of which nerves are located in the pons?

A) V, VI, VII, VIII


B) III, IV, V, VI
C) VI, VII, IX, X

D) IX, X, XI, XII.


E) IV, V, VI, VIII
172. It conveys postganglionic parasympathetic secretory fibers to the parotid
salivary gland:

A) lingual nerve
B) auriculotemporal nerve
C) buccal nerve
D) infraorbital nerve

E) facial nerve
173. Parietal lymph nodes are located
A) in the wall of the pelvic cavity around the large vessels.
B) in the deep ring of the femoral canal medially from the femoral vein
C) in the femoral triangle
D) below the inguinal ligament.
E) in the iliopectineal groove.
174. How is the called right atrio-ventricular valve?
A) seminularis.

B) tricuspidalis
C) bicuspidalis
D) mitralis
E) atrioventricularis sinistra

175. The cerebral peduncles are two thick swellings of the white cord located in the
A) telencephalon
B) metencephalon

C) mesencephalon
D) myelencephalon
E) diencephalon
176. The greater circulation begins in the:

A) right ventricle
B) left ventricle
C) left atrium
D) right atrium
E) aorta

177. Rr. musculares (n. ulnaris) innervate:

A) m. flexor carpi radialis


B) m. extensor carpi ulnaris
C) m. flexor digitorum superficialis
D) m. palmaris longus
E) m. flexor carpi ulnaris
178. It is the branch of a. iliaca externa:

A) a. epigastrica superficialis
B) a. epigastrica inferior
C) a. circumflexa ilium superficialis
D) a. profunda femoris
E) a. pudenda interna
179. The parietal branches of the thoracic aorta are the followings:
I. Rr. bronchiales
II. Rr. mediastinales
III. Superior phrenic arteries
IV. Posterior intercostalis arteries
V. Rr. pericardiaci

A) III,V
B) III,IV
C) IV,V

D) I,III
E) I,II
180. Posterior nasal branches of the pterygopalatine ganglion enter the nasal cavity
through:
A) posterior ethmoid foramen
B) anterior ethmoid foramen
C) infraorbital foramen
D) sphenopalatine foramen
E) olfactory foramina
181. Inferior tympanic artery is the branch of the:

A) a. auricularis posterior
B) a. occipitalis
C) a. pharyngea ascendens

D) a. temporalis superficialis
E) a. maxillaris
182. The branches of the aortic arch:
A) a. subclavia sinistra, a. carotis communis dextra, a. carotis communis sinistra
B) a. carotis communis dextra, a. subclavia dextra, a. carotis interna
C) a. carotis communis sinistra, a. carotis interna, tr. brachiocephalicus
D) a. subclavia dextra, a. carotis externa, tr. brachiocephalicus
E) tr. brachiocephalicus, a. carotis communis sinistra, a. subclavia sinistra

183. Before entering the obturator canal it gives off the ramus pubicus

A) a. obturatoria
B) a. glutea inferior
C) a. glutea superior
D) a. sacralis lateralis
E) a. profunda femoris
184. The reflex center of hearing are:
A) nucleus ruber
B) lower collicules
C) posterior perforated substance
D) upper collicules
E) black substance
185. It does not the visseral branches of the thoracic aorta:

A) aa. oesophageales
B) aa. pericardiaci
C) aa. mediastinales

D) aa. bronchiales
E) aa. phrenicae superiores
186. From which brain vesicle the rhinencephalon does develop?
A) metencephalon
B) diencephalon
C) telencephalon
D) myelencephalon
E) mesencephalon
187. The midbrain includes:
I.Tuber cinereum
II. Pedunculi cerebri
III. Pedunculi cerebellaris superiores
IV. Tectum mesencephali
V. Brachia colliculi superioris
VI. Pedunculi cerebellaris inferiores

A) III;V;VI
B) II;IV;V
C) I;IV;VI
D) I;II;III
E) II;III;IV
188. The branch of the pterygoid part of the maxillary artery is the:

A) a. masseterica
B) a. meningea media
C) a. palatina descendens
D) aa. alveolares anteriores superiors
E) a. infraorbitalis

189. At which level does thoracic duct form according to skeleton?


A) Th11
B) L3
C) L3-L4
D) Th12-L1
E) L5
190. It does not belong to the branches of the mandibular part of the maxillary
artery:

A) a. auricularis profunda
B) a. tympanica anterior
C) a. masseterica
D) a. meningea media

E) a. alveolaris inferior
191. «Ganglion» means:
A) aggregation of neuron’s cell bodies inside of the brain and spinal cord
B) connection between the unipolar and bipolar cells
C) aggregation of the neurons without capsule
D) aggregation of neuron’s cell bodies outside of the brain and spinal cord
E) connection between the multipolar cells
192. It is belonged to the mandibular part of the maxillary artery:

A) a. meningea media
B) a.palatina descendens
C) a.infraorbitalis
D) a. masseterica
E) aa.alveolares superiores anteriores
193. It is located on the anterior part of the temporal lobe under the lentiform
nucleus:
A) claustrum
B) putamen
C) amygdaloid body
D) globus pallidus

E) caudate nucleus
194. It does not belong to the branches of the pterygopalatine ganglion:
A) n.palatinus minor
B) n.palatinus major
C) nasal branches
D) meningeal branch

E) orbital branches
195. The medial branch of the external carotid artery is the :

A) a. pharyngea ascendens
B) a. ophthalmicus
C) a. auricularis posterior
D) a. facialis

E) a. sternocleidomastoidea
196. The nuclei of which nerve is located at the level of the inferior colliculi in the
tegmentum of the cerebral peduncles?

A) V

B) VII
C) IV
D) VI
E) III
197. It doesn’t belong to the muscles of the eyeball:

A) superior rectus
B) medial rectus
C) inferior obliquus
D) levator palpebrae superior
E) medial obliquus
198. It is not branch the mandibular part of the maxillary artery:

A) aa. temporalis profundae


B) a. alveolaris inferior
C) a. auricularis profunda
D) a. meningea media
E) a. tympanica anterior
199. At which level does confluens sinuum compose?

A) foramen magnum
B) internal occipital protuberance

C) inferior border of petrous part of the temporal bone


D) superior border of petrous part of the temporal bone
E) occipital crest
200. The nucleus of the stereognosis is situated in cerebral cortex in the:

A) lobulus parietalis superior


B) gyrus supramarginalis
C) the dorsal part of the gyrus frontalis medius
D) gyrus angularis
E) the dorsal part of the gyrus frontalis inferior

201. The branch of the mandibular part of the maxillary artery is the:
A) a. masseterica
B) a. meningea media
C) a. palatina descendens

D) aa. alveolares anteriores


E) a. infraorbitalis

202. Branches of the femoral artery:


A) a. epigastrica inferior, a. pudenta interna, a. circumflexa iliaca superficialis
B) a. descendens genus, a. vesicalis inferior, a. profunda femoris
C) a.circumflexa iliaca profunda, a. descendens genus, aa. pudendae externae
D) a. epigastrica superficialis, a. circumflexa iliaca profunda, a. uterina
E) a. epigastrica superficialis, a. circumflexa ilium superficialis, a. genus descendens
203. What is the length of the spinal cord in newborns (cm) ?
A) 21
B) 14
C) 16
D) 18
E) 19
204. The branches of the superficial temporal artery is the:
A) rr. parotidei, a. parietalis, a. frontalis
B) a. parietalis,rr. pterygoidei,a. masseterica

C) a. supraorbitalis, a. alveolaris inferior,a. buccalis


D) a. supraorbitalis, a. auricularis profunda, a. frontalis
E) a. masseterica, rr. pterygoidei, a. auricularis profunda, a. alveolaris inferior
205. The branches of the facial artery:

A) a. labialis superior, a. labialis inferior, a. angularis


B) a. labialis superior, a. labialis inferior, a. occipitalis
C) a. labialis inferior, a. masseterica, a. infraorbitalis

D) a. meningea media, a. angularis,a. palatina descendens


E) a. dorsalis nasi, a. maxillaris, a. angularis
206. Midlle temporal artery supply:

A) external acoustic meatus


B) orbicular oculi musculi

C) frontal muscle
D) auricula
E) temporalis muscle

207. Which branch of the radial artery does take place in the cubital network?

A) r. palmaris superficialis
B) r. carpalis palmaris
C) a. recurrens radialis
D) r. carpalis dorsalis
E) a. metacarpea dorsalis
208. From the petrosal part of the internal carotid artery begin:

A) rr. caroticotympanicae
B) a. meningea media
C) a. tympanica anterior
D) a. tympanica superior
E) a. ethmoidalis anterior
209. The marginal mandibular branch of the facial nerve passes to the:

A) m. levator labii superioris


B) m. mentalis
C) m. stylohyoideus
D) m. digastricus
E) m. nasalis
210. The submandibular ganglion located under the:
A) buccal nerve
B) inferior alveolar nerve
C) masseteric nerve
D) mental nerve
E) lingual nerve
211. The branches of the ophthalmic artery:

A) a.infraorbitalis, a.tympanica anterior, a.ethmoidalis anterior


B) a. centralis retina, a.lacrimalis, a.supraorbitalis
C) a.centralis retina, a.cerebri posterior, a.cerebri media

D) a.supraorbitalis, a.supratrochlearis, a.cerebri anterior


E) a.meningea media, a.lacrimalis, a.supratrochlearis

212. They are belonged to the medulla oblongata:


I. Posterior perforated substance
II. Nucleus ruber
III. Thalamus
IV. The olive
V. Tubercle of gracilis nucleus
VI. Tuberculum cuneatum

A) I;III;V
B) IV;V;VI
C) I;II;V
D) II;IV;V
E) III;IV;VI
213. It is the subcortical nuclei of the optic nerve:
I. Corpus geniculatum mediale
II. Corpus geniculatum laterale
III. Colliculi inferiores
IV. Colliculi superiores
V. Pulvinar thalami
A) I, III, V
B) I, IV, V
C) II, IV, V
D) I, II, III

E) II, III, IV
214. Branches of the internal carotid artery in the petrous part:
A) a.meningea media
B) a.ethmoidalis anterior
C) aa.caroticotympanicae
D) a.tympanica anterior

E) a.tympanica superior
215. The posterior spino-cerebellar tract passes by:
A) inferior cerebellar peduncle
B) superior cerebellar peduncle
C) cerebral peduncle

D) capsula interna
E) middle cerebellar peduncle
216. It belongs to the auditory center of the spoken speech:

A) gyrus angularis
B) the posterior part of the gyrus frontalis medius
C) the posterior part of the gyrus temporalis superior and around gyrus supramarginalis
D) the posterior part of the gyrus frontalis inferior
E) the posterior part of the gyrus temporalis superior
217. Which branches of the ulnar artery do supply the elbow joint:

A) a. collateralis ulnaris inferior, a. metacarpea dorsalis


B) a. collateralis ulnaris superior, а. carpalis dorsalis
C) a. recurrens ulnaris, а. carpalis palmaris

D) a. collateralis media, а. carpalis palmaris


E) a. recurrens ulnaris, a. recurrens interossea

218. The parotid lymph nodes do not accept lymph from the:

A) lower teeth
B) external surface of the auricula
C) auditory tube
D) tympanic membrane
E) eyelids
219. The nucleus salivatorius superior is the secretor nucleus of which cranial
nerve?

A) vagus nerve

B) facial nerve
C) glossopharyngeus nerve
D) oculomotorius nerve

E) trigeminal nerve
220. It doesn’t belong to the branches of the deep artery of arm:
A) r. deltoideus
B) a. nutriciae humeri
C) a. collateralis radialis
D) a. collateralis media
E) a. collateralis ulnaris inferior
221. It is belonged to the pterygoid part of maxillary artery:

A) a. infraorbitalis
B) aa. alveolares anteriores superiores
C) palatina descendens
D) a. masseterica
E) a. meningea media
222. The thoracicoacromial artery doesn’t supply:
A) I and II intercostal spaces
B) acromion
C) shoulder joint
D) deltoid muscle
E) pectoral muscle
223. It does not belong to the visceral branches of the internal iliac artery:

A) a. rectalis media
B) a. vesicalis inferior
C) a. uterina
D) a. obturatoria
E) a. pudenda interna
224. Which branch of the radial artery does supply the elbow joint?

A) r. palmaris superficialis
B) r. carpalis palmaris
C) r. carpalis dorsalis

D) а. metacarpea dorsalis
E) а. recurrens radialis

225. Which branches from listed below are belonged to the extracranial part of the
facial nerve?
I. R. stapedius
II. R. stylohyoideus
III. R. petrosus major
IV. R. marginalis mandibulae
V. R. colli
VI. Chorda tympani
A) IV;V;VI
B) I;II;III
C) III;V;VI
D) I; III;VI
E) II;IV;V
226. It does not belong to the branches of the nasociliary nerve:

A) long ciliary nerves


B) the lacrimal nerve
C) anterior ethmoidal nerve
D) infratrochlear nerve
E) posterior ethmoidal nerve

227. These veins are opened into the sagittal sinus:


A) vv. cerebri inferiores
B) v. cerebri magna
C) vv. cerebri superiores
D) vv. cerebelli inferiores

E) vv. cerebelli superiors


228. The pons:
I. Has a ventral and dorsal surface
II. Its dorsal part is formed by elements of the reticular formation
III. It corresponds to the place of hindbrain transition into the middbarin
IV. The gray substance is located on the outside consists of three layers
V. It is a coordination organ
VI. Its longitudinal fibers form the cortico-nuclear, cortico-pontini and cortico-spinal
tract
A) I;II;VI
B) III;V;VI
C) III;IV;V
D) II; IV;VI
E) I;II;III
229. Which muscle of the eyeball is innervated by the abducens nerve?

A) inferior rectus
B) superior obliquus
C) inferior obliquus

D) superior rectus
E) lateral rectus

230. The start from medial fascicle of the brachial plexus:


I. N. ulnaris
II. N. axillaris
III. N. cutaneus brachii medialis
IV. N. radillis
V. N. cutaneus antebrachii medialis
VI. N. subscapulares

A) I;II;III
B) II;IV;VI
C) I;II;VI
D) I;III;V
E) II;III;IV
231. It doesn’t belong to the lumbar plexus:

A) n. cutaneus femoris lateralis


B) n. pudendus
C) n. iliohypogastricus
D) n. obturatorius

E) n. ilioinguinalis
232. The cortex of the auditory analyser is located in the:

A) inferior frontal gyrus


B) superior frontal gyrus
C) angular gyrus
D) transverse temporal gyrus-Heshle’s gyrus
E) middle frontal gyrus
233. Phrenic nerve descends across the anterior surface of the:
A) posterior scalene muscle
B) longus colli muscle
C) anterior scalene muscle

D) longus capitis muscle


E) middle scalene muscle
234. A. gastroepiploica dextra is a branch of:

A) a. gastrica dextra
B) a. gastroduodenalis
C) a. gastrica sinistra
D) a. hepatica communis

E) a. hepatica propria
235. They are innervated by the musculocutaneous nerve:
A) the posterior group muscles of the forearm
B) the anterior group muscles of the arm
C) the skin of the dorsal surface of the forearm
D) the skin of the dorsal surface of the arm

E) the posterior group muscles of the arm


236. Inferior gluteal nerve innervates:

A) internal obturator muscle


B) piriformis muscle
C) gluteus maximus muscle
D) quadratus muscle of the thigh
E) gemellus muscle
237. Between the which intercostal spaces are posterior intercostalis arteries
locate:

A) III-VIII
B) III-XII
C) II-IX
D) IV-X
E) I-VII

238. From which nuclei is originated the fibers of the oculomotor nerve for sphincter
pupillae and ciliary muscles?
A) motor
B) parasympathetic
C) sensory
D) sympathetic
E) sensory and motor
239. These are about the vestibular part of vestibulocochlear nerve:
I. It has the ventral and dorsal nuclei
II. It carries impulses to a spiral ganglion
III. Takes static impulses
IV. It also carries out the vegetative fibes
V. It starts from the ganglion located at the bottom of the internal auditory meatus
VI. It has the medial and lateral nuclei

A) I;II;III
B) IV;V;VI
C) II;IV;V
D) I; III;VI

E) III;V;VI
240. It doesn’t belong to the visceral branches of thoracic aorta:

A) rr. bronchiales
B) rr. esophageales
C) rr. mediastinales
D) aa. intercostales posteriores
E) rr.pericardiaci
241. Where is located the inferior horn of the lateral ventricle?
A) occipital lobe

B) temporal lobe
C) frontal lobe
D) parietal lobe
E) insula

242. The dorsal part of the pons is formed by:

A) cortical-pontini paths
B) cortical-pontini paths
C) reticular formation
D) cortical-nuclear paths
E) basilar groove
243. The parietal branches of the abdominal aorta are the followings:
I. Superior mesenteric artery
II. Renal arteries
III. Aa. lumbales
IV. Aa. phrenicae inferioris
V. A. sacralis mediana
VI. Inferior mesenteric artery
A) I,II,III
B) III,IV,V
C) I,II,VI
D) II,III,V
E) II,IV,VI
244. It doesn’t belong to the branches of the basilar artery:

A) labyrinth artery
B) a.superior cerebelli
C) a.cerebri posterior

D) a.cerebelli anterior inferior


E) a.cerebri anterior

245. To which cerebral vesicle is belonged the medial geniculate body?


A) telencephalon
B) metencephalon
C) diencephalon
D) mesencephalon
E) myelencephalon

246. The unpaired visceral branches of abdominal aorta:


A) tr.coelacus, a. mesenterica superior, a.mesenterica inferior
B) tr.coelacus, a.testicularis, a.renalis
C) a. mesenterica inferior, a. colica dextra, a.suprarenalis media
D) a. colica media, a.suprarenalis inferior, a. mesenterica superior
E) a. colica dextra, a.renalis, aa. lumbales
247. The dorsal scapular nerve supplies:
I. M. levator scapulae
II. M. subclavius
III. M. trapezius
IV. M. supraspinatus
V. Mm. rhomboidei

A) IV,V
B) I,V
C) II,III

D) III,IV
E) I,II
248. Vitreous humor is located behind the:

A) cornea
B) sclera
C) lens
D) retina
E) iris

249. Medial pterygoid nerve doesn’t give branches to the:

A) tensor tympanic membrane muscle


B) temporalis muscle
C) medial pterygoid muscle
D) anterior belly of the digastric muscles
E) tensor palate muscle
250. It does not belong to the sensory branches of the mandibular nerve:

A) buccal nerve
B) meningeal branch
C) lingual nerve
D) auriculotemporal nerve
E) medial pterygoid nerve

251. They are belonged to the parietal branches of the internal iliac artery:

A) a. umbilicalis, a. iliolumbalis
B) a.obturatoria, a. rectalis media
C) a. sacralis lateralis, a. glutea superior
D) a. uterina, a. umbilicalis
E) a. rectalis superior, a. iliolumbalis
252. It does not belong to the branches of the pterygoid part of the maxillary artery:
A) aa. temporalis profundae
B) a. infraorbitalis
C) rr. pterygoidei

D) a. buccinatoria
E) a. masseterica

253. aa. suprarenalis superioresare originated from:

A) aa. lumbales
B) aa. sacralis mediana
C) truncus coeliacus
D) a. lienalis
E) aa. phrenicae inferiores
254. This small nerve emerges from the ventral surface of the brain stem in the
groove between the pons and the medulla oblongata.

A) XII
B) XI
C) I
D) VI
E) II
255. It doesn’t belong to the middle ear:

A) cavitas tympani
B) auditory tube
C) malleus
D) stapes
E) scala vestibuli
256. These nerves belong to the sensory nerves:

A) I, III, V
B) V, VII, IX,
C) I, II, VIII

D) VI, X, XII
E) IV, VII, XI
257. It doesn’t belong to the roots of the brachiocephalic vein:
A) lower thyroid vein
B) vertebral vein
C) internal thoracic vein
D) deep vein of the neck
E) lower epigastric vein
258. Crus cerebri is belonged to:

A) mesencephalon
B) telencephalon
C) diencephalon
D) myelencephalon
E) metencephalon
259. It does not belong to the layers of the eyeball:
A) fibrous
B) retina
C) mucous
D) sclera
E) vasculosa
260. It does not belong to the parietal branches of the internal iliac artery:

A) a. obturatoria
B) a. iliolumbalis
C) a. glutea superior
D) a. vesicalis inferior
E) a. glutea inferior
261. The terminal branches of the external carotid artery is:
I. A. occipitalis
II. A. pharyngea ascendens
III. A. facialis
IV. A. maxillaris
V. A. temporalis superficialis

A) II;V
B) IV;V
C) III;IV

D) I;II
E) I;III
262. It composes the costocervical trunk:

A) inferior thyroid artery


B) deep cervical artery
C) ascending cervical artery
D) suprascapular artery
E) superficial cervical artery

263. These veins are opened into the inferior petrosal sinus:
A) vv. cerebri superiores
B) vv. cerebelli inferiores
C) v.cerebri magna

D) vv. cerebelli superiores


E) vv. cerebri mediae
264. It passes from the foramen suprapiriformis:

A) a. vesicalis inferior
B) a. uterina
C) a. glutea superior
D) a. rectalis media
E) a. pudenda interna
265. What blood flows from the placenta to the fetus through the umbilical vein?

A) mixed
B) mixed with a predominance of venous
C) venous
D) arterial
E) mixed with a predominance of arterial
266. The inferior branch of the oculomotor nerve innervates:

A) inferior and medial rectus muscles, inferior obliquus muscle


B) inferior and medial rectus muscle, lateral rectus muscle
C) superior obliquus and lateral rectus muscle
D) inferior and medial rectus muscles, lateral rectus muscle

E) superior rectus and levator palpebralis superior muscles


267. Runs behind m. psoas major into the iliac fossa where its iliac branch forms an
anastomosis with the deep circumflex iliac artery rising from the external iliac
artery.

A) a. obturatoria
B) a. glutea superior
C) a. sacralis lateralis
D) a. profunda femoris
E) a. glutea inferior
268. A. canalis pterygoidei supplies:
I. upper part of the pharynx
II. Ethmoid cells
III. Auditory tube.
IV. Nasal cavity
V. Maxillary sinus
VI. tympanic cavity

A) I,III,VI
B) I,II,III
C) IV,V,VI
D) III,V,VI

E) II,IV,V
269. Into which vein does open the lower epigastric vein?
A) v. thoracica interna
B) v. аxillaris
C) v. iliaca externa
D) v. iliaca interna
E) v. femoralis
270. Which muscle is supplied by the external branch of the superior laryngeal
nerve?
A) m. arythenoideus obliquus
B) m. arythenoideus posterior
C) m. thyroarythenoideus
D) m. cricoarythenoideus posterior

E) m. cricothyroideus
271. The parietal branches of the thoracic aorta is the:

A) aa. intercostales anteriores, aa. intercostales posteriores


B) aa. intercostales posteriores,aa. phrenicae inferiors
C) aa. phrenicae inferiores, aa. lumbales

D) a. intercostalis suprema, aa. phrenicae inferiors


E) aa. intercostales posteriors, aa. phrenicae superiores
272. Where is located the transverse sinus ?

A) in the inferior concave margin of the falx cerebri


B) on the posterior margin of the cerebellum
C) between the foramen caecum and internal occipital protuberance
D) in the confluence sinuum

E) in the connecting place of the falx cerebri with the tentorium cerebelli
273. It leaves the pelvis through the foramen suprapiriforme and reaches the gluteal
muscle along with the superior gluteal nerve.
A) a. glutea inferior
B) a. glutea superior
C) a. obturatoria
D) a. profunda femoris
E) a. sacralis lateralis
274. The sacral part of the sympathetic trunk consists of the
A) 2 to 5 sacral ganglia
B) 3 to 4 of sacral ganglia
C) 4to 6 ganglia
D) 3 to 6 sacral ganglia
E) 3 to 5 ganglia
275. A. comitans nervi ischiadici is the branch of:
A) a. obturatoria
B) a. sacralis lateralis
C) a. profunda femoris
D) a. glutea inferior
E) a. glutea superior
276. What is the length of the right lymph duct (mm) ?
A) 40-42
B) 20-22
C) 50-52

D) 10-12
E) 30-32
277. Which segments of the spinal cord do have a lateral horn?
A) Thoracic, lumbar and sacral
B) cervical and thoracic
C) ThI-XII and LI-II
D) CVIII, ThI-XII and LI-III
E) Only thoracic
278. Tympanic membrane consist of:

A) it `s whole
B) 4 parts
C) 3 parts
D) it has 2 perforations
E) 2 parts
279. The fibers of the oculomotor nerve that innervate the sphincter pupil and ciliary
body muscles are originated from which nuclei?

A) motor
B) sensory and motor
C) parasympathetic

D) sympathetic and sensory


E) sensory
280. The otic ganglion located just below the:
A) foramen rotundum
B) foramen ovale
C) foramen lacerum
D) sphenopalatine foramen
E) foramen spinosum
281. The anterior wall of the III ventricle is formed by:

I. Commissura anterior
II. Commissura habenularum
III. Lamina terminalis
IV. Substantia perforate posterior
V. Columnae fornicis
VI. Corpus pineale
A) III, IV, V
B) I, III, V

C) II, III, V
D) II, IV, VI
E) I, II, IV
282. Which cranial nerves do start from the myelencephalon?

A) III, IV, XI, XII


B) III, IV, V, VI
C) IX, X, XI, XII

D) III, VI, VII, VIII


E) V, VI, VII, VIII
283. It doesn’t belong to the branches of the sacral plexus:

A) n. gluteus superior
B) n. gluteus inferior
C) n. genitofemoralis
D) n. pudendus

E) r. piriformis
284. Structure, responsible to accommodation, is:

A) iris
B) sclera
C) cornea
D) retina
E) ciliary body
285. The right and left lumbar lymphatic trunks dont collect lymph from the
A) right upper extremite
B) lower extremites
C) pelvis
D) organs of abdominal cavity
E) walls of abdominal cavity

286. The cell body of the first neuron of corticonuclear tract is located in the
A) cerebral cortex of the supramarginal gyrus
B) cerebral cortex of the angular gyrus
C) cerebral cortex of the precentral gyrus
D) cerebral cortex of the parahipocampal gyrus
E) cerebral cortex of the lingual gyrus

287. What happens if the vision center of written speech is destroyed?


A) motor aphasia
B) asteregnosia
C) sensor aphasia
D) agraphia
E) alexia
288. This is belonged to the innervation zone of the phrenicus nerve:

A) pericardium, mediastinal pleura, hepatic ligaments, diaphragm


B) infrahyoid muscles,diаphраgm, mediastinal pleura
C) hepatic ligaments,geniohyoid muscle,sternocleidomastoid muscle

D) sternocleidomastoid muscle, pericardium, mediastinal pleura,diaphrаgm


E) mediastinal pleura,hepatic ligaments,infrahyoid muscles, trapezius muscle
289. They are belonged to the peripheral nervous system:
I. Ganglia
II. Brain
III. Cranial nerves
IV. Plexuses
V. Spinal cord
A) I;III;IV
B) III;IV;V
C) II;III;V
D) II;III;IV
E) I;III;V
290. They are innervated by the femoral nerve:

A) medial group muscles of the thigh


B) anterior group muscles of the leg
C) anterior group muscles of the thigh

D) posterior group muscles of the leg


E) posterior group muscles of the thigh
291. The descending part of the aorta is the:
A) bulbus aortae, arcus aortae
B) pars abdominalis aorta,bulbus aortae
C) pars thoracica aortae, pars abdominalis aorta
D) bifurcatio aortae, pars ascendens
E) isthmus aortae,pars thoracica aortae
292. It is not branches of the pterygo-palatinal part of the maxillary artery:

A) a. canalis pterygoidei
B) a. palatina descendens
C) a. palatina ascendens
D) a. sphenopalatina

E) a. infraorbitalis
293. These are innervated by the obturator nerve:
A) anterior group muscles of the leg
B) medial group muscles of the thigh
C) posterior group muscles of the thigh
D) anterior group muscles of the thigh
E) posterior group muscles of the leg

294. Specify sources of an innervation of the superior rectus eyeball muscle:

A) n. oculomotorius
B) n. trochlearis
C) n. abducens
D) n. glossopharyngeus
E) n. facialis
295. The chorda tympani passes out from the tympanic cavity through:

A) fissura petrosquamosa
B) canalicus tympanicus
C) fissura petrotympanica
D) foramen rotundum
E) foramen stylomastoideum
296. Lesser petrosal nerve is a continuation of:

A) tympanic nerve
B) deep petrosal nerve
C) greater petrosal nerve
D) chorda tympani
E) vagus nerve

297. 1. It belongs to the visual center of the written speech:


A) the posterior part of the gyrus temporalis superior
B) the posterior part of the gyrus frontalis medius
C) the posterior part of the gyrus temporalis superior and around supramarginalis

D) gyrus angularis
E) the posterior part of the gyrus frontalis inferior
298. Which anatomical structure does connect the middle cerebellar peduncles with
cerebellum?
A) thalamus

B) medulla oblongata
C) vermis
D) pons
E) superior medullary velum

299. The parasympathet1. c part (pars parasympathica) of the autonomic nervous


system divides into
A) into 5 parts
B) into 7 parts
C) into 3 parts
D) into 2 parts
E) into 4 parts
300. It supplies the laryngeal mucosa below the vocal cords:
A) internal division of the superior laryngeal nerve
B) external division of the superior laryngeal nerve
C) pharyngeal branches of the glossopharyngeal nerve
D) recurrent laryngeal nerve
E) pharyngeal branches of the vagus nerve
1. Which muscles are innervated by branches exiting from ansa cervicalis?

1. Stylohyoid

2. Omohyoid

3. Mylohyoid

4. Sternothyroid

5. Digastric

6. Sternohyoid

2. Which of the followings are located in midbrain?

I. Substantia grisea centralis

2. Medial nuclei of mamillary body

3. Paraventricular nuclei

4. Nucleus of tuber cinereum

5. Substantia nigra

6. Lateral nuclei of mamillary body

7. Nucleus ruber

3. The anterior spino-cerebellar tract passes by:

A. cerebral peduncle

B. middle cerebellar peduncle

C. superior cerebellar peduncle

D. capsula interna

E. inferior cerebellar peduncle


4. The terminating nuclei of which sensory nerves are not located in the brain
stem?

A. cochlear nerve

B. vestibular nerve

C. sensory part of the trigeminal nerve

D. olfactory nerve

E. optic nerve

5. The infraorbital nerve enters the orbit through the:

A. inferior orbital fissure

B. anterior ethmoid foramen

C. posterior ethmoid foramen

D. foramen ovale

E. superior orbital fissure

6. The retropharyngeal lymph nodes do not accept lymph from the:

A. root of the tongue

B. frontal sinuses

C. lower teeth

E. middle ear

F. nasal part of pharynx


7. It is not supplied by the a. vesicalis inferior:

A. prostate (in male )

B. seminal vesicles (in male )

C. fundus of urinary bladder

D. rectum

E. vagina (in female )

8. They are belonged to the dura mater derivatives:

l. Falx cerebri
ll. Velum medullare superior
Ill. Tuber cinereum
IV. Falx cerebelli
V. Diaphragm of Turkish saddle

A. Il; Ill; IV
B. l; Il; Ill
C. Il; IV; V
D. l; Ill; IV
E. I; IV; V

9. Define tectum mesencephalicus:

6
10. It is the subcortical nuclei of the optic nerve:

l. Corpus geniculatum mediale

ll. Corpus geniculatum laterale

Ill. Colliculi inferiores

IV. Colliculi superiores

V. Pulvinar thalami

A. II, IV, V

B. Il, Ill, IV

C. l, Ill, V

D. l, Il, Ill

E. l, IV, V

11. The nucleus of the auditory analyzer is situated in cerebral cortex in


the:

A. parietal lobe

B. frontal lobe

C. insula

D. temporal lobe

E. occipital lobe
12. It is the subcortical nuclei of the optic nerve:

l. Corpus geniculatum mediale

Il. Corpus geniculatum laterale

Ill. Colliculi inferiores

IV. Colliculi superiores

V. Pulvinar thalami

A. Il, IV, V

B. Il, Ill, IV

C. l, Ill, V

D. l, Il, Ill

E. I, IV, V
14. They belong to basal nuclei:

1. insula

2. internal capsule

3. corpus striatum

4. claustrum

5. external capsule

6. corpus amygdaloideum

7. marginal capsule

16. The descending part of the aorta is the:

A. bulbus aortae, arcus aortae

B. isthmus aortae,pars thoracica aortae

C. pars thoracica aortae, pars abdominalis aorta

D. bifurcatio aortae, pars ascendens

E. pars abdominalis aorta,bulbus aortae


17. Define the cranial nerve roots that emerge between the pyramid and
the olive:

A. XII

B. X

C. VII

D. IX

E. VII

18. Determine compliance:

Trochlear nerve Page 374

Trigeminal nerve Page 483

19. Define the hypoglossal nerve:

12
20. The thoracic part of the sympathetic trunk

A. consist of 12 or 14 ganglia

B. consist of 14 or 15 ganglia

C. consist of 11 or 13 ganglia

D. consist of 15 or 17 ganglia

E. consists of 10 or 12 ganglia

21. How the continuation of the musculocutaneous nerve is called in the

forearm ?

A. the medial cutaneous nerve of the forearm

B. the posterior cutaneous nerve of the forearm

C. the lateral cutaneous nerve of the forearm

D. posterior interosseal nerve

E. anterior interosseal nerve

22. Determine compliance:

Internal thoracic artery Page 691

Lateral thoracic artery Page 699

23. Find the accordance:

superior cerebellar peduncle Page 856

inferior cerebellar peduncle Page 856


24. It refers to the branches of the cervical plexus [[1]], the branches of
the lumbar plexus [[2]] and the branches of the sacral plexus [[3]].

[[1]] Page 451 gl

[[2]] Page 723 gl

[[3]] Page 737 gl

25. Forms the anterior wall of the third ventricle [[1]], posterior wall [[2]],
lateral wall [[3]].

[[1]] lamina cinerea terminalis or page 116

[[2]] epithalamic, habenular commissures and pineal body

[[3]] medial surfaces of the 2 thalami

26. The sacral part of the sympathetic trunk consists of the

A. 3 to 4 of sacral ganglia

B. 2 to 5 sacral ganglia

C. 3 to 5 ganglia

D. 3 to 6 sacral ganglia

E. 4 to 6 ganglia
27. They are belonged to middle ear:

I. Auricula

ll. Tuba auditiva

Ill. Meatus acusticus externus osseus

IV. Meatus acusticus externus cartilagineus

V. Cavitas tympani

A. l; lll

B. Il; lll

C. Ill; lV

D. II; V

E. IV, I

28. Define the long ciliary nerve:

1
29. Define the zygomatic nerve:

12

31. Superior vena cava is formed by the connection of the [[1]] at the level
of the junction of the right I rib and [[2]].

[[1]] right and left brachiocephalic veins Page 566

[[2]] sternum

32. Determine compliance:

To vascular layer of eyeball The vascular layer of the eyeball consists of


the choroid,the ciliary body and the iris Page 336

To fibrous layer of eyeball The fibrous membrane is divided into cornea


and sclera. The border between the cornea and the sclera is the circular
groove of the sclera, sulcus sclerae. Inside the sulcus is the venous sinus
of the sclera (Schlemm's canal). Page 335
33. Determine compliance:

to the branches of internal thoracic artery belong Page 691

to the branches of costocervical trunk belong Page 693

35. It does not belong to the branches of the thyrocervical trunk:

1. Subscapular artery

2. Superior thyroid artery

3. Transverse cervical artery

4. Suprascapular artery

5. Ascending cervical artery

6. Inferior thyroid artery

36. These are belonged to old structures of cerebellum.

1. Dentate nucleus

2. Flocculus

3. Emboliformis nucleus

4. Globosus nucleus

5. Fastigii nucleus (maybe idk this depends on ur luck)

6. Vermis
37. Which nerves belong to the branches of the lumbar plexus?

1. n.femoralis

2. n.ilioinquinalis

3. n.pudendus

4. n.gluteus inferior

5. n.gluteus superior

6. n.iliohypogastricus

42. Find the accordance:

n.petrosus major is the branch of n.facialis

n.tympanicus is the branch of n.glossopharyngeus

45. Sulcus intermedius posterior divides the posterior funiculus into two
parts:

1. Fasciculus cuneatus

2. Lateral pyramidal tract

3. Fasciculus gracilis

4. Anterior pyramidal tract

5. Dorsal spinocerebellar tract or Flechsig’s tract

6. Anterior spinocerebellar tract or Gower’s tract

7. Rubrospinal tract
46. It does not belong to the branches of the thyrocervical trunk:

1. Transverse cervical artery

2. suprascapular artery

3. superior thyroid artery

4. subscapular artery

5. inferior thyroid artery

6. Ascending cervical artery

47. Vascular layer of eyeball consists of:

1. Ciliary body

2. Choroid

3. pigmented layer

4. Iris

5. Conjunctiva

6. Outer nuclear layer

7. Optic disc

48. Determine compliance:

Visceral lymph nodes of the pelvis Lymph nodes located in body cavities
(pectoral cavity, abdominal cavity, pelvic cavity) and close to organs are
called visceral lymph nodes –nоdi lymphatici viscerales

Parietal lymph nodes of the pelvis the nodes located in the walls of these
cavities are called parietal - nodi lymphatici parietals Page 836
49. Facial nerve: (good luck lol)

1. Forms the parotid plexus

2. Divides into extracranial branches in facial canal

3. Innervates the muscles of facial expression

4. The greater petrosal nerve carries sympathetic fibers

5. Innervates the stylopharyngeal muscle

6. Innervates the mylohyoid muscle

7. Innervates the anterior belly of digastric muscle

50. Branches exiting from parotid plexus:

1. Zygomatic branches

2. Communicating branch to glossopharyngeal nerve

3. Cervical branches

4. Stylohyoid branch

5. Buccal branches

6. Communicating branch to tympanic plexus

51. Define the a.gastroepiplocia dextra:

7 idk maybe
53. Determine compliance:

Vestibule Page 782

Cochlear cavity Page 783

54. Define corpus callosum:

1
55. Define the otic ganglion:

13

56. Determine the nucleus ambiguous:

15
57. Determine compliance:

the visual analyzer of written speech is situated in is situated in the angular gyrus
of the inferior parietal lobule. If it is damaged, the ability to read is lost (alexia) but vision is
preserved

the auditory analyzer of spoken speech is located in the posterior part of the
superior temporal gyrus Page 821

58. The structures which take part in formation of posterior wall of Ill
ventricle:

1. claustrum

2. habenular commissure

3. epithalamic commissure

4. velum medullare superius

5. posterior surface of pons

6. pineal body

59. To the short branches of the sacral plexus belong:

1. n.iliohypogastricus

2. n.genitofemoralis

3. n.pudendus

4. n.gluteus inferior

5. n.ilioinguinalis

6. n.gluteus superior
60. These are belonged to basal nuclei:

1. Amygdaloid body

2. Capsula externa

3. Capsula extrema

4. Capsula interna

5. Claustrum

6. Striate body

61. Determine compliance:

The posterior wall of the third ventricle posterior wall is formed by the
epithalamic, habenular commissures and pineal body

The tela chorioidea of the third ventricle is triangle shaped fold and
consists of 2 layers of the pia mater Page 827

62. Determine compliance:

Unpaired visceral branches of the abdominal aorta Page 757

Paired visceral branches of the abdominal aorta Page 765

64. These are belonged to old structures of brain by filogenesis:

1. Caudate nucleus

2. Brainstem

3. Lentiform nuclei

4. Amygdaloid body

5. Claustrum

6. Cerebellum

7. Isocortex
68. Inferior and superior vena cava, coronary sinus are opened into the
[[1]] and pulmonary veins into the [[2]].

[[1]] right atrium

[[2]] left atrium

69. [[1]] refers to the branches of the truncus thyrocervicalis and [[2]] to
truncus costocervicalis.

[[1]] 692

[[2]] 693

70. Determine compliance.

Vestibulocochlear nerve Page 142

Facial nerves good luck

71. the body of the [[1]] lies in the spiral ganglion that contain the [[2]]

[[1]] first neuron

[[2]] bipolar cells


72. [[1]] is belonged to branches of head part of vagus nerve, [[2]] is
belonged to branches of cervical part of vagus nerve and, [[3]] - to the
branches of thoracic part of same nerve.

[[1]] meningeal branch and auricular branch Page 506

[[2]] Pharyngeal branche and superior laryngeal nerve and recurrent


laryngeal nerve and upper cardiac branches Page 507

[[3]] Thoracic cardiac branches and Tracheal branches and Bronchial


branches and Esophageal branches Page 510

74. Find the accordance:

belong to telencephalon The cerebral hemispheres (hemispheriae cerebri)


consist of the pallium, the rhinencephalon and the basal ganglia
(subcortical ganglia).Page 807

belong to diencephalon The diencephalon (Fig. 28) is divided into two


parts: dorsal (phylogenetically younger) part, the thalamencephalon and
ventral (phylogenetically older) part, thehypothalamus. The third ventricle
is a remnant of the primary diencephalic cavity Page 822

75. Determine compliance: Page 704

a.collateraiis ulnaris inferior

76. Centers which are located in the frontal lobe:

1. Auditory analyser of spoken speech.

2. The motor analyzer of speech articulation.

3. Motor analyser of written speech.

4. Auditory analyser.

5. Visual analyser of written speech.


4/20/2021 Midterm exam: İnsan anatomiyası və tibbi terminologiya - 2 (EN) (MPF2): Attempt review

Mənim əsas səhifəm / Mənim fənlərim / İnsan anatomiyası və tibbi terminologiya - 2 (EN) / Ümumi

/ Midterm exam: İnsan anatomiyası və tibbi terminologiya - 2 (EN) (MPF2)

Testə başlandı Tuesday, 20 April 2021, 12:30 PM


State Finished
Sona çatdı Tuesday, 20 April 2021, 12:57 PM
Vaxt keçib 27 dəq. 46 san.
Qiymət 20 (80%) maksimumdan 16

Question 1
Correct

Mark 1.00 out of 1.00

How many lumbar segments does the spinal cord have?

A. 5 

B. 12

C. 8

D. 6

E. 4

Question 2
Incorrect

Mark 0.00 out of 1.00

It happens when the motor analyzer of speech articulation is damaged:

A. apraxia 

B. agraphia

C. tactile agnosia

D. sensory aphasia

E. motor aphasia

https://ets.amu.edu.az/mod/quiz/review.php?attempt=68702&cmid=7722#question-98660-17 1/8
4/20/2021 Midterm exam: İnsan anatomiyası və tibbi terminologiya - 2 (EN) (MPF2): Attempt review

Question 3
Incorrect

Mark 0.00 out of 1.00

The floor of the central part of the lateral ventricle is formed by:
I. Body of caudate nucleus
II. Head of caudate nucleus
III. Thalamus
IV. Corpus fornicis
V. Stria terminalis

A. I; III; V

B. I; II; III 

C. I; III; IV

D. III; IV; V

E. II; III; V

Question 4
Correct

Mark 1.00 out of 1.00

Where is the gyrus parahippocampalis situated?

A. in the posterior part of the gyrus temporalis inferior

B. between the hippocampal and collateral sulcus 

C. between the inferior temporal and occipitotemporalis sulcus

D. between the lateral and superior temporal sulcus

E. between the superior and inferior temporal sulcus

Question 5
Correct

Mark 1.00 out of 1.00

It doesn’t belong to the isthmus rhombencephali:

A. pedunculi cerebellaris superiores and trigonum lemnisci

B. velum medullare inferius 

C. velum medullare superius

D. trigonum lemnisci

E. pedunculi cerebellaris superiores

https://ets.amu.edu.az/mod/quiz/review.php?attempt=68702&cmid=7722#question-98660-17 2/8
4/20/2021 Midterm exam: İnsan anatomiyası və tibbi terminologiya - 2 (EN) (MPF2): Attempt review

Question 6
Correct

Mark 1.00 out of 1.00

In the superior triangle region of the rhomboid fossa is situated the nuclei of the

A. I,II.V,XII cranial nerves

B. V, VI, VII , VIII cranial nerves 

C. V,II,III,VI cranial nerves

D. VIII,VII,III,VI cranial nerves

E. X,XI,XII cranial nerves

Question 7
Correct

Mark 1.00 out of 1.00

Which of the following are the parts of the tympanic membrane?


I. Pars mucosa
II. Pars muscularis
III. Pars tensa
IV. Pars flaccida
V. Pars membranacea

A. IV, V

B. III, V

C. II, IV

D. I, II

E. III, IV 

Question 8
Correct

Mark 1.00 out of 1.00

The glossopharyngeal nerve descends through the upper part of the neck between the:

A. internal carotid artery and external carotid artery

B. internal jugular vein and external carotid artery

C. internal jugular vein and common carotid artery

D. internal jugular vein and external jugular vein

E. internal jugular vein and internal carotid artery 

https://ets.amu.edu.az/mod/quiz/review.php?attempt=68702&cmid=7722#question-98660-17 3/8
4/20/2021 Midterm exam: İnsan anatomiyası və tibbi terminologiya - 2 (EN) (MPF2): Attempt review

Question 9
Correct

Mark 1.00 out of 1.00

The anterior ethmoid nerve enters the cranial cavity through:

A. optic canal

B. anterior ethmoid foramen 

C. olfactory foramina

D. medial side of the fissure orbitalis superior

E. lateral side of the fissure orbitalis inferior

Question 10
Correct

Mark 1.00 out of 1.00

Where is located the superior border of the heart?

A. line connecting upper edges of VII ribs

B. line connecting upper edges the IV ribs

C. line connecting lower edges of the V ribs

D. line connecting lower edges of VI ribs

E. line connecting upper edges of III ribs 

Question 11
Correct

Mark 1.00 out of 1.00

At which level does the external carotid artery divide into two terminating branches?

A. coronoid process of mandible

B. angle of mandible

C. thyroid cartilage

D. neck of mandible 

E. hyoid bone

https://ets.amu.edu.az/mod/quiz/review.php?attempt=68702&cmid=7722#question-98660-17 4/8
4/20/2021 Midterm exam: İnsan anatomiyası və tibbi terminologiya - 2 (EN) (MPF2): Attempt review

Question 12
Incorrect

Mark 0.00 out of 1.00

It doesn’t belong to the branches of thyrocervical trunk:

A. ascending cervical artery

B. subscapular artery

C. superficial cervical artery

D. suprascapular artery 

E. inferior thyroid artery

Question 13
Correct

Mark 1.00 out of 1.00

It is not the branch of the ulnar artery:

A. а. carpalis palmaris

B. а. recurrens ulnaris

C. а. carpalis dorsalis

D. а. interossea communis

E. а. recurrens radialis 

Question 14
Correct

Mark 1.00 out of 1.00

Where is located the inferior sagittal sinus?

A. between the foramen caecum and internal occipital protuberance

B. in the inferior concave margin of the falx cerebri 

C. in the transverse sinus groove of the occipital bone

D. in the connecting place of the falx cerebri with the tentorium cerebelli

E. in the confluence cinuum

https://ets.amu.edu.az/mod/quiz/review.php?attempt=68702&cmid=7722#question-98660-17 5/8
4/20/2021 Midterm exam: İnsan anatomiyası və tibbi terminologiya - 2 (EN) (MPF2): Attempt review

Question 15
Correct

Mark 1.00 out of 1.00

What is called the continuation of the right ascending lumbar vein?

A. accessory hemiazygos vein

B. azygos vein 

C. pericardiaco-phrenic veins

D. internal thoracic vein

E. hemiazygos vein

Question 16
Correct

Mark 1.00 out of 1.00

To the branches of the ophthalmic nerve belong:

1. n. auriculotemporalis

2. n. nasociliaris 

3. n. frontalis 

4. n. petrosus major

5. n. lacrimalis 

6. n. infraorbitalis

Question 17
Incorrect

Mark 0.00 out of 1.00

The anterior wall of the third ventricle is formed by:

1. columns of the fornix

2. epithalamic commissure

3. pineal body

4. thalamus 

5. anterior commissure 

6. terminal lamina 

https://ets.amu.edu.az/mod/quiz/review.php?attempt=68702&cmid=7722#question-98660-17 6/8
4/20/2021 Midterm exam: İnsan anatomiyası və tibbi terminologiya - 2 (EN) (MPF2): Attempt review

Question 18
Correct

Mark 1.00 out of 1.00

Find the accordance:

The superior laryngeal nerve is the branch of vagus nerve 

The tympanic nerve is the branch of glossopharyngeal nerve 

Question 19
Correct

Mark 1.00 out of 1.00

The ulnar artery departs from the [[1]] artery at the level of the [[2]] of the ulna.

[[1]] brachial 

[[2]] coronoid process 

Question 20
Correct

Mark 1.00 out of 1.00

Find the colliculus facialis:

Answer: 6 

◄ Attendance

Keç ...

A. Abdullayev. 7. General overview of the peripheral nervous system. The classification, development, variation, and anomalies of the
cranial nerves. ►

https://ets.amu.edu.az/mod/quiz/review.php?attempt=68702&cmid=7722#question-98660-17 7/8
4/20/2021 Midterm exam: İnsan anatomiyası və tibbi terminologiya - 2 (EN) (MPF2): Attempt review

https://ets.amu.edu.az/mod/quiz/review.php?attempt=68702&cmid=7722#question-98660-17 8/8
4/20/2021 Midterm exam: İnsan anatomiyası və tibbi terminologiya - 2 (EN) (MPF2): Attempt review

Mənim əsas səhifəm / Mənim fənlərim / İnsan anatomiyası və tibbi terminologiya - 2 (EN) / Ümumi

/ Midterm exam: İnsan anatomiyası və tibbi terminologiya - 2 (EN) (MPF2)

Testə başlandı Tuesday, 20 April 2021, 12:30 PM


State Finished
Sona çatdı Tuesday, 20 April 2021, 12:57 PM
Vaxt keçib 27 dəq. 46 san.
Qiymət 20 (80%) maksimumdan 16

Question 1
Correct

Mark 1.00 out of 1.00

How many lumbar segments does the spinal cord have?

A. 5 

B. 12

C. 8

D. 6

E. 4

Question 2
Incorrect

Mark 0.00 out of 1.00

It happens when the motor analyzer of speech articulation is damaged:

A. apraxia 

B. agraphia

C. tactile agnosia

D. sensory aphasia

E. motor aphasia

https://ets.amu.edu.az/mod/quiz/review.php?attempt=68702&cmid=7722#question-98660-17 1/8
4/20/2021 Midterm exam: İnsan anatomiyası və tibbi terminologiya - 2 (EN) (MPF2): Attempt review

Question 3
Incorrect

Mark 0.00 out of 1.00

The floor of the central part of the lateral ventricle is formed by:
I. Body of caudate nucleus
II. Head of caudate nucleus
III. Thalamus
IV. Corpus fornicis
V. Stria terminalis

A. I; III; V

B. I; II; III 

C. I; III; IV

D. III; IV; V

E. II; III; V

Question 4
Correct

Mark 1.00 out of 1.00

Where is the gyrus parahippocampalis situated?

A. in the posterior part of the gyrus temporalis inferior

B. between the hippocampal and collateral sulcus 

C. between the inferior temporal and occipitotemporalis sulcus

D. between the lateral and superior temporal sulcus

E. between the superior and inferior temporal sulcus

Question 5
Correct

Mark 1.00 out of 1.00

It doesn’t belong to the isthmus rhombencephali:

A. pedunculi cerebellaris superiores and trigonum lemnisci

B. velum medullare inferius 

C. velum medullare superius

D. trigonum lemnisci

E. pedunculi cerebellaris superiores

https://ets.amu.edu.az/mod/quiz/review.php?attempt=68702&cmid=7722#question-98660-17 2/8
4/20/2021 Midterm exam: İnsan anatomiyası və tibbi terminologiya - 2 (EN) (MPF2): Attempt review

Question 6
Correct

Mark 1.00 out of 1.00

In the superior triangle region of the rhomboid fossa is situated the nuclei of the

A. I,II.V,XII cranial nerves

B. V, VI, VII , VIII cranial nerves 

C. V,II,III,VI cranial nerves

D. VIII,VII,III,VI cranial nerves

E. X,XI,XII cranial nerves

Question 7
Correct

Mark 1.00 out of 1.00

Which of the following are the parts of the tympanic membrane?


I. Pars mucosa
II. Pars muscularis
III. Pars tensa
IV. Pars flaccida
V. Pars membranacea

A. IV, V

B. III, V

C. II, IV

D. I, II

E. III, IV 

Question 8
Correct

Mark 1.00 out of 1.00

The glossopharyngeal nerve descends through the upper part of the neck between the:

A. internal carotid artery and external carotid artery

B. internal jugular vein and external carotid artery

C. internal jugular vein and common carotid artery

D. internal jugular vein and external jugular vein

E. internal jugular vein and internal carotid artery 

https://ets.amu.edu.az/mod/quiz/review.php?attempt=68702&cmid=7722#question-98660-17 3/8
4/20/2021 Midterm exam: İnsan anatomiyası və tibbi terminologiya - 2 (EN) (MPF2): Attempt review

Question 9
Correct

Mark 1.00 out of 1.00

The anterior ethmoid nerve enters the cranial cavity through:

A. optic canal

B. anterior ethmoid foramen 

C. olfactory foramina

D. medial side of the fissure orbitalis superior

E. lateral side of the fissure orbitalis inferior

Question 10
Correct

Mark 1.00 out of 1.00

Where is located the superior border of the heart?

A. line connecting upper edges of VII ribs

B. line connecting upper edges the IV ribs

C. line connecting lower edges of the V ribs

D. line connecting lower edges of VI ribs

E. line connecting upper edges of III ribs 

Question 11
Correct

Mark 1.00 out of 1.00

At which level does the external carotid artery divide into two terminating branches?

A. coronoid process of mandible

B. angle of mandible

C. thyroid cartilage

D. neck of mandible 

E. hyoid bone

https://ets.amu.edu.az/mod/quiz/review.php?attempt=68702&cmid=7722#question-98660-17 4/8
4/20/2021 Midterm exam: İnsan anatomiyası və tibbi terminologiya - 2 (EN) (MPF2): Attempt review

Question 12
Incorrect

Mark 0.00 out of 1.00

It doesn’t belong to the branches of thyrocervical trunk:

A. ascending cervical artery

B. subscapular artery

C. superficial cervical artery

D. suprascapular artery 

E. inferior thyroid artery

Question 13
Correct

Mark 1.00 out of 1.00

It is not the branch of the ulnar artery:

A. а. carpalis palmaris

B. а. recurrens ulnaris

C. а. carpalis dorsalis

D. а. interossea communis

E. а. recurrens radialis 

Question 14
Correct

Mark 1.00 out of 1.00

Where is located the inferior sagittal sinus?

A. between the foramen caecum and internal occipital protuberance

B. in the inferior concave margin of the falx cerebri 

C. in the transverse sinus groove of the occipital bone

D. in the connecting place of the falx cerebri with the tentorium cerebelli

E. in the confluence cinuum

https://ets.amu.edu.az/mod/quiz/review.php?attempt=68702&cmid=7722#question-98660-17 5/8
4/20/2021 Midterm exam: İnsan anatomiyası və tibbi terminologiya - 2 (EN) (MPF2): Attempt review

Question 15
Correct

Mark 1.00 out of 1.00

What is called the continuation of the right ascending lumbar vein?

A. accessory hemiazygos vein

B. azygos vein 

C. pericardiaco-phrenic veins

D. internal thoracic vein

E. hemiazygos vein

Question 16
Correct

Mark 1.00 out of 1.00

To the branches of the ophthalmic nerve belong:

1. n. auriculotemporalis

2. n. nasociliaris 

3. n. frontalis 

4. n. petrosus major

5. n. lacrimalis 

6. n. infraorbitalis

Question 17
Incorrect

Mark 0.00 out of 1.00

The anterior wall of the third ventricle is formed by:

1. columns of the fornix

2. epithalamic commissure

3. pineal body

4. thalamus 

5. anterior commissure 

6. terminal lamina 

https://ets.amu.edu.az/mod/quiz/review.php?attempt=68702&cmid=7722#question-98660-17 6/8
4/20/2021 Midterm exam: İnsan anatomiyası və tibbi terminologiya - 2 (EN) (MPF2): Attempt review

Question 18
Correct

Mark 1.00 out of 1.00

Find the accordance:

The superior laryngeal nerve is the branch of vagus nerve 

The tympanic nerve is the branch of glossopharyngeal nerve 

Question 19
Correct

Mark 1.00 out of 1.00

The ulnar artery departs from the [[1]] artery at the level of the [[2]] of the ulna.

[[1]] brachial 

[[2]] coronoid process 

Question 20
Correct

Mark 1.00 out of 1.00

Find the colliculus facialis:

Answer: 6 

◄ Attendance

Keç ...

A. Abdullayev. 7. General overview of the peripheral nervous system. The classification, development, variation, and anomalies of the
cranial nerves. ►

https://ets.amu.edu.az/mod/quiz/review.php?attempt=68702&cmid=7722#question-98660-17 7/8
4/20/2021 Midterm exam: İnsan anatomiyası və tibbi terminologiya - 2 (EN) (MPF2): Attempt review

https://ets.amu.edu.az/mod/quiz/review.php?attempt=68702&cmid=7722#question-98660-17 8/8
Mǝnim ǝsas sǝhifǝm / Mǝnim fǝnlǝrim / İnsan anatomiyası vǝ tibbi terminologiya - 2 (EN) / Ümumi
/ Midterm exam: İnsan anatomiyası vǝ tibbi terminologiya - 2 (EN) (MPF2)

Testǝ başlandı Tuesday, 20 April 2021, 12N30 PM


State Finished
Sona çatdı Tuesday, 20 April 2021, 12N57 PM
Vaxt keçib 26 dǝq. 4 san.
Qiymǝt 20 (95%) maksimumdan 19

Question 1 In the medial border of the cerebral peduncle emerges


Correct

Mark 1.00 out A. the trochlear nerve


of 1.00
B. the abducent nerve

C. tthe facial nerve

D. the oculomotor nerve !

E. the trigeminal nerve

Question 2 It happens when the motor analyzer of speech articulation is damaged:


Correct

Mark 1.00 out A. agraphia


of 1.00
B. tactile agnosia

C. sensory aphasia

D. motor aphasia !

E. apraxia

Question 3 The extremal capsule is the white matter separating:


Correct

Mark 1.00 out A. globus pallidus medialis and nucleus caudatus


of 1.00
B. claustrum from cortex of insula !

C. globi pallidus medialis et lateralis

D. globus pallidus lateralis and putamen

E. putamen and claustrum

Question 4 Around the cerebral aqueduct is placed:


Correct

Mark 1.00 out A. substantia grisea centralis !


of 1.00
B. substantia perforata anterior

C. substantia perforata posterior et substantia grisea centralis

D. substantia perforata posterior

E. substantia perforata anterior et substantia grisea centralis


Question 5 The roof of the fourth ventricle in front consists of:
Correct I. Superior cerebellar peduncles
Mark 1.00 out II. Middle cerebellar peduncles
of 1.00 III. Inferior cerebellar peduncles
IV. Velum medullare superius
V. Velum medullare inferius

A. III; V

B. I; II

C. III; IV

D. I; IV !

E. II; III

Question 6 The olive separated from the pyramid by the:


Correct

Mark 1.00 out A. intermediate sulcus


of 1.00
B. anterolateral sulcus !

C. posterior funiculus

D. posterior median sulcus

E. posterolateral sulcus

Question 7 Which cranial nerve receives sound wave stimuli from the small bones of the middle ear, inner ear and
Correct interpret this in the brain as hearing and also control human balance?
Mark 1.00 out
of 1.00 A. olfactory nerve

B. glossopharyngeal nerve

C. intermedius nerve

D. vestibulocochlear nerve !

E. facial nerve

Question 8 It arises from the facial nerve just above the stylomastoid foramen:
Correct

Mark 1.00 out A. the nerve to the stapedius


of 1.00
B. branch to the tympanic plexus

C. greater petrosal nerve

D. lesser petrosal nerve

E. chorda tympani !

Question 9 The vagus nerve passes out of the skull through:


Correct

Mark 1.00 out A. foramen lacerum


of 1.00
B. foramen ovale

C. foramen juguluare !

D. foramen spinosum
D. foramen spinosum

E. foramen rotundum

Question 10 Which layers of the serous pericardium?


Correct

Mark 1.00 out A. mediastinal, costal


of 1.00
B. costal,parietal

C. parietal, costal

D. diaphragmaric,visceral

E. parietal, visceral !

Question 11 Thoracic aorta locates between the:


Correct

Mark 1.00 out A. ThI-ThXII


of 1.00
B. ThIII-ThVIII

C. ThIV-ThX

D. ThVI-ThXII

E. ThIV-ThXII !

Question 12 The branches of the brachiocephalic trunk:


Correct I. A. subclavia dextra
Mark 1.00 out II. A. carotis communis sinistra
of 1.00 III. A. carotis communis dextra
IV. A. subclavia sinistra
V. A. thyroidea ima

A. I;III !

B. III;IV

C. IV;V

D. II;III

E. I;II

Question 13 To which vein does v.basilica open?


Correct

Mark 1.00 out A. axillary


of 1.00
B. external jugular

C. brachial !

D. anterior jugular

E. subclavian

Question 14 It does not belong to the tributaries of the external jugular vein:
Correct

Mark 1.00 out A. v. auricularis posterior


of 1.00
B. v. canalis pterygoidei !
C. v. occipitalis

D. v. suprascapularis

E. v. jugularis anterior

Question 15 What is called the continuation of the left ascending lumbar vein?
Correct

Mark 1.00 out A. internal thoracic vein


of 1.00
B. pericardiaco-phrenic veins

C. accessory hemiazygos vein

D. hemiazygos vein !

E. azygos vein

Question 16 The anterior wall of the third ventricle is formed by:


Correct

Mark 1.00 out 1. epithalamic commissure


of 1.00
2. terminal lamina !

3. anterior commissure !

4. columns of the fornix !

5. pineal body

6. thalamus

Question 17 To the nuclei of the medulla oblongata belong:


Correct

Mark 1.00 out 1. nucleus cuneatus !


of 1.00
2. nucleus ruber

3. nucleus gracilis !

4. nucleus caudatus

5. nucleus dentatus

6. nucleus olivaris !

Question 18 Find the accordance:


Correct

Mark 1.00 out


A. thoracoacromialis is the branch of axillary artery !
of 1.00

A. interossea communis is the branch of ulnar artery !

Question 19 Middle superior alveolar nerve passes in the lateral wall of the [[1]] and innervates the upper [[2]] teeth, gums
Correct and the cheek.
Mark 1.00 out
of 1.00
[[1]] maxillary sinus !

[[2]] premolar !
Question 20 Find the ulnar nerve:
Incorrect

Mark 0.00 out


of 1.00

Answer: 9 "

A. Abdullayev. 7. General overview


of the peripheral nervous system.
◀ Attendance Keç ... The classification, development,
variation, and anomalies of the
cranial nerves. ▶︎
Mǝnim ǝsas sǝhifǝm / Mǝnim fǝnlǝrim / İnsan anatomiyası vǝ tibbi terminologiya - 2 (EN) / Ümumi
/ Midterm exam: İnsan anatomiyası vǝ tibbi terminologiya - 2 (EN) (MPF2)

Testǝ başlandı Tuesday, 20 April 2021, 12N30 PM


State Finished
Sona çatdı Tuesday, 20 April 2021, 12N57 PM
Vaxt keçib 26 dǝq. 4 san.
Qiymǝt 20 (95%) maksimumdan 19

Question 1 In the medial border of the cerebral peduncle emerges


Correct

Mark 1.00 out A. the trochlear nerve


of 1.00
B. the abducent nerve

C. tthe facial nerve

D. the oculomotor nerve !

E. the trigeminal nerve

Question 2 It happens when the motor analyzer of speech articulation is damaged:


Correct

Mark 1.00 out A. agraphia


of 1.00
B. tactile agnosia

C. sensory aphasia

D. motor aphasia !

E. apraxia

Question 3 The extremal capsule is the white matter separating:


Correct

Mark 1.00 out A. globus pallidus medialis and nucleus caudatus


of 1.00
B. claustrum from cortex of insula !

C. globi pallidus medialis et lateralis

D. globus pallidus lateralis and putamen

E. putamen and claustrum

Question 4 Around the cerebral aqueduct is placed:


Correct

Mark 1.00 out A. substantia grisea centralis !


of 1.00
B. substantia perforata anterior

C. substantia perforata posterior et substantia grisea centralis

D. substantia perforata posterior

E. substantia perforata anterior et substantia grisea centralis


Question 5 The roof of the fourth ventricle in front consists of:
Correct I. Superior cerebellar peduncles
Mark 1.00 out II. Middle cerebellar peduncles
of 1.00 III. Inferior cerebellar peduncles
IV. Velum medullare superius
V. Velum medullare inferius

A. III; V

B. I; II

C. III; IV

D. I; IV !

E. II; III

Question 6 The olive separated from the pyramid by the:


Correct

Mark 1.00 out A. intermediate sulcus


of 1.00
B. anterolateral sulcus !

C. posterior funiculus

D. posterior median sulcus

E. posterolateral sulcus

Question 7 Which cranial nerve receives sound wave stimuli from the small bones of the middle ear, inner ear and
Correct interpret this in the brain as hearing and also control human balance?
Mark 1.00 out
of 1.00 A. olfactory nerve

B. glossopharyngeal nerve

C. intermedius nerve

D. vestibulocochlear nerve !

E. facial nerve

Question 8 It arises from the facial nerve just above the stylomastoid foramen:
Correct

Mark 1.00 out A. the nerve to the stapedius


of 1.00
B. branch to the tympanic plexus

C. greater petrosal nerve

D. lesser petrosal nerve

E. chorda tympani !

Question 9 The vagus nerve passes out of the skull through:


Correct

Mark 1.00 out A. foramen lacerum


of 1.00
B. foramen ovale

C. foramen juguluare !

D. foramen spinosum
D. foramen spinosum

E. foramen rotundum

Question 10 Which layers of the serous pericardium?


Correct

Mark 1.00 out A. mediastinal, costal


of 1.00
B. costal,parietal

C. parietal, costal

D. diaphragmaric,visceral

E. parietal, visceral !

Question 11 Thoracic aorta locates between the:


Correct

Mark 1.00 out A. ThI-ThXII


of 1.00
B. ThIII-ThVIII

C. ThIV-ThX

D. ThVI-ThXII

E. ThIV-ThXII !

Question 12 The branches of the brachiocephalic trunk:


Correct I. A. subclavia dextra
Mark 1.00 out II. A. carotis communis sinistra
of 1.00 III. A. carotis communis dextra
IV. A. subclavia sinistra
V. A. thyroidea ima

A. I;III !

B. III;IV

C. IV;V

D. II;III

E. I;II

Question 13 To which vein does v.basilica open?


Correct

Mark 1.00 out A. axillary


of 1.00
B. external jugular

C. brachial !

D. anterior jugular

E. subclavian

Question 14 It does not belong to the tributaries of the external jugular vein:
Correct

Mark 1.00 out A. v. auricularis posterior


of 1.00
B. v. canalis pterygoidei !
C. v. occipitalis

D. v. suprascapularis

E. v. jugularis anterior

Question 15 What is called the continuation of the left ascending lumbar vein?
Correct

Mark 1.00 out A. internal thoracic vein


of 1.00
B. pericardiaco-phrenic veins

C. accessory hemiazygos vein

D. hemiazygos vein !

E. azygos vein

Question 16 The anterior wall of the third ventricle is formed by:


Correct

Mark 1.00 out 1. epithalamic commissure


of 1.00
2. terminal lamina !

3. anterior commissure !

4. columns of the fornix !

5. pineal body

6. thalamus

Question 17 To the nuclei of the medulla oblongata belong:


Correct

Mark 1.00 out 1. nucleus cuneatus !


of 1.00
2. nucleus ruber

3. nucleus gracilis !

4. nucleus caudatus

5. nucleus dentatus

6. nucleus olivaris !

Question 18 Find the accordance:


Correct

Mark 1.00 out


A. thoracoacromialis is the branch of axillary artery !
of 1.00

A. interossea communis is the branch of ulnar artery !

Question 19 Middle superior alveolar nerve passes in the lateral wall of the [[1]] and innervates the upper [[2]] teeth, gums
Correct and the cheek.
Mark 1.00 out
of 1.00
[[1]] maxillary sinus !

[[2]] premolar !
Question 20 Find the ulnar nerve:
Incorrect

Mark 0.00 out


of 1.00

Answer: 9 "

A. Abdullayev. 7. General overview


of the peripheral nervous system.
◀ Attendance Keç ... The classification, development,
variation, and anomalies of the
cranial nerves. ▶︎
4/20/2021 Midterm exam: İnsan anatomiyası və tibbi terminologiya - 2 (EN) (MPF2): Attempt review

Dashboard / My courses / İnsan anatomiyası və tibbi terminologiya - 2 (EN) / General

/ Midterm exam: İnsan anatomiyası və tibbi terminologiya - 2 (EN) (MPF2)

Started on Tuesday, 20 April 2021, 12:30 PM


State Finished
Completed on Tuesday, 20 April 2021, 12:54 PM
Time taken 23 mins 52 secs
Grade 19 out of 20 (95%)

Question 1
Correct

Mark 1.00 out of 1.00

In the posterior funiculus of the spinal cord are located:

A. tractus tectospinalis, tractus vestibulospinalis

B. tractus rubrospinalis, tractus spinocerebellaris dorsalis

C. fasciculus gracilis, tractus tectospinalis

D. tractus vestibulospinalis, tractus spinocerebellaris ventralis

E. fasciculus cuneatus, fasciculus gracilis 

Question 2
Correct

Mark 1.00 out of 1.00

It happens when the motor analyzer of speech articulation is damaged:

A. sensory aphasia

B. agraphia

C. motor aphasia 

D. tactile agnosia

E. apraxia

https://ets.amu.edu.az/mod/quiz/review.php?attempt=68873&cmid=7722 1/8
4/20/2021 Midterm exam: İnsan anatomiyası və tibbi terminologiya - 2 (EN) (MPF2): Attempt review

Question 3
Correct

Mark 1.00 out of 1.00

The extremal capsule is the white matter separating:

A. globus pallidus lateralis and putamen

B. putamen and claustrum

C. claustrum from cortex of insula 

D. globi pallidus medialis et lateralis

E. globus pallidus medialis and nucleus caudatus

Question 4
Correct

Mark 1.00 out of 1.00

It does not belong to epithalamus:

A. epiphysis

B. habenular trigone

C. hypophysis 

D. epithalamic commissure

E. habenular commissure

Question 5
Correct

Mark 1.00 out of 1.00

Where is the vallecula cerebelli located?

A. on the superior surface of cerebellum

B. on the anterior border of cerebellum

C. on the anterior and posterior borders of cerebellum

D. on the inferior surface of cerebellum 

E. on the posterior border of cerebellum

https://ets.amu.edu.az/mod/quiz/review.php?attempt=68873&cmid=7722 2/8
4/20/2021 Midterm exam: İnsan anatomiyası və tibbi terminologiya - 2 (EN) (MPF2): Attempt review

Question 6
Correct

Mark 1.00 out of 1.00

The lower part of the rhomboid fossa is corresponded to:

A. diencephalon

B. mesencephalon

C. medulla oblongata 

D. telencephalon

E. metencephalon

Question 7
Correct

Mark 1.00 out of 1.00

They are belonged to middle ear:


I. Auricula
II. Tuba auditiva
III. Meatus acusticus externus osseus
IV. Meatus acusticus externus cartilagineus
V. Cavitas tympani

A. II;III

B. IV;V

C. III;IV

D. II;V 

E. I;III

Question 8
Correct

Mark 1.00 out of 1.00

The glossopharyngeal nerve descends through the upper part of the neck between the:

A. internal jugular vein and common carotid artery

B. internal jugular vein and external carotid artery

C. internal jugular vein and external jugular vein

D. internal carotid artery and external carotid artery

E. internal jugular vein and internal carotid artery 

https://ets.amu.edu.az/mod/quiz/review.php?attempt=68873&cmid=7722 3/8
4/20/2021 Midterm exam: İnsan anatomiyası və tibbi terminologiya - 2 (EN) (MPF2): Attempt review

Question 9
Correct

Mark 1.00 out of 1.00

It does not belong to the branches of the ophthalmic nerve:


I. N. infaorbitalis
II. N. lacrimalis
III. Rr. pterygopalatini
IV. N. nasociliaris
V. N. frontalis
VI. N. zygomaticus

A. I; III; V

B. II; IV; V

C. I; III; VI 

D. II; III; IV

E. III; IV; VI

Question 10
Correct

Mark 1.00 out of 1.00

Where is located the superior border of the heart?

A. line connecting upper edges of III ribs 

B. line connecting lower edges of the V ribs

C. line connecting upper edges of VII ribs

D. line connecting upper edges the IV ribs

E. line connecting lower edges of VI ribs

Question 11
Correct

Mark 1.00 out of 1.00

Thoracic aorta locates between the:

A. ThI-ThXII

B. ThVI-ThXII

C. ThIV-ThX

D. ThIV-ThXII 

E. ThIII-ThVIII

https://ets.amu.edu.az/mod/quiz/review.php?attempt=68873&cmid=7722 4/8
4/20/2021 Midterm exam: İnsan anatomiyası və tibbi terminologiya - 2 (EN) (MPF2): Attempt review

Question 12
Incorrect

Mark 0.00 out of 1.00

The vertebral artery (a. vertebralis), is the first branch of the

A. subclavian artery

B. superficial cervical artery

C. inferior thyroid artery

D. suprascapular artery

E. subclavian artery 

Question 13
Correct

Mark 1.00 out of 1.00

To which vein does v.cephalica vein open?

A. axillary 

B. anterior jugular

C. brachial

D. external jugular

E. subclavian

Question 14
Correct

Mark 1.00 out of 1.00

At which level does superior vena cava form?

A. I sternocostal synchondrosis 

B. III sternocostal joint

C. IV sternocostal joint

D. sternoclavicular joint

E. II sternocostal joint

https://ets.amu.edu.az/mod/quiz/review.php?attempt=68873&cmid=7722 5/8
4/20/2021 Midterm exam: İnsan anatomiyası və tibbi terminologiya - 2 (EN) (MPF2): Attempt review

Question 15
Correct

Mark 1.00 out of 1.00

This doesn’t belong to the cutaneous branches of the cervical plexus:

A. n. auricularis magnus

B. n. transversus colli

C. n. subclavius 

D. n. occipitalis minor

E. n. supraclavicularis

Question 16
Correct

Mark 1.00 out of 1.00

The anterior wall of the third ventricle is formed by:

1. thalamus

2. anterior commissure 

3. columns of the fornix 

4. pineal body

5. terminal lamina 

6. epithalamic commissure

Question 17
Correct

Mark 1.00 out of 1.00

To the nuclei of the medulla oblongata belong:

1. nucleus cuneatus 

2. nucleus olivaris 

3. nucleus ruber

4. nucleus caudatus

5. nucleus dentatus

6. nucleus gracilis 

https://ets.amu.edu.az/mod/quiz/review.php?attempt=68873&cmid=7722 6/8
4/20/2021 Midterm exam: İnsan anatomiyası və tibbi terminologiya - 2 (EN) (MPF2): Attempt review

Question 18
Correct

Mark 1.00 out of 1.00

Find the accordance:

Gyrus angularis embraces posterior end of the superior temporal sulcus 

Gyrus supramarginalis embraces posterior branch of the lateral sulcus 

Question 19
Correct

Mark 1.00 out of 1.00

The external capsule is a layer of [[1]] matter that is situated between the claustrum and [[2]] nucleus.

[[1]] white 

[[2]] lentiform 

Question 20
Correct

Mark 1.00 out of 1.00

Find the colliculus facialis:

Answer: 6 

◄ Attendance

Jump to...

A. Abdullayev. 7. General overview of the peripheral nervous system. The classification, development, variation, and anomalies of the
cranial nerves. ►

https://ets.amu.edu.az/mod/quiz/review.php?attempt=68873&cmid=7722 7/8
4/20/2021 Midterm exam: İnsan anatomiyası və tibbi terminologiya - 2 (EN) (MPF2): Attempt review

https://ets.amu.edu.az/mod/quiz/review.php?attempt=68873&cmid=7722 8/8
4/20/2021 Midterm exam: İnsan anatomiyası və tibbi terminologiya - 2 (EN) (MPF2): Attempt review

Dashboard / My courses / İnsan anatomiyası və tibbi terminologiya - 2 (EN) / General

/ Midterm exam: İnsan anatomiyası və tibbi terminologiya - 2 (EN) (MPF2)

Started on Tuesday, 20 April 2021, 12:30 PM


State Finished
Completed on Tuesday, 20 April 2021, 12:54 PM
Time taken 23 mins 52 secs
Grade 19 out of 20 (95%)

Question 1
Correct

Mark 1.00 out of 1.00

In the posterior funiculus of the spinal cord are located:

A. tractus tectospinalis, tractus vestibulospinalis

B. tractus rubrospinalis, tractus spinocerebellaris dorsalis

C. fasciculus gracilis, tractus tectospinalis

D. tractus vestibulospinalis, tractus spinocerebellaris ventralis

E. fasciculus cuneatus, fasciculus gracilis 

Question 2
Correct

Mark 1.00 out of 1.00

It happens when the motor analyzer of speech articulation is damaged:

A. sensory aphasia

B. agraphia

C. motor aphasia 

D. tactile agnosia

E. apraxia

https://ets.amu.edu.az/mod/quiz/review.php?attempt=68873&cmid=7722 1/8
4/20/2021 Midterm exam: İnsan anatomiyası və tibbi terminologiya - 2 (EN) (MPF2): Attempt review

Question 3
Correct

Mark 1.00 out of 1.00

The extremal capsule is the white matter separating:

A. globus pallidus lateralis and putamen

B. putamen and claustrum

C. claustrum from cortex of insula 

D. globi pallidus medialis et lateralis

E. globus pallidus medialis and nucleus caudatus

Question 4
Correct

Mark 1.00 out of 1.00

It does not belong to epithalamus:

A. epiphysis

B. habenular trigone

C. hypophysis 

D. epithalamic commissure

E. habenular commissure

Question 5
Correct

Mark 1.00 out of 1.00

Where is the vallecula cerebelli located?

A. on the superior surface of cerebellum

B. on the anterior border of cerebellum

C. on the anterior and posterior borders of cerebellum

D. on the inferior surface of cerebellum 

E. on the posterior border of cerebellum

https://ets.amu.edu.az/mod/quiz/review.php?attempt=68873&cmid=7722 2/8
4/20/2021 Midterm exam: İnsan anatomiyası və tibbi terminologiya - 2 (EN) (MPF2): Attempt review

Question 6
Correct

Mark 1.00 out of 1.00

The lower part of the rhomboid fossa is corresponded to:

A. diencephalon

B. mesencephalon

C. medulla oblongata 

D. telencephalon

E. metencephalon

Question 7
Correct

Mark 1.00 out of 1.00

They are belonged to middle ear:


I. Auricula
II. Tuba auditiva
III. Meatus acusticus externus osseus
IV. Meatus acusticus externus cartilagineus
V. Cavitas tympani

A. II;III

B. IV;V

C. III;IV

D. II;V 

E. I;III

Question 8
Correct

Mark 1.00 out of 1.00

The glossopharyngeal nerve descends through the upper part of the neck between the:

A. internal jugular vein and common carotid artery

B. internal jugular vein and external carotid artery

C. internal jugular vein and external jugular vein

D. internal carotid artery and external carotid artery

E. internal jugular vein and internal carotid artery 

https://ets.amu.edu.az/mod/quiz/review.php?attempt=68873&cmid=7722 3/8
4/20/2021 Midterm exam: İnsan anatomiyası və tibbi terminologiya - 2 (EN) (MPF2): Attempt review

Question 9
Correct

Mark 1.00 out of 1.00

It does not belong to the branches of the ophthalmic nerve:


I. N. infaorbitalis
II. N. lacrimalis
III. Rr. pterygopalatini
IV. N. nasociliaris
V. N. frontalis
VI. N. zygomaticus

A. I; III; V

B. II; IV; V

C. I; III; VI 

D. II; III; IV

E. III; IV; VI

Question 10
Correct

Mark 1.00 out of 1.00

Where is located the superior border of the heart?

A. line connecting upper edges of III ribs 

B. line connecting lower edges of the V ribs

C. line connecting upper edges of VII ribs

D. line connecting upper edges the IV ribs

E. line connecting lower edges of VI ribs

Question 11
Correct

Mark 1.00 out of 1.00

Thoracic aorta locates between the:

A. ThI-ThXII

B. ThVI-ThXII

C. ThIV-ThX

D. ThIV-ThXII 

E. ThIII-ThVIII

https://ets.amu.edu.az/mod/quiz/review.php?attempt=68873&cmid=7722 4/8
4/20/2021 Midterm exam: İnsan anatomiyası və tibbi terminologiya - 2 (EN) (MPF2): Attempt review

Question 12
Incorrect

Mark 0.00 out of 1.00

The vertebral artery (a. vertebralis), is the first branch of the

A. subclavian artery

B. superficial cervical artery

C. inferior thyroid artery

D. suprascapular artery

E. subclavian artery 

Question 13
Correct

Mark 1.00 out of 1.00

To which vein does v.cephalica vein open?

A. axillary 

B. anterior jugular

C. brachial

D. external jugular

E. subclavian

Question 14
Correct

Mark 1.00 out of 1.00

At which level does superior vena cava form?

A. I sternocostal synchondrosis 

B. III sternocostal joint

C. IV sternocostal joint

D. sternoclavicular joint

E. II sternocostal joint

https://ets.amu.edu.az/mod/quiz/review.php?attempt=68873&cmid=7722 5/8
4/20/2021 Midterm exam: İnsan anatomiyası və tibbi terminologiya - 2 (EN) (MPF2): Attempt review

Question 15
Correct

Mark 1.00 out of 1.00

This doesn’t belong to the cutaneous branches of the cervical plexus:

A. n. auricularis magnus

B. n. transversus colli

C. n. subclavius 

D. n. occipitalis minor

E. n. supraclavicularis

Question 16
Correct

Mark 1.00 out of 1.00

The anterior wall of the third ventricle is formed by:

1. thalamus

2. anterior commissure 

3. columns of the fornix 

4. pineal body

5. terminal lamina 

6. epithalamic commissure

Question 17
Correct

Mark 1.00 out of 1.00

To the nuclei of the medulla oblongata belong:

1. nucleus cuneatus 

2. nucleus olivaris 

3. nucleus ruber

4. nucleus caudatus

5. nucleus dentatus

6. nucleus gracilis 

https://ets.amu.edu.az/mod/quiz/review.php?attempt=68873&cmid=7722 6/8
4/20/2021 Midterm exam: İnsan anatomiyası və tibbi terminologiya - 2 (EN) (MPF2): Attempt review

Question 18
Correct

Mark 1.00 out of 1.00

Find the accordance:

Gyrus angularis embraces posterior end of the superior temporal sulcus 

Gyrus supramarginalis embraces posterior branch of the lateral sulcus 

Question 19
Correct

Mark 1.00 out of 1.00

The external capsule is a layer of [[1]] matter that is situated between the claustrum and [[2]] nucleus.

[[1]] white 

[[2]] lentiform 

Question 20
Correct

Mark 1.00 out of 1.00

Find the colliculus facialis:

Answer: 6 

◄ Attendance

Jump to...

A. Abdullayev. 7. General overview of the peripheral nervous system. The classification, development, variation, and anomalies of the
cranial nerves. ►

https://ets.amu.edu.az/mod/quiz/review.php?attempt=68873&cmid=7722 7/8
4/20/2021 Midterm exam: İnsan anatomiyası və tibbi terminologiya - 2 (EN) (MPF2): Attempt review

https://ets.amu.edu.az/mod/quiz/review.php?attempt=68873&cmid=7722 8/8
4/20/2021 Midterm exam: İnsan anatomiyası və tibbi terminologiya - 2 (EN) (MPF2): Attempt review

Mənim əsas səhifəm / Mənim fənlərim / İnsan anatomiyası və tibbi terminologiya - 2 (EN) / Ümumi

/ Midterm exam: İnsan anatomiyası və tibbi terminologiya - 2 (EN) (MPF2)

Testə başlandı Tuesday, 20 April 2021, 12:30 PM


State Finished
Sona çatdı Tuesday, 20 April 2021, 12:57 PM
Vaxt keçib 27 dəq. 46 san.
Qiymət 20 (80%) maksimumdan 16

Question 1
Correct

Mark 1.00 out of 1.00

How many lumbar segments does the spinal cord have?

A. 5 

B. 12

C. 8

D. 6

E. 4

Question 2
Incorrect

Mark 0.00 out of 1.00

It happens when the motor analyzer of speech articulation is damaged:

A. apraxia 

B. agraphia

C. tactile agnosia

D. sensory aphasia

E. motor aphasia

https://ets.amu.edu.az/mod/quiz/review.php?attempt=68702&cmid=7722#question-98660-17 1/8
4/20/2021 Midterm exam: İnsan anatomiyası və tibbi terminologiya - 2 (EN) (MPF2): Attempt review

Question 3
Incorrect

Mark 0.00 out of 1.00

The floor of the central part of the lateral ventricle is formed by:
I. Body of caudate nucleus
II. Head of caudate nucleus
III. Thalamus
IV. Corpus fornicis
V. Stria terminalis

A. I; III; V

B. I; II; III 

C. I; III; IV

D. III; IV; V

E. II; III; V

Question 4
Correct

Mark 1.00 out of 1.00

Where is the gyrus parahippocampalis situated?

A. in the posterior part of the gyrus temporalis inferior

B. between the hippocampal and collateral sulcus 

C. between the inferior temporal and occipitotemporalis sulcus

D. between the lateral and superior temporal sulcus

E. between the superior and inferior temporal sulcus

Question 5
Correct

Mark 1.00 out of 1.00

It doesn’t belong to the isthmus rhombencephali:

A. pedunculi cerebellaris superiores and trigonum lemnisci

B. velum medullare inferius 

C. velum medullare superius

D. trigonum lemnisci

E. pedunculi cerebellaris superiores

https://ets.amu.edu.az/mod/quiz/review.php?attempt=68702&cmid=7722#question-98660-17 2/8
4/20/2021 Midterm exam: İnsan anatomiyası və tibbi terminologiya - 2 (EN) (MPF2): Attempt review

Question 6
Correct

Mark 1.00 out of 1.00

In the superior triangle region of the rhomboid fossa is situated the nuclei of the

A. I,II.V,XII cranial nerves

B. V, VI, VII , VIII cranial nerves 

C. V,II,III,VI cranial nerves

D. VIII,VII,III,VI cranial nerves

E. X,XI,XII cranial nerves

Question 7
Correct

Mark 1.00 out of 1.00

Which of the following are the parts of the tympanic membrane?


I. Pars mucosa
II. Pars muscularis
III. Pars tensa
IV. Pars flaccida
V. Pars membranacea

A. IV, V

B. III, V

C. II, IV

D. I, II

E. III, IV 

Question 8
Correct

Mark 1.00 out of 1.00

The glossopharyngeal nerve descends through the upper part of the neck between the:

A. internal carotid artery and external carotid artery

B. internal jugular vein and external carotid artery

C. internal jugular vein and common carotid artery

D. internal jugular vein and external jugular vein

E. internal jugular vein and internal carotid artery 

https://ets.amu.edu.az/mod/quiz/review.php?attempt=68702&cmid=7722#question-98660-17 3/8
4/20/2021 Midterm exam: İnsan anatomiyası və tibbi terminologiya - 2 (EN) (MPF2): Attempt review

Question 9
Correct

Mark 1.00 out of 1.00

The anterior ethmoid nerve enters the cranial cavity through:

A. optic canal

B. anterior ethmoid foramen 

C. olfactory foramina

D. medial side of the fissure orbitalis superior

E. lateral side of the fissure orbitalis inferior

Question 10
Correct

Mark 1.00 out of 1.00

Where is located the superior border of the heart?

A. line connecting upper edges of VII ribs

B. line connecting upper edges the IV ribs

C. line connecting lower edges of the V ribs

D. line connecting lower edges of VI ribs

E. line connecting upper edges of III ribs 

Question 11
Correct

Mark 1.00 out of 1.00

At which level does the external carotid artery divide into two terminating branches?

A. coronoid process of mandible

B. angle of mandible

C. thyroid cartilage

D. neck of mandible 

E. hyoid bone

https://ets.amu.edu.az/mod/quiz/review.php?attempt=68702&cmid=7722#question-98660-17 4/8
4/20/2021 Midterm exam: İnsan anatomiyası və tibbi terminologiya - 2 (EN) (MPF2): Attempt review

Question 12
Incorrect

Mark 0.00 out of 1.00

It doesn’t belong to the branches of thyrocervical trunk:

A. ascending cervical artery

B. subscapular artery

C. superficial cervical artery

D. suprascapular artery 

E. inferior thyroid artery

Question 13
Correct

Mark 1.00 out of 1.00

It is not the branch of the ulnar artery:

A. а. carpalis palmaris

B. а. recurrens ulnaris

C. а. carpalis dorsalis

D. а. interossea communis

E. а. recurrens radialis 

Question 14
Correct

Mark 1.00 out of 1.00

Where is located the inferior sagittal sinus?

A. between the foramen caecum and internal occipital protuberance

B. in the inferior concave margin of the falx cerebri 

C. in the transverse sinus groove of the occipital bone

D. in the connecting place of the falx cerebri with the tentorium cerebelli

E. in the confluence cinuum

https://ets.amu.edu.az/mod/quiz/review.php?attempt=68702&cmid=7722#question-98660-17 5/8
4/20/2021 Midterm exam: İnsan anatomiyası və tibbi terminologiya - 2 (EN) (MPF2): Attempt review

Question 15
Correct

Mark 1.00 out of 1.00

What is called the continuation of the right ascending lumbar vein?

A. accessory hemiazygos vein

B. azygos vein 

C. pericardiaco-phrenic veins

D. internal thoracic vein

E. hemiazygos vein

Question 16
Correct

Mark 1.00 out of 1.00

To the branches of the ophthalmic nerve belong:

1. n. auriculotemporalis

2. n. nasociliaris 

3. n. frontalis 

4. n. petrosus major

5. n. lacrimalis 

6. n. infraorbitalis

Question 17
Incorrect

Mark 0.00 out of 1.00

The anterior wall of the third ventricle is formed by:

1. columns of the fornix

2. epithalamic commissure

3. pineal body

4. thalamus 

5. anterior commissure 

6. terminal lamina 

https://ets.amu.edu.az/mod/quiz/review.php?attempt=68702&cmid=7722#question-98660-17 6/8
4/20/2021 Midterm exam: İnsan anatomiyası və tibbi terminologiya - 2 (EN) (MPF2): Attempt review

Question 18
Correct

Mark 1.00 out of 1.00

Find the accordance:

The superior laryngeal nerve is the branch of vagus nerve 

The tympanic nerve is the branch of glossopharyngeal nerve 

Question 19
Correct

Mark 1.00 out of 1.00

The ulnar artery departs from the [[1]] artery at the level of the [[2]] of the ulna.

[[1]] brachial 

[[2]] coronoid process 

Question 20
Correct

Mark 1.00 out of 1.00

Find the colliculus facialis:

Answer: 6 

◄ Attendance

Keç ...

A. Abdullayev. 7. General overview of the peripheral nervous system. The classification, development, variation, and anomalies of the
cranial nerves. ►

https://ets.amu.edu.az/mod/quiz/review.php?attempt=68702&cmid=7722#question-98660-17 7/8
4/20/2021 Midterm exam: İnsan anatomiyası və tibbi terminologiya - 2 (EN) (MPF2): Attempt review

https://ets.amu.edu.az/mod/quiz/review.php?attempt=68702&cmid=7722#question-98660-17 8/8
4/21/2021 Midterm exam: İnsan anatomiyası və tibbi terminologiya - 2 (EN) (MPF2): Attempt review

Dashboard / My courses / İnsan anatomiyası və tibbi terminologiya - 2 (EN) / General

/ Midterm exam: İnsan anatomiyası və tibbi terminologiya - 2 (EN) (MPF2)

Started on Tuesday, 20 April 2021, 12:30 PM


State Finished
Completed on Tuesday, 20 April 2021, 12:59 PM
Time taken 29 mins 22 secs
Grade 19 out of 20 (95%)

Question 1
Incorrect

Mark 0.00 out of 1.00

In the posterior funiculus of the spinal cord are located:

A. tractus rubrospinalis, tractus spinocerebellaris dorsalis

B. tractus vestibulospinalis, tractus spinocerebellaris ventralis

C. tractus tectospinalis, tractus vestibulospinalis

D. fasciculus gracilis, tractus tectospinalis 

E. fasciculus cuneatus, fasciculus gracilis

Question 2
Correct

Mark 1.00 out of 1.00

Which from the following gyruses does not belong to the temporal lobe?

A. temporalis inferior

B. temporalis superior

C. temporalis media

D. supramarginal 

E. parahyppocampalis

https://ets.amu.edu.az/mod/quiz/review.php?attempt=68698&cmid=7722 1/8
4/21/2021 Midterm exam: İnsan anatomiyası və tibbi terminologiya - 2 (EN) (MPF2): Attempt review

Question 3
Correct

Mark 1.00 out of 1.00

Posterior end of the thalamus is called:

A. fissura chorioidea

B. tuberculum anterius

C. lamina affixa

D. stria terminalis

E. pulvinar 

Question 4
Correct

Mark 1.00 out of 1.00

It does not belong to epithalamus:

A. hypophysis 

B. epithalamic commissure

C. habenular trigone

D. habenular commissure

E. epiphysis

Question 5
Correct

Mark 1.00 out of 1.00

The dorsal part of metencephalon is:

A. cerebellum 

B. corpus mamillaris

C. pedunculus cerebri

D. pons

E. tuber cinereum

https://ets.amu.edu.az/mod/quiz/review.php?attempt=68698&cmid=7722 2/8
4/21/2021 Midterm exam: İnsan anatomiyası və tibbi terminologiya - 2 (EN) (MPF2): Attempt review

Question 6
Correct

Mark 1.00 out of 1.00

In medulla oblongata is not located:

A. center of vomiting

B. center of swallowing

C. circulatory center

D. respiratory center

E. highest center of vegetative nervous system 

Question 7
Correct

Mark 1.00 out of 1.00

The oculomotor nerve emerges from cranial cavity through:

A. fissura orbitalis superior 

B. canalis opticus

C. foramen ethmoidale anterior

D. foramen rotundum

E. fissura orbitalis inferior

Question 8
Correct

Mark 1.00 out of 1.00

The glossopharyngeal nerve leaves the skull through:

A. canalis hypoglossus

B. foramen jugulare 

C. foramen lacerum

D. foramen rotundum

E. foramen ovale

https://ets.amu.edu.az/mod/quiz/review.php?attempt=68698&cmid=7722 3/8
4/21/2021 Midterm exam: İnsan anatomiyası və tibbi terminologiya - 2 (EN) (MPF2): Attempt review

Question 9
Correct

Mark 1.00 out of 1.00

It does not belong to the branches of the ophthalmic nerve:


I. N. infaorbitalis
II. N. lacrimalis
III. Rr. pterygopalatini
IV. N. nasociliaris
V. N. frontalis
VI. N. zygomaticus

A. III; IV; VI

B. I; III; V

C. I; III; VI 

D. II; III; IV

E. II; IV; V

Question 10
Correct

Mark 1.00 out of 1.00

How many layers can be distinguished in the myocardium of the ventricles?

A. 2

B. 3 

C. 4

D. 1

E. 5

Question 11
Correct

Mark 1.00 out of 1.00

Thoracic aorta locates between the:

A. ThI-ThXII

B. ThIV-ThXII 

C. ThVI-ThXII

D. ThIII-ThVIII

E. ThIV-ThX

https://ets.amu.edu.az/mod/quiz/review.php?attempt=68698&cmid=7722 4/8
4/21/2021 Midterm exam: İnsan anatomiyası və tibbi terminologiya - 2 (EN) (MPF2): Attempt review

Question 12
Correct

Mark 1.00 out of 1.00

It does not compose the thyrocervical trunk:

A. inferior thyroid artery

B. superficial cervical artery

C. suprascapular artery

D. vertebral artery 

E. ascending cervical artery

Question 13
Correct

Mark 1.00 out of 1.00

To which vein does v.cephalica vein open?

A. subclavian

B. axillary 

C. anterior jugular

D. external jugular

E. brachial

Question 14
Correct

Mark 1.00 out of 1.00

The vein which is composed by the connecting occipital and posterior auricular veins:

A. internal jugular vein

B. brachiocephalic vein

C. anterior jugular vein

D. superior venаcava

E. external jugular vein 

https://ets.amu.edu.az/mod/quiz/review.php?attempt=68698&cmid=7722 5/8
4/21/2021 Midterm exam: İnsan anatomiyası və tibbi terminologiya - 2 (EN) (MPF2): Attempt review

Question 15
Correct

Mark 1.00 out of 1.00

Median nerve originates from the

A. axillary nerve

B. musculocutaneous nerve

C. medial and lateral cords of the brachial plexus 

D. radial nerve

E. ulnar nerve

Question 16
Correct

Mark 1.00 out of 1.00

Which muscles are innervated by n. dorsalis scapulae?

1. m. coracobrachialis

2. m. pectoralis major

3. m. latissimus dorsi

4. m. levator scapulae 

5. mm. rhomboidei 

Question 17
Correct

Mark 1.00 out of 1.00

To the branches of the ophthalmic nerve belong:

1. n. petrosus major

2. n. nasociliaris 

3. n. auriculotemporalis

4. n. lacrimalis 

5. n. infraorbitalis

6. n. frontalis 

https://ets.amu.edu.az/mod/quiz/review.php?attempt=68698&cmid=7722 6/8
4/21/2021 Midterm exam: İnsan anatomiyası və tibbi terminologiya - 2 (EN) (MPF2): Attempt review

Question 18
Correct

Mark 1.00 out of 1.00

Find the accordance:

A. interossea communis is the branch of ulnar artery 

A. thoracoacromialis is the branch of axillary artery 

Question 19
Correct

Mark 1.00 out of 1.00

The ulnar artery departs from the [[1]] artery at the level of the [[2]] of the ulna.

[[1]] brachial 

[[2]] coronoid process 

Question 20
Correct

Mark 1.00 out of 1.00

Find n. vagus:

Answer: 11 

◄ Attendance

Jump to...

A. Abdullayev. 7. General overview of the peripheral nervous system. The classification, development, variation, and anomalies of the
cranial nerves. ►

https://ets.amu.edu.az/mod/quiz/review.php?attempt=68698&cmid=7722 7/8
4/21/2021 Midterm exam: İnsan anatomiyası və tibbi terminologiya - 2 (EN) (MPF2): Attempt review

https://ets.amu.edu.az/mod/quiz/review.php?attempt=68698&cmid=7722 8/8
01023ÿ53ÿ01 6789 ÿ ÿÿ 8ÿÿ88ÿ 88ÿÿ0ÿ6ÿ67 !0

34567ÿ49:9ÿ94;6<47 =34567ÿ<45>4?67 =ÿ@59:5ÿ:5:AB76C:9DÿE4ÿA6FF6ÿAG?765B>BH6C:ÿIÿJÿKLMN=ÿO7P76


=ÿ36QAG?7ÿGR:7Sÿ@59:5ÿ:5:AB76C:9DÿE4ÿA6FF6ÿAG?765B>BH6C:ÿIÿJÿKLMNÿK3TUJN

VWXYZ[\]ÿ_
MBAÿCGAÿ:59`G?GQ
3:?aGQÿBPAÿB<ÿbcdd

gGA`GG5ÿA;Gÿ:5AG?6B?ÿh:?AÿB<ÿA;GÿhB59ÿ:5QÿA;Gÿ76QQ>GÿiG?GFG>>:?ÿhGQP5i>GÿG7G?HG9
jcA;Gÿ:FQPiG5Aÿ5G?EG
gcA;GÿBiP>B7BAB?ÿ5G?EG
kcA;GÿA?6HG765:>ÿ5G?EGc
lcA;Gÿ<:i6:>ÿ5G?EG
LcA;GÿA?Bi;>G:?ÿ5G?EG
k>G:?ÿ7Cÿi;B6iG

VWXYZ[\]ÿe
MBAÿCGAÿ:59`G?GQ
3:?aGQÿBPAÿB<ÿbcdd

mAÿ;:hhG59ÿ6<ÿA;Gÿ7BAB?ÿ:5:>CnG?ÿB<ÿ̀?6AAG5ÿ9hGGi;ÿ69ÿQ:7:HGQS
jc7BAB?ÿ:h;:96:
gc:H?:h;6:
kc9G59B?Cÿ:h;:96:
lc:h?:R6:
LcA:iA6>Gÿ:H5B96:
k>G:?ÿ7Cÿi;B6iG

VWXYZ[\]ÿf
MBAÿCGAÿ:59`G?GQ
3:?aGQÿBPAÿB<ÿbcdd

m5ÿ̀;6i;ÿ>BFGÿB<ÿA;GÿiG?GF?:>ÿ;G769h;G?Gÿ69ÿA;GÿiB?hP9ÿ:7CHQ:>B6QGP7ÿ96AP:AGQo
jc<?B5A:>
gcBii6h6A:>
kcAG7hB?:>
lch:?6GA:>
Lc659P>:
k>G:?ÿ7Cÿi;B6iG
p
" #33 $ %$9%$&3 93'%8&3 #$#"#( #)*+1,-./ 89)00001'% 8+2++22- 230
01023ÿ53ÿ01 6789 ÿ ÿÿ 8ÿÿ88ÿ 88ÿÿ0ÿ6ÿ67 !0
3456789:ÿ<
=>?ÿAB?ÿCDEFBGBH
ICGJBHÿ>K?ÿ>LÿMNOO

R?ÿH>BEÿD>?ÿSBT>DUÿ?>ÿBVW?XCTCYKEZ
[NXCSBDKTCGÿ?GWU>DB
\NXCSBDKTCGÿ]>YYWEEKGB
^NBVW?XCTCYW]ÿ]>YYWEEKGB
_NXAV>VXAEWE
`NBVWVXAEWE
^TBCGÿYAÿ]X>W]B

3456789:ÿP
=>?ÿAB?ÿCDEFBGBH
ICGJBHÿ>K?ÿ>LÿMNOO

aXBGBÿWEÿ?XBÿbCTTB]KTCÿ]BGBSBTTWÿT>]C?BHc
[N>Dÿ?XBÿWDLBGW>GÿEKGLC]Bÿ>Lÿ]BGBSBTTKY
\N>Dÿ?XBÿEKVBGW>GÿEKGLC]Bÿ>Lÿ]BGBSBTTKY
^N>Dÿ?XBÿCD?BGW>GÿCDHÿV>E?BGW>GÿS>GHBGEÿ>Lÿ]BGBSBTTKY
_N>Dÿ?XBÿV>E?BGW>GÿS>GHBGÿ>Lÿ]BGBSBTTKY
`N>Dÿ?XBÿCD?BGW>GÿS>GHBGÿ>Lÿ]BGBSBTTKY
^TBCGÿYAÿ]X>W]B

3456789:ÿQ
=>?ÿAB?ÿCDEFBGBH
ICGJBHÿ>K?ÿ>LÿMNOO

dXBÿT>FBGÿVCG?ÿ>Lÿ?XBÿGX>YS>WHÿL>EECÿWEÿ]>GGBEV>DHBHÿ?>Z
[NYBHKTTCÿ>ST>DUC?C
\N?BTBD]BVXCT>D
^NYBEBD]BVXCT>D
_NHWBD]BVXCT>D
`NYB?BD]BVXCT>D
^TBCGÿYAÿ]X>W]B

e
" #33 $ %$9%$&3 93'%8&3 #$#"#( #)*+1,-./ 89)00001'% 8+2++22- 030
01023ÿ53ÿ01 6789 ÿ ÿÿ 8ÿÿ88ÿ 88ÿÿ0ÿ6ÿ67 !0
3456789:ÿ<
=>?ÿAB?ÿCDEFBGBH
ICGJBHÿ>K?ÿ>LÿMNOO

R?ÿH>BEDS?ÿTBU>DVÿ?>ÿ?WBÿT>DAÿUCTAGXD?WY
ZN[CDCUXEÿEB\X[XG[KUCGXEÿ]>E?BGX>G
^N[>[WUBC
_N`BE?XTKUK\
aN[CDCUXEÿEB\X[XG[KUCGXEÿCD?BGX>G
bNK?GX[KUKE
_UBCGÿ\Aÿ[W>X[B

3456789:ÿP
=>?ÿAB?ÿCDEFBGBH
ICGJBHÿ>K?ÿ>LÿMNOO

RD?>ÿW>Fÿ\CDAÿ]CG?EÿH>BEÿ?WBÿLC[XCUÿDBG`BÿHX̀XHBÿC[[>GHXDVÿ?>ÿ?WBÿ?>]>VGC]WAÿY
ZNc
^Nd
_Ne
aNf
bNg
_UBCGÿ\Aÿ[W>X[B

3456789:ÿQ
=>?ÿAB?ÿCDEFBGBH
ICGJBHÿ>K?ÿ>LÿMNOO

R?ÿH>BEÿD>?ÿTBU>DVÿ?>ÿ?WBÿTGCD[WBEÿ>Lÿ?WBÿ[BG`X[CUÿ]CG?ÿ>Lÿ?WBÿ̀CVKEÿDBG`BY
ZNCKGX[KUCGÿTGCD[W
^NEK]BGX>GÿUCGADVBCUÿDBG`B
_N]WCGADVBCUÿTGCD[WBE
aNGGNÿ[CGHXC[Xÿ[BG`X[CUBEÿEK]BGX>GBE
bNGB[KGGBD?ÿUCGADVBCUÿDBG`B
_UBCGÿ\Aÿ[W>X[B

h
" #33 $ %$9%$&3 93'%8&3 #$#"#( #)*+1,-./ 89)00001'% 8+2++22- ,30
01023ÿ53ÿ01 6789 ÿ ÿÿ 8ÿÿ88ÿ 88ÿÿ0ÿ6ÿ67 !0
3456789:ÿ<=
>?@ÿBC@ÿDEFGCHCI
JDHKCIÿ?L@ÿ?MÿNOPP

RSTUSÿVDBCHFÿ?Mÿ@SCÿFCH?LFÿWCHTUDHITLXY
ZOXCITDF@TEDV[ÿU?F@DV
\OWDHTC@DV[ÿ]TFUCHDV
^OWDHTC@DV[ÿU?F@DV
_OITDWSHD`XDHTU[]TFUCHDV
aOU?F@DV[WDHTC@DV
^VCDHÿXBÿUS?TUC

3456789:ÿ<<
>?@ÿBC@ÿDEFGCHCI
JDHKCIÿ?L@ÿ?MÿNOPP

RSCHCÿTFÿV?UD@CIÿU?XX?EÿUDH?@TIÿDH@CHBÿbTMLHUD@T?EY
ZOFLWCHT?Hÿb?HICHÿ?Mÿ@SCÿ@SBH?TIÿUDH@TVD`C
\OTEMCHT?Hÿb?HICHÿ?Mÿ@SCÿ@SBH?TIÿUDH@TVD`C
^OW?F@CHT?Hÿb?HICHÿ?Mÿ@SCÿWDVD@C
_OcÿUCH]TUDVÿ]CH@CbHD
aOH??@ÿ?Mÿ@SCÿ@?E`LC
^VCDHÿXBÿUS?TUC

3456789:ÿ<Q
>?@ÿBC@ÿDEFGCHCI
JDHKCIÿ?L@ÿ?MÿNOPP

dSCÿbHDEUSCFÿ?Mÿ@SCÿbHDUST?UCWSDVTUÿ@HLEKeÿÿ
fOÿZOÿFLbUVD]TDÿICg@HDÿÿ
ffOÿZOÿUDH?@TFÿU?XXLETFÿFTETF@HDÿÿ
fffOÿZOÿUDH?@TFÿU?XXLETFÿICg@HDÿÿ
fcOÿZOÿFLbUVD]TDÿFTETF@HDÿÿ
cOÿZOÿ@SBH?TICDÿTXD
ZOfffhfc
\Ofhfff
^Ofchc
_Ofhff
aOffhfff
^VCDHÿXBÿUS?TUC

i
" #33 $ %$9%$&3 93'%8&3 #$#"#( #)*+1,-./ 89)00001'% 8+2++22- 530
01023ÿ53ÿ01 6789 ÿ ÿÿ 8ÿÿ88ÿ 88ÿÿ0ÿ6ÿ67 !0
3456789:ÿ<=
>?@ÿBC@ÿDEFGCHCI
JDHKCIÿ?L@ÿ?MÿNOPP

S@ÿTFÿE?@ÿ@UCÿVHDEWUÿ?Mÿ@UCÿHDITDXÿDH@CHBY
ZOHOÿWDH[DXTFÿ[DX\DHTF
]OHOÿWDH[DXTFÿI?HFDXTF
^ODOÿ[HTEWC[Fÿ[?XXTWTF
_O`OÿTE@CH?FFCDÿW?\\LETF
aODOÿHCWLHHCEFÿHDITDXTF
^XCDHÿ\BÿWU?TWC

3456789:ÿ<Q
>?@ÿBC@ÿDEFGCHCI
JDHKCIÿ?L@ÿ?MÿNOPP

bUCHCÿTFÿX?WD@CIÿ@UCÿTEMCHT?HÿFDcT@@DXÿFTELFd
ZOVC@GCCEÿ@UCÿM?HD\CEÿWDCWL\ÿDEIÿTE@CHEDXÿ?WWT[T@DXÿ[H?@LVCHDEWC
]OTEÿ@UCÿW?EMXLCEWCÿWTELL\
^OTEÿ@UCÿW?EECW@TEcÿ[XDWCÿ?Mÿ@UCÿMDXeÿWCHCVHTÿGT@Uÿ@UCÿ@CE@?HTL\ÿWCHCVCXXT
_OTEÿ@UCÿ@HDEFfCHFCÿFTELFÿcH??fCÿ?Mÿ@UCÿ?WWT[T@DXÿV?EC
aOTEÿ@UCÿTEMCHT?HÿW?EWDfCÿ\DHcTEÿ?Mÿ@UCÿMDXeÿWCHCVHT
^XCDHÿ\BÿWU?TWC

3456789:ÿ<R
>?@ÿBC@ÿDEFGCHCI
JDHKCIÿ?L@ÿ?MÿNOPP

gUCÿVHDWUTDXÿ[XCeLFÿTFÿM?H\CIÿVBÿ@UC
ZOM?LHÿ@HLEKF
]O@UHCCÿ@HLEKF
^O@G?ÿ@HLEKF
_OFTeÿ@HLEKF
aOMTfCÿ@HLEKF

h
" #33 $ %$9%$&3 93'%8&3 #$#"#( #)*+1,-./ 89)00001'% 8+2++22- -30
01023ÿ53ÿ01 6789 ÿ ÿÿ 8ÿÿ88ÿ 88ÿÿ0ÿ6ÿ67 !0
3456789:ÿ<=
>?@ÿBC@ÿDEFGCHCI
JDHKCIÿ?L@ÿ?MÿNOPP

T?ÿ@UCÿVHDEWUCFÿ?Mÿ@UCÿ?XU@UDYZ[WÿECH\CÿVCY?E]^
NOEOÿYDWH[ZDY[F
_OEOÿ[EMHD?HV[@DY[F
`OEOÿXC@H?FLFÿZDa?H
bOEOÿDLH[WLY?@CZX?HDY[F
cOEOÿMH?E@DY[F
dOEOÿEDF?W[Y[DH[F

3456789:ÿ<Q
>?@ÿBC@ÿDEFGCHCI
JDHKCIÿ?L@ÿ?MÿNOPP

T?ÿ@UCÿVHDEWUCFÿ?Mÿ@UCÿVHDWU[DYÿDH@CHBÿVCY?E]^
NODOÿXH?MLEIDÿVHDWU[[
_ODOÿW?YYD@CHDY[FÿLYEDH[FÿFLXCH[?H
`ODOÿFLVFWDXLYDH[F
bODOÿ@U?HDW?I?HFDY[F
cODOÿ@U?HDW?DWH?Z[DY[F
dODOÿW?YYD@CHDY[FÿLYEDH[Fÿ[EMCH[?H

3456789:ÿ<R
>?@ÿBC@ÿDEFGCHCI
JDHKCIÿ?L@ÿ?MÿNOPP

e[EIÿ@UCÿDWW?HIDEWC^
fBHLFÿDE]LYDH[FÿCZVHDWCF X?F@CH[?HÿCEIÿ?Mÿ@UCÿFLXCH[?Hÿ@CZX?HDYÿFLYWLF
fBHLFÿFLXHDZDH][EDY[FÿCZVHDWCF X?F@CH[?HÿVHDEWUÿ?Mÿ@UCÿYD@CHDYÿFLYWLF

3456789:ÿ<S
>?@ÿBC@ÿDEFGCHCI
JDHKCIÿ?L@ÿ?MÿNOPP

TUCÿ@H?WUYCDHÿECH\CÿCE@CHFÿ@UCÿ?HV[@ÿ@UH?L]Uÿ@UCÿggNhhÿDEIÿ[EECH\D@CFÿ@UCÿgg_hhO
ggNhh FLXCH[?Hÿ?HV[@DYÿM[FFLHC
gg_hh FLXCH[?Hÿ?VY[iLCÿZLFWYC

j
" #33 $ %$9%$&3 93'%8&3 #$#"#( #)*+1,-./ 89)00001'% 8+2++22- *30
01023ÿ53ÿ01 6789 ÿ ÿÿ 8ÿÿ88ÿ 88ÿÿ0ÿ6ÿ67 !0
3456789:ÿ<=
>?@ÿBC@ÿDEFGCHCI
JDHKCIÿ?L@ÿ?MÿNOPP

QREIÿ@SCÿLTEDHÿECHUCV

ÿ
WEFGCHV X

YÿW@@CEIDEZC
[C\ÿOOO
WOÿW]ILTTDBCUOÿXOÿ^CECHDTÿ?UCHURCGÿ?Mÿ@SCÿ_CHR_SCHDTÿECHU?LFÿFBF@C`OÿaSCÿZTDFFRMRZD@R?EbÿICUCT?_`CE@bÿUDHRD@R?EbÿDEIÿDE?`DTRCFÿ?Mÿ@SCÿ
ZHDERDTÿECHUCFOÿc

d
" #33 $ %$9%$&3 93'%8&3 #$#"#( #)*+1,-./ 89)00001'% 8+2++22- 030
2021/4/20 Midterm exam: İnsan anatomiyası və tibbi terminologiya - 2 (EN) (MPF2): Attempt review

Dashboard / My courses / İnsan anatomiyası və tibbi terminologiya - 2 (EN) / General

/ Midterm exam: İnsan anatomiyası və tibbi terminologiya - 2 (EN) (MPF2)

Started on Tuesday, 20 April 2021, 12:30 PM


State Finished
Completed on Tuesday, 20 April 2021, 12:46 PM
Time taken 16 mins 28 secs
Grade 20 out of 20 (100%)

Question 1
Correct

Mark 1.00 out of 1.00

Between the anterior part of the pons and the middle cerebellar peduncle emerges

A. the trigeminal nerve. 

B. the facial nerve

C. the abducent nerve

D. the oculomotor nerve

E. the trochlear nerve

Question 2
Correct

Mark 1.00 out of 1.00

Temporal lobe is separated from the frontal and parietal lobes by the:

A. transversus occipital sulcus

B. parieto-occipital sulcus

C. lateral sulcus 

D. central sulcus

E. postcentral sulcus

Question 3
Correct

Mark 1.00 out of 1.00

It does not belong to lateral ventricle:

A. posterior horn

B. anterior horn

C. central part

D. inferior horn

E. anterior commissure 

https://ets.amu.edu.az/mod/quiz/review.php?attempt=68717&cmid=7722 1/7
2021/4/20 Midterm exam: İnsan anatomiyası və tibbi terminologiya - 2 (EN) (MPF2): Attempt review

Question 4
Correct

Mark 1.00 out of 1.00

Where is the gyrus parahippocampalis situated?

A. in the posterior part of the gyrus temporalis inferior

B. between the hippocampal and collateral sulcus 

C. between the inferior temporal and occipitotemporalis sulcus

D. between the superior and inferior temporal sulcus

E. between the lateral and superior temporal sulcus

Question 5
Correct

Mark 1.00 out of 1.00

The dorsal part of metencephalon is:

A. pedunculus cerebri

B. cerebellum 

C. tuber cinereum

D. corpus mamillaris

E. pons

Question 6
Correct

Mark 1.00 out of 1.00

The lower part of the rhomboid fossa is corresponded to:

A. mesencephalon

B. medulla oblongata 

C. diencephalon

D. metencephalon

E. telencephalon

https://ets.amu.edu.az/mod/quiz/review.php?attempt=68717&cmid=7722 2/7
2021/4/20 Midterm exam: İnsan anatomiyası və tibbi terminologiya - 2 (EN) (MPF2): Attempt review

Question 7
Correct

Mark 1.00 out of 1.00

Which of the following are the parts of the tympanic membrane?


I. Pars mucosa
II. Pars muscularis
III. Pars tensa
IV. Pars flaccida
V. Pars membranacea

A. I, II

B. II, IV

C. IV, V

D. III, IV 

E. III, V

Question 8
Correct

Mark 1.00 out of 1.00

Which cranial nerve provides the human with the ability to smell a wide variety of odors?

A. vestibulocochlear nerve

B. intermedius nerve

C. olfactory nerve 

D. oculomotor nerve

E. optic nerve

Question 9
Correct

Mark 1.00 out of 1.00

Which muscles are innervated by the nervus accesorius?


I. M. sternocleidomastoideus
II. M. trapezius
III. M. digastricus
IV. M. latissimus dorsi
V. M. buccinator

A. II,III

B. IV,V

C. III,IV

D. III,V

E. I,II 

https://ets.amu.edu.az/mod/quiz/review.php?attempt=68717&cmid=7722 3/7
2021/4/20 Midterm exam: İnsan anatomiyası və tibbi terminologiya - 2 (EN) (MPF2): Attempt review

Question 10
Correct

Mark 1.00 out of 1.00

The surfaces of the heart:


I. Oesophagalis
II. Sternocostalis
III. Mediastinalis
IV. Diaphracmatica
V. Trachealis

A. II;III

B. II;IV 

C. I;II

D. IV;V

E. III;IV

Question 11
Correct

Mark 1.00 out of 1.00

Where is located common carotid artery bifurcation?

A. root of the tongue

B. V cervical vertebra

C. posterior border of the palate

D. inferior border of the thyroid cartilage

E. superior border of the thyroid cartilage 

Question 12
Correct

Mark 1.00 out of 1.00

It does not belong the branches of the internal carotid artery:

A. a. cerebri anterior

B. a. cerebri media

C. a. communicans posterior

D. a. ophthalmica

E. a. cerebri posterior 

https://ets.amu.edu.az/mod/quiz/review.php?attempt=68717&cmid=7722 4/7
2021/4/20 Midterm exam: İnsan anatomiyası və tibbi terminologiya - 2 (EN) (MPF2): Attempt review

Question 13
Correct

Mark 1.00 out of 1.00

To which vein does v.cephalica vein open?

A. external jugular

B. anterior jugular

C. brachial

D. subclavian

E. axillary 

Question 14
Correct

Mark 1.00 out of 1.00

Where is located the inferior sagittal sinus?

A. in the connecting place of the falx cerebri with the tentorium cerebelli

B. in the confluence cinuum

C. between the foramen caecum and internal occipital protuberance

D. in the inferior concave margin of the falx cerebri 

E. in the transverse sinus groove of the occipital bone

Question 15
Correct

Mark 1.00 out of 1.00

This doesn’t belong to the cutaneous branches of the cervical plexus:

A. n. occipitalis minor

B. n. supraclavicularis

C. n. subclavius 

D. n. transversus colli

E. n. auricularis magnus

https://ets.amu.edu.az/mod/quiz/review.php?attempt=68717&cmid=7722 5/7
2021/4/20 Midterm exam: İnsan anatomiyası və tibbi terminologiya - 2 (EN) (MPF2): Attempt review

Question 16
Correct

Mark 1.00 out of 1.00

The anterior wall of the third ventricle is formed by:

1. anterior commissure 

2. thalamus

3. epithalamic commissure

4. pineal body

5. columns of the fornix 

6. terminal lamina 

Question 17
Correct

Mark 1.00 out of 1.00

Which muscles are innervated by n. dorsalis scapulae?

1. m. latissimus dorsi

2. m. pectoralis major

3. m. levator scapulae 

4. mm. rhomboidei 

5. m. coracobrachialis

Question 18
Correct

Mark 1.00 out of 1.00

Find the accordance:

The inner ear consists of bony labyrinth and membranous labyrinth 

The external ear includes auricle and external acoustic meatus 

Question 19
Correct

Mark 1.00 out of 1.00

The external capsule is a layer of [[1]] matter that is situated between the claustrum and [[2]] nucleus.

[[1]] white 

[[2]] lentiform 

https://ets.amu.edu.az/mod/quiz/review.php?attempt=68717&cmid=7722 6/7
2021/4/20 Midterm exam: İnsan anatomiyası və tibbi terminologiya - 2 (EN) (MPF2): Attempt review

Question 20
Correct

Mark 1.00 out of 1.00

Find the pons:

Answer: 11 

◄ Attendance

Jump to...

A. Abdullayev. 7. General overview of the peripheral nervous system. The classification, development, variation, and anomalies of the cranial
nerves. ►

https://ets.amu.edu.az/mod/quiz/review.php?attempt=68717&cmid=7722 7/7
4/20/2021 Midterm exam: Human anatomy and medical terminology - 2 (EN) (MPF2): Attempt review

My main page / My subjects / Human Anatomy and Medical Terminology - 2 (EN) / General
/ Midterm exam: Human anatomy and medical terminology - 2 (EN) (MPF2)

The test began Tuesday, 20 April 2021, 12 30 PM


State Finished
It's over Tuesday, 20 April 2021, 12 57 PM
Time has passed 26 minutes 56 sec.
Price 20 ( 95 %) maximum 19
Question 1
Correct
Mark 1.00 out of 1.00

In the posterior funiculus of the spinal cord are located:


A. tectospinalis tract, vestibulospinalis tract
B. rubrospinalis tract, spinocerebellaris dorsal tract
C. fasciculus gracilis, tectospinalis tract
D. vestibulospinalis tract, ventricular spinocerebellaris tract
E.. fasciculus cuneatus, fasciculus gracilis 

Question 2
Correct
Mark 1.00 out of 1.00

Which of the following gyruses does not belong to the temporal lobe?
A. temporalis media
B. parahyppocampalis
C. inferior temporalis
D. superior temporalis
E.. supramarginal 

https://ets.amu.edu.az/mod/quiz/review.php?attempt=68704&cmid=7722 1/7
4/20/2021 Midterm exam: Human anatomy and medical terminology - 2 (EN) (MPF2): Attempt review

Question 3
Correct
Mark 1.00 out of 1.00

The anterior limb of the internal capsule is situated between:


A. putamen and claustrum
B. lentiform nucleus and caudate nucleus 

C. thalamus and lentiform nucleus


D. claustrum and cortex of insula
E.. globi pallidus medialis et lateralis

Question 4
Correct
Mark 1.00 out of 1.00

It does not belong to the epithalamus:


A. epithalamic commissure
B. epiphysis
C. hypophysis 

D. habenular commissure
E.. habenular trigone

Question 5
Correct
Mark 1.00 out of 1.00

The roof of the fourth ventricle in front consists of:


I. Superior cerebellar peduncles
II. Middle cerebellar peduncles
III. Inferior cerebellar peduncles
IV. Velum medullare superius
V. Velum medullare inferius
A. III; V
B. I; IV 

C. II; III
D. I; II
E.. III; IV

https://ets.amu.edu.az/mod/quiz/review.php?attempt=68704&cmid=7722 2/7
4/20/2021 Midterm exam: Human anatomy and medical terminology - 2 (EN) (MPF2): Attempt review

Question 6
Correct
Mark 1.00 out of 1.00

The lower part of the rhomboid fossa is corresponded to:


A. diencephalon
B. metencephalon
C. telencephalon
D. medulla oblongata 

E.. mesencephalon

Question 7
Correct
Mark 1.00 out of 1.00

Where does the optic nerve enter the cranial cavity from?
A. foramen zygomatico-orbitale
B. foramen ethmoidale posterior
C. canalis opticus 

D. foramen ovale
E.. foramen ethmoidae anterior

Question 8
Correct
Mark 1.00 out of 1.00

Which cranial nerve provides the human with the ability to smell a wide variety of odors?
A. intermedius nerve
B. optic nerve
C. oculomotor nerve
D. olfactory nerve 

E.. vestibulocochlear nerve

https://ets.amu.edu.az/mod/quiz/review.php?attempt=68704&cmid=7722 3/7
4/20/2021 Midterm exam: Human anatomy and medical terminology - 2 (EN) (MPF2): Attempt review

Question 9
Correct
Mark 1.00 out of 1.00

It does not belong to the branches of the ophthalmic nerve:


IN infaorbitalis
II. N. lacrimalis
III. Rr. pterygopalatini
IV. N. nasociliaris
VN frontalis
VI. N. zygomaticus
A. III; IV; VI
B. I; III; V
C. II; IV; V
D. I; III; VI 

E.. II; III; IV

Question 10
Correct
Mark 1.00 out of 1.00

Which layers of the serous pericardium?


A. diaphragmaric, visceral
B. mediastinal, costal
C. parietal, visceral 

D. parietal, costal
E.. costal, parietal

Question 11
Correct
Mark 1.00 out of 1.00

The ascending aorta passes to the aortic arch at the level of the:
A. left second sternocostal joint
B. left fourth sternocostal joint
C. left third sternocostal joint
D. right second sternocostal joint 

E.. right third sternocostal joint

https://ets.amu.edu.az/mod/quiz/review.php?attempt=68704&cmid=7722 4/7
4/20/2021 Midterm exam: Human anatomy and medical terminology - 2 (EN) (MPF2): Attempt review

Question 12
Incorrect
Mark 0.00 out of 1.00

The vertebral artery (a. Vertebralis), is the first branch of the


A. subclavian artery
B. superficial cervical artery
C. subclavian artery
D. inferior thyroid artery
E.. suprascapular artery

Question 13
Correct
Mark 1.00 out of 1.00

To which vein does v.basilica open?


A. brachial 

B. anterior jugular
C. subclavian
D. axillary
E.. external jugular

Question 14
Correct
Mark 1.00 out of 1.00

The vein which is composed by the connecting occipital and posterior auricular veins:
A. internal jugular vein
B. brachiocephalic vein
C. superior venacava
D. external jugular vein 

E.. anterior jugular vein

https://ets.amu.edu.az/mod/quiz/review.php?attempt=68704&cmid=7722 5/7
4/20/2021 Midterm exam: Human anatomy and medical terminology - 2 (EN) (MPF2): Attempt review

Question 15
Correct
Mark 1.00 out of 1.00

The azygos vein opens into the vena cava superior at the level of:
A. Th9
B. Th7
C. Th3
D. Th4
E.. Th5 

Question 16
Correct
Mark 1.00 out of 1.00

Which muscles are innervated by n. dorsal scapulae?


1. m. levator scapulae 

2. mm. rhomboidei 

3. m. pectoralis major
4. m. latissimus dorsi
5. m. coracobrachialis

Question 17
Correct
Mark 1.00 out of 1.00

To the nuclei of the medulla oblongata belong:


1. nucleus caudatus
2. nucleus olivaris 

3. nucleus gracilis 

4. nucleus ruber
5. nucleus cuneatus 

6. nucleus dentatus

https://ets.amu.edu.az/mod/quiz/review.php?attempt=68704&cmid=7722 6/7
4/20/2021 Midterm exam: Human anatomy and medical terminology - 2 (EN) (MPF2): Attempt review

Question 18
Correct
Mark 1.00 out of 1.00

Find the according:


The superior laryngeal nerve is the branch of vagus nerve 

The tympanic nerve is the branch of glossopharyngeal nerve 

Question 19
Correct
Mark 1.00 out of 1.00

At the level of the lateral edge of the [[1]] rib, the axillary vein continues into the [[2]] vein.
[[1]] first 

[[2]] subclavian 

Question 20
Correct
Mark 1.00 out of 1.00

Find the pons:

Answer: 11 

◀ Attendance
Go ...
A. Abdullayev. 7. General overview of the peripheral nervous system. The classification, development, variation, and anomalies of the
cranial nerves. ▶

https://ets.amu.edu.az/mod/quiz/review.php?attempt=68704&cmid=7722 7/7
Mǝnim ǝsas sǝhifǝm / Mǝnim fǝnlǝrim / İnsan anatomiyası vǝ tibbi terminologiya - 2 (EN) / Ümumi
/ Midterm exam: İnsan anatomiyası vǝ tibbi terminologiya - 2 (EN) (MPF2)

Testǝ başlandı Tuesday, 20 April 2021, 12N30 PM


State Finished
Sona çatdı Tuesday, 20 April 2021, 12N57 PM
Vaxt keçib 26 dǝq. 4 san.
Qiymǝt 20 (95%) maksimumdan 19

Question 1 In the medial border of the cerebral peduncle emerges


Correct

Mark 1.00 out A. the trochlear nerve


of 1.00
B. the abducent nerve

C. tthe facial nerve

D. the oculomotor nerve !

E. the trigeminal nerve

Question 2 It happens when the motor analyzer of speech articulation is damaged:


Correct

Mark 1.00 out A. agraphia


of 1.00
B. tactile agnosia

C. sensory aphasia

D. motor aphasia !

E. apraxia

Question 3 The extremal capsule is the white matter separating:


Correct

Mark 1.00 out A. globus pallidus medialis and nucleus caudatus


of 1.00
B. claustrum from cortex of insula !

C. globi pallidus medialis et lateralis

D. globus pallidus lateralis and putamen

E. putamen and claustrum

Question 4 Around the cerebral aqueduct is placed:


Correct

Mark 1.00 out A. substantia grisea centralis !


of 1.00
B. substantia perforata anterior

C. substantia perforata posterior et substantia grisea centralis

D. substantia perforata posterior

E. substantia perforata anterior et substantia grisea centralis


Question 5 The roof of the fourth ventricle in front consists of:
Correct I. Superior cerebellar peduncles
Mark 1.00 out II. Middle cerebellar peduncles
of 1.00 III. Inferior cerebellar peduncles
IV. Velum medullare superius
V. Velum medullare inferius

A. III; V

B. I; II

C. III; IV

D. I; IV !

E. II; III

Question 6 The olive separated from the pyramid by the:


Correct

Mark 1.00 out A. intermediate sulcus


of 1.00
B. anterolateral sulcus !

C. posterior funiculus

D. posterior median sulcus

E. posterolateral sulcus

Question 7 Which cranial nerve receives sound wave stimuli from the small bones of the middle ear, inner ear and
Correct interpret this in the brain as hearing and also control human balance?
Mark 1.00 out
of 1.00 A. olfactory nerve

B. glossopharyngeal nerve

C. intermedius nerve

D. vestibulocochlear nerve !

E. facial nerve

Question 8 It arises from the facial nerve just above the stylomastoid foramen:
Correct

Mark 1.00 out A. the nerve to the stapedius


of 1.00
B. branch to the tympanic plexus

C. greater petrosal nerve

D. lesser petrosal nerve

E. chorda tympani !

Question 9 The vagus nerve passes out of the skull through:


Correct

Mark 1.00 out A. foramen lacerum


of 1.00
B. foramen ovale

C. foramen juguluare !

D. foramen spinosum
D. foramen spinosum

E. foramen rotundum

Question 10 Which layers of the serous pericardium?


Correct

Mark 1.00 out A. mediastinal, costal


of 1.00
B. costal,parietal

C. parietal, costal

D. diaphragmaric,visceral

E. parietal, visceral !

Question 11 Thoracic aorta locates between the:


Correct

Mark 1.00 out A. ThI-ThXII


of 1.00
B. ThIII-ThVIII

C. ThIV-ThX

D. ThVI-ThXII

E. ThIV-ThXII !

Question 12 The branches of the brachiocephalic trunk:


Correct I. A. subclavia dextra
Mark 1.00 out II. A. carotis communis sinistra
of 1.00 III. A. carotis communis dextra
IV. A. subclavia sinistra
V. A. thyroidea ima

A. I;III !

B. III;IV

C. IV;V

D. II;III

E. I;II

Question 13 To which vein does v.basilica open?


Correct

Mark 1.00 out A. axillary


of 1.00
B. external jugular

C. brachial !

D. anterior jugular

E. subclavian

Question 14 It does not belong to the tributaries of the external jugular vein:
Correct

Mark 1.00 out A. v. auricularis posterior


of 1.00
B. v. canalis pterygoidei !
C. v. occipitalis

D. v. suprascapularis

E. v. jugularis anterior

Question 15 What is called the continuation of the left ascending lumbar vein?
Correct

Mark 1.00 out A. internal thoracic vein


of 1.00
B. pericardiaco-phrenic veins

C. accessory hemiazygos vein

D. hemiazygos vein !

E. azygos vein

Question 16 The anterior wall of the third ventricle is formed by:


Correct

Mark 1.00 out 1. epithalamic commissure


of 1.00
2. terminal lamina !

3. anterior commissure !

4. columns of the fornix !

5. pineal body

6. thalamus

Question 17 To the nuclei of the medulla oblongata belong:


Correct

Mark 1.00 out 1. nucleus cuneatus !


of 1.00
2. nucleus ruber

3. nucleus gracilis !

4. nucleus caudatus

5. nucleus dentatus

6. nucleus olivaris !

Question 18 Find the accordance:


Correct

Mark 1.00 out


A. thoracoacromialis is the branch of axillary artery !
of 1.00

A. interossea communis is the branch of ulnar artery !

Question 19 Middle superior alveolar nerve passes in the lateral wall of the [[1]] and innervates the upper [[2]] teeth, gums
Correct and the cheek.
Mark 1.00 out
of 1.00
[[1]] maxillary sinus !

[[2]] premolar !
Question 20 Find the ulnar nerve:
Incorrect

Mark 0.00 out


of 1.00

Answer: 9 "

A. Abdullayev. 7. General overview


of the peripheral nervous system.
◀ Attendance Keç ... The classification, development,
variation, and anomalies of the
cranial nerves. ▶︎
4/20/2021 Midterm exam: İnsan anatomiyası və tibbi terminologiya - 2 (EN) (MPF2): Attempt review

Dashboard / My courses / İnsan anatomiyası və tibbi terminologiya - 2 (EN) / General

/ Midterm exam: İnsan anatomiyası və tibbi terminologiya - 2 (EN) (MPF2)

Started on Tuesday, 20 April 2021, 12:31 PM


State Finished
Completed on Tuesday, 20 April 2021, 12:59 PM
Time taken 28 mins 42 secs
Grade 17 out of 20 (83%)

Question 1
Incorrect

Mark 0.00 out of 1.00

On the midline of the anterior surface of the spinal cord is:

A. dorsal (posterior) median sulcus. 

B. ventrolateral grooves

C. dorsolateral grooves

D. fissura orbitalis superior

E. ventral median fissure

Question 2
Correct

Mark 1.00 out of 1.00

Temporal lobe is separated from the frontal and parietal lobes by the:

A. transversus occipital sulcus

B. lateral sulcus 

C. parieto-occipital sulcus

D. central sulcus

E. postcentral sulcus

https://ets.amu.edu.az/mod/quiz/review.php?attempt=69157&cmid=7722#question-99121-5 1/8
4/20/2021 Midterm exam: İnsan anatomiyası və tibbi terminologiya - 2 (EN) (MPF2): Attempt review

Question 3
Correct

Mark 1.00 out of 1.00

The extremal capsule is the white matter separating:

A. claustrum from cortex of insula 

B. globus pallidus lateralis and putamen

C. globi pallidus medialis et lateralis

D. globus pallidus medialis and nucleus caudatus

E. putamen and claustrum

Question 4
Correct

Mark 1.00 out of 1.00

Midbrain in Latin is called:

A. telencephalon

B. diencephalon

C. myelencephalon

D. mesencephalon 

E. metencephalon

Question 5
Incorrect

Mark 0.00 out of 1.00

The trigonum lemnisci is bounded medially by:

A. inferior cerebellar peduncle

B. brachium colliculi superioris

C. brachium colliculi inferioris 

D. superior cerebellar peduncle

E. cerebral peduncle

https://ets.amu.edu.az/mod/quiz/review.php?attempt=69157&cmid=7722#question-99121-5 2/8
4/20/2021 Midterm exam: İnsan anatomiyası və tibbi terminologiya - 2 (EN) (MPF2): Attempt review

Question 6
Correct

Mark 1.00 out of 1.00

The olive separated from the pyramid by the:

A. anterolateral sulcus 

B. posterior median sulcus

C. intermediate sulcus

D. posterior funiculus

E. posterolateral sulcus

Question 7
Correct

Mark 1.00 out of 1.00

Which cranial nerve receives sound wave stimuli from the small bones of the middle ear, inner ear and interpret this in the brain as hearing
and also control human balance?

A. vestibulocochlear nerve 

B. facial nerve

C. glossopharyngeal nerve

D. intermedius nerve

E. olfactory nerve

Question 8
Correct

Mark 1.00 out of 1.00

It arises from the facial nerve just above the stylomastoid foramen:

A. branch to the tympanic plexus

B. lesser petrosal nerve

C. the nerve to the stapedius

D. greater petrosal nerve

E. chorda tympani 

https://ets.amu.edu.az/mod/quiz/review.php?attempt=69157&cmid=7722#question-99121-5 3/8
4/20/2021 Midterm exam: İnsan anatomiyası və tibbi terminologiya - 2 (EN) (MPF2): Attempt review

Question 9
Correct

Mark 1.00 out of 1.00

The vagus nerve passes out of the skull through:

A. foramen juguluare 

B. foramen lacerum

C. foramen rotundum

D. foramen ovale

E. foramen spinosum

Question 10
Correct

Mark 1.00 out of 1.00

Which layers of the serous pericardium?

A. parietal, costal

B. mediastinal, costal

C. diaphragmaric,visceral

D. costal,parietal

E. parietal, visceral 

Question 11
Correct

Mark 1.00 out of 1.00

Where is located common carotid artery bifurcation?

A. superior border of the thyroid cartilage 

B. V cervical vertebra

C. inferior border of the thyroid cartilage

D. posterior border of the palate

E. root of the tongue

https://ets.amu.edu.az/mod/quiz/review.php?attempt=69157&cmid=7722#question-99121-5 4/8
4/20/2021 Midterm exam: İnsan anatomiyası və tibbi terminologiya - 2 (EN) (MPF2): Attempt review

Question 12
Correct

Mark 1.00 out of 1.00

The vertebral artery (a. vertebralis), is the first branch of the

A. subclavian artery

B. superficial cervical artery

C. inferior thyroid artery

D. suprascapular artery

E. subclavian artery 

Question 13
Correct

Mark 1.00 out of 1.00

To which vein does v.cephalica vein open?

A. anterior jugular

B. axillary 

C. external jugular

D. subclavian

E. brachial

Question 14
Correct

Mark 1.00 out of 1.00

The internal jugular vein is a continuation of:

A. inferior petrosal sinus

B. sigmoid sinus 

C. transverse sinus

D. superior petrosal sinus

E. superior sagittal sinus

https://ets.amu.edu.az/mod/quiz/review.php?attempt=69157&cmid=7722#question-99121-5 5/8
4/20/2021 Midterm exam: İnsan anatomiyası və tibbi terminologiya - 2 (EN) (MPF2): Attempt review

Question 15
Correct

Mark 1.00 out of 1.00

The brachial plexus is formed by the

A. two trunks

B. six trunks

C. three trunks 

D. five trunks

E. four trunks

Question 16
Partially correct

Mark 0.67 out of 1.00

The anterior wall of the third ventricle is formed by:

1. epithalamic commissure

2. anterior commissure

3. terminal lamina 

4. thalamus

5. pineal body

6. columns of the fornix 

Question 17
Incorrect

Mark 0.00 out of 1.00

To the nuclei of the medulla oblongata belong:

1. nucleus olivaris 

2. nucleus gracilis

3. nucleus caudatus

4. nucleus ruber 

5. nucleus cuneatus

6. nucleus dentatus

https://ets.amu.edu.az/mod/quiz/review.php?attempt=69157&cmid=7722#question-99121-5 6/8
4/20/2021 Midterm exam: İnsan anatomiyası və tibbi terminologiya - 2 (EN) (MPF2): Attempt review

Question 18
Correct

Mark 1.00 out of 1.00

Find the accordance:

The superior laryngeal nerve is the branch of vagus nerve 

The tympanic nerve is the branch of glossopharyngeal nerve 

Question 19
Correct

Mark 1.00 out of 1.00

At the level of the lateral edge of the [[1]] rib, the axillary vein continues into the [[2]] vein.

[[1]] first 

[[2]] subclavian 

Question 20
Correct

Mark 1.00 out of 1.00

Find n. vagus:

Answer: 11 

◄ Attendance

Jump to...

A. Abdullayev. 7. General overview of the peripheral nervous system. The classification, development, variation, and anomalies of the
cranial nerves. ►

https://ets.amu.edu.az/mod/quiz/review.php?attempt=69157&cmid=7722#question-99121-5 7/8
4/20/2021 Midterm exam: İnsan anatomiyası və tibbi terminologiya - 2 (EN) (MPF2): Attempt review

https://ets.amu.edu.az/mod/quiz/review.php?attempt=69157&cmid=7722#question-99121-5 8/8
20/04/2021 Midterm exam: İnsan anatomiyası və tibbi terminologiya - 2 (EN) (MPF2): Attempt review

Mənim əsas səhifəm / Mənim fənlərim / İnsan anatomiyası və tibbi terminologiya - 2 (EN) / Ümumi

/ Midterm exam: İnsan anatomiyası və tibbi terminologiya - 2 (EN) (MPF2)

Testə başlandı Tuesday, 20 April 2021, 12:30 PM


State Finished
Sona çatdı Tuesday, 20 April 2021, 12:58 PM
Vaxt keçib 27 dəq. 37 san.
Qiymət 20 (68%) maksimumdan 14

Question 1
Correct

Mark 1.00 out of 1.00

The spinal cord is in a topoqaphic proportion are divided into:

A. cervical,lumbar-sacral parts

B. cervical,coccygeal and lumbar parts

C. cervical ,thoracic,sacral parts

D. cervical,thoracic,lumbar parts 

E. cervical,sacral parts

Question 2
Correct

Mark 1.00 out of 1.00

Which from the following gyruses does not belong to the temporal lobe?

A. supramarginal 

B. temporalis superior

C. parahyppocampalis

D. temporalis inferior

E. temporalis media

https://ets.amu.edu.az/mod/quiz/review.php?attempt=69027&cmid=7722 1/7
20/04/2021 Midterm exam: İnsan anatomiyası və tibbi terminologiya - 2 (EN) (MPF2): Attempt review

Question 3
Incorrect

Mark 0.00 out of 1.00

The upper wall or roof of the posterior horns of the lateral ventricle is formed by the lower surfaces of:

A. commissura posterior 

B. indusium griseum

C. forceps minor

D. commisura anterior

E. tapetum

Question 4
Correct

Mark 1.00 out of 1.00

The cerebral peduncles are located:

A. behind of cerebral aqueduct

B. under the tectum mesencephali 

C. laterally of cerebral aqueduct

D. medially of cerebral aqueduct

E. in front of optic chiasm

Question 5
Correct

Mark 1.00 out of 1.00

The roof of the fourth ventricle in front consists of:


I. Superior cerebellar peduncles
II. Middle cerebellar peduncles
III. Inferior cerebellar peduncles
IV. Velum medullare superius
V. Velum medullare inferius

A. I; IV 

B. I; II

C. II; III

D. III; IV

E. III; V

https://ets.amu.edu.az/mod/quiz/review.php?attempt=69027&cmid=7722 2/7
20/04/2021 Midterm exam: İnsan anatomiyası və tibbi terminologiya - 2 (EN) (MPF2): Attempt review

Question 6
Not answered

Marked out of 1.00

What extends from the superior to the inferior angle of the rhomboid fossa?

A. sulcus medianus

B. eminentia medialis

C. sulcus limitans

D. colliculus facialis

E. striae medullares

Question 7
Correct

Mark 1.00 out of 1.00

Which of the following are the parts of the tympanic membrane?


I. Pars mucosa
II. Pars muscularis
III. Pars tensa
IV. Pars flaccida
V. Pars membranacea

A. IV, V

B. II, IV

C. III, IV 

D. I, II

E. III, V

Question 8
Correct

Mark 1.00 out of 1.00

Which cranial nerve provides the human with the ability to smell a wide variety of odors?

A. vestibulocochlear nerve

B. olfactory nerve 

C. oculomotor nerve

D. optic nerve

E. intermedius nerve

https://ets.amu.edu.az/mod/quiz/review.php?attempt=69027&cmid=7722 3/7
20/04/2021 Midterm exam: İnsan anatomiyası və tibbi terminologiya - 2 (EN) (MPF2): Attempt review

Question 9
Correct

Mark 1.00 out of 1.00

Through which opening does the maxillary nerve pass out the cranial cavity?

A. spinosum

B. rotundum 

C. ovale

D. fissura orbitalis superior

E. lacerum

Question 10
Correct

Mark 1.00 out of 1.00

How is the called right atrio-ventricular valve?

A. atrioventricularis sinistra

B. seminularis

C. bicuspidalis

D. mitralis

E. tricuspidalis 

Question 11
Correct

Mark 1.00 out of 1.00

The ascending aorta passes to the aortic arch at the level of the:

A. left third sternocostal joint

B. left fourth sternocostal joint

C. right second sternocostal joint 

D. left second sternocostal joint

E. right third sternocostal joint

https://ets.amu.edu.az/mod/quiz/review.php?attempt=69027&cmid=7722 4/7
20/04/2021 Midterm exam: İnsan anatomiyası və tibbi terminologiya - 2 (EN) (MPF2): Attempt review

Question 12
Incorrect

Mark 0.00 out of 1.00

It doesn’t belong to the branches of thyrocervical trunk:

A. suprascapular artery

B. superficial cervical artery 

C. inferior thyroid artery

D. ascending cervical artery

E. subscapular artery

Question 13
Not answered

Marked out of 1.00

To which vein does v.cephalica vein open?

A. external jugular

B. anterior jugular

C. brachial

D. axillary

E. subclavian

Question 14
Correct

Mark 1.00 out of 1.00

At which level the internal jugular vein connects with the subclavian vein?

A. thyroid cartilage

B. I rib

C. III rib

D. sterno-clavicular joint 

E. II rib

https://ets.amu.edu.az/mod/quiz/review.php?attempt=69027&cmid=7722 5/7
20/04/2021 Midterm exam: İnsan anatomiyası və tibbi terminologiya - 2 (EN) (MPF2): Attempt review

Question 15
Incorrect

Mark 0.00 out of 1.00

The parietal branches of the thoracic aorta are the followings:


I. Rr. bronchiales
II. Rr. mediastinales
III. Superior phrenic arteries
IV. Posterior intercostalis arteries
V. Rr. pericardiaci

A. III,V

B. I,III

C. III,IV

D. I,II 

E. IV,V

Question 16
Correct

Mark 1.00 out of 1.00

The anterior wall of the third ventricle is formed by:

1. anterior commissure 

2. epithalamic commissure

3. terminal lamina 

4. pineal body

5. columns of the fornix 

6. thalamus

Question 17
Correct

Mark 1.00 out of 1.00

To the branches of the ophthalmic nerve belong:

1. n. infraorbitalis

2. n. petrosus major

3. n. nasociliaris 

4. n. lacrimalis 

5. n. auriculotemporalis

6. n. frontalis 

https://ets.amu.edu.az/mod/quiz/review.php?attempt=69027&cmid=7722 6/7
20/04/2021 Midterm exam: İnsan anatomiyası və tibbi terminologiya - 2 (EN) (MPF2): Attempt review

Question 18
Partially correct

Mark 0.50 out of 1.00

Find the accordance:

Gyrus supramarginalis embraces anterior end of the inferior temporal sulcus 

Gyrus angularis embraces posterior end of the superior temporal sulcus 

Question 19
Not answered

Marked out of 1.00

At the level of the lateral edge of the [[1]] rib, the axillary vein continues into the [[2]] vein.

[[1]] Seçin...

[[2]] Seçin...

Question 20
Correct

Mark 1.00 out of 1.00

Find the ulnar nerve:

Answer: 7 

◄ Attendance

Keç ...

A. Abdullayev. 7. General overview of the peripheral nervous system. The classification, development, variation, and anomalies of the
cranial nerves. ►

https://ets.amu.edu.az/mod/quiz/review.php?attempt=69027&cmid=7722 7/7
2021/4/20 Midterm exam: İnsan anatomiyası və tibbi terminologiya - 2 (EN) (MPF2): Attempt review

Mənim əsas səhifəm / Mənim fənlərim / İnsan anatomiyası və tibbi terminologiya - 2 (EN) / Ümumi

/ Midterm exam: İnsan anatomiyası və tibbi terminologiya - 2 (EN) (MPF2)

Testə başlandı Tuesday, 20 April 2021, 12:30 PM


State Finished
Sona çatdı Tuesday, 20 April 2021, 12:59 PM
Vaxt keçib 29 dəq. 14 san.
Qiymət 20 (80%) maksimumdan 16

Question 1
Correct

Mark 1.00 out of 1.00

On the midline of the anterior surface of the spinal cord is:

A. fissura orbitalis superior

B. ventrolateral grooves

C. dorsal (posterior) median sulcus.

D. dorsolateral grooves

E. ventral median fissure 

Question 2
Incorrect

Mark 0.00 out of 1.00

It happens if the motor analyzer of written speech is damaged:

A. apraxia

B. motor aphasia 

C. tactile agnosia

D. sensory aphasia

E. agraphia

https://ets.amu.edu.az/mod/quiz/review.php?attempt=68865&cmid=7722#question-98823-15 1/7
2021/4/20 Midterm exam: İnsan anatomiyası və tibbi terminologiya - 2 (EN) (MPF2): Attempt review

Question 3
Correct

Mark 1.00 out of 1.00

The floor of the central part of the lateral ventricle is formed by:
I. Body of caudate nucleus
II. Head of caudate nucleus
III. Thalamus
IV. Corpus fornicis
V. Stria terminalis

A. I; III; IV

B. II; III; V

C. I; II; III

D. I; III; V 

E. III; IV; V

Question 4
Correct

Mark 1.00 out of 1.00

Midbrain in Latin is called:

A. metencephalon

B. telencephalon

C. mesencephalon 

D. myelencephalon

E. diencephalon

Question 5
Incorrect

Mark 0.00 out of 1.00

The trigonum lemnisci is bounded medially by:

A. cerebral peduncle

B. brachium colliculi superioris

C. brachium colliculi inferioris 

D. superior cerebellar peduncle

E. inferior cerebellar peduncle

https://ets.amu.edu.az/mod/quiz/review.php?attempt=68865&cmid=7722#question-98823-15 2/7
2021/4/20 Midterm exam: İnsan anatomiyası və tibbi terminologiya - 2 (EN) (MPF2): Attempt review

Question 6
Incorrect

Mark 0.00 out of 1.00

In the superior triangle region of the rhomboid fossa is situated the nuclei of the

A. VIII,VII,III,VI cranial nerves 

B. V,II,III,VI cranial nerves

C. X,XI,XII cranial nerves

D. V, VI, VII , VIII cranial nerves

E. I,II.V,XII cranial nerves

Question 7
Correct

Mark 1.00 out of 1.00

The oculomotor nerve emerges from cranial cavity through:

A. foramen ethmoidale anterior

B. canalis opticus

C. fissura orbitalis inferior

D. foramen rotundum

E. fissura orbitalis superior 

Question 8
Correct

Mark 1.00 out of 1.00

The glossopharyngeal nerve leaves the skull through:

A. canalis hypoglossus

B. foramen ovale

C. foramen lacerum

D. foramen rotundum

E. foramen jugulare 

https://ets.amu.edu.az/mod/quiz/review.php?attempt=68865&cmid=7722#question-98823-15 3/7
2021/4/20 Midterm exam: İnsan anatomiyası və tibbi terminologiya - 2 (EN) (MPF2): Attempt review

Question 9
Correct

Mark 1.00 out of 1.00

Which muscles are innervated by the nervus accesorius?


I. M. sternocleidomastoideus
II. M. trapezius
III. M. digastricus
IV. M. latissimus dorsi
V. M. buccinator

A. III,V

B. I,II 

C. II,III

D. III,IV

E. IV,V

Question 10
Correct

Mark 1.00 out of 1.00

Which layers of the serous pericardium?

A. costal,parietal

B. mediastinal, costal

C. parietal, visceral 

D. parietal, costal

E. diaphragmaric,visceral

Question 11
Correct

Mark 1.00 out of 1.00

Where is located common carotid artery bifurcation?

A. superior border of the thyroid cartilage 

B. root of the tongue

C. inferior border of the thyroid cartilage

D. posterior border of the palate

E. V cervical vertebra

https://ets.amu.edu.az/mod/quiz/review.php?attempt=68865&cmid=7722#question-98823-15 4/7
2021/4/20 Midterm exam: İnsan anatomiyası və tibbi terminologiya - 2 (EN) (MPF2): Attempt review

Question 12
Correct

Mark 1.00 out of 1.00

The vertebral artery (a. vertebralis), is the first branch of the

A. suprascapular artery

B. superficial cervical artery

C. subclavian artery 

D. inferior thyroid artery

E. subclavian artery

Question 13
Correct

Mark 1.00 out of 1.00

It is not the branch of the ulnar artery:

A. а. carpalis dorsalis

B. а. carpalis palmaris

C. а. interossea communis

D. а. recurrens radialis 

E. а. recurrens ulnaris

Question 14
Correct

Mark 1.00 out of 1.00

The vein which is composed by the connecting occipital and posterior auricular veins:

A. internal jugular vein

B. superior venаcava

C. anterior jugular vein

D. brachiocephalic vein

E. external jugular vein 

https://ets.amu.edu.az/mod/quiz/review.php?attempt=68865&cmid=7722#question-98823-15 5/7
2021/4/20 Midterm exam: İnsan anatomiyası və tibbi terminologiya - 2 (EN) (MPF2): Attempt review

Question 15
Incorrect

Mark 0.00 out of 1.00

The visceral branches of the thoracic aorta are the followings:


I. Rr. bronchiales
II. Rr. mediastinales
III. Superior phrenic arteries
IV. Posterior intercostalis arteries
V. Rr. pericardiaci

A. II,III,V

B. I,II,V

C. I,II,IV

D. III,IV,V

E. II,III,IV 

Question 16
Correct

Mark 1.00 out of 1.00

Which muscles are innervated by n. dorsalis scapulae?

1. m. pectoralis major

2. m. levator scapulae 

3. m. latissimus dorsi

4. m. coracobrachialis

5. mm. rhomboidei 

Question 17
Correct

Mark 1.00 out of 1.00

To the branches of the brachial artery belong:

1. a. thoracodorsalis

2. a. collateralis ulnaris superior 

3. a. subscapularis

4. a. thoracoacromialis

5. a. collateralis ulnaris inferior 

6. a. profunda brachii 

https://ets.amu.edu.az/mod/quiz/review.php?attempt=68865&cmid=7722#question-98823-15 6/7
2021/4/20 Midterm exam: İnsan anatomiyası və tibbi terminologiya - 2 (EN) (MPF2): Attempt review

Question 18
Correct

Mark 1.00 out of 1.00

Find the accordance:

The external ear includes auricle and external acoustic meatus 

The inner ear consists of bony labyrinth and membranous labyrinth 

Question 19
Correct

Mark 1.00 out of 1.00

The external capsule is a layer of [[1]] matter that is situated between the claustrum and [[2]] nucleus.

[[1]] white 

[[2]] lentiform 

Question 20
Correct

Mark 1.00 out of 1.00

Find the ulnar nerve:

Answer: 7 

◄ Attendance

Keç ...

A. Abdullayev. 7. General overview of the peripheral nervous system. The classification, development, variation, and anomalies of the
cranial nerves. ►

https://ets.amu.edu.az/mod/quiz/review.php?attempt=68865&cmid=7722#question-98823-15 7/7
4/20/2021 Midterm exam: İnsan anatomiyası və tibbi terminologiya - 2 (EN) (MPF2): Attempt review

Mənim əsas səhifəm / Mənim fənlərim / İnsan anatomiyası və tibbi terminologiya - 2 (EN) / Ümumi

/ Midterm exam: İnsan anatomiyası və tibbi terminologiya - 2 (EN) (MPF2)

Testə başlandı Tuesday, 20 April 2021, 12:30 PM


State Finished
Sona çatdı Tuesday, 20 April 2021, 12:56 PM
Vaxt keçib 26 dəq. 15 san.
Qiymət 20 (98%) maksimumdan 20

Question 1
Correct

Mark 1.00 out of 1.00

Vermis is belonged to the:

A. pons

B. midbrain

C. cerebellum 

D. medulla oblongata

E. caudate nucleus

Question 2
Correct

Mark 1.00 out of 1.00

They are situated on the inferior surface of the temporal lobe:


I. Sulcus occipitotemporalis
II. Gyrus temporalis medius
III. Gyri temporales transversi
IV. Sulcus collateralis

A. I; II

B. III; IV

C. II; III

D. II; IV

E. I; IV 

https://ets.amu.edu.az/mod/quiz/review.php?attempt=68726&cmid=7722#question-98698-17 1/7
4/20/2021 Midterm exam: İnsan anatomiyası və tibbi terminologiya - 2 (EN) (MPF2): Attempt review

Question 3
Correct

Mark 1.00 out of 1.00

The extremal capsule is the white matter separating:

A. claustrum from cortex of insula 

B. globi pallidus medialis et lateralis

C. putamen and claustrum

D. globus pallidus lateralis and putamen

E. globus pallidus medialis and nucleus caudatus

Question 4
Correct

Mark 1.00 out of 1.00

Where is the gyrus parahippocampalis situated?

A. in the posterior part of the gyrus temporalis inferior

B. between the hippocampal and collateral sulcus 

C. between the superior and inferior temporal sulcus

D. between the inferior temporal and occipitotemporalis sulcus

E. between the lateral and superior temporal sulcus

Question 5
Correct

Mark 1.00 out of 1.00

Where is the vallecula cerebelli located?

A. on the posterior border of cerebellum

B. on the superior surface of cerebellum

C. on the anterior border of cerebellum

D. on the inferior surface of cerebellum 

E. on the anterior and posterior borders of cerebellum

https://ets.amu.edu.az/mod/quiz/review.php?attempt=68726&cmid=7722#question-98698-17 2/7
4/20/2021 Midterm exam: İnsan anatomiyası və tibbi terminologiya - 2 (EN) (MPF2): Attempt review

Question 6
Correct

Mark 1.00 out of 1.00

What extends from the superior to the inferior angle of the rhomboid fossa?

A. sulcus medianus 

B. sulcus limitans

C. colliculus facialis

D. eminentia medialis

E. striae medullares

Question 7
Correct

Mark 1.00 out of 1.00

Where does the optic nerve enter the cranial cavity from?

A. foramen zygomatico-orbitale

B. foramen ovale

C. foramen ethmoidae anterior

D. foramen ethmoidale posterior

E. canalis opticus 

Question 8
Correct

Mark 1.00 out of 1.00

Which cranial nerve provides the human with the ability to smell a wide variety of odors?

A. optic nerve

B. olfactory nerve 

C. intermedius nerve

D. oculomotor nerve

E. vestibulocochlear nerve

https://ets.amu.edu.az/mod/quiz/review.php?attempt=68726&cmid=7722#question-98698-17 3/7
4/20/2021 Midterm exam: İnsan anatomiyası və tibbi terminologiya - 2 (EN) (MPF2): Attempt review

Question 9
Correct

Mark 1.00 out of 1.00

It does not belong to the branches of the cervical part of the vagus nerve:

A. rr. cardiaci cervicales superiores

B. pharyngeal branches

C. recurrent laryngeal nerve

D. auricular branch 

E. superior laryngeal nerve

Question 10
Correct

Mark 1.00 out of 1.00

How is the called right atrio-ventricular valve?

A. tricuspidalis 

B. atrioventricularis sinistra

C. mitralis

D. bicuspidalis

E. seminularis

Question 11
Correct

Mark 1.00 out of 1.00

The ascending aorta passes to the aortic arch at the level of the:

A. right third sternocostal joint

B. left fourth sternocostal joint

C. left third sternocostal joint

D. right second sternocostal joint 

E. left second sternocostal joint

https://ets.amu.edu.az/mod/quiz/review.php?attempt=68726&cmid=7722#question-98698-17 4/7
4/20/2021 Midterm exam: İnsan anatomiyası və tibbi terminologiya - 2 (EN) (MPF2): Attempt review

Question 12
Correct

Mark 1.00 out of 1.00

It doesn’t belong to the branches of thyrocervical trunk:

A. subscapular artery 

B. inferior thyroid artery

C. suprascapular artery

D. ascending cervical artery

E. superficial cervical artery

Question 13
Correct

Mark 1.00 out of 1.00

To which vein does v.basilica open?

A. subclavian

B. axillary

C. brachial 

D. anterior jugular

E. external jugular

Question 14
Correct

Mark 1.00 out of 1.00

It does not belong to the tributaries of the external jugular vein:

A. v. suprascapularis

B. v. canalis pterygoidei 

C. v. auricularis posterior

D. v. occipitalis

E. v. jugularis anterior

https://ets.amu.edu.az/mod/quiz/review.php?attempt=68726&cmid=7722#question-98698-17 5/7
4/20/2021 Midterm exam: İnsan anatomiyası və tibbi terminologiya - 2 (EN) (MPF2): Attempt review

Question 15
Correct

Mark 1.00 out of 1.00

The visceral branches of the thoracic aorta are the followings:


I. Rr. bronchiales
II. Rr. mediastinales
III. Superior phrenic arteries
IV. Posterior intercostalis arteries
V. Rr. pericardiaci

A. II,III,IV

B. I,II,IV

C. III,IV,V

D. I,II,V 

E. II,III,V

Question 16
Correct

Mark 1.00 out of 1.00

The anterior wall of the third ventricle is formed by:

1. terminal lamina 

2. anterior commissure 

3. columns of the fornix 

4. pineal body

5. thalamus

6. epithalamic commissure

Question 17
Partially correct

Mark 0.67 out of 1.00

To the nuclei of the medulla oblongata belong:

1. nucleus caudatus

2. nucleus olivaris 

3. nucleus dentatus

4. nucleus ruber

5. nucleus gracilis 

6. nucleus cuneatus

https://ets.amu.edu.az/mod/quiz/review.php?attempt=68726&cmid=7722#question-98698-17 6/7
4/20/2021 Midterm exam: İnsan anatomiyası və tibbi terminologiya - 2 (EN) (MPF2): Attempt review

Question 18
Correct

Mark 1.00 out of 1.00

Find the accordance:

A. interossea communis is the branch of ulnar artery 

A. thoracoacromialis is the branch of axillary artery 

Question 19
Correct

Mark 1.00 out of 1.00

At the level of the lateral edge of the [[1]] rib, the axillary vein continues into the [[2]] vein.

[[1]] first 

[[2]] subclavian 

Question 20
Correct

Mark 1.00 out of 1.00

Find the ulnar nerve:

Answer: 7 

◄ Attendance

Keç ...

A. Abdullayev. 7. General overview of the peripheral nervous system. The classification, development, variation, and anomalies of the cranial
nerves. ►

https://ets.amu.edu.az/mod/quiz/review.php?attempt=68726&cmid=7722#question-98698-17 7/7
4/20/2021 Midterm exam: İnsan anatomiyası və tibbi terminologiya - 2 (EN) (MPF2): Attempt review

Dashboard / My courses / İnsan anatomiyası və tibbi terminologiya - 2 (EN) / General

/ Midterm exam: İnsan anatomiyası və tibbi terminologiya - 2 (EN) (MPF2)

Started on Tuesday, 20 April 2021, 12:30 PM


State Finished
Completed on Tuesday, 20 April 2021, 12:54 PM
Time taken 23 mins 52 secs
Grade 19 out of 20 (95%)

Question 1
Correct

Mark 1.00 out of 1.00

In the posterior funiculus of the spinal cord are located:

A. tractus tectospinalis, tractus vestibulospinalis

B. tractus rubrospinalis, tractus spinocerebellaris dorsalis

C. fasciculus gracilis, tractus tectospinalis

D. tractus vestibulospinalis, tractus spinocerebellaris ventralis

E. fasciculus cuneatus, fasciculus gracilis 

Question 2
Correct

Mark 1.00 out of 1.00

It happens when the motor analyzer of speech articulation is damaged:

A. sensory aphasia

B. agraphia

C. motor aphasia 

D. tactile agnosia

E. apraxia

https://ets.amu.edu.az/mod/quiz/review.php?attempt=68873&cmid=7722 1/8
4/20/2021 Midterm exam: İnsan anatomiyası və tibbi terminologiya - 2 (EN) (MPF2): Attempt review

Question 3
Correct

Mark 1.00 out of 1.00

The extremal capsule is the white matter separating:

A. globus pallidus lateralis and putamen

B. putamen and claustrum

C. claustrum from cortex of insula 

D. globi pallidus medialis et lateralis

E. globus pallidus medialis and nucleus caudatus

Question 4
Correct

Mark 1.00 out of 1.00

It does not belong to epithalamus:

A. epiphysis

B. habenular trigone

C. hypophysis 

D. epithalamic commissure

E. habenular commissure

Question 5
Correct

Mark 1.00 out of 1.00

Where is the vallecula cerebelli located?

A. on the superior surface of cerebellum

B. on the anterior border of cerebellum

C. on the anterior and posterior borders of cerebellum

D. on the inferior surface of cerebellum 

E. on the posterior border of cerebellum

https://ets.amu.edu.az/mod/quiz/review.php?attempt=68873&cmid=7722 2/8
4/20/2021 Midterm exam: İnsan anatomiyası və tibbi terminologiya - 2 (EN) (MPF2): Attempt review

Question 6
Correct

Mark 1.00 out of 1.00

The lower part of the rhomboid fossa is corresponded to:

A. diencephalon

B. mesencephalon

C. medulla oblongata 

D. telencephalon

E. metencephalon

Question 7
Correct

Mark 1.00 out of 1.00

They are belonged to middle ear:


I. Auricula
II. Tuba auditiva
III. Meatus acusticus externus osseus
IV. Meatus acusticus externus cartilagineus
V. Cavitas tympani

A. II;III

B. IV;V

C. III;IV

D. II;V 

E. I;III

Question 8
Correct

Mark 1.00 out of 1.00

The glossopharyngeal nerve descends through the upper part of the neck between the:

A. internal jugular vein and common carotid artery

B. internal jugular vein and external carotid artery

C. internal jugular vein and external jugular vein

D. internal carotid artery and external carotid artery

E. internal jugular vein and internal carotid artery 

https://ets.amu.edu.az/mod/quiz/review.php?attempt=68873&cmid=7722 3/8
4/20/2021 Midterm exam: İnsan anatomiyası və tibbi terminologiya - 2 (EN) (MPF2): Attempt review

Question 9
Correct

Mark 1.00 out of 1.00

It does not belong to the branches of the ophthalmic nerve:


I. N. infaorbitalis
II. N. lacrimalis
III. Rr. pterygopalatini
IV. N. nasociliaris
V. N. frontalis
VI. N. zygomaticus

A. I; III; V

B. II; IV; V

C. I; III; VI 

D. II; III; IV

E. III; IV; VI

Question 10
Correct

Mark 1.00 out of 1.00

Where is located the superior border of the heart?

A. line connecting upper edges of III ribs 

B. line connecting lower edges of the V ribs

C. line connecting upper edges of VII ribs

D. line connecting upper edges the IV ribs

E. line connecting lower edges of VI ribs

Question 11
Correct

Mark 1.00 out of 1.00

Thoracic aorta locates between the:

A. ThI-ThXII

B. ThVI-ThXII

C. ThIV-ThX

D. ThIV-ThXII 

E. ThIII-ThVIII

https://ets.amu.edu.az/mod/quiz/review.php?attempt=68873&cmid=7722 4/8
4/20/2021 Midterm exam: İnsan anatomiyası və tibbi terminologiya - 2 (EN) (MPF2): Attempt review

Question 12
Incorrect

Mark 0.00 out of 1.00

The vertebral artery (a. vertebralis), is the first branch of the

A. subclavian artery

B. superficial cervical artery

C. inferior thyroid artery

D. suprascapular artery

E. subclavian artery 

Question 13
Correct

Mark 1.00 out of 1.00

To which vein does v.cephalica vein open?

A. axillary 

B. anterior jugular

C. brachial

D. external jugular

E. subclavian

Question 14
Correct

Mark 1.00 out of 1.00

At which level does superior vena cava form?

A. I sternocostal synchondrosis 

B. III sternocostal joint

C. IV sternocostal joint

D. sternoclavicular joint

E. II sternocostal joint

https://ets.amu.edu.az/mod/quiz/review.php?attempt=68873&cmid=7722 5/8
4/20/2021 Midterm exam: İnsan anatomiyası və tibbi terminologiya - 2 (EN) (MPF2): Attempt review

Question 15
Correct

Mark 1.00 out of 1.00

This doesn’t belong to the cutaneous branches of the cervical plexus:

A. n. auricularis magnus

B. n. transversus colli

C. n. subclavius 

D. n. occipitalis minor

E. n. supraclavicularis

Question 16
Correct

Mark 1.00 out of 1.00

The anterior wall of the third ventricle is formed by:

1. thalamus

2. anterior commissure 

3. columns of the fornix 

4. pineal body

5. terminal lamina 

6. epithalamic commissure

Question 17
Correct

Mark 1.00 out of 1.00

To the nuclei of the medulla oblongata belong:

1. nucleus cuneatus 

2. nucleus olivaris 

3. nucleus ruber

4. nucleus caudatus

5. nucleus dentatus

6. nucleus gracilis 

https://ets.amu.edu.az/mod/quiz/review.php?attempt=68873&cmid=7722 6/8
4/20/2021 Midterm exam: İnsan anatomiyası və tibbi terminologiya - 2 (EN) (MPF2): Attempt review

Question 18
Correct

Mark 1.00 out of 1.00

Find the accordance:

Gyrus angularis embraces posterior end of the superior temporal sulcus 

Gyrus supramarginalis embraces posterior branch of the lateral sulcus 

Question 19
Correct

Mark 1.00 out of 1.00

The external capsule is a layer of [[1]] matter that is situated between the claustrum and [[2]] nucleus.

[[1]] white 

[[2]] lentiform 

Question 20
Correct

Mark 1.00 out of 1.00

Find the colliculus facialis:

Answer: 6 

◄ Attendance

Jump to...

A. Abdullayev. 7. General overview of the peripheral nervous system. The classification, development, variation, and anomalies of the
cranial nerves. ►

https://ets.amu.edu.az/mod/quiz/review.php?attempt=68873&cmid=7722 7/8
4/20/2021 Midterm exam: İnsan anatomiyası və tibbi terminologiya - 2 (EN) (MPF2): Attempt review

https://ets.amu.edu.az/mod/quiz/review.php?attempt=68873&cmid=7722 8/8
4/20/2021 Midterm exam: İnsan anatomiyası və tibbi terminologiya - 2 (EN) (MPF2): Attempt review

Dashboard / My courses / İnsan anatomiyası və tibbi terminologiya - 2 (EN) / General

/ Midterm exam: İnsan anatomiyası və tibbi terminologiya - 2 (EN) (MPF2)

Started on Tuesday, 20 April 2021, 12:30 PM


State Finished
Completed on Tuesday, 20 April 2021, 12:56 PM
Time taken 25 mins 47 secs
Grade 17 out of 20 (85%)

Question 1
Correct

Mark 1.00 out of 1.00

The length of the spinal cord in female is (cm) :

A. 35

B. 32

C. 50

D. 43 

E. 46

Question 2
Correct

Mark 1.00 out of 1.00

The basilar groove is belonged to:

A. midbrain

B. pons 

C. diencephalon

D. lamina terminalis

E. medulla oblongata

https://ets.amu.edu.az/mod/quiz/review.php?attempt=69006&cmid=7722 1/8
4/20/2021 Midterm exam: İnsan anatomiyası və tibbi terminologiya - 2 (EN) (MPF2): Attempt review

Question 3
Correct

Mark 1.00 out of 1.00

The floor of the central part of the lateral ventricle is formed by:
I. Body of caudate nucleus
II. Head of caudate nucleus
III. Thalamus
IV. Corpus fornicis
V. Stria terminalis

A. I; II; III

B. II; III; V

C. III; IV; V

D. I; III; V 

E. I; III; IV

Question 4
Correct

Mark 1.00 out of 1.00

Where is the gyrus parahippocampalis situated?

A. between the superior and inferior temporal sulcus

B. between the lateral and superior temporal sulcus

C. between the hippocampal and collateral sulcus 

D. in the posterior part of the gyrus temporalis inferior

E. between the inferior temporal and occipitotemporalis sulcus

Question 5
Correct

Mark 1.00 out of 1.00

In region of the lateral angles of the rhomboid fossa lays:

A. locus caeruleus

B. eminentia medialis

C. nucleus dorsalis nervi vagi

D. area vestibularis 

E. nucleus ruber

https://ets.amu.edu.az/mod/quiz/review.php?attempt=69006&cmid=7722 2/8
4/20/2021 Midterm exam: İnsan anatomiyası və tibbi terminologiya - 2 (EN) (MPF2): Attempt review

Question 6
Correct

Mark 1.00 out of 1.00

What extends from the superior to the inferior angle of the rhomboid fossa?

A. eminentia medialis

B. striae medullares

C. sulcus medianus 

D. colliculus facialis

E. sulcus limitans

Question 7
Correct

Mark 1.00 out of 1.00

The nucleus of the olfactory analyzer lies in the:

A. medial surface of the occipital lobe

B. superior parietal lobule

C. uncus of the parahippocampal gyrus 

D. middle temporal gyrus

E. superior temporal gyrus

Question 8
Correct

Mark 1.00 out of 1.00

It arises from the facial nerve just above the stylomastoid foramen:

A. lesser petrosal nerve

B. branch to the tympanic plexus

C. the nerve to the stapedius

D. chorda tympani 

E. greater petrosal nerve

https://ets.amu.edu.az/mod/quiz/review.php?attempt=69006&cmid=7722 3/8
4/20/2021 Midterm exam: İnsan anatomiyası və tibbi terminologiya - 2 (EN) (MPF2): Attempt review

Question 9
Correct

Mark 1.00 out of 1.00

It does not belong to the branches of the cervical part of the vagus nerve:

A. pharyngeal branches

B. superior laryngeal nerve

C. recurrent laryngeal nerve

D. rr. cardiaci cervicales superiores

E. auricular branch 

Question 10
Correct

Mark 1.00 out of 1.00

Which layers of the serous pericardium?

A. mediastinal, costal

B. costal,parietal

C. parietal, costal

D. diaphragmaric,visceral

E. parietal, visceral 

Question 11
Incorrect

Mark 0.00 out of 1.00

The first branch of the internal carotid artery is the following:

A. a. ophthalmica 

B. a. cerebri media

C. a. cerebri anterior

D. rr. caroticotympanici

E. a. communicans posterior

https://ets.amu.edu.az/mod/quiz/review.php?attempt=69006&cmid=7722 4/8
4/20/2021 Midterm exam: İnsan anatomiyası və tibbi terminologiya - 2 (EN) (MPF2): Attempt review

Question 12
Correct

Mark 1.00 out of 1.00

It does not belong the branches of the internal carotid artery:

A. a. cerebri posterior 

B. a. ophthalmica

C. a. cerebri anterior

D. a. cerebri media

E. a. communicans posterior

Question 13
Correct

Mark 1.00 out of 1.00

It is not the branch of the radial artery:

A. r. carpalis dorsalis

B. a. recurrens radialis

C. r. carpalis palmaris

D. а. interossea communis 

E. a. princeps pollicis

Question 14
Correct

Mark 1.00 out of 1.00

The vein which is composed by the connecting occipital and posterior auricular veins:

A. superior venаcava

B. anterior jugular vein

C. brachiocephalic vein

D. internal jugular vein

E. external jugular vein 

https://ets.amu.edu.az/mod/quiz/review.php?attempt=69006&cmid=7722 5/8
4/20/2021 Midterm exam: İnsan anatomiyası və tibbi terminologiya - 2 (EN) (MPF2): Attempt review

Question 15
Correct

Mark 1.00 out of 1.00

What is called the continuation of the right ascending lumbar vein?

A. azygos vein 

B. pericardiaco-phrenic veins

C. internal thoracic vein

D. accessory hemiazygos vein

E. hemiazygos vein

Question 16
Incorrect

Mark 0.00 out of 1.00

The anterior wall of the third ventricle is formed by:

1. pineal body 

2. thalamus

3. anterior commissure 

4. columns of the fornix

5. epithalamic commissure

6. terminal lamina 

Question 17
Incorrect

Mark 0.00 out of 1.00

Which muscles are innervated by n. dorsalis scapulae?

1. m. levator scapulae 

2. m. latissimus dorsi

3. m. coracobrachialis

4. m. pectoralis major 

5. mm. rhomboidei 

https://ets.amu.edu.az/mod/quiz/review.php?attempt=69006&cmid=7722 6/8
4/20/2021 Midterm exam: İnsan anatomiyası və tibbi terminologiya - 2 (EN) (MPF2): Attempt review

Question 18
Correct

Mark 1.00 out of 1.00

Find the accordance:

Gyrus angularis embraces posterior end of the superior temporal sulcus 

Gyrus supramarginalis embraces posterior branch of the lateral sulcus 

Question 19
Correct

Mark 1.00 out of 1.00

The trochlear nerve enters the orbit through the [[1]] and innervates the [[2]].

[[1]] superior orbital fissure 

[[2]] superior oblique muscle 

Question 20
Correct

Mark 1.00 out of 1.00

Find n. vagus:

Answer: 11 

◄ Attendance

Jump to...

A. Abdullayev. 7. General overview of the peripheral nervous system. The classification, development, variation, and anomalies of the
cranial nerves. ►

https://ets.amu.edu.az/mod/quiz/review.php?attempt=69006&cmid=7722 7/8
4/20/2021 Midterm exam: İnsan anatomiyası və tibbi terminologiya - 2 (EN) (MPF2): Attempt review

https://ets.amu.edu.az/mod/quiz/review.php?attempt=69006&cmid=7722 8/8
.

G0OdLUCk
1) Find the accordance:

From the lateral cord of the brachial plexus arises n. musculocutaneus

From the posterior cord of the brachial plexus arises n. radialis

2) Find the according

From the lateral cord of the brachial plexus arises N. musculocutaneous

From the posterior cord of the brachial plexus arises N. Radialis

3) It does not belong to the branches of intracranial part of the facial nerve:
A. The nerve to the stapedius
B. Chorda tympani
C. Branch to the tympanic plexus
D. Lesser petrosal nerve
E. Greater petrosal nerve
4) The branch of a.profunda brachii:
A. a. circumflexahumeriposterior
B. a. recurrens radialis
C. a. recurrens ulnaris
D. a. circumflexahumerianterior
E. a. collateralismedia

5) It doesn’t belong to the external ear:


l. Auricula
ll. Tuba auditive
lll. Meatus acusticus externus osseus
lV. Meatus acusticus externus cartilaginous
V. Cavitas tympani

A. lV,V
B. lll,lV
C. ll,lll
D. l,lll
E. ll,V
4/21/2021 Midterm exam: İnsan anatomiyası və tibbi terminologiya - 2 (EN) (MPF2): Attempt review

Dashboard / My courses / İnsan anatomiyası və tibbi terminologiya - 2 (EN) / General

/ Midterm exam: İnsan anatomiyası və tibbi terminologiya - 2 (EN) (MPF2)

Started on Tuesday, 20 April 2021, 12:30 PM


State Finished
Completed on Tuesday, 20 April 2021, 12:59 PM
Time taken 29 mins 22 secs
Grade 19 out of 20 (95%)

Question 1
Incorrect

Mark 0.00 out of 1.00

In the posterior funiculus of the spinal cord are located:

A. tractus rubrospinalis, tractus spinocerebellaris dorsalis

B. tractus vestibulospinalis, tractus spinocerebellaris ventralis

C. tractus tectospinalis, tractus vestibulospinalis

D. fasciculus gracilis, tractus tectospinalis 

E. fasciculus cuneatus, fasciculus gracilis

Question 2
Correct

Mark 1.00 out of 1.00

Which from the following gyruses does not belong to the temporal lobe?

A. temporalis inferior

B. temporalis superior

C. temporalis media

D. supramarginal 

E. parahyppocampalis

https://ets.amu.edu.az/mod/quiz/review.php?attempt=68698&cmid=7722 1/8
4/21/2021 Midterm exam: İnsan anatomiyası və tibbi terminologiya - 2 (EN) (MPF2): Attempt review

Question 3
Correct

Mark 1.00 out of 1.00

Posterior end of the thalamus is called:

A. fissura chorioidea

B. tuberculum anterius

C. lamina affixa

D. stria terminalis

E. pulvinar 

Question 4
Correct

Mark 1.00 out of 1.00

It does not belong to epithalamus:

A. hypophysis 

B. epithalamic commissure

C. habenular trigone

D. habenular commissure

E. epiphysis

Question 5
Correct

Mark 1.00 out of 1.00

The dorsal part of metencephalon is:

A. cerebellum 

B. corpus mamillaris

C. pedunculus cerebri

D. pons

E. tuber cinereum

https://ets.amu.edu.az/mod/quiz/review.php?attempt=68698&cmid=7722 2/8
4/21/2021 Midterm exam: İnsan anatomiyası və tibbi terminologiya - 2 (EN) (MPF2): Attempt review

Question 6
Correct

Mark 1.00 out of 1.00

In medulla oblongata is not located:

A. center of vomiting

B. center of swallowing

C. circulatory center

D. respiratory center

E. highest center of vegetative nervous system 

Question 7
Correct

Mark 1.00 out of 1.00

The oculomotor nerve emerges from cranial cavity through:

A. fissura orbitalis superior 

B. canalis opticus

C. foramen ethmoidale anterior

D. foramen rotundum

E. fissura orbitalis inferior

Question 8
Correct

Mark 1.00 out of 1.00

The glossopharyngeal nerve leaves the skull through:

A. canalis hypoglossus

B. foramen jugulare 

C. foramen lacerum

D. foramen rotundum

E. foramen ovale

https://ets.amu.edu.az/mod/quiz/review.php?attempt=68698&cmid=7722 3/8
4/21/2021 Midterm exam: İnsan anatomiyası və tibbi terminologiya - 2 (EN) (MPF2): Attempt review

Question 9
Correct

Mark 1.00 out of 1.00

It does not belong to the branches of the ophthalmic nerve:


I. N. infaorbitalis
II. N. lacrimalis
III. Rr. pterygopalatini
IV. N. nasociliaris
V. N. frontalis
VI. N. zygomaticus

A. III; IV; VI

B. I; III; V

C. I; III; VI 

D. II; III; IV

E. II; IV; V

Question 10
Correct

Mark 1.00 out of 1.00

How many layers can be distinguished in the myocardium of the ventricles?

A. 2

B. 3 

C. 4

D. 1

E. 5

Question 11
Correct

Mark 1.00 out of 1.00

Thoracic aorta locates between the:

A. ThI-ThXII

B. ThIV-ThXII 

C. ThVI-ThXII

D. ThIII-ThVIII

E. ThIV-ThX

https://ets.amu.edu.az/mod/quiz/review.php?attempt=68698&cmid=7722 4/8
4/21/2021 Midterm exam: İnsan anatomiyası və tibbi terminologiya - 2 (EN) (MPF2): Attempt review

Question 12
Correct

Mark 1.00 out of 1.00

It does not compose the thyrocervical trunk:

A. inferior thyroid artery

B. superficial cervical artery

C. suprascapular artery

D. vertebral artery 

E. ascending cervical artery

Question 13
Correct

Mark 1.00 out of 1.00

To which vein does v.cephalica vein open?

A. subclavian

B. axillary 

C. anterior jugular

D. external jugular

E. brachial

Question 14
Correct

Mark 1.00 out of 1.00

The vein which is composed by the connecting occipital and posterior auricular veins:

A. internal jugular vein

B. brachiocephalic vein

C. anterior jugular vein

D. superior venаcava

E. external jugular vein 

https://ets.amu.edu.az/mod/quiz/review.php?attempt=68698&cmid=7722 5/8
4/21/2021 Midterm exam: İnsan anatomiyası və tibbi terminologiya - 2 (EN) (MPF2): Attempt review

Question 15
Correct

Mark 1.00 out of 1.00

Median nerve originates from the

A. axillary nerve

B. musculocutaneous nerve

C. medial and lateral cords of the brachial plexus 

D. radial nerve

E. ulnar nerve

Question 16
Correct

Mark 1.00 out of 1.00

Which muscles are innervated by n. dorsalis scapulae?

1. m. coracobrachialis

2. m. pectoralis major

3. m. latissimus dorsi

4. m. levator scapulae 

5. mm. rhomboidei 

Question 17
Correct

Mark 1.00 out of 1.00

To the branches of the ophthalmic nerve belong:

1. n. petrosus major

2. n. nasociliaris 

3. n. auriculotemporalis

4. n. lacrimalis 

5. n. infraorbitalis

6. n. frontalis 

https://ets.amu.edu.az/mod/quiz/review.php?attempt=68698&cmid=7722 6/8
4/21/2021 Midterm exam: İnsan anatomiyası və tibbi terminologiya - 2 (EN) (MPF2): Attempt review

Question 18
Correct

Mark 1.00 out of 1.00

Find the accordance:

A. interossea communis is the branch of ulnar artery 

A. thoracoacromialis is the branch of axillary artery 

Question 19
Correct

Mark 1.00 out of 1.00

The ulnar artery departs from the [[1]] artery at the level of the [[2]] of the ulna.

[[1]] brachial 

[[2]] coronoid process 

Question 20
Correct

Mark 1.00 out of 1.00

Find n. vagus:

Answer: 11 

◄ Attendance

Jump to...

A. Abdullayev. 7. General overview of the peripheral nervous system. The classification, development, variation, and anomalies of the
cranial nerves. ►

https://ets.amu.edu.az/mod/quiz/review.php?attempt=68698&cmid=7722 7/8
4/21/2021 Midterm exam: İnsan anatomiyası və tibbi terminologiya - 2 (EN) (MPF2): Attempt review

https://ets.amu.edu.az/mod/quiz/review.php?attempt=68698&cmid=7722 8/8
2021/4/20 Midterm exam: İnsan anatomiyası və tibbi terminologiya - 2 (EN) (MPF2): Attempt review

Mənim əsas səhifəm / Mənim fənlərim / İnsan anatomiyası və tibbi terminologiya - 2 (EN) / Ümumi

/ Midterm exam: İnsan anatomiyası və tibbi terminologiya - 2 (EN) (MPF2)

Testə başlandı Tuesday, 20 April 2021, 12:30 PM


State Finished
Sona çatdı Tuesday, 20 April 2021, 12:59 PM
Vaxt keçib 29 dəq. 14 san.
Qiymət 20 (80%) maksimumdan 16

Question 1
Correct

Mark 1.00 out of 1.00

On the midline of the anterior surface of the spinal cord is:

A. fissura orbitalis superior

B. ventrolateral grooves

C. dorsal (posterior) median sulcus.

D. dorsolateral grooves

E. ventral median fissure 

Question 2
Incorrect

Mark 0.00 out of 1.00

It happens if the motor analyzer of written speech is damaged:

A. apraxia

B. motor aphasia 

C. tactile agnosia

D. sensory aphasia

E. agraphia

https://ets.amu.edu.az/mod/quiz/review.php?attempt=68865&cmid=7722#question-98823-15 1/7
2021/4/20 Midterm exam: İnsan anatomiyası və tibbi terminologiya - 2 (EN) (MPF2): Attempt review

Question 3
Correct

Mark 1.00 out of 1.00

The floor of the central part of the lateral ventricle is formed by:
I. Body of caudate nucleus
II. Head of caudate nucleus
III. Thalamus
IV. Corpus fornicis
V. Stria terminalis

A. I; III; IV

B. II; III; V

C. I; II; III

D. I; III; V 

E. III; IV; V

Question 4
Correct

Mark 1.00 out of 1.00

Midbrain in Latin is called:

A. metencephalon

B. telencephalon

C. mesencephalon 

D. myelencephalon

E. diencephalon

Question 5
Incorrect

Mark 0.00 out of 1.00

The trigonum lemnisci is bounded medially by:

A. cerebral peduncle

B. brachium colliculi superioris

C. brachium colliculi inferioris 

D. superior cerebellar peduncle

E. inferior cerebellar peduncle

https://ets.amu.edu.az/mod/quiz/review.php?attempt=68865&cmid=7722#question-98823-15 2/7
2021/4/20 Midterm exam: İnsan anatomiyası və tibbi terminologiya - 2 (EN) (MPF2): Attempt review

Question 6
Incorrect

Mark 0.00 out of 1.00

In the superior triangle region of the rhomboid fossa is situated the nuclei of the

A. VIII,VII,III,VI cranial nerves 

B. V,II,III,VI cranial nerves

C. X,XI,XII cranial nerves

D. V, VI, VII , VIII cranial nerves

E. I,II.V,XII cranial nerves

Question 7
Correct

Mark 1.00 out of 1.00

The oculomotor nerve emerges from cranial cavity through:

A. foramen ethmoidale anterior

B. canalis opticus

C. fissura orbitalis inferior

D. foramen rotundum

E. fissura orbitalis superior 

Question 8
Correct

Mark 1.00 out of 1.00

The glossopharyngeal nerve leaves the skull through:

A. canalis hypoglossus

B. foramen ovale

C. foramen lacerum

D. foramen rotundum

E. foramen jugulare 

https://ets.amu.edu.az/mod/quiz/review.php?attempt=68865&cmid=7722#question-98823-15 3/7
2021/4/20 Midterm exam: İnsan anatomiyası və tibbi terminologiya - 2 (EN) (MPF2): Attempt review

Question 9
Correct

Mark 1.00 out of 1.00

Which muscles are innervated by the nervus accesorius?


I. M. sternocleidomastoideus
II. M. trapezius
III. M. digastricus
IV. M. latissimus dorsi
V. M. buccinator

A. III,V

B. I,II 

C. II,III

D. III,IV

E. IV,V

Question 10
Correct

Mark 1.00 out of 1.00

Which layers of the serous pericardium?

A. costal,parietal

B. mediastinal, costal

C. parietal, visceral 

D. parietal, costal

E. diaphragmaric,visceral

Question 11
Correct

Mark 1.00 out of 1.00

Where is located common carotid artery bifurcation?

A. superior border of the thyroid cartilage 

B. root of the tongue

C. inferior border of the thyroid cartilage

D. posterior border of the palate

E. V cervical vertebra

https://ets.amu.edu.az/mod/quiz/review.php?attempt=68865&cmid=7722#question-98823-15 4/7
2021/4/20 Midterm exam: İnsan anatomiyası və tibbi terminologiya - 2 (EN) (MPF2): Attempt review

Question 12
Correct

Mark 1.00 out of 1.00

The vertebral artery (a. vertebralis), is the first branch of the

A. suprascapular artery

B. superficial cervical artery

C. subclavian artery 

D. inferior thyroid artery

E. subclavian artery

Question 13
Correct

Mark 1.00 out of 1.00

It is not the branch of the ulnar artery:

A. а. carpalis dorsalis

B. а. carpalis palmaris

C. а. interossea communis

D. а. recurrens radialis 

E. а. recurrens ulnaris

Question 14
Correct

Mark 1.00 out of 1.00

The vein which is composed by the connecting occipital and posterior auricular veins:

A. internal jugular vein

B. superior venаcava

C. anterior jugular vein

D. brachiocephalic vein

E. external jugular vein 

https://ets.amu.edu.az/mod/quiz/review.php?attempt=68865&cmid=7722#question-98823-15 5/7
2021/4/20 Midterm exam: İnsan anatomiyası və tibbi terminologiya - 2 (EN) (MPF2): Attempt review

Question 15
Incorrect

Mark 0.00 out of 1.00

The visceral branches of the thoracic aorta are the followings:


I. Rr. bronchiales
II. Rr. mediastinales
III. Superior phrenic arteries
IV. Posterior intercostalis arteries
V. Rr. pericardiaci

A. II,III,V

B. I,II,V

C. I,II,IV

D. III,IV,V

E. II,III,IV 

Question 16
Correct

Mark 1.00 out of 1.00

Which muscles are innervated by n. dorsalis scapulae?

1. m. pectoralis major

2. m. levator scapulae 

3. m. latissimus dorsi

4. m. coracobrachialis

5. mm. rhomboidei 

Question 17
Correct

Mark 1.00 out of 1.00

To the branches of the brachial artery belong:

1. a. thoracodorsalis

2. a. collateralis ulnaris superior 

3. a. subscapularis

4. a. thoracoacromialis

5. a. collateralis ulnaris inferior 

6. a. profunda brachii 

https://ets.amu.edu.az/mod/quiz/review.php?attempt=68865&cmid=7722#question-98823-15 6/7
2021/4/20 Midterm exam: İnsan anatomiyası və tibbi terminologiya - 2 (EN) (MPF2): Attempt review

Question 18
Correct

Mark 1.00 out of 1.00

Find the accordance:

The external ear includes auricle and external acoustic meatus 

The inner ear consists of bony labyrinth and membranous labyrinth 

Question 19
Correct

Mark 1.00 out of 1.00

The external capsule is a layer of [[1]] matter that is situated between the claustrum and [[2]] nucleus.

[[1]] white 

[[2]] lentiform 

Question 20
Correct

Mark 1.00 out of 1.00

Find the ulnar nerve:

Answer: 7 

◄ Attendance

Keç ...

A. Abdullayev. 7. General overview of the peripheral nervous system. The classification, development, variation, and anomalies of the
cranial nerves. ►

https://ets.amu.edu.az/mod/quiz/review.php?attempt=68865&cmid=7722#question-98823-15 7/7
4/20/2021 Midterm exam: İnsan anatomiyası və tibbi terminologiya - 2 (EN) (MPF2): Attempt review

Dashboard / My courses / İnsan anatomiyası və tibbi terminologiya - 2 (EN) / General

/ Midterm exam: İnsan anatomiyası və tibbi terminologiya - 2 (EN) (MPF2)

Started on Tuesday, 20 April 2021, 12:31 PM


State Finished
Completed on Tuesday, 20 April 2021, 12:59 PM
Time taken 28 mins 42 secs
Grade 17 out of 20 (83%)

Question 1
Incorrect

Mark 0.00 out of 1.00

On the midline of the anterior surface of the spinal cord is:

A. dorsal (posterior) median sulcus. 

B. ventrolateral grooves

C. dorsolateral grooves

D. fissura orbitalis superior

E. ventral median fissure

Question 2
Correct

Mark 1.00 out of 1.00

Temporal lobe is separated from the frontal and parietal lobes by the:

A. transversus occipital sulcus

B. lateral sulcus 

C. parieto-occipital sulcus

D. central sulcus

E. postcentral sulcus

https://ets.amu.edu.az/mod/quiz/review.php?attempt=69157&cmid=7722#question-99121-5 1/8
4/20/2021 Midterm exam: İnsan anatomiyası və tibbi terminologiya - 2 (EN) (MPF2): Attempt review

Question 3
Correct

Mark 1.00 out of 1.00

The extremal capsule is the white matter separating:

A. claustrum from cortex of insula 

B. globus pallidus lateralis and putamen

C. globi pallidus medialis et lateralis

D. globus pallidus medialis and nucleus caudatus

E. putamen and claustrum

Question 4
Correct

Mark 1.00 out of 1.00

Midbrain in Latin is called:

A. telencephalon

B. diencephalon

C. myelencephalon

D. mesencephalon 

E. metencephalon

Question 5
Incorrect

Mark 0.00 out of 1.00

The trigonum lemnisci is bounded medially by:

A. inferior cerebellar peduncle

B. brachium colliculi superioris

C. brachium colliculi inferioris 

D. superior cerebellar peduncle

E. cerebral peduncle

https://ets.amu.edu.az/mod/quiz/review.php?attempt=69157&cmid=7722#question-99121-5 2/8
4/20/2021 Midterm exam: İnsan anatomiyası və tibbi terminologiya - 2 (EN) (MPF2): Attempt review

Question 6
Correct

Mark 1.00 out of 1.00

The olive separated from the pyramid by the:

A. anterolateral sulcus 

B. posterior median sulcus

C. intermediate sulcus

D. posterior funiculus

E. posterolateral sulcus

Question 7
Correct

Mark 1.00 out of 1.00

Which cranial nerve receives sound wave stimuli from the small bones of the middle ear, inner ear and interpret this in the brain as hearing
and also control human balance?

A. vestibulocochlear nerve 

B. facial nerve

C. glossopharyngeal nerve

D. intermedius nerve

E. olfactory nerve

Question 8
Correct

Mark 1.00 out of 1.00

It arises from the facial nerve just above the stylomastoid foramen:

A. branch to the tympanic plexus

B. lesser petrosal nerve

C. the nerve to the stapedius

D. greater petrosal nerve

E. chorda tympani 

https://ets.amu.edu.az/mod/quiz/review.php?attempt=69157&cmid=7722#question-99121-5 3/8
4/20/2021 Midterm exam: İnsan anatomiyası və tibbi terminologiya - 2 (EN) (MPF2): Attempt review

Question 9
Correct

Mark 1.00 out of 1.00

The vagus nerve passes out of the skull through:

A. foramen juguluare 

B. foramen lacerum

C. foramen rotundum

D. foramen ovale

E. foramen spinosum

Question 10
Correct

Mark 1.00 out of 1.00

Which layers of the serous pericardium?

A. parietal, costal

B. mediastinal, costal

C. diaphragmaric,visceral

D. costal,parietal

E. parietal, visceral 

Question 11
Correct

Mark 1.00 out of 1.00

Where is located common carotid artery bifurcation?

A. superior border of the thyroid cartilage 

B. V cervical vertebra

C. inferior border of the thyroid cartilage

D. posterior border of the palate

E. root of the tongue

https://ets.amu.edu.az/mod/quiz/review.php?attempt=69157&cmid=7722#question-99121-5 4/8
4/20/2021 Midterm exam: İnsan anatomiyası və tibbi terminologiya - 2 (EN) (MPF2): Attempt review

Question 12
Correct

Mark 1.00 out of 1.00

The vertebral artery (a. vertebralis), is the first branch of the

A. subclavian artery

B. superficial cervical artery

C. inferior thyroid artery

D. suprascapular artery

E. subclavian artery 

Question 13
Correct

Mark 1.00 out of 1.00

To which vein does v.cephalica vein open?

A. anterior jugular

B. axillary 

C. external jugular

D. subclavian

E. brachial

Question 14
Correct

Mark 1.00 out of 1.00

The internal jugular vein is a continuation of:

A. inferior petrosal sinus

B. sigmoid sinus 

C. transverse sinus

D. superior petrosal sinus

E. superior sagittal sinus

https://ets.amu.edu.az/mod/quiz/review.php?attempt=69157&cmid=7722#question-99121-5 5/8
4/20/2021 Midterm exam: İnsan anatomiyası və tibbi terminologiya - 2 (EN) (MPF2): Attempt review

Question 15
Correct

Mark 1.00 out of 1.00

The brachial plexus is formed by the

A. two trunks

B. six trunks

C. three trunks 

D. five trunks

E. four trunks

Question 16
Partially correct

Mark 0.67 out of 1.00

The anterior wall of the third ventricle is formed by:

1. epithalamic commissure

2. anterior commissure

3. terminal lamina 

4. thalamus

5. pineal body

6. columns of the fornix 

Question 17
Incorrect

Mark 0.00 out of 1.00

To the nuclei of the medulla oblongata belong:

1. nucleus olivaris 

2. nucleus gracilis

3. nucleus caudatus

4. nucleus ruber 

5. nucleus cuneatus

6. nucleus dentatus

https://ets.amu.edu.az/mod/quiz/review.php?attempt=69157&cmid=7722#question-99121-5 6/8
4/20/2021 Midterm exam: İnsan anatomiyası və tibbi terminologiya - 2 (EN) (MPF2): Attempt review

Question 18
Correct

Mark 1.00 out of 1.00

Find the accordance:

The superior laryngeal nerve is the branch of vagus nerve 

The tympanic nerve is the branch of glossopharyngeal nerve 

Question 19
Correct

Mark 1.00 out of 1.00

At the level of the lateral edge of the [[1]] rib, the axillary vein continues into the [[2]] vein.

[[1]] first 

[[2]] subclavian 

Question 20
Correct

Mark 1.00 out of 1.00

Find n. vagus:

Answer: 11 

◄ Attendance

Jump to...

A. Abdullayev. 7. General overview of the peripheral nervous system. The classification, development, variation, and anomalies of the
cranial nerves. ►

https://ets.amu.edu.az/mod/quiz/review.php?attempt=69157&cmid=7722#question-99121-5 7/8
4/20/2021 Midterm exam: İnsan anatomiyası və tibbi terminologiya - 2 (EN) (MPF2): Attempt review

https://ets.amu.edu.az/mod/quiz/review.php?attempt=69157&cmid=7722#question-99121-5 8/8
4/20/2021 Midterm exam: İnsan anatomiyası və tibbi terminologiya - 2 (EN) (MPF2): Attempt review

Mənim əsas səhifəm / Mənim fənlərim / İnsan anatomiyası və tibbi terminologiya - 2 (EN) / Ümumi

/ Midterm exam: İnsan anatomiyası və tibbi terminologiya - 2 (EN) (MPF2)

Testə başlandı Tuesday, 20 April 2021, 12:30 PM


State Finished
Sona çatdı Tuesday, 20 April 2021, 12:56 PM
Vaxt keçib 26 dəq. 15 san.
Qiymət 20 (98%) maksimumdan 20

Question 1
Correct

Mark 1.00 out of 1.00

Vermis is belonged to the:

A. pons

B. midbrain

C. cerebellum 

D. medulla oblongata

E. caudate nucleus

Question 2
Correct

Mark 1.00 out of 1.00

They are situated on the inferior surface of the temporal lobe:


I. Sulcus occipitotemporalis
II. Gyrus temporalis medius
III. Gyri temporales transversi
IV. Sulcus collateralis

A. I; II

B. III; IV

C. II; III

D. II; IV

E. I; IV 

https://ets.amu.edu.az/mod/quiz/review.php?attempt=68726&cmid=7722#question-98698-17 1/7
4/20/2021 Midterm exam: İnsan anatomiyası və tibbi terminologiya - 2 (EN) (MPF2): Attempt review

Question 3
Correct

Mark 1.00 out of 1.00

The extremal capsule is the white matter separating:

A. claustrum from cortex of insula 

B. globi pallidus medialis et lateralis

C. putamen and claustrum

D. globus pallidus lateralis and putamen

E. globus pallidus medialis and nucleus caudatus

Question 4
Correct

Mark 1.00 out of 1.00

Where is the gyrus parahippocampalis situated?

A. in the posterior part of the gyrus temporalis inferior

B. between the hippocampal and collateral sulcus 

C. between the superior and inferior temporal sulcus

D. between the inferior temporal and occipitotemporalis sulcus

E. between the lateral and superior temporal sulcus

Question 5
Correct

Mark 1.00 out of 1.00

Where is the vallecula cerebelli located?

A. on the posterior border of cerebellum

B. on the superior surface of cerebellum

C. on the anterior border of cerebellum

D. on the inferior surface of cerebellum 

E. on the anterior and posterior borders of cerebellum

https://ets.amu.edu.az/mod/quiz/review.php?attempt=68726&cmid=7722#question-98698-17 2/7
4/20/2021 Midterm exam: İnsan anatomiyası və tibbi terminologiya - 2 (EN) (MPF2): Attempt review

Question 6
Correct

Mark 1.00 out of 1.00

What extends from the superior to the inferior angle of the rhomboid fossa?

A. sulcus medianus 

B. sulcus limitans

C. colliculus facialis

D. eminentia medialis

E. striae medullares

Question 7
Correct

Mark 1.00 out of 1.00

Where does the optic nerve enter the cranial cavity from?

A. foramen zygomatico-orbitale

B. foramen ovale

C. foramen ethmoidae anterior

D. foramen ethmoidale posterior

E. canalis opticus 

Question 8
Correct

Mark 1.00 out of 1.00

Which cranial nerve provides the human with the ability to smell a wide variety of odors?

A. optic nerve

B. olfactory nerve 

C. intermedius nerve

D. oculomotor nerve

E. vestibulocochlear nerve

https://ets.amu.edu.az/mod/quiz/review.php?attempt=68726&cmid=7722#question-98698-17 3/7
4/20/2021 Midterm exam: İnsan anatomiyası və tibbi terminologiya - 2 (EN) (MPF2): Attempt review

Question 9
Correct

Mark 1.00 out of 1.00

It does not belong to the branches of the cervical part of the vagus nerve:

A. rr. cardiaci cervicales superiores

B. pharyngeal branches

C. recurrent laryngeal nerve

D. auricular branch 

E. superior laryngeal nerve

Question 10
Correct

Mark 1.00 out of 1.00

How is the called right atrio-ventricular valve?

A. tricuspidalis 

B. atrioventricularis sinistra

C. mitralis

D. bicuspidalis

E. seminularis

Question 11
Correct

Mark 1.00 out of 1.00

The ascending aorta passes to the aortic arch at the level of the:

A. right third sternocostal joint

B. left fourth sternocostal joint

C. left third sternocostal joint

D. right second sternocostal joint 

E. left second sternocostal joint

https://ets.amu.edu.az/mod/quiz/review.php?attempt=68726&cmid=7722#question-98698-17 4/7
4/20/2021 Midterm exam: İnsan anatomiyası və tibbi terminologiya - 2 (EN) (MPF2): Attempt review

Question 12
Correct

Mark 1.00 out of 1.00

It doesn’t belong to the branches of thyrocervical trunk:

A. subscapular artery 

B. inferior thyroid artery

C. suprascapular artery

D. ascending cervical artery

E. superficial cervical artery

Question 13
Correct

Mark 1.00 out of 1.00

To which vein does v.basilica open?

A. subclavian

B. axillary

C. brachial 

D. anterior jugular

E. external jugular

Question 14
Correct

Mark 1.00 out of 1.00

It does not belong to the tributaries of the external jugular vein:

A. v. suprascapularis

B. v. canalis pterygoidei 

C. v. auricularis posterior

D. v. occipitalis

E. v. jugularis anterior

https://ets.amu.edu.az/mod/quiz/review.php?attempt=68726&cmid=7722#question-98698-17 5/7
4/20/2021 Midterm exam: İnsan anatomiyası və tibbi terminologiya - 2 (EN) (MPF2): Attempt review

Question 15
Correct

Mark 1.00 out of 1.00

The visceral branches of the thoracic aorta are the followings:


I. Rr. bronchiales
II. Rr. mediastinales
III. Superior phrenic arteries
IV. Posterior intercostalis arteries
V. Rr. pericardiaci

A. II,III,IV

B. I,II,IV

C. III,IV,V

D. I,II,V 

E. II,III,V

Question 16
Correct

Mark 1.00 out of 1.00

The anterior wall of the third ventricle is formed by:

1. terminal lamina 

2. anterior commissure 

3. columns of the fornix 

4. pineal body

5. thalamus

6. epithalamic commissure

Question 17
Partially correct

Mark 0.67 out of 1.00

To the nuclei of the medulla oblongata belong:

1. nucleus caudatus

2. nucleus olivaris 

3. nucleus dentatus

4. nucleus ruber

5. nucleus gracilis 

6. nucleus cuneatus

https://ets.amu.edu.az/mod/quiz/review.php?attempt=68726&cmid=7722#question-98698-17 6/7
4/20/2021 Midterm exam: İnsan anatomiyası və tibbi terminologiya - 2 (EN) (MPF2): Attempt review

Question 18
Correct

Mark 1.00 out of 1.00

Find the accordance:

A. interossea communis is the branch of ulnar artery 

A. thoracoacromialis is the branch of axillary artery 

Question 19
Correct

Mark 1.00 out of 1.00

At the level of the lateral edge of the [[1]] rib, the axillary vein continues into the [[2]] vein.

[[1]] first 

[[2]] subclavian 

Question 20
Correct

Mark 1.00 out of 1.00

Find the ulnar nerve:

Answer: 7 

◄ Attendance

Keç ...

A. Abdullayev. 7. General overview of the peripheral nervous system. The classification, development, variation, and anomalies of the cranial
nerves. ►

https://ets.amu.edu.az/mod/quiz/review.php?attempt=68726&cmid=7722#question-98698-17 7/7
2021/4/20 Midterm exam: İnsan anatomiyası və tibbi terminologiya - 2 (EN) (MPF2): Attempt review

Dashboard / My courses / İnsan anatomiyası və tibbi terminologiya - 2 (EN) / General

/ Midterm exam: İnsan anatomiyası və tibbi terminologiya - 2 (EN) (MPF2)

Started on Tuesday, 20 April 2021, 12:30 PM


State Finished
Completed on Tuesday, 20 April 2021, 12:46 PM
Time taken 16 mins 28 secs
Grade 20 out of 20 (100%)

Question 1
Correct

Mark 1.00 out of 1.00

Between the anterior part of the pons and the middle cerebellar peduncle emerges

A. the trigeminal nerve. 

B. the facial nerve

C. the abducent nerve

D. the oculomotor nerve

E. the trochlear nerve

Question 2
Correct

Mark 1.00 out of 1.00

Temporal lobe is separated from the frontal and parietal lobes by the:

A. transversus occipital sulcus

B. parieto-occipital sulcus

C. lateral sulcus 

D. central sulcus

E. postcentral sulcus

Question 3
Correct

Mark 1.00 out of 1.00

It does not belong to lateral ventricle:

A. posterior horn

B. anterior horn

C. central part

D. inferior horn

E. anterior commissure 

https://ets.amu.edu.az/mod/quiz/review.php?attempt=68717&cmid=7722 1/7
2021/4/20 Midterm exam: İnsan anatomiyası və tibbi terminologiya - 2 (EN) (MPF2): Attempt review

Question 4
Correct

Mark 1.00 out of 1.00

Where is the gyrus parahippocampalis situated?

A. in the posterior part of the gyrus temporalis inferior

B. between the hippocampal and collateral sulcus 

C. between the inferior temporal and occipitotemporalis sulcus

D. between the superior and inferior temporal sulcus

E. between the lateral and superior temporal sulcus

Question 5
Correct

Mark 1.00 out of 1.00

The dorsal part of metencephalon is:

A. pedunculus cerebri

B. cerebellum 

C. tuber cinereum

D. corpus mamillaris

E. pons

Question 6
Correct

Mark 1.00 out of 1.00

The lower part of the rhomboid fossa is corresponded to:

A. mesencephalon

B. medulla oblongata 

C. diencephalon

D. metencephalon

E. telencephalon

https://ets.amu.edu.az/mod/quiz/review.php?attempt=68717&cmid=7722 2/7
2021/4/20 Midterm exam: İnsan anatomiyası və tibbi terminologiya - 2 (EN) (MPF2): Attempt review

Question 7
Correct

Mark 1.00 out of 1.00

Which of the following are the parts of the tympanic membrane?


I. Pars mucosa
II. Pars muscularis
III. Pars tensa
IV. Pars flaccida
V. Pars membranacea

A. I, II

B. II, IV

C. IV, V

D. III, IV 

E. III, V

Question 8
Correct

Mark 1.00 out of 1.00

Which cranial nerve provides the human with the ability to smell a wide variety of odors?

A. vestibulocochlear nerve

B. intermedius nerve

C. olfactory nerve 

D. oculomotor nerve

E. optic nerve

Question 9
Correct

Mark 1.00 out of 1.00

Which muscles are innervated by the nervus accesorius?


I. M. sternocleidomastoideus
II. M. trapezius
III. M. digastricus
IV. M. latissimus dorsi
V. M. buccinator

A. II,III

B. IV,V

C. III,IV

D. III,V

E. I,II 

https://ets.amu.edu.az/mod/quiz/review.php?attempt=68717&cmid=7722 3/7
2021/4/20 Midterm exam: İnsan anatomiyası və tibbi terminologiya - 2 (EN) (MPF2): Attempt review

Question 10
Correct

Mark 1.00 out of 1.00

The surfaces of the heart:


I. Oesophagalis
II. Sternocostalis
III. Mediastinalis
IV. Diaphracmatica
V. Trachealis

A. II;III

B. II;IV 

C. I;II

D. IV;V

E. III;IV

Question 11
Correct

Mark 1.00 out of 1.00

Where is located common carotid artery bifurcation?

A. root of the tongue

B. V cervical vertebra

C. posterior border of the palate

D. inferior border of the thyroid cartilage

E. superior border of the thyroid cartilage 

Question 12
Correct

Mark 1.00 out of 1.00

It does not belong the branches of the internal carotid artery:

A. a. cerebri anterior

B. a. cerebri media

C. a. communicans posterior

D. a. ophthalmica

E. a. cerebri posterior 

https://ets.amu.edu.az/mod/quiz/review.php?attempt=68717&cmid=7722 4/7
2021/4/20 Midterm exam: İnsan anatomiyası və tibbi terminologiya - 2 (EN) (MPF2): Attempt review

Question 13
Correct

Mark 1.00 out of 1.00

To which vein does v.cephalica vein open?

A. external jugular

B. anterior jugular

C. brachial

D. subclavian

E. axillary 

Question 14
Correct

Mark 1.00 out of 1.00

Where is located the inferior sagittal sinus?

A. in the connecting place of the falx cerebri with the tentorium cerebelli

B. in the confluence cinuum

C. between the foramen caecum and internal occipital protuberance

D. in the inferior concave margin of the falx cerebri 

E. in the transverse sinus groove of the occipital bone

Question 15
Correct

Mark 1.00 out of 1.00

This doesn’t belong to the cutaneous branches of the cervical plexus:

A. n. occipitalis minor

B. n. supraclavicularis

C. n. subclavius 

D. n. transversus colli

E. n. auricularis magnus

https://ets.amu.edu.az/mod/quiz/review.php?attempt=68717&cmid=7722 5/7
2021/4/20 Midterm exam: İnsan anatomiyası və tibbi terminologiya - 2 (EN) (MPF2): Attempt review

Question 16
Correct

Mark 1.00 out of 1.00

The anterior wall of the third ventricle is formed by:

1. anterior commissure 

2. thalamus

3. epithalamic commissure

4. pineal body

5. columns of the fornix 

6. terminal lamina 

Question 17
Correct

Mark 1.00 out of 1.00

Which muscles are innervated by n. dorsalis scapulae?

1. m. latissimus dorsi

2. m. pectoralis major

3. m. levator scapulae 

4. mm. rhomboidei 

5. m. coracobrachialis

Question 18
Correct

Mark 1.00 out of 1.00

Find the accordance:

The inner ear consists of bony labyrinth and membranous labyrinth 

The external ear includes auricle and external acoustic meatus 

Question 19
Correct

Mark 1.00 out of 1.00

The external capsule is a layer of [[1]] matter that is situated between the claustrum and [[2]] nucleus.

[[1]] white 

[[2]] lentiform 

https://ets.amu.edu.az/mod/quiz/review.php?attempt=68717&cmid=7722 6/7
2021/4/20 Midterm exam: İnsan anatomiyası və tibbi terminologiya - 2 (EN) (MPF2): Attempt review

Question 20
Correct

Mark 1.00 out of 1.00

Find the pons:

Answer: 11 

◄ Attendance

Jump to...

A. Abdullayev. 7. General overview of the peripheral nervous system. The classification, development, variation, and anomalies of the cranial
nerves. ►

https://ets.amu.edu.az/mod/quiz/review.php?attempt=68717&cmid=7722 7/7
4/20/2021 Midterm exam: İnsan anatomiyası və tibbi terminologiya - 2 (EN) (MPF2): Attempt review

Mənim əsas səhifəm / Mənim fənlərim / İnsan anatomiyası və tibbi terminologiya - 2 (EN) / Ümumi

/ Midterm exam: İnsan anatomiyası və tibbi terminologiya - 2 (EN) (MPF2)

Testə başlandı Tuesday, 20 April 2021, 12:30 PM


State Finished
Sona çatdı Tuesday, 20 April 2021, 12:57 PM
Vaxt keçib 27 dəq. 46 san.
Qiymət 20 (80%) maksimumdan 16

Question 1
Correct

Mark 1.00 out of 1.00

How many lumbar segments does the spinal cord have?

A. 5 

B. 12

C. 8

D. 6

E. 4

Question 2
Incorrect

Mark 0.00 out of 1.00

It happens when the motor analyzer of speech articulation is damaged:

A. apraxia 

B. agraphia

C. tactile agnosia

D. sensory aphasia

E. motor aphasia

https://ets.amu.edu.az/mod/quiz/review.php?attempt=68702&cmid=7722#question-98660-17 1/8
4/20/2021 Midterm exam: İnsan anatomiyası və tibbi terminologiya - 2 (EN) (MPF2): Attempt review

Question 3
Incorrect

Mark 0.00 out of 1.00

The floor of the central part of the lateral ventricle is formed by:
I. Body of caudate nucleus
II. Head of caudate nucleus
III. Thalamus
IV. Corpus fornicis
V. Stria terminalis

A. I; III; V

B. I; II; III 

C. I; III; IV

D. III; IV; V

E. II; III; V

Question 4
Correct

Mark 1.00 out of 1.00

Where is the gyrus parahippocampalis situated?

A. in the posterior part of the gyrus temporalis inferior

B. between the hippocampal and collateral sulcus 

C. between the inferior temporal and occipitotemporalis sulcus

D. between the lateral and superior temporal sulcus

E. between the superior and inferior temporal sulcus

Question 5
Correct

Mark 1.00 out of 1.00

It doesn’t belong to the isthmus rhombencephali:

A. pedunculi cerebellaris superiores and trigonum lemnisci

B. velum medullare inferius 

C. velum medullare superius

D. trigonum lemnisci

E. pedunculi cerebellaris superiores

https://ets.amu.edu.az/mod/quiz/review.php?attempt=68702&cmid=7722#question-98660-17 2/8
4/20/2021 Midterm exam: İnsan anatomiyası və tibbi terminologiya - 2 (EN) (MPF2): Attempt review

Question 6
Correct

Mark 1.00 out of 1.00

In the superior triangle region of the rhomboid fossa is situated the nuclei of the

A. I,II.V,XII cranial nerves

B. V, VI, VII , VIII cranial nerves 

C. V,II,III,VI cranial nerves

D. VIII,VII,III,VI cranial nerves

E. X,XI,XII cranial nerves

Question 7
Correct

Mark 1.00 out of 1.00

Which of the following are the parts of the tympanic membrane?


I. Pars mucosa
II. Pars muscularis
III. Pars tensa
IV. Pars flaccida
V. Pars membranacea

A. IV, V

B. III, V

C. II, IV

D. I, II

E. III, IV 

Question 8
Correct

Mark 1.00 out of 1.00

The glossopharyngeal nerve descends through the upper part of the neck between the:

A. internal carotid artery and external carotid artery

B. internal jugular vein and external carotid artery

C. internal jugular vein and common carotid artery

D. internal jugular vein and external jugular vein

E. internal jugular vein and internal carotid artery 

https://ets.amu.edu.az/mod/quiz/review.php?attempt=68702&cmid=7722#question-98660-17 3/8
4/20/2021 Midterm exam: İnsan anatomiyası və tibbi terminologiya - 2 (EN) (MPF2): Attempt review

Question 9
Correct

Mark 1.00 out of 1.00

The anterior ethmoid nerve enters the cranial cavity through:

A. optic canal

B. anterior ethmoid foramen 

C. olfactory foramina

D. medial side of the fissure orbitalis superior

E. lateral side of the fissure orbitalis inferior

Question 10
Correct

Mark 1.00 out of 1.00

Where is located the superior border of the heart?

A. line connecting upper edges of VII ribs

B. line connecting upper edges the IV ribs

C. line connecting lower edges of the V ribs

D. line connecting lower edges of VI ribs

E. line connecting upper edges of III ribs 

Question 11
Correct

Mark 1.00 out of 1.00

At which level does the external carotid artery divide into two terminating branches?

A. coronoid process of mandible

B. angle of mandible

C. thyroid cartilage

D. neck of mandible 

E. hyoid bone

https://ets.amu.edu.az/mod/quiz/review.php?attempt=68702&cmid=7722#question-98660-17 4/8
4/20/2021 Midterm exam: İnsan anatomiyası və tibbi terminologiya - 2 (EN) (MPF2): Attempt review

Question 12
Incorrect

Mark 0.00 out of 1.00

It doesn’t belong to the branches of thyrocervical trunk:

A. ascending cervical artery

B. subscapular artery

C. superficial cervical artery

D. suprascapular artery 

E. inferior thyroid artery

Question 13
Correct

Mark 1.00 out of 1.00

It is not the branch of the ulnar artery:

A. а. carpalis palmaris

B. а. recurrens ulnaris

C. а. carpalis dorsalis

D. а. interossea communis

E. а. recurrens radialis 

Question 14
Correct

Mark 1.00 out of 1.00

Where is located the inferior sagittal sinus?

A. between the foramen caecum and internal occipital protuberance

B. in the inferior concave margin of the falx cerebri 

C. in the transverse sinus groove of the occipital bone

D. in the connecting place of the falx cerebri with the tentorium cerebelli

E. in the confluence cinuum

https://ets.amu.edu.az/mod/quiz/review.php?attempt=68702&cmid=7722#question-98660-17 5/8
4/20/2021 Midterm exam: İnsan anatomiyası və tibbi terminologiya - 2 (EN) (MPF2): Attempt review

Question 15
Correct

Mark 1.00 out of 1.00

What is called the continuation of the right ascending lumbar vein?

A. accessory hemiazygos vein

B. azygos vein 

C. pericardiaco-phrenic veins

D. internal thoracic vein

E. hemiazygos vein

Question 16
Correct

Mark 1.00 out of 1.00

To the branches of the ophthalmic nerve belong:

1. n. auriculotemporalis

2. n. nasociliaris 

3. n. frontalis 

4. n. petrosus major

5. n. lacrimalis 

6. n. infraorbitalis

Question 17
Incorrect

Mark 0.00 out of 1.00

The anterior wall of the third ventricle is formed by:

1. columns of the fornix

2. epithalamic commissure

3. pineal body

4. thalamus 

5. anterior commissure 

6. terminal lamina 

https://ets.amu.edu.az/mod/quiz/review.php?attempt=68702&cmid=7722#question-98660-17 6/8
4/20/2021 Midterm exam: İnsan anatomiyası və tibbi terminologiya - 2 (EN) (MPF2): Attempt review

Question 18
Correct

Mark 1.00 out of 1.00

Find the accordance:

The superior laryngeal nerve is the branch of vagus nerve 

The tympanic nerve is the branch of glossopharyngeal nerve 

Question 19
Correct

Mark 1.00 out of 1.00

The ulnar artery departs from the [[1]] artery at the level of the [[2]] of the ulna.

[[1]] brachial 

[[2]] coronoid process 

Question 20
Correct

Mark 1.00 out of 1.00

Find the colliculus facialis:

Answer: 6 

◄ Attendance

Keç ...

A. Abdullayev. 7. General overview of the peripheral nervous system. The classification, development, variation, and anomalies of the
cranial nerves. ►

https://ets.amu.edu.az/mod/quiz/review.php?attempt=68702&cmid=7722#question-98660-17 7/8
4/20/2021 Midterm exam: İnsan anatomiyası və tibbi terminologiya - 2 (EN) (MPF2): Attempt review

https://ets.amu.edu.az/mod/quiz/review.php?attempt=68702&cmid=7722#question-98660-17 8/8
Mǝnim ǝsas sǝhifǝm / Mǝnim fǝnlǝrim / İnsan anatomiyası vǝ tibbi terminologiya - 2 (EN) / Ümumi
/ Midterm exam: İnsan anatomiyası vǝ tibbi terminologiya - 2 (EN) (MPF2)

Testǝ başlandı Tuesday, 20 April 2021, 12N30 PM


State Finished
Sona çatdı Tuesday, 20 April 2021, 12N57 PM
Vaxt keçib 26 dǝq. 4 san.
Qiymǝt 20 (95%) maksimumdan 19

Question 1 In the medial border of the cerebral peduncle emerges


Correct

Mark 1.00 out A. the trochlear nerve


of 1.00
B. the abducent nerve

C. tthe facial nerve

D. the oculomotor nerve !

E. the trigeminal nerve

Question 2 It happens when the motor analyzer of speech articulation is damaged:


Correct

Mark 1.00 out A. agraphia


of 1.00
B. tactile agnosia

C. sensory aphasia

D. motor aphasia !

E. apraxia

Question 3 The extremal capsule is the white matter separating:


Correct

Mark 1.00 out A. globus pallidus medialis and nucleus caudatus


of 1.00
B. claustrum from cortex of insula !

C. globi pallidus medialis et lateralis

D. globus pallidus lateralis and putamen

E. putamen and claustrum

Question 4 Around the cerebral aqueduct is placed:


Correct

Mark 1.00 out A. substantia grisea centralis !


of 1.00
B. substantia perforata anterior

C. substantia perforata posterior et substantia grisea centralis

D. substantia perforata posterior

E. substantia perforata anterior et substantia grisea centralis


Question 5 The roof of the fourth ventricle in front consists of:
Correct I. Superior cerebellar peduncles
Mark 1.00 out II. Middle cerebellar peduncles
of 1.00 III. Inferior cerebellar peduncles
IV. Velum medullare superius
V. Velum medullare inferius

A. III; V

B. I; II

C. III; IV

D. I; IV !

E. II; III

Question 6 The olive separated from the pyramid by the:


Correct

Mark 1.00 out A. intermediate sulcus


of 1.00
B. anterolateral sulcus !

C. posterior funiculus

D. posterior median sulcus

E. posterolateral sulcus

Question 7 Which cranial nerve receives sound wave stimuli from the small bones of the middle ear, inner ear and
Correct interpret this in the brain as hearing and also control human balance?
Mark 1.00 out
of 1.00 A. olfactory nerve

B. glossopharyngeal nerve

C. intermedius nerve

D. vestibulocochlear nerve !

E. facial nerve

Question 8 It arises from the facial nerve just above the stylomastoid foramen:
Correct

Mark 1.00 out A. the nerve to the stapedius


of 1.00
B. branch to the tympanic plexus

C. greater petrosal nerve

D. lesser petrosal nerve

E. chorda tympani !

Question 9 The vagus nerve passes out of the skull through:


Correct

Mark 1.00 out A. foramen lacerum


of 1.00
B. foramen ovale

C. foramen juguluare !

D. foramen spinosum
D. foramen spinosum

E. foramen rotundum

Question 10 Which layers of the serous pericardium?


Correct

Mark 1.00 out A. mediastinal, costal


of 1.00
B. costal,parietal

C. parietal, costal

D. diaphragmaric,visceral

E. parietal, visceral !

Question 11 Thoracic aorta locates between the:


Correct

Mark 1.00 out A. ThI-ThXII


of 1.00
B. ThIII-ThVIII

C. ThIV-ThX

D. ThVI-ThXII

E. ThIV-ThXII !

Question 12 The branches of the brachiocephalic trunk:


Correct I. A. subclavia dextra
Mark 1.00 out II. A. carotis communis sinistra
of 1.00 III. A. carotis communis dextra
IV. A. subclavia sinistra
V. A. thyroidea ima

A. I;III !

B. III;IV

C. IV;V

D. II;III

E. I;II

Question 13 To which vein does v.basilica open?


Correct

Mark 1.00 out A. axillary


of 1.00
B. external jugular

C. brachial !

D. anterior jugular

E. subclavian

Question 14 It does not belong to the tributaries of the external jugular vein:
Correct

Mark 1.00 out A. v. auricularis posterior


of 1.00
B. v. canalis pterygoidei !
C. v. occipitalis

D. v. suprascapularis

E. v. jugularis anterior

Question 15 What is called the continuation of the left ascending lumbar vein?
Correct

Mark 1.00 out A. internal thoracic vein


of 1.00
B. pericardiaco-phrenic veins

C. accessory hemiazygos vein

D. hemiazygos vein !

E. azygos vein

Question 16 The anterior wall of the third ventricle is formed by:


Correct

Mark 1.00 out 1. epithalamic commissure


of 1.00
2. terminal lamina !

3. anterior commissure !

4. columns of the fornix !

5. pineal body

6. thalamus

Question 17 To the nuclei of the medulla oblongata belong:


Correct

Mark 1.00 out 1. nucleus cuneatus !


of 1.00
2. nucleus ruber

3. nucleus gracilis !

4. nucleus caudatus

5. nucleus dentatus

6. nucleus olivaris !

Question 18 Find the accordance:


Correct

Mark 1.00 out


A. thoracoacromialis is the branch of axillary artery !
of 1.00

A. interossea communis is the branch of ulnar artery !

Question 19 Middle superior alveolar nerve passes in the lateral wall of the [[1]] and innervates the upper [[2]] teeth, gums
Correct and the cheek.
Mark 1.00 out
of 1.00
[[1]] maxillary sinus !

[[2]] premolar !
Question 20 Find the ulnar nerve:
Incorrect

Mark 0.00 out


of 1.00

Answer: 9 "

A. Abdullayev. 7. General overview


of the peripheral nervous system.
◀ Attendance Keç ... The classification, development,
variation, and anomalies of the
cranial nerves. ▶︎
4/20/2021 Midterm exam: İnsan anatomiyası və tibbi terminologiya - 2 (EN) (MPF2): Attempt review

Dashboard / My courses / İnsan anatomiyası və tibbi terminologiya - 2 (EN) / General

/ Midterm exam: İnsan anatomiyası və tibbi terminologiya - 2 (EN) (MPF2)

Started on Tuesday, 20 April 2021, 12:30 PM


State Finished
Completed on Tuesday, 20 April 2021, 12:54 PM
Time taken 23 mins 52 secs
Grade 19 out of 20 (95%)

Question 1
Correct

Mark 1.00 out of 1.00

In the posterior funiculus of the spinal cord are located:

A. tractus tectospinalis, tractus vestibulospinalis

B. tractus rubrospinalis, tractus spinocerebellaris dorsalis

C. fasciculus gracilis, tractus tectospinalis

D. tractus vestibulospinalis, tractus spinocerebellaris ventralis

E. fasciculus cuneatus, fasciculus gracilis 

Question 2
Correct

Mark 1.00 out of 1.00

It happens when the motor analyzer of speech articulation is damaged:

A. sensory aphasia

B. agraphia

C. motor aphasia 

D. tactile agnosia

E. apraxia

https://ets.amu.edu.az/mod/quiz/review.php?attempt=68873&cmid=7722 1/8
4/20/2021 Midterm exam: İnsan anatomiyası və tibbi terminologiya - 2 (EN) (MPF2): Attempt review

Question 3
Correct

Mark 1.00 out of 1.00

The extremal capsule is the white matter separating:

A. globus pallidus lateralis and putamen

B. putamen and claustrum

C. claustrum from cortex of insula 

D. globi pallidus medialis et lateralis

E. globus pallidus medialis and nucleus caudatus

Question 4
Correct

Mark 1.00 out of 1.00

It does not belong to epithalamus:

A. epiphysis

B. habenular trigone

C. hypophysis 

D. epithalamic commissure

E. habenular commissure

Question 5
Correct

Mark 1.00 out of 1.00

Where is the vallecula cerebelli located?

A. on the superior surface of cerebellum

B. on the anterior border of cerebellum

C. on the anterior and posterior borders of cerebellum

D. on the inferior surface of cerebellum 

E. on the posterior border of cerebellum

https://ets.amu.edu.az/mod/quiz/review.php?attempt=68873&cmid=7722 2/8
4/20/2021 Midterm exam: İnsan anatomiyası və tibbi terminologiya - 2 (EN) (MPF2): Attempt review

Question 6
Correct

Mark 1.00 out of 1.00

The lower part of the rhomboid fossa is corresponded to:

A. diencephalon

B. mesencephalon

C. medulla oblongata 

D. telencephalon

E. metencephalon

Question 7
Correct

Mark 1.00 out of 1.00

They are belonged to middle ear:


I. Auricula
II. Tuba auditiva
III. Meatus acusticus externus osseus
IV. Meatus acusticus externus cartilagineus
V. Cavitas tympani

A. II;III

B. IV;V

C. III;IV

D. II;V 

E. I;III

Question 8
Correct

Mark 1.00 out of 1.00

The glossopharyngeal nerve descends through the upper part of the neck between the:

A. internal jugular vein and common carotid artery

B. internal jugular vein and external carotid artery

C. internal jugular vein and external jugular vein

D. internal carotid artery and external carotid artery

E. internal jugular vein and internal carotid artery 

https://ets.amu.edu.az/mod/quiz/review.php?attempt=68873&cmid=7722 3/8
4/20/2021 Midterm exam: İnsan anatomiyası və tibbi terminologiya - 2 (EN) (MPF2): Attempt review

Question 9
Correct

Mark 1.00 out of 1.00

It does not belong to the branches of the ophthalmic nerve:


I. N. infaorbitalis
II. N. lacrimalis
III. Rr. pterygopalatini
IV. N. nasociliaris
V. N. frontalis
VI. N. zygomaticus

A. I; III; V

B. II; IV; V

C. I; III; VI 

D. II; III; IV

E. III; IV; VI

Question 10
Correct

Mark 1.00 out of 1.00

Where is located the superior border of the heart?

A. line connecting upper edges of III ribs 

B. line connecting lower edges of the V ribs

C. line connecting upper edges of VII ribs

D. line connecting upper edges the IV ribs

E. line connecting lower edges of VI ribs

Question 11
Correct

Mark 1.00 out of 1.00

Thoracic aorta locates between the:

A. ThI-ThXII

B. ThVI-ThXII

C. ThIV-ThX

D. ThIV-ThXII 

E. ThIII-ThVIII

https://ets.amu.edu.az/mod/quiz/review.php?attempt=68873&cmid=7722 4/8
4/20/2021 Midterm exam: İnsan anatomiyası və tibbi terminologiya - 2 (EN) (MPF2): Attempt review

Question 12
Incorrect

Mark 0.00 out of 1.00

The vertebral artery (a. vertebralis), is the first branch of the

A. subclavian artery

B. superficial cervical artery

C. inferior thyroid artery

D. suprascapular artery

E. subclavian artery 

Question 13
Correct

Mark 1.00 out of 1.00

To which vein does v.cephalica vein open?

A. axillary 

B. anterior jugular

C. brachial

D. external jugular

E. subclavian

Question 14
Correct

Mark 1.00 out of 1.00

At which level does superior vena cava form?

A. I sternocostal synchondrosis 

B. III sternocostal joint

C. IV sternocostal joint

D. sternoclavicular joint

E. II sternocostal joint

https://ets.amu.edu.az/mod/quiz/review.php?attempt=68873&cmid=7722 5/8
4/20/2021 Midterm exam: İnsan anatomiyası və tibbi terminologiya - 2 (EN) (MPF2): Attempt review

Question 15
Correct

Mark 1.00 out of 1.00

This doesn’t belong to the cutaneous branches of the cervical plexus:

A. n. auricularis magnus

B. n. transversus colli

C. n. subclavius 

D. n. occipitalis minor

E. n. supraclavicularis

Question 16
Correct

Mark 1.00 out of 1.00

The anterior wall of the third ventricle is formed by:

1. thalamus

2. anterior commissure 

3. columns of the fornix 

4. pineal body

5. terminal lamina 

6. epithalamic commissure

Question 17
Correct

Mark 1.00 out of 1.00

To the nuclei of the medulla oblongata belong:

1. nucleus cuneatus 

2. nucleus olivaris 

3. nucleus ruber

4. nucleus caudatus

5. nucleus dentatus

6. nucleus gracilis 

https://ets.amu.edu.az/mod/quiz/review.php?attempt=68873&cmid=7722 6/8
4/20/2021 Midterm exam: İnsan anatomiyası və tibbi terminologiya - 2 (EN) (MPF2): Attempt review

Question 18
Correct

Mark 1.00 out of 1.00

Find the accordance:

Gyrus angularis embraces posterior end of the superior temporal sulcus 

Gyrus supramarginalis embraces posterior branch of the lateral sulcus 

Question 19
Correct

Mark 1.00 out of 1.00

The external capsule is a layer of [[1]] matter that is situated between the claustrum and [[2]] nucleus.

[[1]] white 

[[2]] lentiform 

Question 20
Correct

Mark 1.00 out of 1.00

Find the colliculus facialis:

Answer: 6 

◄ Attendance

Jump to...

A. Abdullayev. 7. General overview of the peripheral nervous system. The classification, development, variation, and anomalies of the
cranial nerves. ►

https://ets.amu.edu.az/mod/quiz/review.php?attempt=68873&cmid=7722 7/8
4/20/2021 Midterm exam: İnsan anatomiyası və tibbi terminologiya - 2 (EN) (MPF2): Attempt review

https://ets.amu.edu.az/mod/quiz/review.php?attempt=68873&cmid=7722 8/8
4/20/2021 Midterm exam: İnsan anatomiyası və tibbi terminologiya - 2 (EN) (MPF2): Attempt review

Dashboard / My courses / İnsan anatomiyası və tibbi terminologiya - 2 (EN) / General

/ Midterm exam: İnsan anatomiyası və tibbi terminologiya - 2 (EN) (MPF2)

Started on Tuesday, 20 April 2021, 12:30 PM


State Finished
Completed on Tuesday, 20 April 2021, 12:56 PM
Time taken 25 mins 47 secs
Grade 17 out of 20 (85%)

Question 1
Correct

Mark 1.00 out of 1.00

The length of the spinal cord in female is (cm) :

A. 35

B. 32

C. 50

D. 43 

E. 46

Question 2
Correct

Mark 1.00 out of 1.00

The basilar groove is belonged to:

A. midbrain

B. pons 

C. diencephalon

D. lamina terminalis

E. medulla oblongata

https://ets.amu.edu.az/mod/quiz/review.php?attempt=69006&cmid=7722 1/8
4/20/2021 Midterm exam: İnsan anatomiyası və tibbi terminologiya - 2 (EN) (MPF2): Attempt review

Question 3
Correct

Mark 1.00 out of 1.00

The floor of the central part of the lateral ventricle is formed by:
I. Body of caudate nucleus
II. Head of caudate nucleus
III. Thalamus
IV. Corpus fornicis
V. Stria terminalis

A. I; II; III

B. II; III; V

C. III; IV; V

D. I; III; V 

E. I; III; IV

Question 4
Correct

Mark 1.00 out of 1.00

Where is the gyrus parahippocampalis situated?

A. between the superior and inferior temporal sulcus

B. between the lateral and superior temporal sulcus

C. between the hippocampal and collateral sulcus 

D. in the posterior part of the gyrus temporalis inferior

E. between the inferior temporal and occipitotemporalis sulcus

Question 5
Correct

Mark 1.00 out of 1.00

In region of the lateral angles of the rhomboid fossa lays:

A. locus caeruleus

B. eminentia medialis

C. nucleus dorsalis nervi vagi

D. area vestibularis 

E. nucleus ruber

https://ets.amu.edu.az/mod/quiz/review.php?attempt=69006&cmid=7722 2/8
4/20/2021 Midterm exam: İnsan anatomiyası və tibbi terminologiya - 2 (EN) (MPF2): Attempt review

Question 6
Correct

Mark 1.00 out of 1.00

What extends from the superior to the inferior angle of the rhomboid fossa?

A. eminentia medialis

B. striae medullares

C. sulcus medianus 

D. colliculus facialis

E. sulcus limitans

Question 7
Correct

Mark 1.00 out of 1.00

The nucleus of the olfactory analyzer lies in the:

A. medial surface of the occipital lobe

B. superior parietal lobule

C. uncus of the parahippocampal gyrus 

D. middle temporal gyrus

E. superior temporal gyrus

Question 8
Correct

Mark 1.00 out of 1.00

It arises from the facial nerve just above the stylomastoid foramen:

A. lesser petrosal nerve

B. branch to the tympanic plexus

C. the nerve to the stapedius

D. chorda tympani 

E. greater petrosal nerve

https://ets.amu.edu.az/mod/quiz/review.php?attempt=69006&cmid=7722 3/8
4/20/2021 Midterm exam: İnsan anatomiyası və tibbi terminologiya - 2 (EN) (MPF2): Attempt review

Question 9
Correct

Mark 1.00 out of 1.00

It does not belong to the branches of the cervical part of the vagus nerve:

A. pharyngeal branches

B. superior laryngeal nerve

C. recurrent laryngeal nerve

D. rr. cardiaci cervicales superiores

E. auricular branch 

Question 10
Correct

Mark 1.00 out of 1.00

Which layers of the serous pericardium?

A. mediastinal, costal

B. costal,parietal

C. parietal, costal

D. diaphragmaric,visceral

E. parietal, visceral 

Question 11
Incorrect

Mark 0.00 out of 1.00

The first branch of the internal carotid artery is the following:

A. a. ophthalmica 

B. a. cerebri media

C. a. cerebri anterior

D. rr. caroticotympanici

E. a. communicans posterior

https://ets.amu.edu.az/mod/quiz/review.php?attempt=69006&cmid=7722 4/8
4/20/2021 Midterm exam: İnsan anatomiyası və tibbi terminologiya - 2 (EN) (MPF2): Attempt review

Question 12
Correct

Mark 1.00 out of 1.00

It does not belong the branches of the internal carotid artery:

A. a. cerebri posterior 

B. a. ophthalmica

C. a. cerebri anterior

D. a. cerebri media

E. a. communicans posterior

Question 13
Correct

Mark 1.00 out of 1.00

It is not the branch of the radial artery:

A. r. carpalis dorsalis

B. a. recurrens radialis

C. r. carpalis palmaris

D. а. interossea communis 

E. a. princeps pollicis

Question 14
Correct

Mark 1.00 out of 1.00

The vein which is composed by the connecting occipital and posterior auricular veins:

A. superior venаcava

B. anterior jugular vein

C. brachiocephalic vein

D. internal jugular vein

E. external jugular vein 

https://ets.amu.edu.az/mod/quiz/review.php?attempt=69006&cmid=7722 5/8
4/20/2021 Midterm exam: İnsan anatomiyası və tibbi terminologiya - 2 (EN) (MPF2): Attempt review

Question 15
Correct

Mark 1.00 out of 1.00

What is called the continuation of the right ascending lumbar vein?

A. azygos vein 

B. pericardiaco-phrenic veins

C. internal thoracic vein

D. accessory hemiazygos vein

E. hemiazygos vein

Question 16
Incorrect

Mark 0.00 out of 1.00

The anterior wall of the third ventricle is formed by:

1. pineal body 

2. thalamus

3. anterior commissure 

4. columns of the fornix

5. epithalamic commissure

6. terminal lamina 

Question 17
Incorrect

Mark 0.00 out of 1.00

Which muscles are innervated by n. dorsalis scapulae?

1. m. levator scapulae 

2. m. latissimus dorsi

3. m. coracobrachialis

4. m. pectoralis major 

5. mm. rhomboidei 

https://ets.amu.edu.az/mod/quiz/review.php?attempt=69006&cmid=7722 6/8
4/20/2021 Midterm exam: İnsan anatomiyası və tibbi terminologiya - 2 (EN) (MPF2): Attempt review

Question 18
Correct

Mark 1.00 out of 1.00

Find the accordance:

Gyrus angularis embraces posterior end of the superior temporal sulcus 

Gyrus supramarginalis embraces posterior branch of the lateral sulcus 

Question 19
Correct

Mark 1.00 out of 1.00

The trochlear nerve enters the orbit through the [[1]] and innervates the [[2]].

[[1]] superior orbital fissure 

[[2]] superior oblique muscle 

Question 20
Correct

Mark 1.00 out of 1.00

Find n. vagus:

Answer: 11 

◄ Attendance

Jump to...

A. Abdullayev. 7. General overview of the peripheral nervous system. The classification, development, variation, and anomalies of the
cranial nerves. ►

https://ets.amu.edu.az/mod/quiz/review.php?attempt=69006&cmid=7722 7/8
4/20/2021 Midterm exam: İnsan anatomiyası və tibbi terminologiya - 2 (EN) (MPF2): Attempt review

https://ets.amu.edu.az/mod/quiz/review.php?attempt=69006&cmid=7722 8/8
20/04/2021 Midterm exam: İnsan anatomiyası və tibbi terminologiya - 2 (EN) (MPF2): Attempt review

Mənim əsas səhifəm / Mənim fənlərim / İnsan anatomiyası və tibbi terminologiya - 2 (EN) / Ümumi

/ Midterm exam: İnsan anatomiyası və tibbi terminologiya - 2 (EN) (MPF2)

Testə başlandı Tuesday, 20 April 2021, 12:30 PM


State Finished
Sona çatdı Tuesday, 20 April 2021, 12:58 PM
Vaxt keçib 27 dəq. 37 san.
Qiymət 20 (68%) maksimumdan 14

Question 1
Correct

Mark 1.00 out of 1.00

The spinal cord is in a topoqaphic proportion are divided into:

A. cervical,lumbar-sacral parts

B. cervical,coccygeal and lumbar parts

C. cervical ,thoracic,sacral parts

D. cervical,thoracic,lumbar parts 

E. cervical,sacral parts

Question 2
Correct

Mark 1.00 out of 1.00

Which from the following gyruses does not belong to the temporal lobe?

A. supramarginal 

B. temporalis superior

C. parahyppocampalis

D. temporalis inferior

E. temporalis media

https://ets.amu.edu.az/mod/quiz/review.php?attempt=69027&cmid=7722 1/7
20/04/2021 Midterm exam: İnsan anatomiyası və tibbi terminologiya - 2 (EN) (MPF2): Attempt review

Question 3
Incorrect

Mark 0.00 out of 1.00

The upper wall or roof of the posterior horns of the lateral ventricle is formed by the lower surfaces of:

A. commissura posterior 

B. indusium griseum

C. forceps minor

D. commisura anterior

E. tapetum

Question 4
Correct

Mark 1.00 out of 1.00

The cerebral peduncles are located:

A. behind of cerebral aqueduct

B. under the tectum mesencephali 

C. laterally of cerebral aqueduct

D. medially of cerebral aqueduct

E. in front of optic chiasm

Question 5
Correct

Mark 1.00 out of 1.00

The roof of the fourth ventricle in front consists of:


I. Superior cerebellar peduncles
II. Middle cerebellar peduncles
III. Inferior cerebellar peduncles
IV. Velum medullare superius
V. Velum medullare inferius

A. I; IV 

B. I; II

C. II; III

D. III; IV

E. III; V

https://ets.amu.edu.az/mod/quiz/review.php?attempt=69027&cmid=7722 2/7
20/04/2021 Midterm exam: İnsan anatomiyası və tibbi terminologiya - 2 (EN) (MPF2): Attempt review

Question 6
Not answered

Marked out of 1.00

What extends from the superior to the inferior angle of the rhomboid fossa?

A. sulcus medianus

B. eminentia medialis

C. sulcus limitans

D. colliculus facialis

E. striae medullares

Question 7
Correct

Mark 1.00 out of 1.00

Which of the following are the parts of the tympanic membrane?


I. Pars mucosa
II. Pars muscularis
III. Pars tensa
IV. Pars flaccida
V. Pars membranacea

A. IV, V

B. II, IV

C. III, IV 

D. I, II

E. III, V

Question 8
Correct

Mark 1.00 out of 1.00

Which cranial nerve provides the human with the ability to smell a wide variety of odors?

A. vestibulocochlear nerve

B. olfactory nerve 

C. oculomotor nerve

D. optic nerve

E. intermedius nerve

https://ets.amu.edu.az/mod/quiz/review.php?attempt=69027&cmid=7722 3/7
20/04/2021 Midterm exam: İnsan anatomiyası və tibbi terminologiya - 2 (EN) (MPF2): Attempt review

Question 9
Correct

Mark 1.00 out of 1.00

Through which opening does the maxillary nerve pass out the cranial cavity?

A. spinosum

B. rotundum 

C. ovale

D. fissura orbitalis superior

E. lacerum

Question 10
Correct

Mark 1.00 out of 1.00

How is the called right atrio-ventricular valve?

A. atrioventricularis sinistra

B. seminularis

C. bicuspidalis

D. mitralis

E. tricuspidalis 

Question 11
Correct

Mark 1.00 out of 1.00

The ascending aorta passes to the aortic arch at the level of the:

A. left third sternocostal joint

B. left fourth sternocostal joint

C. right second sternocostal joint 

D. left second sternocostal joint

E. right third sternocostal joint

https://ets.amu.edu.az/mod/quiz/review.php?attempt=69027&cmid=7722 4/7
20/04/2021 Midterm exam: İnsan anatomiyası və tibbi terminologiya - 2 (EN) (MPF2): Attempt review

Question 12
Incorrect

Mark 0.00 out of 1.00

It doesn’t belong to the branches of thyrocervical trunk:

A. suprascapular artery

B. superficial cervical artery 

C. inferior thyroid artery

D. ascending cervical artery

E. subscapular artery

Question 13
Not answered

Marked out of 1.00

To which vein does v.cephalica vein open?

A. external jugular

B. anterior jugular

C. brachial

D. axillary

E. subclavian

Question 14
Correct

Mark 1.00 out of 1.00

At which level the internal jugular vein connects with the subclavian vein?

A. thyroid cartilage

B. I rib

C. III rib

D. sterno-clavicular joint 

E. II rib

https://ets.amu.edu.az/mod/quiz/review.php?attempt=69027&cmid=7722 5/7
20/04/2021 Midterm exam: İnsan anatomiyası və tibbi terminologiya - 2 (EN) (MPF2): Attempt review

Question 15
Incorrect

Mark 0.00 out of 1.00

The parietal branches of the thoracic aorta are the followings:


I. Rr. bronchiales
II. Rr. mediastinales
III. Superior phrenic arteries
IV. Posterior intercostalis arteries
V. Rr. pericardiaci

A. III,V

B. I,III

C. III,IV

D. I,II 

E. IV,V

Question 16
Correct

Mark 1.00 out of 1.00

The anterior wall of the third ventricle is formed by:

1. anterior commissure 

2. epithalamic commissure

3. terminal lamina 

4. pineal body

5. columns of the fornix 

6. thalamus

Question 17
Correct

Mark 1.00 out of 1.00

To the branches of the ophthalmic nerve belong:

1. n. infraorbitalis

2. n. petrosus major

3. n. nasociliaris 

4. n. lacrimalis 

5. n. auriculotemporalis

6. n. frontalis 

https://ets.amu.edu.az/mod/quiz/review.php?attempt=69027&cmid=7722 6/7
20/04/2021 Midterm exam: İnsan anatomiyası və tibbi terminologiya - 2 (EN) (MPF2): Attempt review

Question 18
Partially correct

Mark 0.50 out of 1.00

Find the accordance:

Gyrus supramarginalis embraces anterior end of the inferior temporal sulcus 

Gyrus angularis embraces posterior end of the superior temporal sulcus 

Question 19
Not answered

Marked out of 1.00

At the level of the lateral edge of the [[1]] rib, the axillary vein continues into the [[2]] vein.

[[1]] Seçin...

[[2]] Seçin...

Question 20
Correct

Mark 1.00 out of 1.00

Find the ulnar nerve:

Answer: 7 

◄ Attendance

Keç ...

A. Abdullayev. 7. General overview of the peripheral nervous system. The classification, development, variation, and anomalies of the
cranial nerves. ►

https://ets.amu.edu.az/mod/quiz/review.php?attempt=69027&cmid=7722 7/7
4/20/2021 Midterm exam: Human anatomy and medical terminology - 2 (EN) (MPF2): Attempt review

My main page / My subjects / Human Anatomy and Medical Terminology - 2 (EN) / General
/ Midterm exam: Human anatomy and medical terminology - 2 (EN) (MPF2)

The test began Tuesday, 20 April 2021, 12 30 PM


State Finished
It's over Tuesday, 20 April 2021, 12 57 PM
Time has passed 26 minutes 56 sec.
Price 20 ( 95 %) maximum 19
Question 1
Correct
Mark 1.00 out of 1.00

In the posterior funiculus of the spinal cord are located:


A. tectospinalis tract, vestibulospinalis tract
B. rubrospinalis tract, spinocerebellaris dorsal tract
C. fasciculus gracilis, tectospinalis tract
D. vestibulospinalis tract, ventricular spinocerebellaris tract
E.. fasciculus cuneatus, fasciculus gracilis 

Question 2
Correct
Mark 1.00 out of 1.00

Which of the following gyruses does not belong to the temporal lobe?
A. temporalis media
B. parahyppocampalis
C. inferior temporalis
D. superior temporalis
E.. supramarginal 

https://ets.amu.edu.az/mod/quiz/review.php?attempt=68704&cmid=7722 1/7
4/20/2021 Midterm exam: Human anatomy and medical terminology - 2 (EN) (MPF2): Attempt review

Question 3
Correct
Mark 1.00 out of 1.00

The anterior limb of the internal capsule is situated between:


A. putamen and claustrum
B. lentiform nucleus and caudate nucleus 

C. thalamus and lentiform nucleus


D. claustrum and cortex of insula
E.. globi pallidus medialis et lateralis

Question 4
Correct
Mark 1.00 out of 1.00

It does not belong to the epithalamus:


A. epithalamic commissure
B. epiphysis
C. hypophysis 

D. habenular commissure
E.. habenular trigone

Question 5
Correct
Mark 1.00 out of 1.00

The roof of the fourth ventricle in front consists of:


I. Superior cerebellar peduncles
II. Middle cerebellar peduncles
III. Inferior cerebellar peduncles
IV. Velum medullare superius
V. Velum medullare inferius
A. III; V
B. I; IV 

C. II; III
D. I; II
E.. III; IV

https://ets.amu.edu.az/mod/quiz/review.php?attempt=68704&cmid=7722 2/7
4/20/2021 Midterm exam: Human anatomy and medical terminology - 2 (EN) (MPF2): Attempt review

Question 6
Correct
Mark 1.00 out of 1.00

The lower part of the rhomboid fossa is corresponded to:


A. diencephalon
B. metencephalon
C. telencephalon
D. medulla oblongata 

E.. mesencephalon

Question 7
Correct
Mark 1.00 out of 1.00

Where does the optic nerve enter the cranial cavity from?
A. foramen zygomatico-orbitale
B. foramen ethmoidale posterior
C. canalis opticus 

D. foramen ovale
E.. foramen ethmoidae anterior

Question 8
Correct
Mark 1.00 out of 1.00

Which cranial nerve provides the human with the ability to smell a wide variety of odors?
A. intermedius nerve
B. optic nerve
C. oculomotor nerve
D. olfactory nerve 

E.. vestibulocochlear nerve

https://ets.amu.edu.az/mod/quiz/review.php?attempt=68704&cmid=7722 3/7
4/20/2021 Midterm exam: Human anatomy and medical terminology - 2 (EN) (MPF2): Attempt review

Question 9
Correct
Mark 1.00 out of 1.00

It does not belong to the branches of the ophthalmic nerve:


IN infaorbitalis
II. N. lacrimalis
III. Rr. pterygopalatini
IV. N. nasociliaris
VN frontalis
VI. N. zygomaticus
A. III; IV; VI
B. I; III; V
C. II; IV; V
D. I; III; VI 

E.. II; III; IV

Question 10
Correct
Mark 1.00 out of 1.00

Which layers of the serous pericardium?


A. diaphragmaric, visceral
B. mediastinal, costal
C. parietal, visceral 

D. parietal, costal
E.. costal, parietal

Question 11
Correct
Mark 1.00 out of 1.00

The ascending aorta passes to the aortic arch at the level of the:
A. left second sternocostal joint
B. left fourth sternocostal joint
C. left third sternocostal joint
D. right second sternocostal joint 

E.. right third sternocostal joint

https://ets.amu.edu.az/mod/quiz/review.php?attempt=68704&cmid=7722 4/7
4/20/2021 Midterm exam: Human anatomy and medical terminology - 2 (EN) (MPF2): Attempt review

Question 12
Incorrect
Mark 0.00 out of 1.00

The vertebral artery (a. Vertebralis), is the first branch of the


A. subclavian artery
B. superficial cervical artery
C. subclavian artery
D. inferior thyroid artery
E.. suprascapular artery

Question 13
Correct
Mark 1.00 out of 1.00

To which vein does v.basilica open?


A. brachial 

B. anterior jugular
C. subclavian
D. axillary
E.. external jugular

Question 14
Correct
Mark 1.00 out of 1.00

The vein which is composed by the connecting occipital and posterior auricular veins:
A. internal jugular vein
B. brachiocephalic vein
C. superior venacava
D. external jugular vein 

E.. anterior jugular vein

https://ets.amu.edu.az/mod/quiz/review.php?attempt=68704&cmid=7722 5/7
4/20/2021 Midterm exam: Human anatomy and medical terminology - 2 (EN) (MPF2): Attempt review

Question 15
Correct
Mark 1.00 out of 1.00

The azygos vein opens into the vena cava superior at the level of:
A. Th9
B. Th7
C. Th3
D. Th4
E.. Th5 

Question 16
Correct
Mark 1.00 out of 1.00

Which muscles are innervated by n. dorsal scapulae?


1. m. levator scapulae 

2. mm. rhomboidei 

3. m. pectoralis major
4. m. latissimus dorsi
5. m. coracobrachialis

Question 17
Correct
Mark 1.00 out of 1.00

To the nuclei of the medulla oblongata belong:


1. nucleus caudatus
2. nucleus olivaris 

3. nucleus gracilis 

4. nucleus ruber
5. nucleus cuneatus 

6. nucleus dentatus

https://ets.amu.edu.az/mod/quiz/review.php?attempt=68704&cmid=7722 6/7
4/20/2021 Midterm exam: Human anatomy and medical terminology - 2 (EN) (MPF2): Attempt review

Question 18
Correct
Mark 1.00 out of 1.00

Find the according:


The superior laryngeal nerve is the branch of vagus nerve 

The tympanic nerve is the branch of glossopharyngeal nerve 

Question 19
Correct
Mark 1.00 out of 1.00

At the level of the lateral edge of the [[1]] rib, the axillary vein continues into the [[2]] vein.
[[1]] first 

[[2]] subclavian 

Question 20
Correct
Mark 1.00 out of 1.00

Find the pons:

Answer: 11 

◀ Attendance
Go ...
A. Abdullayev. 7. General overview of the peripheral nervous system. The classification, development, variation, and anomalies of the
cranial nerves. ▶

https://ets.amu.edu.az/mod/quiz/review.php?attempt=68704&cmid=7722 7/7
01023ÿ53ÿ01 6789 ÿ ÿÿ 8ÿÿ88ÿ 88ÿÿ0ÿ6ÿ67 !0

34567ÿ49:9ÿ94;6<47 =34567ÿ<45>4?67 =ÿ@59:5ÿ:5:AB76C:9DÿE4ÿA6FF6ÿAG?765B>BH6C:ÿIÿJÿKLMN=ÿO7P76


=ÿ36QAG?7ÿGR:7Sÿ@59:5ÿ:5:AB76C:9DÿE4ÿA6FF6ÿAG?765B>BH6C:ÿIÿJÿKLMNÿK3TUJN

VWXYZ[\]ÿ_
MBAÿCGAÿ:59`G?GQ
3:?aGQÿBPAÿB<ÿbcdd

gGA`GG5ÿA;Gÿ:5AG?6B?ÿh:?AÿB<ÿA;GÿhB59ÿ:5QÿA;Gÿ76QQ>GÿiG?GFG>>:?ÿhGQP5i>GÿG7G?HG9
jcA;Gÿ:FQPiG5Aÿ5G?EG
gcA;GÿBiP>B7BAB?ÿ5G?EG
kcA;GÿA?6HG765:>ÿ5G?EGc
lcA;Gÿ<:i6:>ÿ5G?EG
LcA;GÿA?Bi;>G:?ÿ5G?EG
k>G:?ÿ7Cÿi;B6iG

VWXYZ[\]ÿe
MBAÿCGAÿ:59`G?GQ
3:?aGQÿBPAÿB<ÿbcdd

mAÿ;:hhG59ÿ6<ÿA;Gÿ7BAB?ÿ:5:>CnG?ÿB<ÿ̀?6AAG5ÿ9hGGi;ÿ69ÿQ:7:HGQS
jc7BAB?ÿ:h;:96:
gc:H?:h;6:
kc9G59B?Cÿ:h;:96:
lc:h?:R6:
LcA:iA6>Gÿ:H5B96:
k>G:?ÿ7Cÿi;B6iG

VWXYZ[\]ÿf
MBAÿCGAÿ:59`G?GQ
3:?aGQÿBPAÿB<ÿbcdd

m5ÿ̀;6i;ÿ>BFGÿB<ÿA;GÿiG?GF?:>ÿ;G769h;G?Gÿ69ÿA;GÿiB?hP9ÿ:7CHQ:>B6QGP7ÿ96AP:AGQo
jc<?B5A:>
gcBii6h6A:>
kcAG7hB?:>
lch:?6GA:>
Lc659P>:
k>G:?ÿ7Cÿi;B6iG
p
" #33 $ %$9%$&3 93'%8&3 #$#"#( #)*+1,-./ 89)00001'% 8+2++22- 230
01023ÿ53ÿ01 6789 ÿ ÿÿ 8ÿÿ88ÿ 88ÿÿ0ÿ6ÿ67 !0
3456789:ÿ<
=>?ÿAB?ÿCDEFBGBH
ICGJBHÿ>K?ÿ>LÿMNOO

R?ÿH>BEÿD>?ÿSBT>DUÿ?>ÿBVW?XCTCYKEZ
[NXCSBDKTCGÿ?GWU>DB
\NXCSBDKTCGÿ]>YYWEEKGB
^NBVW?XCTCYW]ÿ]>YYWEEKGB
_NXAV>VXAEWE
`NBVWVXAEWE
^TBCGÿYAÿ]X>W]B

3456789:ÿP
=>?ÿAB?ÿCDEFBGBH
ICGJBHÿ>K?ÿ>LÿMNOO

aXBGBÿWEÿ?XBÿbCTTB]KTCÿ]BGBSBTTWÿT>]C?BHc
[N>Dÿ?XBÿWDLBGW>GÿEKGLC]Bÿ>Lÿ]BGBSBTTKY
\N>Dÿ?XBÿEKVBGW>GÿEKGLC]Bÿ>Lÿ]BGBSBTTKY
^N>Dÿ?XBÿCD?BGW>GÿCDHÿV>E?BGW>GÿS>GHBGEÿ>Lÿ]BGBSBTTKY
_N>Dÿ?XBÿV>E?BGW>GÿS>GHBGÿ>Lÿ]BGBSBTTKY
`N>Dÿ?XBÿCD?BGW>GÿS>GHBGÿ>Lÿ]BGBSBTTKY
^TBCGÿYAÿ]X>W]B

3456789:ÿQ
=>?ÿAB?ÿCDEFBGBH
ICGJBHÿ>K?ÿ>LÿMNOO

dXBÿT>FBGÿVCG?ÿ>Lÿ?XBÿGX>YS>WHÿL>EECÿWEÿ]>GGBEV>DHBHÿ?>Z
[NYBHKTTCÿ>ST>DUC?C
\N?BTBD]BVXCT>D
^NYBEBD]BVXCT>D
_NHWBD]BVXCT>D
`NYB?BD]BVXCT>D
^TBCGÿYAÿ]X>W]B

e
" #33 $ %$9%$&3 93'%8&3 #$#"#( #)*+1,-./ 89)00001'% 8+2++22- 030
01023ÿ53ÿ01 6789 ÿ ÿÿ 8ÿÿ88ÿ 88ÿÿ0ÿ6ÿ67 !0
3456789:ÿ<
=>?ÿAB?ÿCDEFBGBH
ICGJBHÿ>K?ÿ>LÿMNOO

R?ÿH>BEDS?ÿTBU>DVÿ?>ÿ?WBÿT>DAÿUCTAGXD?WY
ZN[CDCUXEÿEB\X[XG[KUCGXEÿ]>E?BGX>G
^N[>[WUBC
_N`BE?XTKUK\
aN[CDCUXEÿEB\X[XG[KUCGXEÿCD?BGX>G
bNK?GX[KUKE
_UBCGÿ\Aÿ[W>X[B

3456789:ÿP
=>?ÿAB?ÿCDEFBGBH
ICGJBHÿ>K?ÿ>LÿMNOO

RD?>ÿW>Fÿ\CDAÿ]CG?EÿH>BEÿ?WBÿLC[XCUÿDBG`BÿHX̀XHBÿC[[>GHXDVÿ?>ÿ?WBÿ?>]>VGC]WAÿY
ZNc
^Nd
_Ne
aNf
bNg
_UBCGÿ\Aÿ[W>X[B

3456789:ÿQ
=>?ÿAB?ÿCDEFBGBH
ICGJBHÿ>K?ÿ>LÿMNOO

R?ÿH>BEÿD>?ÿTBU>DVÿ?>ÿ?WBÿTGCD[WBEÿ>Lÿ?WBÿ[BG`X[CUÿ]CG?ÿ>Lÿ?WBÿ̀CVKEÿDBG`BY
ZNCKGX[KUCGÿTGCD[W
^NEK]BGX>GÿUCGADVBCUÿDBG`B
_N]WCGADVBCUÿTGCD[WBE
aNGGNÿ[CGHXC[Xÿ[BG`X[CUBEÿEK]BGX>GBE
bNGB[KGGBD?ÿUCGADVBCUÿDBG`B
_UBCGÿ\Aÿ[W>X[B

h
" #33 $ %$9%$&3 93'%8&3 #$#"#( #)*+1,-./ 89)00001'% 8+2++22- ,30
01023ÿ53ÿ01 6789 ÿ ÿÿ 8ÿÿ88ÿ 88ÿÿ0ÿ6ÿ67 !0
3456789:ÿ<=
>?@ÿBC@ÿDEFGCHCI
JDHKCIÿ?L@ÿ?MÿNOPP

RSTUSÿVDBCHFÿ?Mÿ@SCÿFCH?LFÿWCHTUDHITLXY
ZOXCITDF@TEDV[ÿU?F@DV
\OWDHTC@DV[ÿ]TFUCHDV
^OWDHTC@DV[ÿU?F@DV
_OITDWSHD`XDHTU[]TFUCHDV
aOU?F@DV[WDHTC@DV
^VCDHÿXBÿUS?TUC

3456789:ÿ<<
>?@ÿBC@ÿDEFGCHCI
JDHKCIÿ?L@ÿ?MÿNOPP

RSCHCÿTFÿV?UD@CIÿU?XX?EÿUDH?@TIÿDH@CHBÿbTMLHUD@T?EY
ZOFLWCHT?Hÿb?HICHÿ?Mÿ@SCÿ@SBH?TIÿUDH@TVD`C
\OTEMCHT?Hÿb?HICHÿ?Mÿ@SCÿ@SBH?TIÿUDH@TVD`C
^OW?F@CHT?Hÿb?HICHÿ?Mÿ@SCÿWDVD@C
_OcÿUCH]TUDVÿ]CH@CbHD
aOH??@ÿ?Mÿ@SCÿ@?E`LC
^VCDHÿXBÿUS?TUC

3456789:ÿ<Q
>?@ÿBC@ÿDEFGCHCI
JDHKCIÿ?L@ÿ?MÿNOPP

dSCÿbHDEUSCFÿ?Mÿ@SCÿbHDUST?UCWSDVTUÿ@HLEKeÿÿ
fOÿZOÿFLbUVD]TDÿICg@HDÿÿ
ffOÿZOÿUDH?@TFÿU?XXLETFÿFTETF@HDÿÿ
fffOÿZOÿUDH?@TFÿU?XXLETFÿICg@HDÿÿ
fcOÿZOÿFLbUVD]TDÿFTETF@HDÿÿ
cOÿZOÿ@SBH?TICDÿTXD
ZOfffhfc
\Ofhfff
^Ofchc
_Ofhff
aOffhfff
^VCDHÿXBÿUS?TUC

i
" #33 $ %$9%$&3 93'%8&3 #$#"#( #)*+1,-./ 89)00001'% 8+2++22- 530
01023ÿ53ÿ01 6789 ÿ ÿÿ 8ÿÿ88ÿ 88ÿÿ0ÿ6ÿ67 !0
3456789:ÿ<=
>?@ÿBC@ÿDEFGCHCI
JDHKCIÿ?L@ÿ?MÿNOPP

S@ÿTFÿE?@ÿ@UCÿVHDEWUÿ?Mÿ@UCÿHDITDXÿDH@CHBY
ZOHOÿWDH[DXTFÿ[DX\DHTF
]OHOÿWDH[DXTFÿI?HFDXTF
^ODOÿ[HTEWC[Fÿ[?XXTWTF
_O`OÿTE@CH?FFCDÿW?\\LETF
aODOÿHCWLHHCEFÿHDITDXTF
^XCDHÿ\BÿWU?TWC

3456789:ÿ<Q
>?@ÿBC@ÿDEFGCHCI
JDHKCIÿ?L@ÿ?MÿNOPP

bUCHCÿTFÿX?WD@CIÿ@UCÿTEMCHT?HÿFDcT@@DXÿFTELFd
ZOVC@GCCEÿ@UCÿM?HD\CEÿWDCWL\ÿDEIÿTE@CHEDXÿ?WWT[T@DXÿ[H?@LVCHDEWC
]OTEÿ@UCÿW?EMXLCEWCÿWTELL\
^OTEÿ@UCÿW?EECW@TEcÿ[XDWCÿ?Mÿ@UCÿMDXeÿWCHCVHTÿGT@Uÿ@UCÿ@CE@?HTL\ÿWCHCVCXXT
_OTEÿ@UCÿ@HDEFfCHFCÿFTELFÿcH??fCÿ?Mÿ@UCÿ?WWT[T@DXÿV?EC
aOTEÿ@UCÿTEMCHT?HÿW?EWDfCÿ\DHcTEÿ?Mÿ@UCÿMDXeÿWCHCVHT
^XCDHÿ\BÿWU?TWC

3456789:ÿ<R
>?@ÿBC@ÿDEFGCHCI
JDHKCIÿ?L@ÿ?MÿNOPP

gUCÿVHDWUTDXÿ[XCeLFÿTFÿM?H\CIÿVBÿ@UC
ZOM?LHÿ@HLEKF
]O@UHCCÿ@HLEKF
^O@G?ÿ@HLEKF
_OFTeÿ@HLEKF
aOMTfCÿ@HLEKF

h
" #33 $ %$9%$&3 93'%8&3 #$#"#( #)*+1,-./ 89)00001'% 8+2++22- -30
01023ÿ53ÿ01 6789 ÿ ÿÿ 8ÿÿ88ÿ 88ÿÿ0ÿ6ÿ67 !0
3456789:ÿ<=
>?@ÿBC@ÿDEFGCHCI
JDHKCIÿ?L@ÿ?MÿNOPP

T?ÿ@UCÿVHDEWUCFÿ?Mÿ@UCÿ?XU@UDYZ[WÿECH\CÿVCY?E]^
NOEOÿYDWH[ZDY[F
_OEOÿ[EMHD?HV[@DY[F
`OEOÿXC@H?FLFÿZDa?H
bOEOÿDLH[WLY?@CZX?HDY[F
cOEOÿMH?E@DY[F
dOEOÿEDF?W[Y[DH[F

3456789:ÿ<Q
>?@ÿBC@ÿDEFGCHCI
JDHKCIÿ?L@ÿ?MÿNOPP

T?ÿ@UCÿVHDEWUCFÿ?Mÿ@UCÿVHDWU[DYÿDH@CHBÿVCY?E]^
NODOÿXH?MLEIDÿVHDWU[[
_ODOÿW?YYD@CHDY[FÿLYEDH[FÿFLXCH[?H
`ODOÿFLVFWDXLYDH[F
bODOÿ@U?HDW?I?HFDY[F
cODOÿ@U?HDW?DWH?Z[DY[F
dODOÿW?YYD@CHDY[FÿLYEDH[Fÿ[EMCH[?H

3456789:ÿ<R
>?@ÿBC@ÿDEFGCHCI
JDHKCIÿ?L@ÿ?MÿNOPP

e[EIÿ@UCÿDWW?HIDEWC^
fBHLFÿDE]LYDH[FÿCZVHDWCF X?F@CH[?HÿCEIÿ?Mÿ@UCÿFLXCH[?Hÿ@CZX?HDYÿFLYWLF
fBHLFÿFLXHDZDH][EDY[FÿCZVHDWCF X?F@CH[?HÿVHDEWUÿ?Mÿ@UCÿYD@CHDYÿFLYWLF

3456789:ÿ<S
>?@ÿBC@ÿDEFGCHCI
JDHKCIÿ?L@ÿ?MÿNOPP

TUCÿ@H?WUYCDHÿECH\CÿCE@CHFÿ@UCÿ?HV[@ÿ@UH?L]Uÿ@UCÿggNhhÿDEIÿ[EECH\D@CFÿ@UCÿgg_hhO
ggNhh FLXCH[?Hÿ?HV[@DYÿM[FFLHC
gg_hh FLXCH[?Hÿ?VY[iLCÿZLFWYC

j
" #33 $ %$9%$&3 93'%8&3 #$#"#( #)*+1,-./ 89)00001'% 8+2++22- *30
01023ÿ53ÿ01 6789 ÿ ÿÿ 8ÿÿ88ÿ 88ÿÿ0ÿ6ÿ67 !0
3456789:ÿ<=
>?@ÿBC@ÿDEFGCHCI
JDHKCIÿ?L@ÿ?MÿNOPP

QREIÿ@SCÿLTEDHÿECHUCV

ÿ
WEFGCHV X

YÿW@@CEIDEZC
[C\ÿOOO
WOÿW]ILTTDBCUOÿXOÿ^CECHDTÿ?UCHURCGÿ?Mÿ@SCÿ_CHR_SCHDTÿECHU?LFÿFBF@C`OÿaSCÿZTDFFRMRZD@R?EbÿICUCT?_`CE@bÿUDHRD@R?EbÿDEIÿDE?`DTRCFÿ?Mÿ@SCÿ
ZHDERDTÿECHUCFOÿc

d
" #33 $ %$9%$&3 93'%8&3 #$#"#( #)*+1,-./ 89)00001'% 8+2++22- 030
4/20/2021 Midterm exam: Human anatomy and medical terminology - 2 (EN) (MPF2): Attempt review

My main page / My subjects / Human Anatomy and Medical Terminology - 2 (EN) / General
/ Midterm exam: Human anatomy and medical terminology - 2 (EN) (MPF2)

The test began Tuesday, 20 April 2021, 12 30 PM


State Finished
It's over Tuesday, 20 April 2021, 12 57 PM
Time has passed 26 minutes 56 sec.
Price 20 ( 95 %) maximum 19
Question 1
Correct
Mark 1.00 out of 1.00

In the posterior funiculus of the spinal cord are located:


A. tectospinalis tract, vestibulospinalis tract
B. rubrospinalis tract, spinocerebellaris dorsal tract
C. fasciculus gracilis, tectospinalis tract
D. vestibulospinalis tract, ventricular spinocerebellaris tract
E.. fasciculus cuneatus, fasciculus gracilis 

Question 2
Correct
Mark 1.00 out of 1.00

Which of the following gyruses does not belong to the temporal lobe?
A. temporalis media
B. parahyppocampalis
C. inferior temporalis
D. superior temporalis
E.. supramarginal 

https://ets.amu.edu.az/mod/quiz/review.php?attempt=68704&cmid=7722 1/7
4/20/2021 Midterm exam: Human anatomy and medical terminology - 2 (EN) (MPF2): Attempt review

Question 3
Correct
Mark 1.00 out of 1.00

The anterior limb of the internal capsule is situated between:


A. putamen and claustrum
B. lentiform nucleus and caudate nucleus 

C. thalamus and lentiform nucleus


D. claustrum and cortex of insula
E.. globi pallidus medialis et lateralis

Question 4
Correct
Mark 1.00 out of 1.00

It does not belong to the epithalamus:


A. epithalamic commissure
B. epiphysis
C. hypophysis 

D. habenular commissure
E.. habenular trigone

Question 5
Correct
Mark 1.00 out of 1.00

The roof of the fourth ventricle in front consists of:


I. Superior cerebellar peduncles
II. Middle cerebellar peduncles
III. Inferior cerebellar peduncles
IV. Velum medullare superius
V. Velum medullare inferius
A. III; V
B. I; IV 

C. II; III
D. I; II
E.. III; IV

https://ets.amu.edu.az/mod/quiz/review.php?attempt=68704&cmid=7722 2/7
4/20/2021 Midterm exam: Human anatomy and medical terminology - 2 (EN) (MPF2): Attempt review

Question 6
Correct
Mark 1.00 out of 1.00

The lower part of the rhomboid fossa is corresponded to:


A. diencephalon
B. metencephalon
C. telencephalon
D. medulla oblongata 

E.. mesencephalon

Question 7
Correct
Mark 1.00 out of 1.00

Where does the optic nerve enter the cranial cavity from?
A. foramen zygomatico-orbitale
B. foramen ethmoidale posterior
C. canalis opticus 

D. foramen ovale
E.. foramen ethmoidae anterior

Question 8
Correct
Mark 1.00 out of 1.00

Which cranial nerve provides the human with the ability to smell a wide variety of odors?
A. intermedius nerve
B. optic nerve
C. oculomotor nerve
D. olfactory nerve 

E.. vestibulocochlear nerve

https://ets.amu.edu.az/mod/quiz/review.php?attempt=68704&cmid=7722 3/7
4/20/2021 Midterm exam: Human anatomy and medical terminology - 2 (EN) (MPF2): Attempt review

Question 9
Correct
Mark 1.00 out of 1.00

It does not belong to the branches of the ophthalmic nerve:


IN infaorbitalis
II. N. lacrimalis
III. Rr. pterygopalatini
IV. N. nasociliaris
VN frontalis
VI. N. zygomaticus
A. III; IV; VI
B. I; III; V
C. II; IV; V
D. I; III; VI 

E.. II; III; IV

Question 10
Correct
Mark 1.00 out of 1.00

Which layers of the serous pericardium?


A. diaphragmaric, visceral
B. mediastinal, costal
C. parietal, visceral 

D. parietal, costal
E.. costal, parietal

Question 11
Correct
Mark 1.00 out of 1.00

The ascending aorta passes to the aortic arch at the level of the:
A. left second sternocostal joint
B. left fourth sternocostal joint
C. left third sternocostal joint
D. right second sternocostal joint 

E.. right third sternocostal joint

https://ets.amu.edu.az/mod/quiz/review.php?attempt=68704&cmid=7722 4/7
4/20/2021 Midterm exam: Human anatomy and medical terminology - 2 (EN) (MPF2): Attempt review

Question 12
Incorrect
Mark 0.00 out of 1.00

The vertebral artery (a. Vertebralis), is the first branch of the


A. subclavian artery
B. superficial cervical artery
C. subclavian artery
D. inferior thyroid artery
E.. suprascapular artery

Question 13
Correct
Mark 1.00 out of 1.00

To which vein does v.basilica open?


A. brachial 

B. anterior jugular
C. subclavian
D. axillary
E.. external jugular

Question 14
Correct
Mark 1.00 out of 1.00

The vein which is composed by the connecting occipital and posterior auricular veins:
A. internal jugular vein
B. brachiocephalic vein
C. superior venacava
D. external jugular vein 

E.. anterior jugular vein

https://ets.amu.edu.az/mod/quiz/review.php?attempt=68704&cmid=7722 5/7
4/20/2021 Midterm exam: Human anatomy and medical terminology - 2 (EN) (MPF2): Attempt review

Question 15
Correct
Mark 1.00 out of 1.00

The azygos vein opens into the vena cava superior at the level of:
A. Th9
B. Th7
C. Th3
D. Th4
E.. Th5 

Question 16
Correct
Mark 1.00 out of 1.00

Which muscles are innervated by n. dorsal scapulae?


1. m. levator scapulae 

2. mm. rhomboidei 

3. m. pectoralis major
4. m. latissimus dorsi
5. m. coracobrachialis

Question 17
Correct
Mark 1.00 out of 1.00

To the nuclei of the medulla oblongata belong:


1. nucleus caudatus
2. nucleus olivaris 

3. nucleus gracilis 

4. nucleus ruber
5. nucleus cuneatus 

6. nucleus dentatus

https://ets.amu.edu.az/mod/quiz/review.php?attempt=68704&cmid=7722 6/7
4/20/2021 Midterm exam: Human anatomy and medical terminology - 2 (EN) (MPF2): Attempt review

Question 18
Correct
Mark 1.00 out of 1.00

Find the according:


The superior laryngeal nerve is the branch of vagus nerve 

The tympanic nerve is the branch of glossopharyngeal nerve 

Question 19
Correct
Mark 1.00 out of 1.00

At the level of the lateral edge of the [[1]] rib, the axillary vein continues into the [[2]] vein.
[[1]] first 

[[2]] subclavian 

Question 20
Correct
Mark 1.00 out of 1.00

Find the pons:

Answer: 11 

◀ Attendance
Go ...
A. Abdullayev. 7. General overview of the peripheral nervous system. The classification, development, variation, and anomalies of the
cranial nerves. ▶

https://ets.amu.edu.az/mod/quiz/review.php?attempt=68704&cmid=7722 7/7
1. The left branch of middle colic artery anastomoses with a. colica sinistra to form:
A) Riolan arch
B) deep palmar arch
C) superficial palmar arch
D) plantar arch
E) Williz circle
2. It is the branch of a. profunda femoris:
A) a. epigastrica inferior
B) a. circumflexa ilium profunda
C) a. obturatoria
D) a. circumflexa femoris lateralis
E) a. umbilicalis
3. V. posterior ventriculi sinistri passes into:
A) atrium dextrum
B) ventriculus dexter
C) coronary sinus
D) atrium sinistrum
E) ventriculus sinistrum
4. The third neurons of the conducting tract of the olfactory analyzer are located in:

1) Olfactory bulb
2) Posterior perforated substance
3) Anterior perforated substance
4) Olfactory triangle
5) Septum pellucidum
A) 2; 3; 5
B) 1; 4; 5
C) 3; 4; 5
D) 2; 3; 4
E) 1; 2; 4

5. To the parietal branches of vena cava inferior belong:


A) v. mesenterica superior
B) vv. hepatici
C) v. mesenterica inferior
D) vv. lumbales
E) v. renalis
6. Which cranial nerves contain the parasympathetic preganglionic fibres?
A) II, VI, VII, XI
B) III, VI, X, XI
C) I, III, VII, IX
D) IV, V, VIII, XII
E) III, VII, IX, X
7. The branch of the cervical plexus which divides into medial, intermediate and
lateral branches:
A) great auricular nerve
B) phrenic nerve
C) supraclavicular nerves
D) transverse nerve of neck
E) lesser occipital nerve
8. Inferior ulnar collateral artery anastomoses with:
A) common interosseus artery
B) anterior branch of ulnar recurrent artery
C) collateral radial artery
D) recurrent radial artery
E) posterior branch of ulnar recurrent artery
9. What surround the atrioventricular orifices between the muscular layers of atria
and ventricles?
A) septum interventriculare et interatriale
B) tunica mucosa et septum interatriale
C) anulus fibrosus dexter et septum interventriculare
D) anuli fibrosi dexter et sinister
E) tunica mucosa et septum interventriculare
10. Which cranial nerve doesn’t form a trunk?
A) facial nerve
B) olfactory nerves
C) glossopharyngeal nerve
D) abducent nerve
E) optic nerve
11. Which of the following belong to the facial artery?
A) aa. alveolares superiores posteriores
B) r. massetericus
C) aa. labiales inferior et superior
D) a. ophthalmica
E) rr. dorsales linguae
12. Which cranial nerve has not parasympathetic nucleus?
A) IX
B) V
C) VII
D) X

E) III
13. It belongs to the branches of the superior thyroid artery:
A) r. suprahyoideus
B) r. mastoideus
C) a. palatina ascendens
D) a. profunda linguae
E) r. infrahyoideus
14. Pupilla locates in the center of:
A) iris
B) retina
C) sclera
D) choroid
E) cornea
15. The carotid branch is the branch of:
A) hypoglossal nerve
B) facial nerve
C) accessory nerve
D) glossopharyngeal nerve
E) vestibulocochlear nerve
16. V. rectalis superior drains into the portal vein system through:
A) v. iliocolica
B) v. mesenterica superior
C) v. mesenterica inferior
D) v. iliaca externa
E) v. lienalis
17. The superior and inferior colliculi of the midbrain are connected with the spinal
cord by:
A) tractus corticonuclearis
B) tractus tectospinalis
C) tractus corticopontinus
D) tractus nucleocorticalis
E) tractus corticocerebellaris
18. From the n. peroneus communis arises:
A) n. cutaneus dorsalis medialis
B) n. cutaneus surae medialis
C) n. cutaneus dorsalis intermedius
D) n. digiti secundi medialis
E) n. cutaneus surae lateralis
19. How many layers does the myocardium of ventricles form?
A) 4
B) 3
C) 6
D) 5
E) 2
20. N. interosseus antebrachii anterior is the branch of:
A) n. thoracodorsalis
B) n. musculocutaneus
C) n. radialis
D) n. axillaris
E) n. medianus
21. The superior branch of the oculomotor nerve innervates:
1) M. rectus lateralis
2) M. rectus superior
3) M. obliquus superior
4) M. levator palpebrae superioris

A) 1; 3
B) 2; 3
C) 1; 2
D) 3; 4
E) 2; 4
22. The branch of a. mesenterica inferior is:
A) a. colica media
B) a. colica dextra
C) a. rectalis inferior
D) a. umbilicalis
E) a. colica sinistra
23. The trochlear nerve innervates:
A) medial rectus muscle
B) superior rectus muscle
C) inferior rectus muscle
D) levator palpebrae superioris muscle
E) superior oblique muscle
24. It belongs to the parietal branches of thoracic aorta:
A) oesophageal branches
B) pericardial branches
C) bronchial branches
D) superior phrenic artery
E) mediastinal branches
25. Via the quadrilateral foramen passes:
A) a. circumflexa scapulae
B) a. collateralis ulnaris superior
C) a. recurrens ulnaris
D) a. collateralis media
E) a. circumflexa humeri posterior
26. It doesn't belong to derivatives of the coeliac plexus:
A) splenic plexus
B) prostatic plexus
C) gastric plexus
D) hepatic plexus
E) suprarenal plexus
27. The upper (superior) wall of tympanic cavity is called:
A) paries jugularis
B) paries tegmentalis
C) paries membranaceus
D) paries labyrinthicus
E) paries mastoideus
28. A. circumflexa ilium profunda is the branch of:
A) iliolumbar artery
B) inferior gluteal artery
C) superior gluteal artery
D) lateral sacral artery
E) external iliac artery
29. The superior cervical ganglion lies in front of the transverse process of:
A) II-III cervical vertebrae
B) VI cervical vertebra
C) IV-V cervical vertebrae
D) V-VI cervical vertebrae
E) IV cervical vertebra
30. The medial wall of tympanic cavity is called:
A) paries membranaceus
B) paries tegmentalis
C) paries mastoideus
D) paries jugularis
E) paries labyrinthicus
31. Which cranial nerve belongs to the sensory nerves?
A) n. facialis
B) n. trigeminus
C) n. olfactorius
D) n. vagus
E) n. hypoglossus
32. The round window is closed by:
A) secondary tympanic membrane
B) base of stapes
C) incus
D) head of malleus
E) head of stapes
33. The branches of the inferior epigastric artery are:
1) ramus pubicus
2) a. glutea superior
3) a. obturatoria
4) a. cremasterica
A) 1; 2
B) 2; 3
C) 3; 4
D) 1; 4
E) 2; 4
34. On the medial wall of the tympanic cavity are located:
1) Promontorium
2) Aditus ad antrum
3) Pars flaccida
4) Fenestra vestibuli
5) Pars tensa
A) 1; 3
B) 4; 5
C) 2; 5
D) 3; 4
E) 1; 4
35. A. interossea communis is the branch of:
A) a. ulnaris
B) a. carpalis dorsalis
C) a. carpalis palmaris
D) a. radialis
E) a. recurrens radialis
36. The muscles of the urogenital diaphragm are innervated by:
A) tibial nerve
B) femoral nerve
C) saphenous nerve
D) perineal nerve
E) obturator nerve
37. Which of the following belongs to membranous labyrinth?
A) auditory tube
B) utriculus
C) cochlea
D) external acoustic meatus
E) vestibule
38. Which of the following belong to bone labyrinth?
1) Vestibule
2) Utriculus
3) Cochlea
4) Semicircular canals
5) Sacculus
A) 1; 3; 4
B) 3; 4; 5
C) 1; 2; 3
D) 2; 3; 5
E) 1; 4; 5
39. What belong to the short branches of the sacral plexus?
1) N. gluteus superior
2) N. cutaneus femoris posterior
3) N. ischiadicus
4) N. gluteus inferior
A) 2; 3
B) 1; 3
C) 3; 4
D) 2; 4
E) 1; 4
40. In what part of the internal ear is spiral organ (organ of hearing) located?
A) sacculus
B) ductus endolymphaticus
C) ductus cochlearis
D) utriculus
E) recessus sphericus
41. Which artery belongs to the pterygoid portion of the maxillary artery?
A) a. infraorbitalis
B) a. masseterica
C) a. palatina descendens
D) a. meningea media
E) a. alveolaris inferior
42. It is not the branch of truncus thyrocervicalis:
A) a. cervicalis superficialis
B) a. thyroidea inferior
C) a. tympanica anterior
D) a. suprascapularis
E) a. cervicalis ascendens
43. The axons of II neurons of the conducting tract the olfactory analyzer pass as a
component of:

A) olfactory tract
B) intermediate bundle
C) medial bundle
D) olfactory nerves
E) lateral bundle

44. The secretory part of the chorda tympani innervates:


1) Parotid gland
2) Submandibular gland
3) Sublingual gland
4) Nasal glands
5) Palatine glands
A) 1; 2
B) 2; 3
C) 1; 4
D) 4; 5
E) 3; 4
45. Arteria transversa cervicis supplies:
A) m. sternocleidomastoideus
B) mm. infrahyoidei
C) mm. scaleni
D) muscles and skin of the back
E) m. rectus abdominis
46. N. thoracicus longus supplies:
A) m. serratus anterior
B) m. subclavius
C) m. levator scapulae
D) m. pectoralis major
E) m. latissimus dorsi
47. Which of the following belongs to the branches of the intracranial part of the
facial nerve?
A) r. buccales
B) n. petrosus major
C) r. marginalis mandibulae
D) r. colli
E) n. auricularis posterior
48. What leaves the tympanic cavity through the petrotympanic fissure?
A) n. petrosus major
B) rr. zygomatici
C) nn. palatini
D) rr. temporales
E) chorda tympani
49. Which nerves form n. suralis?
A) n. cutaneus surae medialis and n. peroneus profundus
B) n. cutaneus surae medialis and n. peroneus superficialis
C) n. fibularis profundus and n. peroneus superficialis
D) n. tibialis and n. saphenus
E) n. cutaneus surae lateralis and n. cutaneus surae medialis
50. Which arteries supply the rectum?
A) a. mesenterica superior and a. rectalis media
B) a. rectalis media and a. vesicalis superior
C) a. mesenterica inferior and a. pudenda externa
D) a. rectalis media, a. pudenda externa and a. vesicalis inferior
E) a. mesenterica inferior, a. rectalis media and a. pudenda interna
51. Which nerve takes part in the innervation of the mucous membrane of tympanic
cavity?
A) n. accessorius
B) n. trochlearis
C) n. oculomotorius
D) n. hypoglossus
E) n. facialis
52. A. princeps pollicis is the branch of:
A) a. ulnaris
B) a. radialis
C) a. interossea anterior
D) a. interossea posterior
E) a. brachialis
53. The glossopharyngeal nerve leaves the skull through:
A) foramen spinosum
B) foramen ovale
C) jugular foramen
D) hypoglossal canal
E) foramen rotundum
54. Which of the following does not belong to the branches of the facial nerve which
arise from the parotid plexus?
A) r. buccales
B) r. colli
C) r. temporales
D) r. marginalis mandibulae
E) r. stapedius
55. Which branch of the glossopharyngeal nerve contains sensory and secretory
(parasympathetic) fibers?
A) communicating
B) stylopharyngeal
C) tonsillar
D) tympanic nerve
E) pharyngeal
56. The medial half of m. flexor digitorum profundus and m. flexor carpi ulnaris are
innervated by:
A) n. ulnaris
B) n. musculocutaneus
C) n. radialis
D) n. axillaris
E) n. medianus
57. The parasympathetic nucleus of the glossopharyngeal nerve is:
A) dorsal nucleus
B) sensory nucleus
C) inferior salivary nucleus
D) ambiguus nucleus
E) superior salivary nucleus
58. Cranial nerve which innervates the posterior 1/3 part of the tongue is:
A) III
B) V
C) IX
D) VII
E) IV
59. Which cranial nerve consists of cranial and spinal roots?
A) IX
B) X
C) VI
D) V
E) XI
60. Which cranial nerves belong to the mixed nerves?
A) I, III, V
B) VI, X, XI
C) I, II, VIII
D) VI, XI, XII
E) V, VII, IX, X
61. It belongs to dermis:
A) basal layer
B) granular layer
C) spinous layer
D) cornified layer
E) reticular layer
62. Epidermis of the skin consists of:
A) five-layered epithelium
B) seven-layered epithelium
C) two-layered epithelium
D) three-layered epithelium
E) eight-layered epithelium
63. It is not the branch of the medial cord of the brachial plexus:
A) n. axillaris
B) n. ulnaris
C) n. cutaneus brachii medialis
D) n. cutaneus antebrachii medialis
E) medial root of n. medianus
64. It belongs to the branches of the frontal nerve:
A) posterior ethmoidal nerve
B) supratrochlear nerve
C) infratrochlear nerve
D) connecting branch
E) anterior ethmoidal nerve
65. The glossopharyngeal nerve gives preganglionic parasympathetic fibers to:
A) pterygopalatinal ganglion
B) otic ganglion
C) ciliary ganglion
D) semilunar ganglion
E) submandibular ganglion
66. It gives off the internal medial and internal lateral nasal branches:
A) infratrochlear nerve
B) connecting branch
C) posterior ethmoidal nerve
D) supraorbital nerve
E) anterior ethmoidal nerve
67. The visceral branch of a. iliaca interna is:
A) a. obturatoria
B) a. glutea inferior
C) a. iliolumbalis
D) a. glutea superior
E) a. uterina
68. It doesn’t belong to the motor branches of the mandibular nerve:
A) nerve to lateral pterygoid muscle
B) masseteric nerve
C) nerve to medial pterygoid muscle
D) buccal nerve
E) deep temporal nerves
69. It isn't the branch of the nasociliary nerve:
A) supraorbital nerve
B) long ciliary nerves
C) anterior ethmoidal nerve
D) infratrochlear nerve
E) posterior ethmoidal nerve
70. N. infraorbitalis gives off the sensory fibers to:
1) lower eyelid
2) upper eyelid
3) upper lip
4) lower lip
5) skin of the external nose
A) 1; 3; 5
B) 1; 2; 4
C) 3; 4; 5
D) 1; 2; 3
E) 2; 4; 5
71. The motor branches of the mandibular nerve innervate:
1) muscles of mastication
2) muscles of facial expression
3) tensor tympani and tensor veli palatini muscles
4) mylohyoid and anterior belly of digastric muscles
5) pharyngeal muscles
A) 1; 2; 5
B) 3; 4; 5
C) 1; 3; 4
D) 2; 4; 5
E) 2; 3; 4
72. Find the ductus thoracicus:

A) 4
B) 5
C) 3
D) 2
E) 1
73. It is not branch of thoracic part of aorta:
A) aa. intercostales posteriores
B) rr. bronchiales
C) a. phrenica superior
D) rr. oesophageales
E) aa. phrenicae inferiores
74. Inferior alveolar nerve innervates:
A) stylohyoid muscle
B) masseter muscle
C) mylohyoid muscle
D) medial pterygoid muscle
E) geniohyoid muscle
75. The middle superior alveolar nerve from the maxillary nerve innervates:
A) upper first molars
B) upper incisors
C) upper canine
D) upper premolars
E) upper second molars
76. In males the inferior vesical artery gives off branches to:
1) Upper part of urinary bladder
2) Seminal vesicles
3) Prostate
4) Fundus of urinary bladder
5) Inferior part of rectum
A) 3; 4; 5
B) 1; 3; 4
C) 2; 4; 5
D) 2; 3; 4
E) 1; 4; 5
77. How is one of the terminal branches of a. poplitea called?
A) a. tibialis posterior
B) a. malleolares anterior lateralis
C) a. circumflexa iliaca profunda
D) a malleolares anterior medialis
E) a. profunda femoris
78. Which of the following belong to the ganglionic branches of the maxillary nerve?
1) n. massetericus
2) rr. orbitales
3) rr. nasales posteriores superiores
4) n. petrosus minor
5) nn. palatini major et minor
A) 2; 3; 5
B) 1; 4; 5
C) 2; 4; 5
D) 1; 3; 5
E) 1; 2; 4
79. Find n. frontalis:

A) 1
B) 2
C) 5
D) 4
E) 3
80. Which of the following open to the brachiocephalic vein?
1) V. thyroidea inferior
2) V. retromandibularis
3) V. vertebralis
4) V. thoracica interna
5) V. azygos
A) 3; 4; 5
B) 2; 4; 5
C) 1; 2; 5
D) 2; 3; 4
E) 1; 3; 4
81. It leaves the skull through foramen ovale:
A) n. opticus
B) n. oculomotorius
C) n. mandibularis
D) n. ophthalmicus
E) n. maxillaris
82. Mammary gland consists of:
A) 15-20 lobules
B) 30-35 lobules
C) 5-10 lobules
D) 25-35 lobules
E) 1-5 lobules
83. It leaves the skull through the foramen rotundum:
A) n. opticus
B) n. mandibularis
C) n. ophthalmicus
D) n. maxillaris
E) n. oculomotorius
84. A. rectalis inferior is the branch of:
A) external iliac artery
B) internal pudendal artery
C) inferior mesenteric artery
D) internal iliac artery
E) inferior epigastric artery
85. Which of the following belong to the branches of the anterior vagal trunk?
1) Anterior gastric
2) Posterior gastric
3) Hepatic
4) Celiac trunk
5) Intestinal
A) 1; 2
B) 1; 3
C) 3; 4
D) 4; 5
E) 2; 5
86. Which layers of myocardium are common for both ventricles?
A) external and middle
B) external and inner
C) middle and inner
D) transverse and vertical
E) oblique and transverse
87. What does n. hypoglossus innervate?
1) Muscles of the tongue except m. palatoglossus
2) Laryngeal muscles
3) Geniohyoid muscle
4) Mylohyoid muscle
5) Pectoralis major muscle
A) 2; 4
B) 1; 3
C) 2; 5
D) 1; 5
E) 3; 4
88. Which of the following joins with the vagus nerve?
A) internal branches of the accessory nerve
B) external branches of the accessory nerve
C) ventral branches of the spinal nerve
D) dorsal branches of the spinal nerve
E) communicating branches of the spinal nerve
89. The gluteus maximus muscle is innervated by:
A) inferior gluteal nerve
B) piriformis nerve
C) superior gluteal nerve
D) perineal nerve
E) pudendus nerve
90. N. peroneus superficialis is divided into:
A) nn. digitales plantares proprii
B) n. plantaris medialis and n. plantaris lateralis
C) nn. digitales plantares communes
D) n. cutaneus dorsalis medialis and n. cutaneus dorsalis intermedius
E) n. cutaneus surae lateralis and n. cutaneus surae medialis
91. The internal branch (division) of the superior laryngeal nerve supplies:
A) mucous membrane of infraglottic part of larynx
B) m. levator veli palatini
C) laryngeal mucosa above the vocal cords
D) trachea, root of the tongue
E) cricothyroid muscle, root of the tongue
92. Sternocleidomastoid muscle is innervated by:
1) glossopharyngeal nerve
2) accessory nerve
3) cervical plexus
4) hypoglossus nerve
5) facial nerve
A) 2; 3
B) 1; 3
C) 1; 5
D) 4; 5
E) 2; 4
93. The dorsal branch of posterior intercostal arteries supplies:
A) bronchi
B) thymus
C) muscles and skin of the back
D) pericardium
E) esophagus
94. Which branch of a. poplitea branches out in cruciate ligaments of the knee joint?
A) a. genu inferior lateralis
B) a. genu media
C) a. genu inferior medialis
D) a. genu superior lateralis
E) a. genu superior medialis
95. Which cranial nerve superiorly bounds Pyragoff’s triangle in the neck?
A) V
B) X
C) IX
D) VI
E) XII
96. The trapezius muscle is innervated by:
A) n. glossopharyngeus
B) n. accessorius
C) n. vagus
D) n. trigeminus
E) n. vestibulocochlearis
97. Which artery belongs to the mandibular portion of the maxillary artery?
A) a. masseterica
B) aa. alveolares superiores anteriores
C) a. infraorbitalis
D) a. meningea media
E) a. palatina descendens
98. The external branch of the superior laryngeal nerve supplies:
A) cricothyroid muscle
B) vocal muscle
C) oblique arytenoid muscle
D) transverse arytenoid muscle
E) thyro-arytenoid muscle
99. To the branches of radial artery belong:
A) r. carpalis palmaris and a. collateralis ulnaris inferior
B) r. palmaris profundus and a. interossea communis
C) a. princeps pollicis and r. palmaris superficialis
D) a. interossea anterior and a. collateralis radialis
E) r. carpalis dorsalis and a. collateralis ulnaris superior
100. The middle cardiac vein locates in:
A) anterior interventricular groove
B) coronary groove
C) right pulmonary surface
D) left pulmonary surface
E) posterior interventricular groove
101. It doesn’t belong to the thoracic part of vagus nerve:
A) tracheal branches
B) bronchial branches
C) esophageal branches
D) upper cervical cardiac branches
E) thoracic cardiac branches
102. The pectinate muscles are formed by:
A) superficial layer of myocardium of right atrium
B) circular bundles of deep layer of myocardium of right atrium
C) vertical bundles of deep layer of myocardium of atria
D) circular bundles of deep layer of myocardium of left atrium
E) superficial layer of myocardium of left atrium
103. It does not belong to the valves of the heart:
A) interventricular
B) tricuspid
C) mitralis
D) aortic
E) valve of pulmonary trunk
104. Tuberculum intervenosum is located in:
A) ventriculus sinister
B) ventriculus dexter
C) atrium sinistrum
D) atrium dextrum
E) aorta descendens
105. What open into the right atrium of the heart?
1) superior vena cava
2) vv. pulmonaria
3) coronary sinus
4) inferior vena cava
5) v. cordis magna
6) v. cordis parva
A) 3; 5; 6
B) 2; 3; 5
C) 2; 4; 6
D) 1; 3; 4
E) 1; 2; 4
106. The fibers of lateral lemniscus end in:
1) pulvinar
2) superior colliculi
3) inferior colliculi
4) lateral geniculate body
5) medial geniculate body
A) 2; 4
B) 1; 3
C) 3; 5
D) 4; 5
E) 1; 2
107. A. dorsalis nasi is the branch of:
A) a. ophthalmica
B) a. tympanica anterior
C) a. temporalis superficialis
D) a. auricularis profunda
E) a. pharyngea ascendens
108. Find the auricula dextra:

A) 4
B) 5
C) 3
D) 2
E) 1
109. Find v. cava superior:
A) 2
B) 5
C) 3
D) 1
E) 4
110. Right suprarenal vein opens to:
A) epigastric veins
B) inferior vena cava
C) prepyloric vein
D) paraumbilical veins
E) superior phrenicus vein
111. A. recurrens tibialis posterior gives off branches to:
A) m. triceps surae
B) m. tibialis posterior
C) mm. tibialis anterior and posterior
D) knee joint
E) m. quadriceps femoris
112. Which of the following belong to the surfaces of the heart?
1) Oesophageal
2) Sternocostal
3) Mediastinal
4) Diaphragmatic
5) Tracheal
A) 1; 4
B) 2; 3
C) 3; 5
D) 1; 5
E) 2; 4
113. In the left ventricle of the heart are located:
A) trabeculae carneae, m. papillaris posterior, fossa ovalis
B) chordae tendinae, m. papillaris posterior, trabeculae carneae
C) m. papillaris septalis, m. papillaris anterior
D) m. papillaris anterior, chordae tendinae, fossa ovalis
E) m. papillaris septalis, trabeculae carneae
114. To the branches of a. carotis interna belong:
1) A. cerebelli superior
2) A. cerebri media
3) A. basilaris
4) A. ophthalmica
5) A. communicans posterior
A) 2; 4; 5
B) 1; 2; 3
C) 3; 4; 5
D) 2; 3; 5
E) 1; 2; 4
115. The weight of the heart in females is (gr):
A) 250-270
B) 150-200
C) 100-150
D) 120-130
E) 200-220
116. The inferior mesenteric artery branches off on the level of the inferior edge of:
A) first lumbar vertebra
B) twelfth thoracic vertebra
C) eleventh thoracic vertebra
D) third lumbar vertebra
E) fifth lumbar vertebra
117. The tricuspid valve of the heart is located in:
A) left atrioventricular ostium
B) coronary sinus
C) pulmonary trunk ostium
D) aortal ostium
E) right atrioventricular ostium
118. Through it blood flows from plexus thyroideus impar:
A) internal thoracic vein
B) inferior thyroid vein
C) deep cervical vein
D) superior intercostal vein
E) vertebral vein
119. The layers of the serous pericardium are:
A) diaphragmatic and visceral
B) mediastinal and costal
C) costal and parietal
D) parietal and costal
E) parietal and visceral
120. The lacrimal gland is divided into two parts by tendon of:
A) levator palpebrae superioris muscle
B) superior rectus muscle
C) superior oblique muscle
D) medial rectus muscle
E) inferior rectus muscle
121. The transverse sinus of pericardium is limited anteriorly and superiorly by:
1) initial part of aorta
2) anterior surface of right atrium
3) pulmonary trunk
4) inferior vena cava
5) anterior surface of left atrium
A) 1; 2
B) 3; 4
C) 4; 5
D) 2; 5
E) 1; 3
122. Layers of wall of the heart:
A) tunica fibrosa, myocardium, endocardium
B) epicardium, tunica fibrosa, pericardium
C) tunica mucosa, endocardium, myocardium
D) epicardium, myocardium, endocardium
E) epicardium, tunica mucosa, tunica fibrosa
123. The auricular branch of the vagus nerve exits through:
A) fissura petrosquamosa
B) foramen stylomastoideum
C) fissura petrotympanica
D) fissura tympanosquamosa
E) fissura tympanomastoidea
124. Truncus costocervicalis gives off:
A) a. pericardiacophrenica
B) a. intercostalis suprema
C) rr. intercostales anteriores
D) a. musculophrenica
E) rr. mediastinales
125. It belongs to the conducting system of the heart:
A) nodus sinuatrialis
B) vortex cordis
C) valva atrioventricularis dextra
D) apex cordis
E) valva atrioventricularis sinistra
126. The cusps of left atrioventricular valve of the heart are called:
A) cuspis dexter et sinister
B) cuspis sinister et septalis
C) cuspis anterior et posterior
D) cuspis sinister et posterior
E) cuspis dexter et septalis
127. The 8th layer of the retina is called:
A) stratum nucleare internum
B) stratum nucleare externum
C) stratum ganglionare nervi optici
D) rod and cones cells
E) stratum plexiforme externum
128. The continuation of the right coronary artery is known as:
A) a. interventricularis anterior
B) r. circumflexus
C) r. lateralis
D) a. interventricularis posterior
E) r. superior
129. Anterior jugular vein is formed from small veins of:
A) buccal region
B) submandibular region
C) mental region
D) sternocleidomastoid region
E) subclavian region
130. Which of the following open directly into right atrium of the heart?
A) v. cardiaca media
B) vv. cardiacae minimae
C) v. posterior ventriculi sinistri
D) v. cardiaca magna
E) v. cardiaca раrvа
131. It belongs to the branches of the frontal nerve:
A) anterior ethmoidal nerve
B) posterior ethmoidal nerve
C) infratrochlear nerve
D) connecting branch
E) supraorbital nerve
132. The coronary sinus of the heart opens into:
A) right atrium
B) vortex cordis
C) left ventricle
D) right ventricle
E) left atrium
133. The short articular branch of the sacral plexus innervates:
A) hip joint
B) m. quadratus femoris
C) m. gemellus superior
D) m. gemellus inferior
E) knee joint
134. It locates at the right side from midline:
A) right auricle
B) left ventricle
C) interatrial septum
D) interventricular septum
E) most part of left atrium
135. The superior ophthalmic vein opens into:
A) sinus petrosus superior
B) sinus petrosus inferior
C) sinus transversus
D) sinus cavernosus
E) sinus occipitalis
136. The tones of aortic valve are listened:
A) at the base of xiphoid process
B) in II intercostal space, at the left border of sternum
C) in left V intercostal space
D) in left VI intercostal space
E) in II intercostal space, at the right border of sternum
137. The abducens nerve innervates:
A) lateral rectus muscle
B) superior oblique muscle
C) superior rectus muscle
D) inferior rectus muscle
E) medial rectus muscle
138. The tones of tricuspidal valve are listened:
A) in left V intercostal space
B) in II intercostal space, right border of sternum
C) in left VI intercostal space
D) in II intercostal space, left border of sternum
E) at the base of xiphoid process
139. Arcus aortae continuous with the descending aorta at the level of:
A) fifth thoracic vertebra
B) sixth thoracic vertebra
C) third thoracic vertebra
D) fourth thoracic vertebra
E) seventh thoracic vertebra
140. How many parts does a. vertebralis have?
A) 5
B) 1
C) 2
D) 4
E) 3
141. Radial artery in the proximal part of the forearm is located between:
A) brachioradialis and palmaris longus muscles
B) pronator teres and brachioradialis muscles
C) flexor carpi ulnaris and palmaris longus muscles
D) brachioradialis and flexor carpi ulnaris muscles
E) pronator teres and flexor carpi ulnaris muscles
142. Stylomastoid artery is the branch of:
A) lingual artery
B) posterior auricular artery
C) superior thyroid artery
D) facial artery
E) internal carotid artery
143. Determine the portions of the maxillary artery:
1) cranial
2) mandibular
3) cervical
4) pterygoid
5) pterygopalatine
A) 2; 4; 5
B) 1; 3; 5
C) 3; 4; 5
D) 2; 3; 5
E) 1; 2; 4
144. The femoral artery passes into the popliteal fossa through:
A) canalis adductorius
B) foramen ischiadicus major
C) apertura suprapiriformis
D) foramen ischiadicus minor
E) canalis obturatorius
145. The anterior branch of a. carotis externa is:
A) a. maxillaris
B) a. mandibularis
C) a. ulnaris
D) a. thyroidea inferior
E) a. facialis
146. To which cranial nerve does the nucleus ambiguus belong?
A) V, VI, IX
B) VII, IX, X
C) IX, X, XI
D) III, IV, VII
E) X, XI, XII
147. The branch of the pterygoid portion of maxillary artery is:
A) a. supraobitalis
B) a. pharyngea ascendens
C) a. palatina descendens
D) a. masseterica
E) a. sphenopalatina
148. A. pudenda interna passes out of the pelvis through:
A) foramen suprapiriforme
B) inguinal canal
C) popliteal fossa
D) foramen infrapiriforme
E) canalis obturatorius
149. Descending upon the tuberosity of the maxilla, a. alveolaris superior posterior
(branch of maxillary artery) supply:
A) parotid gland
B) tongue
C) molar and premolar teeth
D) frontal sinus
E) eyeball
150. It descends along the canalis cruropopliteus:
A) a. tibialis anterior
B) a. obturatoria
C) a. tibialis posterior
D) a. arcuata
E) a. femoralis
151. A. transversa faciei is the branch of:
A) a. maxillaris
B) a. auricularis posterior
C) a. occipitalis
D) a. temporalis superficialis
E) a. facialis
152. Transverse sinus of pericardium is limited posteriorly by:
1) superior vena cava
2) anterior surface of right atrium
3) inferior vena cava
4) pulmonary trunk
5) anterior surface of left atrium
A) 4; 5
B) 2; 4
C) 1; 2
D) 1; 3
E) 3; 5
153. A. tympanica inferior is the branch of:
A) a. occipitalis
B) a. facialis
C) a. pharyngea ascendens
D) a. maxillaris
E) a. auricularis posterior
154. Muscular branches of which nerve innervate posterior group muscles of the
arm?
A) ulnar nerve
B) median nerve
C) dorsal scapular nerve
D) radial nerve
E) suprascapular nerve
155. R. mylohyoideus is the branch of:
A) a. temporalis superficialis
B) aa. alveolares superiores posteriores
C) a. alveolaris inferior
D) a. meningea media
E) a. auricularis profundus
156. A. laryngea superior is the branch of:
A) a. facialis
B) a. occipitalis
C) a. thyroidea superior
D) a. lingualis
E) a. carotis communis
157. It is branch of truncus coeliacus:
A) aa. gastricae breves
B) a. gastroepiploica sinistra
C) a. gastroduodenalis
D) a. gastrica sinistra
E) rr. pancreaticae
158. It doesn’t belong to the ganglionic branches of the maxillary nerve:
A) rr. nasales posteriores superiores
B) n. palatinus major
C) n. petrosus minor
D) nn. palatini minores
E) rr. orbitales
159. The lower (inferior) wall of tympanic cavity is called:
A) paries membranaceus
B) paries tegmentalis
C) paries mastoideus
D) paries labyrinthicus
E) paries jugularis
160. A. angularis is the branch of:
A) a. infraorbitalis
B) a. lacrimalis
C) a. supraorbitalis
D) a. facialis
E) a. cerebri posterior
161. A. metatarsea dorsalis prima is the branch of:
A) a. dorsalis pedis
B) aa. metatarsales plantares
C) aa. tarsales mediales
D) aa. tarsales laterales
E) aa. malleolares lateralis et medialis
162. A. infraorbitalis supplies:
1) m. rectus inferior
2) glandula lacrimalis
3) m. obliquus inferior
4) m. rectus superior
5) m. obliquus superior
A) 2; 3
B) 1; 3
C) 2; 5
D) 3; 4
E) 4; 5
163. M. levator scapulae is innervated by:
A) n. axillaris
B) n. subclavius
C) n. dorsalis scapulae
D) n. thoracodorsalis
E) n. subscapularis
164. To the branches a. basilaris belongs:
A) a. cerebelli superior
B) a. ophthalmica
C) a. сеrеbri anterior
D) a. сеrеbri media
E) a. choroidea anterior
165. The length of the right lymphatic duct is (mm):
A) 3-5
B) 20-22
C) 16-18
D) 10-12
E) 24-26
166. In formation of the arterial circle of Willis take part:
A) anterior inferior cerebellar arteries
B) posterior choroidal arteries
C) anterior cerebral arteries
D) posterior inferior cerebellar arteries
E) anterior choroidal arteries
167. What belong to the paired visceral branches of the abdominal aorta?
A) a. testicularis,a. mesenterica superior, a. suprarenalis inferior
B) a. suprarenalis inferior, a. gastrica sinistra, a. lienalis
C) a. renalis, a. testicularis, a. suprarenalis media
D) a. suprarenalis superior, a. mesenterica superior, a. testicularis
E) a. renalis, a. lienalis, a. suprarenalis media
168. The splenic artery gives off:
1) aa. gastricae breves
2) a. colica sinistra
3) rami pancreatici
4) a. gastro-epiploica sinistra
5) a. gastrica dextra
A) 1; 2; 3
B) 2; 3; 4
C) 3; 4; 5
D) 2; 4; 5
E) 1; 3; 4
169. Two anterior cerebral arteries are connected to each other by:
A) a. cerebri posterior
B) a. communicans posterior
C) a. spinalis posterior
D) a. communicans anterior
E) a. spinalis anterior
170. Arteria coronaria sinistra separates from:
A) right sinus of aorta
B) arcus аоrtaе
C) ventriculus dexter
D) truncus pulmonalis
E) left sinus of aorta
171. The descending genicular artery exites through:
A) anterior wall of canalis adductorius with n. suralis
B) anterior wall of canalis cruropopliteus with n. suralis
C) anterior wall of canalis adductorius with n. saphenus
D) anterior wall of canalis cruropopliteus with n. tibialis
E) posterior wall of canalis cruropopliteus with n. saphenus
172. The thoracic duct begins at the level of:
A) XII thoracic-I lumbar vertebrae
B) I-II lumbar vertebrae
C) II-III lumbar vertebrae
D) X-XI thoracic vertebrae
E) XI-XII thoracic vertebrae
173. Number 1 refers to:

A) a. centralis retinae
B) a. supratrochlearis
C) aa. ciliares posteriores
D) a. lacrimalis
E) aa. ethmoidales anterior et posterior
174. The largest branch of the axillary artery is:
A) lateral thoracic artery
B) anterior circumflex artery of humerus
C) thoracoacromial artery
D) subscapular artery
E) superior thoracic artery
175. Which of the following does not belong to the branches of the vagus nerve?
A) rr. pharyngei
B) n. laryngeus superior
C) r. meningeus
D) n. lingualis
E) r. auricularis
176. What open into the left atrium of the heart?
A) pulmonary veins
B) superior vena cava
C) inferior vena cava
D) azygos and hemiazygos veins
E) coronary sinus
177. Brachial artery continues to the axillary artery at:
A) superior border of pectoralis minor muscle
B) inferior border of supraspinatus muscle
C) inferior border of the pectoralis major muscle
D) superior border of supraspinatus muscle
E) superior border of deltoideus muscle
178. It is the continuation of the anterior tibial artery on the foot:
A) a. plantaris lateralis
B) a. plantaris profunda
C) a. dorsalis pedis
D) a. fibularis
E) a. plantaris medialis
179. Superior ulnar collateral artery anastomoses with:
A) collateral radial artery
B) posterior branch of ulnar recurrent artery
C) dorsal carpal branch of radial artery
D) palmar carpal branch of radial artery
E) middle collateral artery
180. The tones of pulmonary trunk valve are listened:
A) at the base of xiphoid process
B) in left V intercostal space
C) in II intercostal space, at the right border of sternum
D) in II intercostal space, at the left border of sternum
E) in left VI intercostal space
181. A. cerebelli inferior posterior is the branch of:
A) a. lacrimalis
B) a. vertebralis
C) a. cerebri posterior
D) a. cerebri anterior
E) a. cerebri media
182. The lateral deep cervical lymph nodes are located around:
A) external jugular vein
B) internal carotid artery
C) external carotid artery
D) internal jugular vein
E) common carotid artery
183. A. princeps pollicis supplies:
1) palmar surface of the thumb
2) the radial side of the index finger
3) medial side of palmar surface of fourth finger
4) lateral side of palmar surface of third finger
5) medial side of palmar surface of third finger
A) 1; 4
B) 1; 2
C) 2; 3
D) 3; 5
E) 4; 5
184. Find nn. alveolares superiores posteriores:

A) 4
B) 3
C) 5
D) 2
E) 1
185. Which of the following belong to the extracranial tributaries of the internal
jugular vein?
1) V. thyroidea superior
2) V. thoracica interna
3) V. lingualis
4) V. vertebralis
5) V. azygos
A) 1; 3
B) 1; 2
C) 3; 4
D) 4; 5
E) 2; 5
186.
187. Which artery takes place in the formation of acromial rete?
A) a. thoracoacromialis
B) a. subscapularis
C) a. alveolaris inferior
D) a. thoracica suprema
E) a. meningea media
188. The posterior wall of tympanic cavity is called:
A) paries membranaceus
B) paries mastoideus
C) paries labyrinthicus
D) paries tegmentalis
E) paries jugularis
189. Which ganglion is located on the lateral side of optic nerve?
A) pterygopalatinal ganglion
B) submandibular ganglion
C) ciliary ganglion
D) otic ganglion
E) sublingual ganglion
190. Which of the following are supplied by ileocolic artery?
1) sigmoid colon
2) descending colon
3) distal part of the ileum
4) caecum
5) rectum
A) 3; 5
B) 2; 5
C) 3; 4
D) 2; 4
E) 1; 2
191. Truncus costocervicalis supplies:
A) trapezius muscle
B) latissimus dorsi muscle
C) muscles of the first and second intercostal spaces
D) erector spinae muscle
E) rectus abdominis muscle
192. The radial artery penetrates the palm through:
A) third interosseous space
B) fourth interosseous space
C) first interosseous space
D) second interosseous space
E) second and third interosseous spaces
193. How many cm below the inguinal ligament does a. profunda femoris originates
from the posterior side of a. femoralis?
A) 7 to 8
B) 6 to 7
C) 11 to 12
D) 4 to 5
E) 9 to 10
194. The abducens nerve enters the orbit through:
A) sphenopalatine foramen
B) jugular foramen
C) foramen ovale
D) superior orbital fissure
E) foramen rotundum
195. A. thyroidea inferior is the branch of:
A) arteria vertebralis
B) truncus costocervicalis
C) arteria carotis interna
D) truncus thyrocervicalis
E) arteria occipitalis
196. To the branches of ulnar artery belong:
A) a. interossea communis
B) a. collateralis media
C) a. metacarpalis dorsalis I
D) a. thoracoacromialis
E) a. princeps pollicis
197. The branch of a. profunda brachii is:
A) a. circumflexa humeri posterior
B) a. recurrens ulnaris
C) a. collateralis media
D) a. circumflexa humeri anterior
E) a. recurrens radialis
198. The brachiocephalic trunk divides into:
A) right and left subclavian arteries
B) right and left external carotid arteries
C) right common carotid and right subclavian arteries
D) right coronary and right common carotid arteries
E) right and left coronary arteries
199. The upper molar teeth are innervated by:
A) middle superior alveolar nerve from maxillary nerve
B) anterior superior alveolar nerve from maxillary nerve
C) anterior inferior alveolar branch from mandibular nerve
D) posterior superior alveolar nerve from maxillary nerve
E) middle inferior alveolar branch from mandibular nerve
200. To the branches of superior cervical ganglion belongs:
A) n. caroticus internus
B) n. splanchnicus major
C) n. splanchnicus minor
D) n. vertebralis
E) plexus subclavius
201. It passes to the dorsal surface of the scapula through foramen trilaterum:
A) a. radialis
B) a. brachialis
C) a. subclavia
D) a. ulnaris
E) a.circumflexa scapulae
202. The nerve of pterygoid canal is formed by:
A) deep petrosal and tympanic nerves
B) greater petrosal and deep petrosal nerves
C) lesser petrosal and deep petrosal nerves
D) greater petrosal and stapedius nerves
E) greater petrosal and auriculotemporal nerves
203. To the branches of a. profunda femoris belong:
A) a. obturatoria, a. circumflexa femoris medialis, a. tibialis anterior
B) a. tibialis posterior, a. circumflexa femoris medialis
C) a. recurrens tibialis anterior, a. circumflexa femoris lateralis
D) aa. perforantes, a. obturatoria, a. tibialis posterior
E) a. circumflexa femoris medialis, a. circumflexa femoris lateralis, aa. perforantes
204. It gives off the muscular branches to the deep muscles of the forearm:
A) a. interossea anterior
B) a. subclavia
C) a. axillaris
D) a. thoracoacromialis
E) a. subscapularis
205. The ventral and dorsal nuclei are:
A) secondary neurons of the hearing analyzer
B) third neurons of the hearing analyzer
C) first neurons of the hearing analyzer
D) cortical centers of the hearing analyzer
E) subcortical centers of the hearing analyzer
206. Deep branch of the lateral plantar nerve doesn’t innervate:
A) lateral head of m. flexor hallucis brevis
B) dorsal interossei muscles
C) adductor hallucis muscle
D) abductor hallucis muscle
E) third and fourth lumbricales muscles
207. The communicating branch from cervical plexus is to:
A) glossopharyngeal nerve
B) vestibulocochlear nerve
C) vagus nerve
D) hypoglossal nerve
E) facial nerve
208. The deep palmar arch is formed by anastomosing of:
A) subscapular artery with the deep artery of arm
B) end of radial artery with the deep palmar branch of ulnar artery
C) terminal part of ulnar artery with the superficial palmar branch of radial artery
D) superior collateral ulnar artery with the inferior collateral ulnar artery
E) anterior circumflex humeral artery with the posterior circumflex humeral artery
209. A. thoracica interna does not supply:
A) intercostal muscles
B) pericardium
C) mm. scaleni
D) thymus
E) diaphragm
210. Superior vena cava is formed by the connection of:
A) hemyazygos and right highest intercostal veins
B) facial and left brachiocephalic veins
C) right brachiocephalic and right intercostal veins
D) azygos, hemiazygos and retromandibular veins
E) right and left brachiocephalic veins
211. A. ovarica begins from:
A) a. pudenda interna
B) a. rectalis superior
C) abdominal aorta
D) a. iliaca communis
E) a. iliaca interna
212. A. submentalis is the branch of:
A) a. facialis
B) a. occipitalis
C) a. temporalis superficialis
D) a. lingualis
E) a. laryngea superior
213. The suprascapular nerve supplies:
A) m. pectoralis major
B) m. pectoralis minor
C) m. subclavius
D) m. serratus anterior
E) m. supraspinatus
214. It starts from the jugular foramen, as an extension of the sigmoid sinus of the
dura mater:
A) internal thoracic vein
B) vertebral vein
C) superior intercostal vein
D) deep cervical vein
E) internal jugular vein
215. A. cervicalis profunda is the branch of:
A) a. carotis interna
B) a. thyroidea superior
C) truncus thyrocervicalis
D) truncus costocervicalis
E) a. carotis externa
216. Which branch of the ophthalmic nerve innervates the anterior part of the nasal
cavity?
A) n. infratrochlearis and nn. ciliares longi
B) nn. ciliares longi
C) n. infratrochlearis
D) r. communicans
E) n. ethmoidalis anterior
217. The left gastric artery gives off branches to:
1) stomach
2) superior part of duodenum
3) abdominal part of the oesophagus
4) left part of omentum majus
A) 3; 4
B) 2; 4
C) 1; 2
D) 1; 3
E) 2; 3
218. Which sinus lies on the lateral side of the sella turcica of the sphenoid bone?
A) sinus sagittalis superior
B) sinus transversus
C) sinus rectus
D) sinus sagittalis inferior
E) sinus cavernosus
219. Which branch of a. axillaris gives off the branches to the shoulder joint?
A) a. circumflexa humeri anterior
B) a. subscapularis
C) a. thoracodorsalis
D) a. circumflexa scapulae
E) a. thoracica lateralis
220. Vеnа cardiaca media opens into:
A) vеntriсulus sinister
B) atrium sinistrum
C) ventriculus dexter
D) atrium dexter
E) coronary sinus
221. Anterior temporal diploic vein opens into:
A) vena cerebri magna
B) superior petrosus sinus
C) sphenoparietal sinus
D) inferior petrosus sinus
E) transverse sinus
222. Which of the following belong to the subcortical hearing centers?
1) Lateral geniculate body
2) Medial geniculate body
3) Inferior colliculi
4) Superior colliculi
5) Thalamus
A) 2; 4
B) 1; 3
C) 1; 5
D) 4; 5
E) 2; 3
223. It arises from the posterior cord of the brachial plexus:
A) n. musculocutaneus
B) n. cutaneus brachii medialis
C) n. ulnaris
D) n. cutaneus antebrachii medialis
E) n. radialis
224. The lumbar arteries supply:
1) kidneys
2) skin of the back
3) muscles of the back
4) mesentery of small intestine
5) spinal cord
A) 1; 2; 3
B) 2; 3; 5
C) 2; 4; 5
D) 1; 2; 4
E) 3; 4; 5
225. The basilic vein flows into:
A) axillary vein
B) anterior jugular vein
C) external jugular vein
D) subclavian vein
E) brachial vein
226. Which of the following belong to the terminal branches of the posterior tibial
artery?
A) a. tarsalis lateralis and a. metatarsea prima
B) a. peronea and a. arcuata
C) aa. plantares medialis and lateralis
D) aa. tarsales lateralis and medialis
E) aa. recurrens tibialis anterior and posterior
227. It belongs to the mixed branches of the cervical plexus:
A) lesser occipital nerve
B) great auricular nerve
C) supraclavicular nerves
D) phrenic nerve
E) transverse nerve of the neck
228. The olfactory nerve passes through:

A) anterior ethmoidal foramen


B) posterior ethmoidal foramen
C) superior orbital fissure
D) inferior orbital fissure
E) cribriform plate of ethmoid bone
229. To the paired veins belong:
A) azygos and hemiazygos veins
B) cephalic and basilic veins
C) radial and ulnar veins
D) internal jugular and right brachiocephalic veins
E) left brachiocephalic and inferior cava veins
230. N. musculocutaneus innervates:
A) m. biceps, m. coracobrachialis, m. brachialis
B) skin of dorsal surface of arm
C) m. extensor carpi radialis longus
D) skin of dorsal surface of forearm
E) m. triceps brachii, m. anconeus
231. The communicating branch from cervical plexus is to:
A) glossopharyngeal nerve
B) vagus nerve
C) vestibulocochlear nerve
D) accessory nerve
E) facial nerve
232. It is branch of a. gastroduodenalis:
A) a. suprarenalis media
B) a. mesenterica superior
C) a. gastrica sinistra
D) a. gastroepiploica dextra
E) a. gastroepiploica sinistra
233. From sigmoid lymph nodes the lymph flows into:
A) deep inguinal lymph nodes
B) lower mesenteric lymph nodes
C) superior mesenteric lymph nodes
D) celiac lymph nodes
E) superficial inguinal lymph nodes
234. Which of the following belong to the parietal branches of internal iliac artery?
1) A. iliolumbalis
2) A. vesicalis inferior
3) A. glutea superior
4) A. rectalis media
5) A. obturatoria
A) 1; 3; 5
B) 3; 4; 5
C) 2; 3; 5
D) 1; 4; 5
E) 2; 3; 4
235. Posterior branch of the first cervical spinal nerve is called:
A) n. auricularis magnus
B) n. suboccipitalis
C) n. occipitalis minor
D) n. phrenicus
E) n. hypoglossus
236. It is not the branch of a. thoracica interna:
A) a. epigastrica superior
B) a. musculophrenica
C) rr. sternales
D) a. subscapularis
E) a. pericardiacophrenica
237. It locates at the right side from midline:
A) lesser part of left atrium
B) interventricular septum
C) right auricle
D) left ventricle
E) most part of left atrium
238. The teres minor muscle and deltoid muscle are innervated by:
A) n. subscapularis
B) n. axillaris
C) n. subclavius
D) n. dorsalis scapulae
E) n. thoracodorsalis
239. The saphenous nerve (subcutaneous) is the branch of:
A) n. pudendus
B) n. obturatorius
C) n. genitofemoralis
D) n. femoralis
E) n. ischiadicus
240. Which nerve passes through the quadrangular foramen?
A) n. axillaris
B) n. medianus
C) n. subclavius
D) n. ulnaris
E) n. musculocutaneus
241. In the abdominal cavity to the visceral lymph nodes belong:
A) lumbar lymph nodes
B) inferior diaphragmatic lymph nodes
C) celiac lymph nodes
D) parasternal lymph nodes
E) inferior epigastric lymph nodes
242. The subscapular nerve supplies:
A) pectoralis minor muscle
B) subclavius muscle
C) pectoralis major muscle
D) teres major muscle
E) serratus anterior muscle
243. The ansa cervicalis doesn’t innervate:
A) m. omohyoideus
B) m. thyrohyoideus
C) m. sternohyoideus
D) m. mylohyoideus
E) m. sternothyroideus
244. N. thoracodorsalis supplies:
A) m. pectoralis major
B) m. serratus anterior
C) m. levator scapulae
D) m. latissimus dorsi
E) m. deltoideus
245. Number 5 refers to:
A) n. vestibulocochlearis
B) ductus semicircularis anterior
C) tuba auditiva
D) ductus semicircularis lateralis
E) meatus acusticus externus
246. The vagina in female is suppied by:
A) a. iliaca externa
B) a. obturatoria
C) a. vesicalis inferior
D) a. testicularis
E) a. femoralis
247. It passes into the canalis musculoperoneus inferior:
A) anterior tibial artery
B) femoral artery
C) obturatory artery
D) arcuate artery
E) peroneal artery
248. It arises from the lateral cord of the brachial plexus:
A) n. ulnaris
B) n. cutaneus brachii medialis
C) n. musculocutaneus
D) n. cutaneus antebrachii medialis
E) n. radialis
249. The mammary glands locate between:
A) 2-4 ribs
B) 9-10 ribs
C) 6-9 ribs
D) 3-6 ribs
E) 1-3 ribs
250. From which ganglia of the sympathetic trunk does n. splanchnicus major
arise?
A) 10th to 12th thoracic ganglia
B) 1st to 2nd lumbar ganglia
C) 5th to 9th thoracic ganglia
D) 1st to 4th thoracic ganglia
E) 12th thoracic to 1st lumbar ganglia
251. Accompanied by deep brachial artery the radial nerve passes into:
A) humeromuscular canal
B) foramen trilaterum
C) median groove
D) foramen quadrilaterum
E) supraclavicular region
252. It does not belong to the short branches of the brachial plexus:
A) subscapular nerve
B) suprascapular nerve
C) median nerve
D) subclavius nerve
E) dorsal scapular nerve
253. The weight of the heart in males is (gr):
A) 440
B) 260
C) 220
D) 420
E) 300
254. Find the tuba auditiva:

A) 4
B) 5
C) 2
D) 3
E) 1
255. N. cutaneus brachii lateralis superior is the branch of:
A) n. radialis
B) n. musculocutaneus
C) n. medianus
D) n. thoracodorsalis
E) n. axillaris
256. The posterior branch of the obturator nerve gives off branches to:
1) M. obturatorius internus
2) M. rectus abdominis
3) M. adductor brevis
4) Hip joint
5) M. adductor magnus
6) M. piriformis
A) 1; 2; 6
B) 1; 2; 4
C) 2; 3; 6
D) 2; 3; 5
E) 3; 4; 5
257. Which of the following depart from the posterior intercostal arteries?
1) Thymic branches
2) Dorsal branch
3) Lateral and medial cutaneous branches
4) Inferior phrenic artery
5) Suprema intercostal arteries
A) 2; 3
B) 3; 4
C) 4; 5
D) 1; 2
E) 1; 3
258. A. pericardiacophrenica is the branch of:
A) a. thoracica interna
B) a. subclavia
C) pars thoracica aortae
D) truncus thyrocervicalis
E) a. occipitalis
259. To the parietal branches of the thoracic part of aorta belong:
A) rr. pericardiaci
B) rr. bronchiales
C) rr. oesophageales
D) aa. intercostales posteriores
E) rr. mediastinales
260. It doesn’t belong to the parasympathetic part of the intermedio-facial nerve:
A) superior salivary nucleus
B) submandibular vegetative ganglion
C) inferior salivary nucleus
D) pterygopalatine vegetative ganglion
E) sublingual vegetative ganglion
261. It belongs to the visceral branches of thoracic aorta:
A) superior phrenic artery
B) inferior phrenic artery
C) oesophageal branches
D) anterior intercostal arteries
E) posterior intercostal arteries
262. The central processes of I neurons of the conducting tract of the olfactory
analyzer forms:

A) lateral bundle
B) intermediate bundle
C) olfactory tract
D) olfactory nerves
E) medial bundle

263. M. infraspinatus is innervated by:


A) n. subscapularis
B) n. subclavius
C) n. thoracodorsalis
D) n. axillaris
E) n. suprascapularis
264. A. palatina ascendens is the branch of:
A) a. pharyngea ascendens
B) a. thyroidea inferior
C) a. facialis
D) a. occipitalis
E) a. thyroidea superior
265. Into v. azygos drains:
A) v. mesenterica superior
B) v. hemiazygos
C) v. porta
D) v. mesenterica inferior
E) v. sacralis mediana
266. A. genus superior medialis is branch of:
A) a. tibialis anterior
B) a. femoralis
C) a. peronea
D) a. poplitea
E) a. tibialis posterior
267. The branch of a. subscapularis is:
A) a. collateralis ulnaris superior
B) a. circumflexa scapulae
C) a. circumflexa humeri posterior
D) a. recurrens ulnaris
E) a. collateralis media
268. The anterior cutaneous branches of the thoracic nerves supply:
A) skin of lateral wall of chest and abdomen
B) subcostal muscles
C) skin on anterior surface of thorax and abdomen
D) transversus thoracis muscle
E) anterior and lateral group muscles of abdomen
269. Into the transverse sinus drains:
A) sinus sagittalis inferioris
B) sinus rectus
C) sinus sphenoparietalis
D) sinus petrosus inferioris
E) sinus cavernosus
270. The cusps of the tricuspid (right atrioventricular) valve of the heart are called:
A) cuspis dexter, sinister et posterior
B) cuspis dexter et septalis
C) cuspis dexter et sinister
D) cuspis dexter, sinister et anterior
E) cuspis anterior, posterior et septalis
271. To the branches of the extracranial part of the facial nerve before parotid gland
belongs:
A) posterior auricular nerve
B) nerve to the stapedius
C) branch to the tympanic plexus
D) chorda tympani
E) greater petrosal nerve
272. Abdominal part of oesophagus is supplied by:
A) a. pericardiacophrenica
B) a. gastriса dextra
C) a. thoracica interna
D) a. gastriса sinistra
E) a. renalis
273. V. obliqua atrii sinistri opens into:
A) atrium sinistrum
B) ventriculus dexter
C) coronary sinus
D) ventriculus sinistrum
E) atrium dextrum
274. It doesn’t belong to the portions of the internal carotid artery:
A) cavernous
B) pterygoid
C) petrous
D) cerebral
E) cervical
275. Arteria thoracoacromialis is the branch of:
A) a. subclavia
B) a. subscapularis
C) a. brachialis
D) a. ulnaris
E) a. axillaris
276. It is branch of a. hepatica communis:
A) a. mesenterica inferior
B) a. gastroduodenalis
C) a. gastroepiploica sinistra
D) a. mesenterica superior
E) a. gastrica sinistra
277. Preganglionic parasympathetic fibers of the greater petrosal nerve pass to:
A) sublingual ganglion
B) otic ganglion
C) ciliary ganglion
D) submandibular ganglion
E) pterygopalatine ganglion
278. A. cystica begins from:
A) splenic artery
B) gastroduodenal artery
C) left gastric artery
D) left branch of a. hepatica communis
E) right branch of a. hepatica propria
279. Find n. ophthalmicus:

A) 4
B) 5
C) 1
D) 3
E) 2
280. The anterior branch of the obturatory nerve doesn’t innervate:
A) adductor longus muscle
B) adductor brevis muscle
C) pectineus muscle
D) gracilis muscle
E) biceps femoris muscle
281. The right and left lumbar lymphatic trunks collect lymph from:
1) lower extremities
2) right upper limb, right part of the head and neck
3) organs and walls of the abdominal and pelvic cavities
4) walls of the right part of the thoracic cavity
5) left upper limb, left part of the head and neck
A) 2; 5
B) 1; 4
C) 3; 5
D) 1; 3
E) 2; 4
282. The renal artery branches off the aorta at the level of:
A) fourth lumbar vertebra
B) third lumbar vertebra
C) eleventh thoracic vertebra
D) twelfth thoracic vertebra
E) second lumbar vertebra
283. The mucosal glands of the nose and palate are innervated by postganglionic
parasympathetic fibres originating from:
A) sublingual ganglion
B) pterygopalatine ganglion
C) ciliary ganglion
D) submandibular ganglion
E) submandibular and sublingual ganglia
284. Ovarium is supplied by:
A) a. mesenterica inferior
B) a. uterina
C) a. iliaca communis
D) a. vesicalis inferior
E) a. rectalis superior
285. The inferior phrenic artery supplies:
1) adrenal gland
2) kidneys
3) lumbar part of the diaphragm
4) spinal cord
A) 1; 2
B) 2; 4
C) 2; 3
D) 3; 4
E) 1; 3
286. The superficial palmar arch is formed by anastomosing of:
A) superficial palmar branch of radial artery with the terminal part of ulnar artery
B) end of radial artery with the deep palmar branch of ulnar artery
C) superior collateral ulnar artery with the inferior collateral ulnar artery
D) anterior circumflex humeral artery with the posterior circumflex humeral artery
E) subscapular artery with the deep artery of arm
287. The perichoroidal space is between the:

A) sclera and retina

B) choroid and iris

C) retina and ciliary body


D) cornea and sclera

E) sclera and choroid


288. M. stapedius is innervated by:
A) n. glossopharyngeus
B) n. auricularis magnus
C) n. facialis
D) n. auriculotemporalis
E) n. vestibulocochlearis
289. Aa. suprarenalis superiores arise from:
A) a. phrenica inferior
B) a. phrenica superior
C) a. splenica
D) a. mesenterica superior
E) a. mesenterica inferior
290. N. cutaneus brachii posterior is the branch of:
A) n. axillaris
B) n. medianus
C) n. musculocutaneus
D) n. radialis
E) n. thoracodorsalis
291. It is situated between the layers of the dura mater forming the cavernous sinus:
A) external carotid artery
B) anterior tympanic artery
C) anterior ethmoidal artery
D) middle cerebral artery
E) internal carotid artery
292. Which of the following belong to the visceral branches of internal iliac artery?
1) A. iliolumbalis
2) A. vesicalis inferior
3) A. glutea superior
4) A. rectalis media
5) A. obturatoria
A) 2; 4
B) 1; 5
C) 1; 4
D) 2; 3
E) 3; 5
293. How many layers does the myocardium of atria form?
A) 3
B) 5
C) 4
D) 2
E) 6
294. The fibrous membrane of the eyeball is divided into:

1) Choroid
2) Cornea
3) Ciliary body
4) Sclera
5) Iris
A) 3; 5
B) 2; 4
C) 4; 5
D) 1; 3
E) 1; 2

295. The uterine tube is supplied by:


A) a. uterina
B) a. iliaca communis
C) a. vesicalis inferior
D) a. mesenterica inferior
E) a. rectalis superior
296. Which sinus lies in the attached margin of the falx cerebelli to the internal
occipital crest?
A) sinus rectus
B) sinus transversus
C) sinus sagittalis inferior
D) sinus sagittalis superior
E) sinus occipitalis
297. The parts of lacrimal gland are:
1) Pars orbitalis
2) Pars nasalis
3) Pars palpebralis
4) Pars frontalis
5) Pars zygomaticus
A) 2; 5
B) 2; 4
C) 1; 3
D) 3; 4
E) 1; 5
298. It belongs to dermis:
A) basal layer
B) papillary layer
C) spinous layer
D) cornified layer
E) granular layer
299. A. ductus deferentis in males originates from:
A) a. sacralis media
B) a. sacralis lateralis
C) a. umbilicalis
D) a. iliaca interna
E) a. glutea inferior
300. The zygomatic nerve gives off the parasympathetic postganglionic fibers by
anastomoses with:
A) lacrimal nerve
B) anterior ethmoid nerve
C) infratrochlear nerve
D) frontal nerve
E) posterior ethmoid nerve
301. The portal vein is located in:
A) lig. gastrolienale
B) lig. hepatoduodenale
C) lig. hepatorenale
D) lig. hepatogastricum
E) lig. gastrophrenicum
302. What artery leaves the pelvis through the foramen suprapiriforme?
A) a. iliolumbalis
B) a. obturatoria
C) a. glutea inferior
D) a. glutea superior
E) a. pudenda interna
303. In the canalis musculoperoneus superior, n. peroneus superficialis gives off
muscular branches to:
A) peroneus longus and brevis muscles
B) extensor hallucis longus and tibialis anterior muscles
C) tibialis anterior muscle
D) extensor digitorum brevis muscle
E) extensor hallucis longus muscle
304. In front of the promontory and the last lumbar vertebrae is located:
A) plexus renalis
B) plexus hypogastricus superior
C) plexus uretericus
D) plexus rectalis superior
E) plexus hypogastricus inferior
305. It is not the branch of the lumbar plexus:
A) n. iliohypogastricus
B) n. genitofemoralis
C) n. cutaneus femoris lateralis
D) n. ilioinguinalis
E) n. cutaneus femoris posterior
306. A. epigastrica inferior is the branch of:
A) iliolumbar artery
B) lateral sacral artery
C) inferior gluteal artery
D) obturator artery
E) external iliac artery
307. To the superior vena cava opens:
A) vv. lumbales
B) v. mesenterica superior
C) v. mesenterica inferior
D) v. porta
E) v. azygos
308. To the branches of femoral artery belong:
A) inferior epigastric, internal pudendal, deep circumflex ilium arteries
B) superficial epigastric, deep circumflex iliac, uterine arteries
C) middle genicular, inferior vesical, profunda femoris arteries
D) deep circumflex iliac, inferior epigastric, external pudendal arteries
E) superficial epigastric, superficial circumflex iliac, descending genicular arteries
309. To the branches of femoral artery belong:
A) a. circumflexa ilium profunda
B) aa. pudendae externae
C) a. epigastrica inferior
D) a. obturatoria
E) a. umbilicalis
310. To the branches of a. рrоfundа brachii belong:
A) a. circumflexa scapulae and a. circumflexa humeri posterior
B) a. collateralis ulnaris inferior and a. thoracoacromialis
C) a. collateralis media and a. collateralis radialis
D) a. collateralis ulnaris superior and a. subscapularis
E) a. circumflexa scapulae and a. subscapularis
311. A. cervicalis ascendens is the branch of:
A) a. thyroidea superior
B) a. carotis interna
C) truncus costocervicalis
D) a. thyroidea inferior
E) truncus thyrocervicalis
312. A. circumflexa femoris medialis is the branch of:
A) a. iliaca interna
B) a. poplitea
C) a. epigastrica inferior
D) a. profunda femoris
E) a. iliaca externa
313. The branch of the cervical plexus which divides into superior and inferior
branches:
A) transverse nerve of the neck
B) supraclavicular nerves
C) great auricular nerve
D) lesser occipital nerve
E) phrenic nerve
314. Nn. cardiaci thoracici arise from:
A) 5th, 6th and 7th sympathetic thoracic ganglia
B) 9th, 10th and 11th sympathetic thoracic ganglia
C) 2nd, 3rd, 4th and 5th sympathetic thoracic ganglia
D) 7th, 8th and 9th sympathetic thoracic ganglia
E) 1st and 2nd sympathetic thoracic ganglia
315. It innervates the sphenoid sinus:
A) supraorbital nerve
B) infratrochlear nerve
C) posterior ethmoidal nerve
D) connecting branch
E) anterior ethmoidal nerve
316. It does not belong to the branches of a. femoralis:
A) superficial circumflex iliac artery
B) superficial epigastric artery
C) descending genicular artery
D) profunda femoris artery
E) deep circumflex iliac artery
317. Unpaired visceral branches of abdominal aorta are:
A) a. mesenterica suреrior, a. lienalis and a. renalis
B) a. gastrica dextra, a. lienalis and a. suprarenalis
C) a. gastrica sinistra, a. gastrica dextra and a. phrenicae inferiores
D) a. renalis, a. mesenterica suреrior and a. mesenterica inferior
E) truncus coeliacus, a. mesenterica suреrior and a. mesenterica inferior
318. A. descendens genus supplies:
A) skin of anterior and medial surfaces of leg
B) skin of anterior surface of thigh
C) posterior group muscles and skin of thigh
D) posterior group muscles of leg
E) m. vastus medialis and knee joint
319. What passes through the lacuna vasorum?
A) a. femoralis
B) a. tibialis posterior
C) a. poplitea
D) a. tibialis anterior
E) a. fibularis
320. The portal vein is formed by:
A) splenic vein, superior and inferior mesenteric veins
B) renal, inferior mesenteric and azygos veins
C) lumbales, renal and superior mesenteric veins
D) renal, superior mesenteric and azygos veins
E) splenic, superior mesenteric and lumbar veins
321. From the main trunk of the n. peroneus communis arises the muscular branch
to:
A) obturatorius internus muscle
B) short head of biceps femoris muscle
C) semitendinosus muscle
D) semimembranosus muscle
E) psoas major muscle
322. It belongs to the long branch of the sacral plexus:
A) n. gluteus superior
B) n. ischiadicus
C) n. pudendus
D) n. perinealis
E) n. gluteus inferior
323. It doesn’t belong to the branches of a. dorsalis pedis:
A) aa. tarseae mediales
B) a. arcuata
C) a. tarsea lateralis
D) a. metatarsea dorsalis prima
E) a. plantaris medialis
324. What are the branches of the coeliac trunk?
A) left gastric, superior mesenteric and inferior mesenteric arteries
B) proper hepatic, left colic and left gastric arteries
C) right gastric, superior mesenteric and inferior pancreatoduodenal arteries
D) superior mesenteric, pancreatic and right colic arteries
E) left gastric, common hepatic and splenic arteries
325. A. recurrens tibialis posterior is the branch of:
A) a. arcuata
B) a. femoralis
C) a. poplitea
D) a. tibialis anterior
E) a. dorsalis pedis
326. The branches of a. mesenterica superior to jejunum and ileum are:
A) intestinal arteries
B) middle colic artery
C) right colic artery
D) middle and inferior rectal arteries
E) left colic artery
327. The terminal branch of external carotid artery is:
A) a. maxillaris
B) a. pharyngea ascendens
C) a. infraorbitalis
D) a. facialis
E) a. supraobitalis
328. A. cerebelli inferior anterior is the branch of:
A) a. cerebri posterior
B) a. occipitalis
C) a. basilaris
D) a. cerebri media
E) a. facialis
329. The branch of a. tibialis posterior is:
A) femoral artery
B) arcuate artery
C) peroneal artery
D) posterior tibial recurrent artery
E) anterior tibial recurrent artery
330. Nervus phrenicus innervates:
A) mm. infrahyoidei
B) m. geniohyoideus
C) m. trapezius
D) m. sternocleidomastoideus
E) diaphragmatic musculature
331. A. tympanica anterior enters the tympanic cavity through:
A) caroticotympanic canaliculi
B) external carotid foramen
C) petrotympanic fissure
D) stylomastoid foramen
E) tympanomastoid fissura
332. N. vagus leaves the skull through:
A) hypoglossal canal
B) foramen ovale
C) foramen spinosum
D) jugular foramen
E) foramen rotundum
333. It doesn't belong to the branches of the anterior tibial artery:
A) a. recurrens tibialis posterior
B) a. recurrens tibialis anterior
C) a. malleolaris anterior lateralis
D) a. arcuata
E) a. malleolaris anterior medialis
334. The parts of the interventricular septum of the heart are:
A) membranous and muscular
B) pulmonary and mediastinal
C) costal and mediastinal
D) costal and pulmonary
E) membranous and mediastinal
335. How many ganglia does the sympathetic trunk consist of?
A) 20-25
B) 18-20
C) 10-12
D) 25-27
E) 28-30
336. To the visseral branches of the inferior vena cava belong:
1) Renal veins
2) Lumbar veins
3) Right suprarenal vein
4) Inferior phrenic veins
5) Hepatic veins
A) 1; 3; 5
B) 3; 4; 5
C) 2; 3; 5
D) 1; 2; 4
E) 2; 3; 4
337. Which cranial nerves belong to the motor nerves?
A) V, VII, IX, X
B) I, III, V
C) I, II, VIII
D) VI, X, XII
E) III, IV, VI, XI, XII
338. A. arcuata is the branch of:
A) aa. tarsales mediales
B) aa. tarsales laterales
C) a. dorsalis pedis
D) aa. malleolares lateralis and medialis
E) aa. metatarsales plantares
339. It does not belong to the branches of the internal iliac artery:
A) a. sacralis mediana
B) a. pudenda interna
C) a. uterina
D) a. rectalis media
E) a. iliolumbalis
340. V. saphena magna drains into:
A) v. saphena magna
B) v. femoralis
C) vv. marginales mediales
D) femoral vein
E) vv. lumbales
341. Which arteries form arcus plantaris?
A) arcuate artery and medial plantar artery
B) lateral plantar artery and deep plantar branch
C) arcuate artery and lateral malleolar artery
D) fibular artery and medial malleolar artery
E) lateral and medial malleolar arteries
342. N. genitofemoralis emerges from:
A) brachial plexus
B) lumbar plexus
C) coccygeal plexus
D) intercostal nerves
E) cervical plexus
343. Which parasympathetic ganglion is located just below the foramen ovale and is
medial to the mandibular nerve?
A) pterygopalatinal ganglion
B) otic ganglion
C) submandibular ganglion
D) ciliary ganglion
E) sublingual ganglion
344. The vascular layer of the eyeball consists of:

1) Choroid
2) Cornea
3) Ciliary body
4) Sclera
5) Iris
A) 2; 4; 5
B) 1; 3; 5
C) 2; 3; 4
D) 3; 4; 5
E) 1; 2; 4

345. It belongs to the long branches of the brachial plexus:


A) n. musculocutaneus
B) n. dorsalis scapulae
C) n. suprascapularis
D) n. subscapularis
E) n. subclavius
346. Which of the following are innervated by obturatory nerve?
1) Mm. adductor longus and brevis
2) Skin of the lateral surface of the thigh
3) Skin of the medial surface of the thigh
4) Hip joint
5) M. rectus abdominis
A) 1; 3; 4
B) 3; 4; 5
C) 1; 2; 4
D) 2; 3; 4
E) 2; 4; 5
347. To the parietal branches of abdominal aorta belong:
A) a. mesenterica suреtior and a. mesenterica inferior
B) a. mеsеntеriса inferior and a. iliaca interna
C) a. testicularis and a. ovarica
D) aa. lumbalеs and a. phrenica inferior
E) a. mеsеntеriса inferior and a. renalis
348. The aorta carries blood from:
A) right ventricle of heart
B) right auricle of heart
C) left ventricle of heart
D) left auricle of heart
E) apex of the heart
349. How is the largest skin branch of the femoral nerve called?
A) saphenous nerve
B) obturator nerve
C) inferior gluteal nerve
D) pudendal nerve
E) superior gluteal nerve
350. The right lymphatic duct is formed by connection of:
A) right and left jugular trunks
B) right subclavian, intestinal and left broncho-mediastinal trunks
C) right subclavian, right jugular and right broncho-mediastinal trunks
D) right and left lumbar trunks
E) intestinal and right broncho-mediastinal trunks
351. Determine the branch of splenic artery:
A) a. hepatica communis
B) a. gastroepiploica sinistra
C) a. pancreatoduodenalis superior
D) a. gastroduodenalis
E) a. gastica dextra
352. The inferior sagittal sinus lies:
A) on the lateral side of the sella turcica
B) at the junction of the falx cerebri with the tentorium cerebelli
C) in groove for transverse sinus of occipital bone
D) in the posterior margin of tentorium cerebelli
E) in the free lower margin of the falx cerebri
353. M. tensor fasciae latae is innervated by:
A) perineal nerve
B) superior gluteal nerve
C) inferior gluteal nerve
D) piriformis nerve
E) pudendus nerve
354. This nerve exits from the dorsal aspect of brainstem:
A) oculomotor nerve
B) vagus nerve
C) abducens nerve
D) trochlear nerve
E) facial nerve
355. How is the external layer of myocardium of the ventricles called?
A) oblique
B) vertical
C) radial
D) irregular
E) transverse
356. Which lymph nodes are located on lateral edge of trapezius muscle?
A) submental
B) parotid
C) occipital
D) prelaryngeal
E) facial
357. What belong to the branches of n. cutaneus femoris posterior?
1) Nn. clunium inferiores
2) N. cutaneus surae lateralis
3) N. cutaneus surae medialis
4) Rr. perineales
5) Nn. digitales dorsales pedis
A) 4; 5
B) 2; 4
C) 1; 3
D) 3; 5
E) 1; 4
358. A. brachialis is located in:
A) median groove
B) groove lateral to biceps brachii muscle
C) radial groove
D) groove medial to biceps brachii muscle
E) interscalene space
359. Sural nerve winds around the lateral maleolus from the back and then
continues as:
A) n. gluteus inferior
B) n. cutaneous femoris posterior
C) n. cutaneous dorsalis lateralis
D) rr. cutanei cruri medialis
E) n. gluteus superior
360. The lateral wall of tympanic cavity is called:
A) paries membranaceus
B) paries jugularis
C) paries mastoideus
D) paries labyrinthicus
E) paries tegmentalis
361. A. pudenda interna is the branch of:
A) a. iliaca communis
B) a. pudenda interna
C) a. mesenterica inferior
D) a. iliaca interna
E) a. femoralis
362. It does not belong to the cutaneous branches of the cervical plexus:
A) nn. supraclaviculares
B) n. transversus colli
C) n. occipitalis minor
D) n. occipitalis major
E) n. auricularis magnus
363. N. suralis supplies the skin of:
A) medial malleolus
B) medial and lateral malleoli
C) dorsal lateral aspect of foot
D) medial border of foot
E) plantar surface of foot
364. The ilioinguinal nerve is branch of:
A) plexus lumbalis
B) plexus cervicalis
C) plexus coccygeus
D) plexus sacralis
E) plexus sacralis and plexus coccygeus
365. The superior sagittal sinus lies:
A) in the posterior margin of tentorium cerebelli
B) on the lateral side of the sella turcica
C) at the junction of the falx cerebri with the tentorium cerebelli
D) in the upper fixed margin of falx cerebri
E) in groove for transverse sinus of occipital bone
366. N. tibialis doesn’t innervate:
A) m. soleus
B) m. flexor hallucis longus
C) m. gastrocnemius
D) short head of m. biceps femoris
E) m. tibialis posterior
367. It is the short branch of the brachial plexus:
A) n. radialis
B) n. suprascapularis
C) n. medianus
D) n. ulnaris
E) n. musculocutaneus
368. Which of the following does not belong to the branches of the intercostal
nerves?
A) r. cutanei anteriores pectoralis and abdominales
B) r. pleuralis
C) r. peritonealis
D) r. oesophagei
E) r. cutanei laterales pectorales and abdominales
369. The cephalic vein flows into:
A) brachial vein
B) subclavian vein
C) external jugular vein
D) anterior jugular vein
E) axillary vein
370. The thoracic duct forms an arch at the level of:
A) IV-V cervical vertebrae
B) V-VII cervical vertebrae
C) II-III cervical vertebrae
D) V-VI thoracic vertebrae
E) VII-VIII thoracic vertebrae
371. Which arteries belong to the pterygopalatine portion of the maxillary artery?
A) a. infraorbitalis, a. sphenopalatina, a. palatina descendens
B) a. masseterica, a. meningea media, a. submentalis
C) a. alveolaris inferior, a. palatina descendens, a. tympanica anterior
D) a. meningea media, aa. temporales profundae, a. lacrimalis
E) a. palatina ascendens, a. tympanica anterior, a. sphenopalatina
372. The m. transversus abdominis, mm. obliquus abdominis internus and externus
are innervated by muscular branches of:
A) ilioinguinal nerve
B) genitofemoral nerve
C) inferior rectal nerve
D) perineal nerve
E) obturatory nerve
373. N. saphenus innervates:
1) skin of the medial surface of the leg
2) lateral edge of the foot
3) lateral malleolus
4) medial edge of the foot
A) 2; 3
B) 1; 2
C) 3; 4
D) 1; 4
E) 2; 4
374. Number 2 refers to:
A) truncus lumbalis sinister
B) truncus intestinalis
C) cysterna chili
D) ductus thoracicus
E) truncus lumbalis dexter
375. They participate in the formation of the dorsal carpal and palmar carpal rete:
1) r.carpalis palmaris of a. radialis
2) a. recurrens ulnaris
3) r.carpalis dorsalis of a. radialis
4) a. recurrens radialis
A) 3; 4
B) 1; 3
C) 2; 3
D) 2; 4
E) 1; 4
376. From the upper extremities the lymph flows through:
A) lumbar trunk
B) bronchomediastinal trunk
C) intestinal trunk
D) jugular trunk
E) subclavian trunk
377. The superior hypogastric plexus is divided into right and left hypogastric
nerves:
A) below the promontorium
B) above the promontorium
C) at level of V lumbar vertebra
D) between IV and V lumbar vertebrae
E) at level of IV lumbar vertebra
378. The ventral (anterior) branches of which spinal nerves don’t form the plexus?
A) lumbar nerves
B) sacral nerves
C) cervical nerves
D) coccygeal nerves
E) thoracic nerves
379. Posterior branch of what cervical nerve is motor?
A) fourth
B) seventh
C) second
D) first
E) third
380. To the parietal lymph nodes of the pectoral cavity belong:
A) bronchopulmonary
B) upper tracheobronchial
C) anterior mediastinal
D) lower tracheobronchial
E) parasternal
381. The I neuron of the statokinetic analyzer locates in:
A) mastoid antrum
B) internal acoustic meatus
C) auditory tube
D) vestibule
E) external acoustic meatus
382. R. palmaris profundus is the branch of:
A) a. ulnaris
B) a. collateralis media
C) a. radialis
D) a. radialis recurrens
E) a. brachialis
383. In the abdominal cavity to the parietal lymph nodes belong:
A) gastric lymph nodes
B) pancreatic lymph nodes
C) lumbar lymph nodes
D) celiac lymph nodes
E) hepatic lymph nodes
384. The brachiocephalic vein is formed by the fusion of:
A) facial vein with external jugular vein
B) retromandibular vein with external jugular vein
C) internal jugular vein with the subclavian vein
D) subclavian vein with external jugular vein
E) azygos vein with hemiazygos vein
385. The phrenic nerve innervates:
1) pericardium
2) m. scalenus posterior
3) plevra
4) ligaments of the stomach
5) hepatic ligaments
6) m. scalenus medius
A) 3; 4; 6
B) 2; 3; 4
C) 1; 2; 5
D) 1; 3; 5
E) 4; 5; 6
386.
387. The sinuatrial node of the heart is located in the wall of:
A) left atrium
B) left ventricule
C) right atrium
D) interventricular septum
E) left auricle
388. The ganglion stellatum is formed by fusion of:
A) superior cervical ganglion and first thoracic sympathetic ganglion
B) inferior cervical ganglion and first thoracic sympathetic ganglion
C) inferior cervical ganglion and third thoracic sympathetic ganglion
D) inferior cervical ganglion and fifth thoracic sympathetic ganglion
E) middle cervical ganglion and first thoracic sympathetic ganglion
389. The oval window is covered by:
A) secondary tympanic membrane
B) incus
C) head of malleus
D) tympanic membrane
E) base of stapes
390. From which ganglia of the sympathetic trunk does n. splanchnicus minor
arise?
A) 10th and 11th thoracic ganglia
B) 12th thoracic and 1st lumbar ganglia
C) 8th and 9th thoracic ganglia
D) 1st and 2nd lumbar ganglia
E) 7th and 8th thoracic ganglia
391. A. pharyngea ascendens does not supply:
A) tongue
B) tympanic cavity
C) dura mater
D) m. constrictores pharyngis medius and inferior
E) tonsil
392. The inferior branch of the oculomotor nerve innervates:
1) Inferior rectus muscle
2) Lateral rectus muscle
3) Inferior oblique muscle
4) Medial rectus muscle
5) Levator palpebrae superioris muscle

A) 2; 4; 5
B) 2; 3; 4
C) 1; 3; 4
D) 3; 4; 5
E) 1; 2; 4

393. Which of the following does the superior laryngeal nerve innervate?
A) esophagus, m. levator veli palatini
B) external acoustic meatus
C) trachea, esophagus
D) laryngeal mucosa above vocal cords
E) laryngeal mucosa below vocal cords, trachea
394. On m. levator ani is located:
A) plexus renalis
B) plexus hypogastricus superior
C) plexus uretericus
D) plexus lienalis
E) plexus hypogastricus inferior
395. It belongs to the parasympathetic part of the glossopharyngeal nerve:
A) nucleus salivatorius inferius
B) nucleus salivatorius superius
C) pterygopalatine ganglion
D) submandibular ganglion
E) sublingual ganglion
396. V. saphena parva drains into:
A) vv. marginales mediales
B) v. saphena magna
C) vv. lumbales
D) v. femoralis
E) v. poplitea
397. The sciatic nerve gives off muscular branches to:
1) M. obturator internus
2) M. psoas major
3) M. sartorius
4) Mm. gemelli
A) 2; 3
B) 1; 2
C) 3; 4
D) 1; 4
E) 2; 4
398. The parasympathetic postganglionic secretory fibers from the otic ganglion
reach:
A) thyroid gland
B) submandibular salivary gland
C) sublingual salivary gland
D) parotid salivary gland
E) adrenal gland
399. The chorda tympani is the branch of:
A) n. trigeminus
B) n. facialis
C) n. vestibulocochlearis
D) n. vagus
E) n. glossopharyngeus
400. From the head and neck the lymph flows through:
A) bronchomediastinal trunk
B) jugular trunk
C) subclavian trunk
D) lumbar trunk
E) intestinal trunk
401. Which cranial nerves belong to the sensory nerves?
A) I, II, VIII
B) I, III, V
C) IV, VII, XI
D) VI, X, XII
E) VII, IX, X
4/21/2021 Midterm exam: İnsan anatomiyası və tibbi terminologiya - 2 (EN) (MPF2): Attempt review

Dashboard / My courses / İnsan anatomiyası və tibbi terminologiya - 2 (EN) / General

/ Midterm exam: İnsan anatomiyası və tibbi terminologiya - 2 (EN) (MPF2)

Started on Tuesday, 20 April 2021, 12:30 PM


State Finished
Completed on Tuesday, 20 April 2021, 12:59 PM
Time taken 29 mins 22 secs
Grade 19 out of 20 (95%)

Question 1
Incorrect

Mark 0.00 out of 1.00

In the posterior funiculus of the spinal cord are located:

A. tractus rubrospinalis, tractus spinocerebellaris dorsalis

B. tractus vestibulospinalis, tractus spinocerebellaris ventralis

C. tractus tectospinalis, tractus vestibulospinalis

D. fasciculus gracilis, tractus tectospinalis 

E. fasciculus cuneatus, fasciculus gracilis

Question 2
Correct

Mark 1.00 out of 1.00

Which from the following gyruses does not belong to the temporal lobe?

A. temporalis inferior

B. temporalis superior

C. temporalis media

D. supramarginal 

E. parahyppocampalis

https://ets.amu.edu.az/mod/quiz/review.php?attempt=68698&cmid=7722 1/8
4/21/2021 Midterm exam: İnsan anatomiyası və tibbi terminologiya - 2 (EN) (MPF2): Attempt review

Question 3
Correct

Mark 1.00 out of 1.00

Posterior end of the thalamus is called:

A. fissura chorioidea

B. tuberculum anterius

C. lamina affixa

D. stria terminalis

E. pulvinar 

Question 4
Correct

Mark 1.00 out of 1.00

It does not belong to epithalamus:

A. hypophysis 

B. epithalamic commissure

C. habenular trigone

D. habenular commissure

E. epiphysis

Question 5
Correct

Mark 1.00 out of 1.00

The dorsal part of metencephalon is:

A. cerebellum 

B. corpus mamillaris

C. pedunculus cerebri

D. pons

E. tuber cinereum

https://ets.amu.edu.az/mod/quiz/review.php?attempt=68698&cmid=7722 2/8
4/21/2021 Midterm exam: İnsan anatomiyası və tibbi terminologiya - 2 (EN) (MPF2): Attempt review

Question 6
Correct

Mark 1.00 out of 1.00

In medulla oblongata is not located:

A. center of vomiting

B. center of swallowing

C. circulatory center

D. respiratory center

E. highest center of vegetative nervous system 

Question 7
Correct

Mark 1.00 out of 1.00

The oculomotor nerve emerges from cranial cavity through:

A. fissura orbitalis superior 

B. canalis opticus

C. foramen ethmoidale anterior

D. foramen rotundum

E. fissura orbitalis inferior

Question 8
Correct

Mark 1.00 out of 1.00

The glossopharyngeal nerve leaves the skull through:

A. canalis hypoglossus

B. foramen jugulare 

C. foramen lacerum

D. foramen rotundum

E. foramen ovale

https://ets.amu.edu.az/mod/quiz/review.php?attempt=68698&cmid=7722 3/8
4/21/2021 Midterm exam: İnsan anatomiyası və tibbi terminologiya - 2 (EN) (MPF2): Attempt review

Question 9
Correct

Mark 1.00 out of 1.00

It does not belong to the branches of the ophthalmic nerve:


I. N. infaorbitalis
II. N. lacrimalis
III. Rr. pterygopalatini
IV. N. nasociliaris
V. N. frontalis
VI. N. zygomaticus

A. III; IV; VI

B. I; III; V

C. I; III; VI 

D. II; III; IV

E. II; IV; V

Question 10
Correct

Mark 1.00 out of 1.00

How many layers can be distinguished in the myocardium of the ventricles?

A. 2

B. 3 

C. 4

D. 1

E. 5

Question 11
Correct

Mark 1.00 out of 1.00

Thoracic aorta locates between the:

A. ThI-ThXII

B. ThIV-ThXII 

C. ThVI-ThXII

D. ThIII-ThVIII

E. ThIV-ThX

https://ets.amu.edu.az/mod/quiz/review.php?attempt=68698&cmid=7722 4/8
4/21/2021 Midterm exam: İnsan anatomiyası və tibbi terminologiya - 2 (EN) (MPF2): Attempt review

Question 12
Correct

Mark 1.00 out of 1.00

It does not compose the thyrocervical trunk:

A. inferior thyroid artery

B. superficial cervical artery

C. suprascapular artery

D. vertebral artery 

E. ascending cervical artery

Question 13
Correct

Mark 1.00 out of 1.00

To which vein does v.cephalica vein open?

A. subclavian

B. axillary 

C. anterior jugular

D. external jugular

E. brachial

Question 14
Correct

Mark 1.00 out of 1.00

The vein which is composed by the connecting occipital and posterior auricular veins:

A. internal jugular vein

B. brachiocephalic vein

C. anterior jugular vein

D. superior venаcava

E. external jugular vein 

https://ets.amu.edu.az/mod/quiz/review.php?attempt=68698&cmid=7722 5/8
4/21/2021 Midterm exam: İnsan anatomiyası və tibbi terminologiya - 2 (EN) (MPF2): Attempt review

Question 15
Correct

Mark 1.00 out of 1.00

Median nerve originates from the

A. axillary nerve

B. musculocutaneous nerve

C. medial and lateral cords of the brachial plexus 

D. radial nerve

E. ulnar nerve

Question 16
Correct

Mark 1.00 out of 1.00

Which muscles are innervated by n. dorsalis scapulae?

1. m. coracobrachialis

2. m. pectoralis major

3. m. latissimus dorsi

4. m. levator scapulae 

5. mm. rhomboidei 

Question 17
Correct

Mark 1.00 out of 1.00

To the branches of the ophthalmic nerve belong:

1. n. petrosus major

2. n. nasociliaris 

3. n. auriculotemporalis

4. n. lacrimalis 

5. n. infraorbitalis

6. n. frontalis 

https://ets.amu.edu.az/mod/quiz/review.php?attempt=68698&cmid=7722 6/8
4/21/2021 Midterm exam: İnsan anatomiyası və tibbi terminologiya - 2 (EN) (MPF2): Attempt review

Question 18
Correct

Mark 1.00 out of 1.00

Find the accordance:

A. interossea communis is the branch of ulnar artery 

A. thoracoacromialis is the branch of axillary artery 

Question 19
Correct

Mark 1.00 out of 1.00

The ulnar artery departs from the [[1]] artery at the level of the [[2]] of the ulna.

[[1]] brachial 

[[2]] coronoid process 

Question 20
Correct

Mark 1.00 out of 1.00

Find n. vagus:

Answer: 11 

◄ Attendance

Jump to...

A. Abdullayev. 7. General overview of the peripheral nervous system. The classification, development, variation, and anomalies of the
cranial nerves. ►

https://ets.amu.edu.az/mod/quiz/review.php?attempt=68698&cmid=7722 7/8
4/21/2021 Midterm exam: İnsan anatomiyası və tibbi terminologiya - 2 (EN) (MPF2): Attempt review

https://ets.amu.edu.az/mod/quiz/review.php?attempt=68698&cmid=7722 8/8
4/20/2021 Midterm exam: Human anatomy and medical terminology - 2 (EN) (MPF2): Attempt review

My main page / My subjects / Human Anatomy and Medical Terminology - 2 (EN) / General
/ Midterm exam: Human anatomy and medical terminology - 2 (EN) (MPF2)

The test began Tuesday, 20 April 2021, 12 30 PM


State Finished
It's over Tuesday, 20 April 2021, 12 57 PM
Time has passed 26 minutes 56 sec.
Price 20 ( 95 %) maximum 19
Question 1
Correct
Mark 1.00 out of 1.00

In the posterior funiculus of the spinal cord are located:


A. tectospinalis tract, vestibulospinalis tract
B. rubrospinalis tract, spinocerebellaris dorsal tract
C. fasciculus gracilis, tectospinalis tract
D. vestibulospinalis tract, ventricular spinocerebellaris tract
E.. fasciculus cuneatus, fasciculus gracilis 

Question 2
Correct
Mark 1.00 out of 1.00

Which of the following gyruses does not belong to the temporal lobe?
A. temporalis media
B. parahyppocampalis
C. inferior temporalis
D. superior temporalis
E.. supramarginal 

https://ets.amu.edu.az/mod/quiz/review.php?attempt=68704&cmid=7722 1/7
4/20/2021 Midterm exam: Human anatomy and medical terminology - 2 (EN) (MPF2): Attempt review

Question 3
Correct
Mark 1.00 out of 1.00

The anterior limb of the internal capsule is situated between:


A. putamen and claustrum
B. lentiform nucleus and caudate nucleus 

C. thalamus and lentiform nucleus


D. claustrum and cortex of insula
E.. globi pallidus medialis et lateralis

Question 4
Correct
Mark 1.00 out of 1.00

It does not belong to the epithalamus:


A. epithalamic commissure
B. epiphysis
C. hypophysis 

D. habenular commissure
E.. habenular trigone

Question 5
Correct
Mark 1.00 out of 1.00

The roof of the fourth ventricle in front consists of:


I. Superior cerebellar peduncles
II. Middle cerebellar peduncles
III. Inferior cerebellar peduncles
IV. Velum medullare superius
V. Velum medullare inferius
A. III; V
B. I; IV 

C. II; III
D. I; II
E.. III; IV

https://ets.amu.edu.az/mod/quiz/review.php?attempt=68704&cmid=7722 2/7
4/20/2021 Midterm exam: Human anatomy and medical terminology - 2 (EN) (MPF2): Attempt review

Question 6
Correct
Mark 1.00 out of 1.00

The lower part of the rhomboid fossa is corresponded to:


A. diencephalon
B. metencephalon
C. telencephalon
D. medulla oblongata 

E.. mesencephalon

Question 7
Correct
Mark 1.00 out of 1.00

Where does the optic nerve enter the cranial cavity from?
A. foramen zygomatico-orbitale
B. foramen ethmoidale posterior
C. canalis opticus 

D. foramen ovale
E.. foramen ethmoidae anterior

Question 8
Correct
Mark 1.00 out of 1.00

Which cranial nerve provides the human with the ability to smell a wide variety of odors?
A. intermedius nerve
B. optic nerve
C. oculomotor nerve
D. olfactory nerve 

E.. vestibulocochlear nerve

https://ets.amu.edu.az/mod/quiz/review.php?attempt=68704&cmid=7722 3/7
4/20/2021 Midterm exam: Human anatomy and medical terminology - 2 (EN) (MPF2): Attempt review

Question 9
Correct
Mark 1.00 out of 1.00

It does not belong to the branches of the ophthalmic nerve:


IN infaorbitalis
II. N. lacrimalis
III. Rr. pterygopalatini
IV. N. nasociliaris
VN frontalis
VI. N. zygomaticus
A. III; IV; VI
B. I; III; V
C. II; IV; V
D. I; III; VI 

E.. II; III; IV

Question 10
Correct
Mark 1.00 out of 1.00

Which layers of the serous pericardium?


A. diaphragmaric, visceral
B. mediastinal, costal
C. parietal, visceral 

D. parietal, costal
E.. costal, parietal

Question 11
Correct
Mark 1.00 out of 1.00

The ascending aorta passes to the aortic arch at the level of the:
A. left second sternocostal joint
B. left fourth sternocostal joint
C. left third sternocostal joint
D. right second sternocostal joint 

E.. right third sternocostal joint

https://ets.amu.edu.az/mod/quiz/review.php?attempt=68704&cmid=7722 4/7
4/20/2021 Midterm exam: Human anatomy and medical terminology - 2 (EN) (MPF2): Attempt review

Question 12
Incorrect
Mark 0.00 out of 1.00

The vertebral artery (a. Vertebralis), is the first branch of the


A. subclavian artery
B. superficial cervical artery
C. subclavian artery
D. inferior thyroid artery
E.. suprascapular artery

Question 13
Correct
Mark 1.00 out of 1.00

To which vein does v.basilica open?


A. brachial 

B. anterior jugular
C. subclavian
D. axillary
E.. external jugular

Question 14
Correct
Mark 1.00 out of 1.00

The vein which is composed by the connecting occipital and posterior auricular veins:
A. internal jugular vein
B. brachiocephalic vein
C. superior venacava
D. external jugular vein 

E.. anterior jugular vein

https://ets.amu.edu.az/mod/quiz/review.php?attempt=68704&cmid=7722 5/7
4/20/2021 Midterm exam: Human anatomy and medical terminology - 2 (EN) (MPF2): Attempt review

Question 15
Correct
Mark 1.00 out of 1.00

The azygos vein opens into the vena cava superior at the level of:
A. Th9
B. Th7
C. Th3
D. Th4
E.. Th5 

Question 16
Correct
Mark 1.00 out of 1.00

Which muscles are innervated by n. dorsal scapulae?


1. m. levator scapulae 

2. mm. rhomboidei 

3. m. pectoralis major
4. m. latissimus dorsi
5. m. coracobrachialis

Question 17
Correct
Mark 1.00 out of 1.00

To the nuclei of the medulla oblongata belong:


1. nucleus caudatus
2. nucleus olivaris 

3. nucleus gracilis 

4. nucleus ruber
5. nucleus cuneatus 

6. nucleus dentatus

https://ets.amu.edu.az/mod/quiz/review.php?attempt=68704&cmid=7722 6/7
4/20/2021 Midterm exam: Human anatomy and medical terminology - 2 (EN) (MPF2): Attempt review

Question 18
Correct
Mark 1.00 out of 1.00

Find the according:


The superior laryngeal nerve is the branch of vagus nerve 

The tympanic nerve is the branch of glossopharyngeal nerve 

Question 19
Correct
Mark 1.00 out of 1.00

At the level of the lateral edge of the [[1]] rib, the axillary vein continues into the [[2]] vein.
[[1]] first 

[[2]] subclavian 

Question 20
Correct
Mark 1.00 out of 1.00

Find the pons:

Answer: 11 

◀ Attendance
Go ...
A. Abdullayev. 7. General overview of the peripheral nervous system. The classification, development, variation, and anomalies of the
cranial nerves. ▶

https://ets.amu.edu.az/mod/quiz/review.php?attempt=68704&cmid=7722 7/7
4/20/2021 Midterm exam: İnsan anatomiyası və tibbi terminologiya - 2 (EN) (MPF2): Attempt review

Dashboard / My courses / İnsan anatomiyası və tibbi terminologiya - 2 (EN) / General

/ Midterm exam: İnsan anatomiyası və tibbi terminologiya - 2 (EN) (MPF2)

Started on Tuesday, 20 April 2021, 12:31 PM


State Finished
Completed on Tuesday, 20 April 2021, 12:59 PM
Time taken 28 mins 42 secs
Grade 17 out of 20 (83%)

Question 1
Incorrect

Mark 0.00 out of 1.00

On the midline of the anterior surface of the spinal cord is:

A. dorsal (posterior) median sulcus. 

B. ventrolateral grooves

C. dorsolateral grooves

D. fissura orbitalis superior

E. ventral median fissure

Question 2
Correct

Mark 1.00 out of 1.00

Temporal lobe is separated from the frontal and parietal lobes by the:

A. transversus occipital sulcus

B. lateral sulcus 

C. parieto-occipital sulcus

D. central sulcus

E. postcentral sulcus

https://ets.amu.edu.az/mod/quiz/review.php?attempt=69157&cmid=7722#question-99121-5 1/8
4/20/2021 Midterm exam: İnsan anatomiyası və tibbi terminologiya - 2 (EN) (MPF2): Attempt review

Question 3
Correct

Mark 1.00 out of 1.00

The extremal capsule is the white matter separating:

A. claustrum from cortex of insula 

B. globus pallidus lateralis and putamen

C. globi pallidus medialis et lateralis

D. globus pallidus medialis and nucleus caudatus

E. putamen and claustrum

Question 4
Correct

Mark 1.00 out of 1.00

Midbrain in Latin is called:

A. telencephalon

B. diencephalon

C. myelencephalon

D. mesencephalon 

E. metencephalon

Question 5
Incorrect

Mark 0.00 out of 1.00

The trigonum lemnisci is bounded medially by:

A. inferior cerebellar peduncle

B. brachium colliculi superioris

C. brachium colliculi inferioris 

D. superior cerebellar peduncle

E. cerebral peduncle

https://ets.amu.edu.az/mod/quiz/review.php?attempt=69157&cmid=7722#question-99121-5 2/8
4/20/2021 Midterm exam: İnsan anatomiyası və tibbi terminologiya - 2 (EN) (MPF2): Attempt review

Question 6
Correct

Mark 1.00 out of 1.00

The olive separated from the pyramid by the:

A. anterolateral sulcus 

B. posterior median sulcus

C. intermediate sulcus

D. posterior funiculus

E. posterolateral sulcus

Question 7
Correct

Mark 1.00 out of 1.00

Which cranial nerve receives sound wave stimuli from the small bones of the middle ear, inner ear and interpret this in the brain as hearing
and also control human balance?

A. vestibulocochlear nerve 

B. facial nerve

C. glossopharyngeal nerve

D. intermedius nerve

E. olfactory nerve

Question 8
Correct

Mark 1.00 out of 1.00

It arises from the facial nerve just above the stylomastoid foramen:

A. branch to the tympanic plexus

B. lesser petrosal nerve

C. the nerve to the stapedius

D. greater petrosal nerve

E. chorda tympani 

https://ets.amu.edu.az/mod/quiz/review.php?attempt=69157&cmid=7722#question-99121-5 3/8
4/20/2021 Midterm exam: İnsan anatomiyası və tibbi terminologiya - 2 (EN) (MPF2): Attempt review

Question 9
Correct

Mark 1.00 out of 1.00

The vagus nerve passes out of the skull through:

A. foramen juguluare 

B. foramen lacerum

C. foramen rotundum

D. foramen ovale

E. foramen spinosum

Question 10
Correct

Mark 1.00 out of 1.00

Which layers of the serous pericardium?

A. parietal, costal

B. mediastinal, costal

C. diaphragmaric,visceral

D. costal,parietal

E. parietal, visceral 

Question 11
Correct

Mark 1.00 out of 1.00

Where is located common carotid artery bifurcation?

A. superior border of the thyroid cartilage 

B. V cervical vertebra

C. inferior border of the thyroid cartilage

D. posterior border of the palate

E. root of the tongue

https://ets.amu.edu.az/mod/quiz/review.php?attempt=69157&cmid=7722#question-99121-5 4/8
4/20/2021 Midterm exam: İnsan anatomiyası və tibbi terminologiya - 2 (EN) (MPF2): Attempt review

Question 12
Correct

Mark 1.00 out of 1.00

The vertebral artery (a. vertebralis), is the first branch of the

A. subclavian artery

B. superficial cervical artery

C. inferior thyroid artery

D. suprascapular artery

E. subclavian artery 

Question 13
Correct

Mark 1.00 out of 1.00

To which vein does v.cephalica vein open?

A. anterior jugular

B. axillary 

C. external jugular

D. subclavian

E. brachial

Question 14
Correct

Mark 1.00 out of 1.00

The internal jugular vein is a continuation of:

A. inferior petrosal sinus

B. sigmoid sinus 

C. transverse sinus

D. superior petrosal sinus

E. superior sagittal sinus

https://ets.amu.edu.az/mod/quiz/review.php?attempt=69157&cmid=7722#question-99121-5 5/8
4/20/2021 Midterm exam: İnsan anatomiyası və tibbi terminologiya - 2 (EN) (MPF2): Attempt review

Question 15
Correct

Mark 1.00 out of 1.00

The brachial plexus is formed by the

A. two trunks

B. six trunks

C. three trunks 

D. five trunks

E. four trunks

Question 16
Partially correct

Mark 0.67 out of 1.00

The anterior wall of the third ventricle is formed by:

1. epithalamic commissure

2. anterior commissure

3. terminal lamina 

4. thalamus

5. pineal body

6. columns of the fornix 

Question 17
Incorrect

Mark 0.00 out of 1.00

To the nuclei of the medulla oblongata belong:

1. nucleus olivaris 

2. nucleus gracilis

3. nucleus caudatus

4. nucleus ruber 

5. nucleus cuneatus

6. nucleus dentatus

https://ets.amu.edu.az/mod/quiz/review.php?attempt=69157&cmid=7722#question-99121-5 6/8
4/20/2021 Midterm exam: İnsan anatomiyası və tibbi terminologiya - 2 (EN) (MPF2): Attempt review

Question 18
Correct

Mark 1.00 out of 1.00

Find the accordance:

The superior laryngeal nerve is the branch of vagus nerve 

The tympanic nerve is the branch of glossopharyngeal nerve 

Question 19
Correct

Mark 1.00 out of 1.00

At the level of the lateral edge of the [[1]] rib, the axillary vein continues into the [[2]] vein.

[[1]] first 

[[2]] subclavian 

Question 20
Correct

Mark 1.00 out of 1.00

Find n. vagus:

Answer: 11 

◄ Attendance

Jump to...

A. Abdullayev. 7. General overview of the peripheral nervous system. The classification, development, variation, and anomalies of the
cranial nerves. ►

https://ets.amu.edu.az/mod/quiz/review.php?attempt=69157&cmid=7722#question-99121-5 7/8
4/20/2021 Midterm exam: İnsan anatomiyası və tibbi terminologiya - 2 (EN) (MPF2): Attempt review

https://ets.amu.edu.az/mod/quiz/review.php?attempt=69157&cmid=7722#question-99121-5 8/8
Mənim əsas səhifəm / Mənim fənlərim / İnsan anatomiyası və tibbi terminologiya - 2 (EN) / Ümumi
/ Midterm exam: İnsan anatomiyası və tibbi terminologiya - 2 (EN) (MPF2)

Testə başlandı Thursday, 22 April 2021, 12:30 PM


State Finished
Sona çatdı Thursday, 22 April 2021, 12:58 PM
Vaxt keçib 28 dəq. 23 san.
Qiymət 20 (100%) maksimumdan 20

Question 1 The number of the segments of the spinal cord:


Correct

Mark 1.00 out of A. 27


1.00
B. 28

C. 31 

D. 29

E. 33

Question 2 It happens when the motor analyzer of speech articulation is damaged:


Correct

Mark 1.00 out of A. agraphia


1.00
B. sensory aphasia

C. apraxia

D. tactile agnosia

E. motor aphasia 

Question 3 The upper wall or roof of the posterior horns of the lateral ventricle is formed by the lower surfaces of:
Correct

Mark 1.00 out of A. indusium griseum


1.00
B. forceps minor

C. tapetum 

D. commissura posterior

E. commisura anterior

Question 4 Which wall of III ventricle does the medial surface of thalamus form?
Correct

Mark 1.00 out of A. superior


1.00
B. anterior

C. posterior

D. inferior

E. lateral 

Question 5 The trigonum lemnisci is bounded medially by:


Correct

Mark 1.00 out of A. brachium colliculi inferioris


1.00
B. superior cerebellar peduncle 

C. inferior cerebellar peduncle

D. brachium colliculi superioris

E. cerebral peduncle

Question 6 In medulla oblongata is not located:


Correct

Mark 1.00 out of A. center of swallowing


1.00
B. highest center of vegetative nervous system 

C. center of vomiting

D. respiratory center

E. circulatory center

Question 7 It doesn’t belong to the bony labyrinth:


Correct

Mark 1.00 out of A. vestibulum


1.00
B. canalis semicircularis anterior

C. utriculus 

D. cochlea

E. canalis semicircularis posterior

Question 8 Into how many parts does the facial nerve divide according to the topography :
Correct

Mark 1.00 out of A. 4


1.00
B. 6

C. 3

D. 2 

E. 5

Question 9 Which muscles are innervated by the nervus accesorius?


Correct I. M. sternocleidomastoideus
Mark 1.00 out of II. M. trapezius
1.00 III. M. digastricus
IV. M. latissimus dorsi
V. M. buccinator

A. III,IV

B. IV,V

C. I,II 

D. III,V

E. II,III

Question 10 Where is located the superior border of the heart?


Correct

Mark 1.00 out of A. line connecting upper edges of VII ribs


1.00
B. line connecting upper edges the IV ribs

C. line connecting lower edges of the V ribs

D. line connecting lower edges of VI ribs

E. line connecting upper edges of III ribs 

Question 11 The first branch of the internal carotid artery is the following:
Correct

Mark 1.00 out of A. a. cerebri media


1.00
B. rr. caroticotympanici 

C. a. communicans posterior

D. a. ophthalmica

E. a. cerebri anterior

Question 12 The branches of the brachiocephalic trunk:


Correct I. A. subclavia dextra
Mark 1.00 out of II. A. carotis communis sinistra
1.00 III. A. carotis communis dextra
IV. A. subclavia sinistra
V. A. thyroidea ima

A. III;IV

B. I;III 

C. II;III

D. IV;V

E. I;II

Question 13 It is not the branch of the ulnar artery:


Correct

Mark 1.00 out of A. а. recurrens ulnaris


1.00
B. а. recurrens radialis 

C. а. carpalis dorsalis

D. а. interossea communis

E. а. carpalis palmaris

Question 14 The internal jugular vein is a continuation of:


Correct

Mark 1.00 out of A. sigmoid sinus 


1.00
B. superior sagittal sinus

C. inferior petrosal sinus

D. superior petrosal sinus

E. transverse sinus

Question 15 The azygos vein opens into the vena cava superior at the level of:
C t
Correct

Mark 1.00 out of A. Th4


1.00
B. Th7

C. Th9

D. Th5 

E. Th3

Question 16 Which muscles are innervated by n. dorsalis scapulae?


Correct

Mark 1.00 out of 1. m. latissimus dorsi


1.00
2. m. levator scapulae 

3. m. coracobrachialis

4. mm. rhomboidei 

5. m. pectoralis major

Question 17 To the branches of the ophthalmic nerve belong:


Correct

Mark 1.00 out of 1. n. frontalis 


1.00
2. n. nasociliaris 

3. n. infraorbitalis

4. n. auriculotemporalis

5. n. petrosus major

6. n. lacrimalis 

Question 18 Find the accordance:


Correct

Mark 1.00 out of


The external ear includes auricle and external acoustic meatus 
1.00

The inner ear consists of bony labyrinth and membranous labyrinth 

Question 19 The trochlear nerve enters the orbit through the [[1]] and innervates the [[2]].
Correct

Mark 1.00 out of


[[1]] superior orbital fissure 
1.00

[[2]] superior oblique muscle 

Question 20 Find the ulnar nerve:


Correct

Mark 1.00 out of


1.00
Answer: 7 

A. Abdullayev. 7. General overview of


the peripheral nervous system. The
◄ Attendance Keç ... classification, development, variation,
and anomalies of the cranial nerves.

4/20/2021 Midterm exam: İnsan anatomiyası və tibbi terminologiya - 2 (EN) (MPF2): Attempt review

Mənim əsas səhifəm / Mənim fənlərim / İnsan anatomiyası və tibbi terminologiya - 2 (EN) / Ümumi

/ Midterm exam: İnsan anatomiyası və tibbi terminologiya - 2 (EN) (MPF2)

Testə başlandı Tuesday, 20 April 2021, 12:30 PM


State Finished
Sona çatdı Tuesday, 20 April 2021, 12:56 PM
Vaxt keçib 26 dəq. 15 san.
Qiymət 20 (98%) maksimumdan 20

Question 1
Correct

Mark 1.00 out of 1.00

Vermis is belonged to the:

A. pons

B. midbrain

C. cerebellum 

D. medulla oblongata

E. caudate nucleus

Question 2
Correct

Mark 1.00 out of 1.00

They are situated on the inferior surface of the temporal lobe:


I. Sulcus occipitotemporalis
II. Gyrus temporalis medius
III. Gyri temporales transversi
IV. Sulcus collateralis

A. I; II

B. III; IV

C. II; III

D. II; IV

E. I; IV 

https://ets.amu.edu.az/mod/quiz/review.php?attempt=68726&cmid=7722#question-98698-17 1/7
4/20/2021 Midterm exam: İnsan anatomiyası və tibbi terminologiya - 2 (EN) (MPF2): Attempt review

Question 3
Correct

Mark 1.00 out of 1.00

The extremal capsule is the white matter separating:

A. claustrum from cortex of insula 

B. globi pallidus medialis et lateralis

C. putamen and claustrum

D. globus pallidus lateralis and putamen

E. globus pallidus medialis and nucleus caudatus

Question 4
Correct

Mark 1.00 out of 1.00

Where is the gyrus parahippocampalis situated?

A. in the posterior part of the gyrus temporalis inferior

B. between the hippocampal and collateral sulcus 

C. between the superior and inferior temporal sulcus

D. between the inferior temporal and occipitotemporalis sulcus

E. between the lateral and superior temporal sulcus

Question 5
Correct

Mark 1.00 out of 1.00

Where is the vallecula cerebelli located?

A. on the posterior border of cerebellum

B. on the superior surface of cerebellum

C. on the anterior border of cerebellum

D. on the inferior surface of cerebellum 

E. on the anterior and posterior borders of cerebellum

https://ets.amu.edu.az/mod/quiz/review.php?attempt=68726&cmid=7722#question-98698-17 2/7
4/20/2021 Midterm exam: İnsan anatomiyası və tibbi terminologiya - 2 (EN) (MPF2): Attempt review

Question 6
Correct

Mark 1.00 out of 1.00

What extends from the superior to the inferior angle of the rhomboid fossa?

A. sulcus medianus 

B. sulcus limitans

C. colliculus facialis

D. eminentia medialis

E. striae medullares

Question 7
Correct

Mark 1.00 out of 1.00

Where does the optic nerve enter the cranial cavity from?

A. foramen zygomatico-orbitale

B. foramen ovale

C. foramen ethmoidae anterior

D. foramen ethmoidale posterior

E. canalis opticus 

Question 8
Correct

Mark 1.00 out of 1.00

Which cranial nerve provides the human with the ability to smell a wide variety of odors?

A. optic nerve

B. olfactory nerve 

C. intermedius nerve

D. oculomotor nerve

E. vestibulocochlear nerve

https://ets.amu.edu.az/mod/quiz/review.php?attempt=68726&cmid=7722#question-98698-17 3/7
4/20/2021 Midterm exam: İnsan anatomiyası və tibbi terminologiya - 2 (EN) (MPF2): Attempt review

Question 9
Correct

Mark 1.00 out of 1.00

It does not belong to the branches of the cervical part of the vagus nerve:

A. rr. cardiaci cervicales superiores

B. pharyngeal branches

C. recurrent laryngeal nerve

D. auricular branch 

E. superior laryngeal nerve

Question 10
Correct

Mark 1.00 out of 1.00

How is the called right atrio-ventricular valve?

A. tricuspidalis 

B. atrioventricularis sinistra

C. mitralis

D. bicuspidalis

E. seminularis

Question 11
Correct

Mark 1.00 out of 1.00

The ascending aorta passes to the aortic arch at the level of the:

A. right third sternocostal joint

B. left fourth sternocostal joint

C. left third sternocostal joint

D. right second sternocostal joint 

E. left second sternocostal joint

https://ets.amu.edu.az/mod/quiz/review.php?attempt=68726&cmid=7722#question-98698-17 4/7
4/20/2021 Midterm exam: İnsan anatomiyası və tibbi terminologiya - 2 (EN) (MPF2): Attempt review

Question 12
Correct

Mark 1.00 out of 1.00

It doesn’t belong to the branches of thyrocervical trunk:

A. subscapular artery 

B. inferior thyroid artery

C. suprascapular artery

D. ascending cervical artery

E. superficial cervical artery

Question 13
Correct

Mark 1.00 out of 1.00

To which vein does v.basilica open?

A. subclavian

B. axillary

C. brachial 

D. anterior jugular

E. external jugular

Question 14
Correct

Mark 1.00 out of 1.00

It does not belong to the tributaries of the external jugular vein:

A. v. suprascapularis

B. v. canalis pterygoidei 

C. v. auricularis posterior

D. v. occipitalis

E. v. jugularis anterior

https://ets.amu.edu.az/mod/quiz/review.php?attempt=68726&cmid=7722#question-98698-17 5/7
4/20/2021 Midterm exam: İnsan anatomiyası və tibbi terminologiya - 2 (EN) (MPF2): Attempt review

Question 15
Correct

Mark 1.00 out of 1.00

The visceral branches of the thoracic aorta are the followings:


I. Rr. bronchiales
II. Rr. mediastinales
III. Superior phrenic arteries
IV. Posterior intercostalis arteries
V. Rr. pericardiaci

A. II,III,IV

B. I,II,IV

C. III,IV,V

D. I,II,V 

E. II,III,V

Question 16
Correct

Mark 1.00 out of 1.00

The anterior wall of the third ventricle is formed by:

1. terminal lamina 

2. anterior commissure 

3. columns of the fornix 

4. pineal body

5. thalamus

6. epithalamic commissure

Question 17
Partially correct

Mark 0.67 out of 1.00

To the nuclei of the medulla oblongata belong:

1. nucleus caudatus

2. nucleus olivaris 

3. nucleus dentatus

4. nucleus ruber

5. nucleus gracilis 

6. nucleus cuneatus

https://ets.amu.edu.az/mod/quiz/review.php?attempt=68726&cmid=7722#question-98698-17 6/7
4/20/2021 Midterm exam: İnsan anatomiyası və tibbi terminologiya - 2 (EN) (MPF2): Attempt review

Question 18
Correct

Mark 1.00 out of 1.00

Find the accordance:

A. interossea communis is the branch of ulnar artery 

A. thoracoacromialis is the branch of axillary artery 

Question 19
Correct

Mark 1.00 out of 1.00

At the level of the lateral edge of the [[1]] rib, the axillary vein continues into the [[2]] vein.

[[1]] first 

[[2]] subclavian 

Question 20
Correct

Mark 1.00 out of 1.00

Find the ulnar nerve:

Answer: 7 

◄ Attendance

Keç ...

A. Abdullayev. 7. General overview of the peripheral nervous system. The classification, development, variation, and anomalies of the cranial
nerves. ►

https://ets.amu.edu.az/mod/quiz/review.php?attempt=68726&cmid=7722#question-98698-17 7/7
4/20/2021 Midterm exam: İnsan anatomiyası və tibbi terminologiya - 2 (EN) (MPF2): Attempt review

Dashboard / My courses / İnsan anatomiyası və tibbi terminologiya - 2 (EN) / General

/ Midterm exam: İnsan anatomiyası və tibbi terminologiya - 2 (EN) (MPF2)

Started on Tuesday, 20 April 2021, 12:31 PM


State Finished
Completed on Tuesday, 20 April 2021, 12:59 PM
Time taken 28 mins 42 secs
Grade 17 out of 20 (83%)

Question 1
Incorrect

Mark 0.00 out of 1.00

On the midline of the anterior surface of the spinal cord is:

A. dorsal (posterior) median sulcus. 

B. ventrolateral grooves

C. dorsolateral grooves

D. fissura orbitalis superior

E. ventral median fissure

Question 2
Correct

Mark 1.00 out of 1.00

Temporal lobe is separated from the frontal and parietal lobes by the:

A. transversus occipital sulcus

B. lateral sulcus 

C. parieto-occipital sulcus

D. central sulcus

E. postcentral sulcus

https://ets.amu.edu.az/mod/quiz/review.php?attempt=69157&cmid=7722#question-99121-5 1/8
4/20/2021 Midterm exam: İnsan anatomiyası və tibbi terminologiya - 2 (EN) (MPF2): Attempt review

Question 3
Correct

Mark 1.00 out of 1.00

The extremal capsule is the white matter separating:

A. claustrum from cortex of insula 

B. globus pallidus lateralis and putamen

C. globi pallidus medialis et lateralis

D. globus pallidus medialis and nucleus caudatus

E. putamen and claustrum

Question 4
Correct

Mark 1.00 out of 1.00

Midbrain in Latin is called:

A. telencephalon

B. diencephalon

C. myelencephalon

D. mesencephalon 

E. metencephalon

Question 5
Incorrect

Mark 0.00 out of 1.00

The trigonum lemnisci is bounded medially by:

A. inferior cerebellar peduncle

B. brachium colliculi superioris

C. brachium colliculi inferioris 

D. superior cerebellar peduncle

E. cerebral peduncle

https://ets.amu.edu.az/mod/quiz/review.php?attempt=69157&cmid=7722#question-99121-5 2/8
4/20/2021 Midterm exam: İnsan anatomiyası və tibbi terminologiya - 2 (EN) (MPF2): Attempt review

Question 6
Correct

Mark 1.00 out of 1.00

The olive separated from the pyramid by the:

A. anterolateral sulcus 

B. posterior median sulcus

C. intermediate sulcus

D. posterior funiculus

E. posterolateral sulcus

Question 7
Correct

Mark 1.00 out of 1.00

Which cranial nerve receives sound wave stimuli from the small bones of the middle ear, inner ear and interpret this in the brain as hearing
and also control human balance?

A. vestibulocochlear nerve 

B. facial nerve

C. glossopharyngeal nerve

D. intermedius nerve

E. olfactory nerve

Question 8
Correct

Mark 1.00 out of 1.00

It arises from the facial nerve just above the stylomastoid foramen:

A. branch to the tympanic plexus

B. lesser petrosal nerve

C. the nerve to the stapedius

D. greater petrosal nerve

E. chorda tympani 

https://ets.amu.edu.az/mod/quiz/review.php?attempt=69157&cmid=7722#question-99121-5 3/8
4/20/2021 Midterm exam: İnsan anatomiyası və tibbi terminologiya - 2 (EN) (MPF2): Attempt review

Question 9
Correct

Mark 1.00 out of 1.00

The vagus nerve passes out of the skull through:

A. foramen juguluare 

B. foramen lacerum

C. foramen rotundum

D. foramen ovale

E. foramen spinosum

Question 10
Correct

Mark 1.00 out of 1.00

Which layers of the serous pericardium?

A. parietal, costal

B. mediastinal, costal

C. diaphragmaric,visceral

D. costal,parietal

E. parietal, visceral 

Question 11
Correct

Mark 1.00 out of 1.00

Where is located common carotid artery bifurcation?

A. superior border of the thyroid cartilage 

B. V cervical vertebra

C. inferior border of the thyroid cartilage

D. posterior border of the palate

E. root of the tongue

https://ets.amu.edu.az/mod/quiz/review.php?attempt=69157&cmid=7722#question-99121-5 4/8
4/20/2021 Midterm exam: İnsan anatomiyası və tibbi terminologiya - 2 (EN) (MPF2): Attempt review

Question 12
Correct

Mark 1.00 out of 1.00

The vertebral artery (a. vertebralis), is the first branch of the

A. subclavian artery

B. superficial cervical artery

C. inferior thyroid artery

D. suprascapular artery

E. subclavian artery 

Question 13
Correct

Mark 1.00 out of 1.00

To which vein does v.cephalica vein open?

A. anterior jugular

B. axillary 

C. external jugular

D. subclavian

E. brachial

Question 14
Correct

Mark 1.00 out of 1.00

The internal jugular vein is a continuation of:

A. inferior petrosal sinus

B. sigmoid sinus 

C. transverse sinus

D. superior petrosal sinus

E. superior sagittal sinus

https://ets.amu.edu.az/mod/quiz/review.php?attempt=69157&cmid=7722#question-99121-5 5/8
4/20/2021 Midterm exam: İnsan anatomiyası və tibbi terminologiya - 2 (EN) (MPF2): Attempt review

Question 15
Correct

Mark 1.00 out of 1.00

The brachial plexus is formed by the

A. two trunks

B. six trunks

C. three trunks 

D. five trunks

E. four trunks

Question 16
Partially correct

Mark 0.67 out of 1.00

The anterior wall of the third ventricle is formed by:

1. epithalamic commissure

2. anterior commissure

3. terminal lamina 

4. thalamus

5. pineal body

6. columns of the fornix 

Question 17
Incorrect

Mark 0.00 out of 1.00

To the nuclei of the medulla oblongata belong:

1. nucleus olivaris 

2. nucleus gracilis

3. nucleus caudatus

4. nucleus ruber 

5. nucleus cuneatus

6. nucleus dentatus

https://ets.amu.edu.az/mod/quiz/review.php?attempt=69157&cmid=7722#question-99121-5 6/8
4/20/2021 Midterm exam: İnsan anatomiyası və tibbi terminologiya - 2 (EN) (MPF2): Attempt review

Question 18
Correct

Mark 1.00 out of 1.00

Find the accordance:

The superior laryngeal nerve is the branch of vagus nerve 

The tympanic nerve is the branch of glossopharyngeal nerve 

Question 19
Correct

Mark 1.00 out of 1.00

At the level of the lateral edge of the [[1]] rib, the axillary vein continues into the [[2]] vein.

[[1]] first 

[[2]] subclavian 

Question 20
Correct

Mark 1.00 out of 1.00

Find n. vagus:

Answer: 11 

◄ Attendance

Jump to...

A. Abdullayev. 7. General overview of the peripheral nervous system. The classification, development, variation, and anomalies of the
cranial nerves. ►

https://ets.amu.edu.az/mod/quiz/review.php?attempt=69157&cmid=7722#question-99121-5 7/8
4/20/2021 Midterm exam: İnsan anatomiyası və tibbi terminologiya - 2 (EN) (MPF2): Attempt review

https://ets.amu.edu.az/mod/quiz/review.php?attempt=69157&cmid=7722#question-99121-5 8/8
4/20/2021 Midterm exam: İnsan anatomiyası və tibbi terminologiya - 2 (EN) (MPF2): Attempt review

Dashboard / My courses / İnsan anatomiyası və tibbi terminologiya - 2 (EN) / General

/ Midterm exam: İnsan anatomiyası və tibbi terminologiya - 2 (EN) (MPF2)

Started on Tuesday, 20 April 2021, 12:30 PM


State Finished
Completed on Tuesday, 20 April 2021, 12:56 PM
Time taken 25 mins 47 secs
Grade 17 out of 20 (85%)

Question 1
Correct

Mark 1.00 out of 1.00

The length of the spinal cord in female is (cm) :

A. 35

B. 32

C. 50

D. 43 

E. 46

Question 2
Correct

Mark 1.00 out of 1.00

The basilar groove is belonged to:

A. midbrain

B. pons 

C. diencephalon

D. lamina terminalis

E. medulla oblongata

https://ets.amu.edu.az/mod/quiz/review.php?attempt=69006&cmid=7722 1/8
4/20/2021 Midterm exam: İnsan anatomiyası və tibbi terminologiya - 2 (EN) (MPF2): Attempt review

Question 3
Correct

Mark 1.00 out of 1.00

The floor of the central part of the lateral ventricle is formed by:
I. Body of caudate nucleus
II. Head of caudate nucleus
III. Thalamus
IV. Corpus fornicis
V. Stria terminalis

A. I; II; III

B. II; III; V

C. III; IV; V

D. I; III; V 

E. I; III; IV

Question 4
Correct

Mark 1.00 out of 1.00

Where is the gyrus parahippocampalis situated?

A. between the superior and inferior temporal sulcus

B. between the lateral and superior temporal sulcus

C. between the hippocampal and collateral sulcus 

D. in the posterior part of the gyrus temporalis inferior

E. between the inferior temporal and occipitotemporalis sulcus

Question 5
Correct

Mark 1.00 out of 1.00

In region of the lateral angles of the rhomboid fossa lays:

A. locus caeruleus

B. eminentia medialis

C. nucleus dorsalis nervi vagi

D. area vestibularis 

E. nucleus ruber

https://ets.amu.edu.az/mod/quiz/review.php?attempt=69006&cmid=7722 2/8
4/20/2021 Midterm exam: İnsan anatomiyası və tibbi terminologiya - 2 (EN) (MPF2): Attempt review

Question 6
Correct

Mark 1.00 out of 1.00

What extends from the superior to the inferior angle of the rhomboid fossa?

A. eminentia medialis

B. striae medullares

C. sulcus medianus 

D. colliculus facialis

E. sulcus limitans

Question 7
Correct

Mark 1.00 out of 1.00

The nucleus of the olfactory analyzer lies in the:

A. medial surface of the occipital lobe

B. superior parietal lobule

C. uncus of the parahippocampal gyrus 

D. middle temporal gyrus

E. superior temporal gyrus

Question 8
Correct

Mark 1.00 out of 1.00

It arises from the facial nerve just above the stylomastoid foramen:

A. lesser petrosal nerve

B. branch to the tympanic plexus

C. the nerve to the stapedius

D. chorda tympani 

E. greater petrosal nerve

https://ets.amu.edu.az/mod/quiz/review.php?attempt=69006&cmid=7722 3/8
4/20/2021 Midterm exam: İnsan anatomiyası və tibbi terminologiya - 2 (EN) (MPF2): Attempt review

Question 9
Correct

Mark 1.00 out of 1.00

It does not belong to the branches of the cervical part of the vagus nerve:

A. pharyngeal branches

B. superior laryngeal nerve

C. recurrent laryngeal nerve

D. rr. cardiaci cervicales superiores

E. auricular branch 

Question 10
Correct

Mark 1.00 out of 1.00

Which layers of the serous pericardium?

A. mediastinal, costal

B. costal,parietal

C. parietal, costal

D. diaphragmaric,visceral

E. parietal, visceral 

Question 11
Incorrect

Mark 0.00 out of 1.00

The first branch of the internal carotid artery is the following:

A. a. ophthalmica 

B. a. cerebri media

C. a. cerebri anterior

D. rr. caroticotympanici

E. a. communicans posterior

https://ets.amu.edu.az/mod/quiz/review.php?attempt=69006&cmid=7722 4/8
4/20/2021 Midterm exam: İnsan anatomiyası və tibbi terminologiya - 2 (EN) (MPF2): Attempt review

Question 12
Correct

Mark 1.00 out of 1.00

It does not belong the branches of the internal carotid artery:

A. a. cerebri posterior 

B. a. ophthalmica

C. a. cerebri anterior

D. a. cerebri media

E. a. communicans posterior

Question 13
Correct

Mark 1.00 out of 1.00

It is not the branch of the radial artery:

A. r. carpalis dorsalis

B. a. recurrens radialis

C. r. carpalis palmaris

D. а. interossea communis 

E. a. princeps pollicis

Question 14
Correct

Mark 1.00 out of 1.00

The vein which is composed by the connecting occipital and posterior auricular veins:

A. superior venаcava

B. anterior jugular vein

C. brachiocephalic vein

D. internal jugular vein

E. external jugular vein 

https://ets.amu.edu.az/mod/quiz/review.php?attempt=69006&cmid=7722 5/8
4/20/2021 Midterm exam: İnsan anatomiyası və tibbi terminologiya - 2 (EN) (MPF2): Attempt review

Question 15
Correct

Mark 1.00 out of 1.00

What is called the continuation of the right ascending lumbar vein?

A. azygos vein 

B. pericardiaco-phrenic veins

C. internal thoracic vein

D. accessory hemiazygos vein

E. hemiazygos vein

Question 16
Incorrect

Mark 0.00 out of 1.00

The anterior wall of the third ventricle is formed by:

1. pineal body 

2. thalamus

3. anterior commissure 

4. columns of the fornix

5. epithalamic commissure

6. terminal lamina 

Question 17
Incorrect

Mark 0.00 out of 1.00

Which muscles are innervated by n. dorsalis scapulae?

1. m. levator scapulae 

2. m. latissimus dorsi

3. m. coracobrachialis

4. m. pectoralis major 

5. mm. rhomboidei 

https://ets.amu.edu.az/mod/quiz/review.php?attempt=69006&cmid=7722 6/8
4/20/2021 Midterm exam: İnsan anatomiyası və tibbi terminologiya - 2 (EN) (MPF2): Attempt review

Question 18
Correct

Mark 1.00 out of 1.00

Find the accordance:

Gyrus angularis embraces posterior end of the superior temporal sulcus 

Gyrus supramarginalis embraces posterior branch of the lateral sulcus 

Question 19
Correct

Mark 1.00 out of 1.00

The trochlear nerve enters the orbit through the [[1]] and innervates the [[2]].

[[1]] superior orbital fissure 

[[2]] superior oblique muscle 

Question 20
Correct

Mark 1.00 out of 1.00

Find n. vagus:

Answer: 11 

◄ Attendance

Jump to...

A. Abdullayev. 7. General overview of the peripheral nervous system. The classification, development, variation, and anomalies of the
cranial nerves. ►

https://ets.amu.edu.az/mod/quiz/review.php?attempt=69006&cmid=7722 7/8
4/20/2021 Midterm exam: İnsan anatomiyası və tibbi terminologiya - 2 (EN) (MPF2): Attempt review

https://ets.amu.edu.az/mod/quiz/review.php?attempt=69006&cmid=7722 8/8
2021/4/20 Midterm exam: İnsan anatomiyası və tibbi terminologiya - 2 (EN) (MPF2): Attempt review

Dashboard / My courses / İnsan anatomiyası və tibbi terminologiya - 2 (EN) / General

/ Midterm exam: İnsan anatomiyası və tibbi terminologiya - 2 (EN) (MPF2)

Started on Tuesday, 20 April 2021, 12:30 PM


State Finished
Completed on Tuesday, 20 April 2021, 12:46 PM
Time taken 16 mins 28 secs
Grade 20 out of 20 (100%)

Question 1
Correct

Mark 1.00 out of 1.00

Between the anterior part of the pons and the middle cerebellar peduncle emerges

A. the trigeminal nerve. 

B. the facial nerve

C. the abducent nerve

D. the oculomotor nerve

E. the trochlear nerve

Question 2
Correct

Mark 1.00 out of 1.00

Temporal lobe is separated from the frontal and parietal lobes by the:

A. transversus occipital sulcus

B. parieto-occipital sulcus

C. lateral sulcus 

D. central sulcus

E. postcentral sulcus

Question 3
Correct

Mark 1.00 out of 1.00

It does not belong to lateral ventricle:

A. posterior horn

B. anterior horn

C. central part

D. inferior horn

E. anterior commissure 

https://ets.amu.edu.az/mod/quiz/review.php?attempt=68717&cmid=7722 1/7
2021/4/20 Midterm exam: İnsan anatomiyası və tibbi terminologiya - 2 (EN) (MPF2): Attempt review

Question 4
Correct

Mark 1.00 out of 1.00

Where is the gyrus parahippocampalis situated?

A. in the posterior part of the gyrus temporalis inferior

B. between the hippocampal and collateral sulcus 

C. between the inferior temporal and occipitotemporalis sulcus

D. between the superior and inferior temporal sulcus

E. between the lateral and superior temporal sulcus

Question 5
Correct

Mark 1.00 out of 1.00

The dorsal part of metencephalon is:

A. pedunculus cerebri

B. cerebellum 

C. tuber cinereum

D. corpus mamillaris

E. pons

Question 6
Correct

Mark 1.00 out of 1.00

The lower part of the rhomboid fossa is corresponded to:

A. mesencephalon

B. medulla oblongata 

C. diencephalon

D. metencephalon

E. telencephalon

https://ets.amu.edu.az/mod/quiz/review.php?attempt=68717&cmid=7722 2/7
2021/4/20 Midterm exam: İnsan anatomiyası və tibbi terminologiya - 2 (EN) (MPF2): Attempt review

Question 7
Correct

Mark 1.00 out of 1.00

Which of the following are the parts of the tympanic membrane?


I. Pars mucosa
II. Pars muscularis
III. Pars tensa
IV. Pars flaccida
V. Pars membranacea

A. I, II

B. II, IV

C. IV, V

D. III, IV 

E. III, V

Question 8
Correct

Mark 1.00 out of 1.00

Which cranial nerve provides the human with the ability to smell a wide variety of odors?

A. vestibulocochlear nerve

B. intermedius nerve

C. olfactory nerve 

D. oculomotor nerve

E. optic nerve

Question 9
Correct

Mark 1.00 out of 1.00

Which muscles are innervated by the nervus accesorius?


I. M. sternocleidomastoideus
II. M. trapezius
III. M. digastricus
IV. M. latissimus dorsi
V. M. buccinator

A. II,III

B. IV,V

C. III,IV

D. III,V

E. I,II 

https://ets.amu.edu.az/mod/quiz/review.php?attempt=68717&cmid=7722 3/7
2021/4/20 Midterm exam: İnsan anatomiyası və tibbi terminologiya - 2 (EN) (MPF2): Attempt review

Question 10
Correct

Mark 1.00 out of 1.00

The surfaces of the heart:


I. Oesophagalis
II. Sternocostalis
III. Mediastinalis
IV. Diaphracmatica
V. Trachealis

A. II;III

B. II;IV 

C. I;II

D. IV;V

E. III;IV

Question 11
Correct

Mark 1.00 out of 1.00

Where is located common carotid artery bifurcation?

A. root of the tongue

B. V cervical vertebra

C. posterior border of the palate

D. inferior border of the thyroid cartilage

E. superior border of the thyroid cartilage 

Question 12
Correct

Mark 1.00 out of 1.00

It does not belong the branches of the internal carotid artery:

A. a. cerebri anterior

B. a. cerebri media

C. a. communicans posterior

D. a. ophthalmica

E. a. cerebri posterior 

https://ets.amu.edu.az/mod/quiz/review.php?attempt=68717&cmid=7722 4/7
2021/4/20 Midterm exam: İnsan anatomiyası və tibbi terminologiya - 2 (EN) (MPF2): Attempt review

Question 13
Correct

Mark 1.00 out of 1.00

To which vein does v.cephalica vein open?

A. external jugular

B. anterior jugular

C. brachial

D. subclavian

E. axillary 

Question 14
Correct

Mark 1.00 out of 1.00

Where is located the inferior sagittal sinus?

A. in the connecting place of the falx cerebri with the tentorium cerebelli

B. in the confluence cinuum

C. between the foramen caecum and internal occipital protuberance

D. in the inferior concave margin of the falx cerebri 

E. in the transverse sinus groove of the occipital bone

Question 15
Correct

Mark 1.00 out of 1.00

This doesn’t belong to the cutaneous branches of the cervical plexus:

A. n. occipitalis minor

B. n. supraclavicularis

C. n. subclavius 

D. n. transversus colli

E. n. auricularis magnus

https://ets.amu.edu.az/mod/quiz/review.php?attempt=68717&cmid=7722 5/7
2021/4/20 Midterm exam: İnsan anatomiyası və tibbi terminologiya - 2 (EN) (MPF2): Attempt review

Question 16
Correct

Mark 1.00 out of 1.00

The anterior wall of the third ventricle is formed by:

1. anterior commissure 

2. thalamus

3. epithalamic commissure

4. pineal body

5. columns of the fornix 

6. terminal lamina 

Question 17
Correct

Mark 1.00 out of 1.00

Which muscles are innervated by n. dorsalis scapulae?

1. m. latissimus dorsi

2. m. pectoralis major

3. m. levator scapulae 

4. mm. rhomboidei 

5. m. coracobrachialis

Question 18
Correct

Mark 1.00 out of 1.00

Find the accordance:

The inner ear consists of bony labyrinth and membranous labyrinth 

The external ear includes auricle and external acoustic meatus 

Question 19
Correct

Mark 1.00 out of 1.00

The external capsule is a layer of [[1]] matter that is situated between the claustrum and [[2]] nucleus.

[[1]] white 

[[2]] lentiform 

https://ets.amu.edu.az/mod/quiz/review.php?attempt=68717&cmid=7722 6/7
2021/4/20 Midterm exam: İnsan anatomiyası və tibbi terminologiya - 2 (EN) (MPF2): Attempt review

Question 20
Correct

Mark 1.00 out of 1.00

Find the pons:

Answer: 11 

◄ Attendance

Jump to...

A. Abdullayev. 7. General overview of the peripheral nervous system. The classification, development, variation, and anomalies of the cranial
nerves. ►

https://ets.amu.edu.az/mod/quiz/review.php?attempt=68717&cmid=7722 7/7
4/21/2021 Midterm exam: İnsan anatomiyası və tibbi terminologiya - 2 (EN) (MPF2): Attempt review

Dashboard / My courses / İnsan anatomiyası və tibbi terminologiya - 2 (EN) / General

/ Midterm exam: İnsan anatomiyası və tibbi terminologiya - 2 (EN) (MPF2)

Started on Tuesday, 20 April 2021, 12:30 PM


State Finished
Completed on Tuesday, 20 April 2021, 12:59 PM
Time taken 29 mins 22 secs
Grade 19 out of 20 (95%)

Question 1
Incorrect

Mark 0.00 out of 1.00

In the posterior funiculus of the spinal cord are located:

A. tractus rubrospinalis, tractus spinocerebellaris dorsalis

B. tractus vestibulospinalis, tractus spinocerebellaris ventralis

C. tractus tectospinalis, tractus vestibulospinalis

D. fasciculus gracilis, tractus tectospinalis 

E. fasciculus cuneatus, fasciculus gracilis

Question 2
Correct

Mark 1.00 out of 1.00

Which from the following gyruses does not belong to the temporal lobe?

A. temporalis inferior

B. temporalis superior

C. temporalis media

D. supramarginal 

E. parahyppocampalis

https://ets.amu.edu.az/mod/quiz/review.php?attempt=68698&cmid=7722 1/8
4/21/2021 Midterm exam: İnsan anatomiyası və tibbi terminologiya - 2 (EN) (MPF2): Attempt review

Question 3
Correct

Mark 1.00 out of 1.00

Posterior end of the thalamus is called:

A. fissura chorioidea

B. tuberculum anterius

C. lamina affixa

D. stria terminalis

E. pulvinar 

Question 4
Correct

Mark 1.00 out of 1.00

It does not belong to epithalamus:

A. hypophysis 

B. epithalamic commissure

C. habenular trigone

D. habenular commissure

E. epiphysis

Question 5
Correct

Mark 1.00 out of 1.00

The dorsal part of metencephalon is:

A. cerebellum 

B. corpus mamillaris

C. pedunculus cerebri

D. pons

E. tuber cinereum

https://ets.amu.edu.az/mod/quiz/review.php?attempt=68698&cmid=7722 2/8
4/21/2021 Midterm exam: İnsan anatomiyası və tibbi terminologiya - 2 (EN) (MPF2): Attempt review

Question 6
Correct

Mark 1.00 out of 1.00

In medulla oblongata is not located:

A. center of vomiting

B. center of swallowing

C. circulatory center

D. respiratory center

E. highest center of vegetative nervous system 

Question 7
Correct

Mark 1.00 out of 1.00

The oculomotor nerve emerges from cranial cavity through:

A. fissura orbitalis superior 

B. canalis opticus

C. foramen ethmoidale anterior

D. foramen rotundum

E. fissura orbitalis inferior

Question 8
Correct

Mark 1.00 out of 1.00

The glossopharyngeal nerve leaves the skull through:

A. canalis hypoglossus

B. foramen jugulare 

C. foramen lacerum

D. foramen rotundum

E. foramen ovale

https://ets.amu.edu.az/mod/quiz/review.php?attempt=68698&cmid=7722 3/8
4/21/2021 Midterm exam: İnsan anatomiyası və tibbi terminologiya - 2 (EN) (MPF2): Attempt review

Question 9
Correct

Mark 1.00 out of 1.00

It does not belong to the branches of the ophthalmic nerve:


I. N. infaorbitalis
II. N. lacrimalis
III. Rr. pterygopalatini
IV. N. nasociliaris
V. N. frontalis
VI. N. zygomaticus

A. III; IV; VI

B. I; III; V

C. I; III; VI 

D. II; III; IV

E. II; IV; V

Question 10
Correct

Mark 1.00 out of 1.00

How many layers can be distinguished in the myocardium of the ventricles?

A. 2

B. 3 

C. 4

D. 1

E. 5

Question 11
Correct

Mark 1.00 out of 1.00

Thoracic aorta locates between the:

A. ThI-ThXII

B. ThIV-ThXII 

C. ThVI-ThXII

D. ThIII-ThVIII

E. ThIV-ThX

https://ets.amu.edu.az/mod/quiz/review.php?attempt=68698&cmid=7722 4/8
4/21/2021 Midterm exam: İnsan anatomiyası və tibbi terminologiya - 2 (EN) (MPF2): Attempt review

Question 12
Correct

Mark 1.00 out of 1.00

It does not compose the thyrocervical trunk:

A. inferior thyroid artery

B. superficial cervical artery

C. suprascapular artery

D. vertebral artery 

E. ascending cervical artery

Question 13
Correct

Mark 1.00 out of 1.00

To which vein does v.cephalica vein open?

A. subclavian

B. axillary 

C. anterior jugular

D. external jugular

E. brachial

Question 14
Correct

Mark 1.00 out of 1.00

The vein which is composed by the connecting occipital and posterior auricular veins:

A. internal jugular vein

B. brachiocephalic vein

C. anterior jugular vein

D. superior venаcava

E. external jugular vein 

https://ets.amu.edu.az/mod/quiz/review.php?attempt=68698&cmid=7722 5/8
4/21/2021 Midterm exam: İnsan anatomiyası və tibbi terminologiya - 2 (EN) (MPF2): Attempt review

Question 15
Correct

Mark 1.00 out of 1.00

Median nerve originates from the

A. axillary nerve

B. musculocutaneous nerve

C. medial and lateral cords of the brachial plexus 

D. radial nerve

E. ulnar nerve

Question 16
Correct

Mark 1.00 out of 1.00

Which muscles are innervated by n. dorsalis scapulae?

1. m. coracobrachialis

2. m. pectoralis major

3. m. latissimus dorsi

4. m. levator scapulae 

5. mm. rhomboidei 

Question 17
Correct

Mark 1.00 out of 1.00

To the branches of the ophthalmic nerve belong:

1. n. petrosus major

2. n. nasociliaris 

3. n. auriculotemporalis

4. n. lacrimalis 

5. n. infraorbitalis

6. n. frontalis 

https://ets.amu.edu.az/mod/quiz/review.php?attempt=68698&cmid=7722 6/8
4/21/2021 Midterm exam: İnsan anatomiyası və tibbi terminologiya - 2 (EN) (MPF2): Attempt review

Question 18
Correct

Mark 1.00 out of 1.00

Find the accordance:

A. interossea communis is the branch of ulnar artery 

A. thoracoacromialis is the branch of axillary artery 

Question 19
Correct

Mark 1.00 out of 1.00

The ulnar artery departs from the [[1]] artery at the level of the [[2]] of the ulna.

[[1]] brachial 

[[2]] coronoid process 

Question 20
Correct

Mark 1.00 out of 1.00

Find n. vagus:

Answer: 11 

◄ Attendance

Jump to...

A. Abdullayev. 7. General overview of the peripheral nervous system. The classification, development, variation, and anomalies of the
cranial nerves. ►

https://ets.amu.edu.az/mod/quiz/review.php?attempt=68698&cmid=7722 7/8
4/21/2021 Midterm exam: İnsan anatomiyası və tibbi terminologiya - 2 (EN) (MPF2): Attempt review

https://ets.amu.edu.az/mod/quiz/review.php?attempt=68698&cmid=7722 8/8
20/04/2021 Midterm exam: İnsan anatomiyası və tibbi terminologiya - 2 (EN) (MPF2): Attempt review

Mənim əsas səhifəm / Mənim fənlərim / İnsan anatomiyası və tibbi terminologiya - 2 (EN) / Ümumi

/ Midterm exam: İnsan anatomiyası və tibbi terminologiya - 2 (EN) (MPF2)

Testə başlandı Tuesday, 20 April 2021, 12:30 PM


State Finished
Sona çatdı Tuesday, 20 April 2021, 12:58 PM
Vaxt keçib 27 dəq. 37 san.
Qiymət 20 (68%) maksimumdan 14

Question 1
Correct

Mark 1.00 out of 1.00

The spinal cord is in a topoqaphic proportion are divided into:

A. cervical,lumbar-sacral parts

B. cervical,coccygeal and lumbar parts

C. cervical ,thoracic,sacral parts

D. cervical,thoracic,lumbar parts 

E. cervical,sacral parts

Question 2
Correct

Mark 1.00 out of 1.00

Which from the following gyruses does not belong to the temporal lobe?

A. supramarginal 

B. temporalis superior

C. parahyppocampalis

D. temporalis inferior

E. temporalis media

https://ets.amu.edu.az/mod/quiz/review.php?attempt=69027&cmid=7722 1/7
20/04/2021 Midterm exam: İnsan anatomiyası və tibbi terminologiya - 2 (EN) (MPF2): Attempt review

Question 3
Incorrect

Mark 0.00 out of 1.00

The upper wall or roof of the posterior horns of the lateral ventricle is formed by the lower surfaces of:

A. commissura posterior 

B. indusium griseum

C. forceps minor

D. commisura anterior

E. tapetum

Question 4
Correct

Mark 1.00 out of 1.00

The cerebral peduncles are located:

A. behind of cerebral aqueduct

B. under the tectum mesencephali 

C. laterally of cerebral aqueduct

D. medially of cerebral aqueduct

E. in front of optic chiasm

Question 5
Correct

Mark 1.00 out of 1.00

The roof of the fourth ventricle in front consists of:


I. Superior cerebellar peduncles
II. Middle cerebellar peduncles
III. Inferior cerebellar peduncles
IV. Velum medullare superius
V. Velum medullare inferius

A. I; IV 

B. I; II

C. II; III

D. III; IV

E. III; V

https://ets.amu.edu.az/mod/quiz/review.php?attempt=69027&cmid=7722 2/7
20/04/2021 Midterm exam: İnsan anatomiyası və tibbi terminologiya - 2 (EN) (MPF2): Attempt review

Question 6
Not answered

Marked out of 1.00

What extends from the superior to the inferior angle of the rhomboid fossa?

A. sulcus medianus

B. eminentia medialis

C. sulcus limitans

D. colliculus facialis

E. striae medullares

Question 7
Correct

Mark 1.00 out of 1.00

Which of the following are the parts of the tympanic membrane?


I. Pars mucosa
II. Pars muscularis
III. Pars tensa
IV. Pars flaccida
V. Pars membranacea

A. IV, V

B. II, IV

C. III, IV 

D. I, II

E. III, V

Question 8
Correct

Mark 1.00 out of 1.00

Which cranial nerve provides the human with the ability to smell a wide variety of odors?

A. vestibulocochlear nerve

B. olfactory nerve 

C. oculomotor nerve

D. optic nerve

E. intermedius nerve

https://ets.amu.edu.az/mod/quiz/review.php?attempt=69027&cmid=7722 3/7
20/04/2021 Midterm exam: İnsan anatomiyası və tibbi terminologiya - 2 (EN) (MPF2): Attempt review

Question 9
Correct

Mark 1.00 out of 1.00

Through which opening does the maxillary nerve pass out the cranial cavity?

A. spinosum

B. rotundum 

C. ovale

D. fissura orbitalis superior

E. lacerum

Question 10
Correct

Mark 1.00 out of 1.00

How is the called right atrio-ventricular valve?

A. atrioventricularis sinistra

B. seminularis

C. bicuspidalis

D. mitralis

E. tricuspidalis 

Question 11
Correct

Mark 1.00 out of 1.00

The ascending aorta passes to the aortic arch at the level of the:

A. left third sternocostal joint

B. left fourth sternocostal joint

C. right second sternocostal joint 

D. left second sternocostal joint

E. right third sternocostal joint

https://ets.amu.edu.az/mod/quiz/review.php?attempt=69027&cmid=7722 4/7
20/04/2021 Midterm exam: İnsan anatomiyası və tibbi terminologiya - 2 (EN) (MPF2): Attempt review

Question 12
Incorrect

Mark 0.00 out of 1.00

It doesn’t belong to the branches of thyrocervical trunk:

A. suprascapular artery

B. superficial cervical artery 

C. inferior thyroid artery

D. ascending cervical artery

E. subscapular artery

Question 13
Not answered

Marked out of 1.00

To which vein does v.cephalica vein open?

A. external jugular

B. anterior jugular

C. brachial

D. axillary

E. subclavian

Question 14
Correct

Mark 1.00 out of 1.00

At which level the internal jugular vein connects with the subclavian vein?

A. thyroid cartilage

B. I rib

C. III rib

D. sterno-clavicular joint 

E. II rib

https://ets.amu.edu.az/mod/quiz/review.php?attempt=69027&cmid=7722 5/7
20/04/2021 Midterm exam: İnsan anatomiyası və tibbi terminologiya - 2 (EN) (MPF2): Attempt review

Question 15
Incorrect

Mark 0.00 out of 1.00

The parietal branches of the thoracic aorta are the followings:


I. Rr. bronchiales
II. Rr. mediastinales
III. Superior phrenic arteries
IV. Posterior intercostalis arteries
V. Rr. pericardiaci

A. III,V

B. I,III

C. III,IV

D. I,II 

E. IV,V

Question 16
Correct

Mark 1.00 out of 1.00

The anterior wall of the third ventricle is formed by:

1. anterior commissure 

2. epithalamic commissure

3. terminal lamina 

4. pineal body

5. columns of the fornix 

6. thalamus

Question 17
Correct

Mark 1.00 out of 1.00

To the branches of the ophthalmic nerve belong:

1. n. infraorbitalis

2. n. petrosus major

3. n. nasociliaris 

4. n. lacrimalis 

5. n. auriculotemporalis

6. n. frontalis 

https://ets.amu.edu.az/mod/quiz/review.php?attempt=69027&cmid=7722 6/7
20/04/2021 Midterm exam: İnsan anatomiyası və tibbi terminologiya - 2 (EN) (MPF2): Attempt review

Question 18
Partially correct

Mark 0.50 out of 1.00

Find the accordance:

Gyrus supramarginalis embraces anterior end of the inferior temporal sulcus 

Gyrus angularis embraces posterior end of the superior temporal sulcus 

Question 19
Not answered

Marked out of 1.00

At the level of the lateral edge of the [[1]] rib, the axillary vein continues into the [[2]] vein.

[[1]] Seçin...

[[2]] Seçin...

Question 20
Correct

Mark 1.00 out of 1.00

Find the ulnar nerve:

Answer: 7 

◄ Attendance

Keç ...

A. Abdullayev. 7. General overview of the peripheral nervous system. The classification, development, variation, and anomalies of the
cranial nerves. ►

https://ets.amu.edu.az/mod/quiz/review.php?attempt=69027&cmid=7722 7/7
4/20/2021 Midterm exam: İnsan anatomiyası və tibbi terminologiya - 2 (EN) (MPF2): Attempt review

Mənim əsas səhifəm / Mənim fənlərim / İnsan anatomiyası və tibbi terminologiya - 2 (EN) / Ümumi

/ Midterm exam: İnsan anatomiyası və tibbi terminologiya - 2 (EN) (MPF2)

Testə başlandı Tuesday, 20 April 2021, 12:30 PM


State Finished
Sona çatdı Tuesday, 20 April 2021, 12:56 PM
Vaxt keçib 26 dəq. 15 san.
Qiymət 20 (98%) maksimumdan 20

Question 1
Correct

Mark 1.00 out of 1.00

Vermis is belonged to the:

A. pons

B. midbrain

C. cerebellum 

D. medulla oblongata

E. caudate nucleus

Question 2
Correct

Mark 1.00 out of 1.00

They are situated on the inferior surface of the temporal lobe:


I. Sulcus occipitotemporalis
II. Gyrus temporalis medius
III. Gyri temporales transversi
IV. Sulcus collateralis

A. I; II

B. III; IV

C. II; III

D. II; IV

E. I; IV 

https://ets.amu.edu.az/mod/quiz/review.php?attempt=68726&cmid=7722#question-98698-17 1/7
4/20/2021 Midterm exam: İnsan anatomiyası və tibbi terminologiya - 2 (EN) (MPF2): Attempt review

Question 3
Correct

Mark 1.00 out of 1.00

The extremal capsule is the white matter separating:

A. claustrum from cortex of insula 

B. globi pallidus medialis et lateralis

C. putamen and claustrum

D. globus pallidus lateralis and putamen

E. globus pallidus medialis and nucleus caudatus

Question 4
Correct

Mark 1.00 out of 1.00

Where is the gyrus parahippocampalis situated?

A. in the posterior part of the gyrus temporalis inferior

B. between the hippocampal and collateral sulcus 

C. between the superior and inferior temporal sulcus

D. between the inferior temporal and occipitotemporalis sulcus

E. between the lateral and superior temporal sulcus

Question 5
Correct

Mark 1.00 out of 1.00

Where is the vallecula cerebelli located?

A. on the posterior border of cerebellum

B. on the superior surface of cerebellum

C. on the anterior border of cerebellum

D. on the inferior surface of cerebellum 

E. on the anterior and posterior borders of cerebellum

https://ets.amu.edu.az/mod/quiz/review.php?attempt=68726&cmid=7722#question-98698-17 2/7
4/20/2021 Midterm exam: İnsan anatomiyası və tibbi terminologiya - 2 (EN) (MPF2): Attempt review

Question 6
Correct

Mark 1.00 out of 1.00

What extends from the superior to the inferior angle of the rhomboid fossa?

A. sulcus medianus 

B. sulcus limitans

C. colliculus facialis

D. eminentia medialis

E. striae medullares

Question 7
Correct

Mark 1.00 out of 1.00

Where does the optic nerve enter the cranial cavity from?

A. foramen zygomatico-orbitale

B. foramen ovale

C. foramen ethmoidae anterior

D. foramen ethmoidale posterior

E. canalis opticus 

Question 8
Correct

Mark 1.00 out of 1.00

Which cranial nerve provides the human with the ability to smell a wide variety of odors?

A. optic nerve

B. olfactory nerve 

C. intermedius nerve

D. oculomotor nerve

E. vestibulocochlear nerve

https://ets.amu.edu.az/mod/quiz/review.php?attempt=68726&cmid=7722#question-98698-17 3/7
4/20/2021 Midterm exam: İnsan anatomiyası və tibbi terminologiya - 2 (EN) (MPF2): Attempt review

Question 9
Correct

Mark 1.00 out of 1.00

It does not belong to the branches of the cervical part of the vagus nerve:

A. rr. cardiaci cervicales superiores

B. pharyngeal branches

C. recurrent laryngeal nerve

D. auricular branch 

E. superior laryngeal nerve

Question 10
Correct

Mark 1.00 out of 1.00

How is the called right atrio-ventricular valve?

A. tricuspidalis 

B. atrioventricularis sinistra

C. mitralis

D. bicuspidalis

E. seminularis

Question 11
Correct

Mark 1.00 out of 1.00

The ascending aorta passes to the aortic arch at the level of the:

A. right third sternocostal joint

B. left fourth sternocostal joint

C. left third sternocostal joint

D. right second sternocostal joint 

E. left second sternocostal joint

https://ets.amu.edu.az/mod/quiz/review.php?attempt=68726&cmid=7722#question-98698-17 4/7
4/20/2021 Midterm exam: İnsan anatomiyası və tibbi terminologiya - 2 (EN) (MPF2): Attempt review

Question 12
Correct

Mark 1.00 out of 1.00

It doesn’t belong to the branches of thyrocervical trunk:

A. subscapular artery 

B. inferior thyroid artery

C. suprascapular artery

D. ascending cervical artery

E. superficial cervical artery

Question 13
Correct

Mark 1.00 out of 1.00

To which vein does v.basilica open?

A. subclavian

B. axillary

C. brachial 

D. anterior jugular

E. external jugular

Question 14
Correct

Mark 1.00 out of 1.00

It does not belong to the tributaries of the external jugular vein:

A. v. suprascapularis

B. v. canalis pterygoidei 

C. v. auricularis posterior

D. v. occipitalis

E. v. jugularis anterior

https://ets.amu.edu.az/mod/quiz/review.php?attempt=68726&cmid=7722#question-98698-17 5/7
4/20/2021 Midterm exam: İnsan anatomiyası və tibbi terminologiya - 2 (EN) (MPF2): Attempt review

Question 15
Correct

Mark 1.00 out of 1.00

The visceral branches of the thoracic aorta are the followings:


I. Rr. bronchiales
II. Rr. mediastinales
III. Superior phrenic arteries
IV. Posterior intercostalis arteries
V. Rr. pericardiaci

A. II,III,IV

B. I,II,IV

C. III,IV,V

D. I,II,V 

E. II,III,V

Question 16
Correct

Mark 1.00 out of 1.00

The anterior wall of the third ventricle is formed by:

1. terminal lamina 

2. anterior commissure 

3. columns of the fornix 

4. pineal body

5. thalamus

6. epithalamic commissure

Question 17
Partially correct

Mark 0.67 out of 1.00

To the nuclei of the medulla oblongata belong:

1. nucleus caudatus

2. nucleus olivaris 

3. nucleus dentatus

4. nucleus ruber

5. nucleus gracilis 

6. nucleus cuneatus

https://ets.amu.edu.az/mod/quiz/review.php?attempt=68726&cmid=7722#question-98698-17 6/7
4/20/2021 Midterm exam: İnsan anatomiyası və tibbi terminologiya - 2 (EN) (MPF2): Attempt review

Question 18
Correct

Mark 1.00 out of 1.00

Find the accordance:

A. interossea communis is the branch of ulnar artery 

A. thoracoacromialis is the branch of axillary artery 

Question 19
Correct

Mark 1.00 out of 1.00

At the level of the lateral edge of the [[1]] rib, the axillary vein continues into the [[2]] vein.

[[1]] first 

[[2]] subclavian 

Question 20
Correct

Mark 1.00 out of 1.00

Find the ulnar nerve:

Answer: 7 

◄ Attendance

Keç ...

A. Abdullayev. 7. General overview of the peripheral nervous system. The classification, development, variation, and anomalies of the cranial
nerves. ►

https://ets.amu.edu.az/mod/quiz/review.php?attempt=68726&cmid=7722#question-98698-17 7/7
2021/4/20 Midterm exam: İnsan anatomiyası və tibbi terminologiya - 2 (EN) (MPF2): Attempt review

Mənim əsas səhifəm / Mənim fənlərim / İnsan anatomiyası və tibbi terminologiya - 2 (EN) / Ümumi

/ Midterm exam: İnsan anatomiyası və tibbi terminologiya - 2 (EN) (MPF2)

Testə başlandı Tuesday, 20 April 2021, 12:30 PM


State Finished
Sona çatdı Tuesday, 20 April 2021, 12:59 PM
Vaxt keçib 29 dəq. 14 san.
Qiymət 20 (80%) maksimumdan 16

Question 1
Correct

Mark 1.00 out of 1.00

On the midline of the anterior surface of the spinal cord is:

A. fissura orbitalis superior

B. ventrolateral grooves

C. dorsal (posterior) median sulcus.

D. dorsolateral grooves

E. ventral median fissure 

Question 2
Incorrect

Mark 0.00 out of 1.00

It happens if the motor analyzer of written speech is damaged:

A. apraxia

B. motor aphasia 

C. tactile agnosia

D. sensory aphasia

E. agraphia

https://ets.amu.edu.az/mod/quiz/review.php?attempt=68865&cmid=7722#question-98823-15 1/7
2021/4/20 Midterm exam: İnsan anatomiyası və tibbi terminologiya - 2 (EN) (MPF2): Attempt review

Question 3
Correct

Mark 1.00 out of 1.00

The floor of the central part of the lateral ventricle is formed by:
I. Body of caudate nucleus
II. Head of caudate nucleus
III. Thalamus
IV. Corpus fornicis
V. Stria terminalis

A. I; III; IV

B. II; III; V

C. I; II; III

D. I; III; V 

E. III; IV; V

Question 4
Correct

Mark 1.00 out of 1.00

Midbrain in Latin is called:

A. metencephalon

B. telencephalon

C. mesencephalon 

D. myelencephalon

E. diencephalon

Question 5
Incorrect

Mark 0.00 out of 1.00

The trigonum lemnisci is bounded medially by:

A. cerebral peduncle

B. brachium colliculi superioris

C. brachium colliculi inferioris 

D. superior cerebellar peduncle

E. inferior cerebellar peduncle

https://ets.amu.edu.az/mod/quiz/review.php?attempt=68865&cmid=7722#question-98823-15 2/7
2021/4/20 Midterm exam: İnsan anatomiyası və tibbi terminologiya - 2 (EN) (MPF2): Attempt review

Question 6
Incorrect

Mark 0.00 out of 1.00

In the superior triangle region of the rhomboid fossa is situated the nuclei of the

A. VIII,VII,III,VI cranial nerves 

B. V,II,III,VI cranial nerves

C. X,XI,XII cranial nerves

D. V, VI, VII , VIII cranial nerves

E. I,II.V,XII cranial nerves

Question 7
Correct

Mark 1.00 out of 1.00

The oculomotor nerve emerges from cranial cavity through:

A. foramen ethmoidale anterior

B. canalis opticus

C. fissura orbitalis inferior

D. foramen rotundum

E. fissura orbitalis superior 

Question 8
Correct

Mark 1.00 out of 1.00

The glossopharyngeal nerve leaves the skull through:

A. canalis hypoglossus

B. foramen ovale

C. foramen lacerum

D. foramen rotundum

E. foramen jugulare 

https://ets.amu.edu.az/mod/quiz/review.php?attempt=68865&cmid=7722#question-98823-15 3/7
2021/4/20 Midterm exam: İnsan anatomiyası və tibbi terminologiya - 2 (EN) (MPF2): Attempt review

Question 9
Correct

Mark 1.00 out of 1.00

Which muscles are innervated by the nervus accesorius?


I. M. sternocleidomastoideus
II. M. trapezius
III. M. digastricus
IV. M. latissimus dorsi
V. M. buccinator

A. III,V

B. I,II 

C. II,III

D. III,IV

E. IV,V

Question 10
Correct

Mark 1.00 out of 1.00

Which layers of the serous pericardium?

A. costal,parietal

B. mediastinal, costal

C. parietal, visceral 

D. parietal, costal

E. diaphragmaric,visceral

Question 11
Correct

Mark 1.00 out of 1.00

Where is located common carotid artery bifurcation?

A. superior border of the thyroid cartilage 

B. root of the tongue

C. inferior border of the thyroid cartilage

D. posterior border of the palate

E. V cervical vertebra

https://ets.amu.edu.az/mod/quiz/review.php?attempt=68865&cmid=7722#question-98823-15 4/7
2021/4/20 Midterm exam: İnsan anatomiyası və tibbi terminologiya - 2 (EN) (MPF2): Attempt review

Question 12
Correct

Mark 1.00 out of 1.00

The vertebral artery (a. vertebralis), is the first branch of the

A. suprascapular artery

B. superficial cervical artery

C. subclavian artery 

D. inferior thyroid artery

E. subclavian artery

Question 13
Correct

Mark 1.00 out of 1.00

It is not the branch of the ulnar artery:

A. а. carpalis dorsalis

B. а. carpalis palmaris

C. а. interossea communis

D. а. recurrens radialis 

E. а. recurrens ulnaris

Question 14
Correct

Mark 1.00 out of 1.00

The vein which is composed by the connecting occipital and posterior auricular veins:

A. internal jugular vein

B. superior venаcava

C. anterior jugular vein

D. brachiocephalic vein

E. external jugular vein 

https://ets.amu.edu.az/mod/quiz/review.php?attempt=68865&cmid=7722#question-98823-15 5/7
2021/4/20 Midterm exam: İnsan anatomiyası və tibbi terminologiya - 2 (EN) (MPF2): Attempt review

Question 15
Incorrect

Mark 0.00 out of 1.00

The visceral branches of the thoracic aorta are the followings:


I. Rr. bronchiales
II. Rr. mediastinales
III. Superior phrenic arteries
IV. Posterior intercostalis arteries
V. Rr. pericardiaci

A. II,III,V

B. I,II,V

C. I,II,IV

D. III,IV,V

E. II,III,IV 

Question 16
Correct

Mark 1.00 out of 1.00

Which muscles are innervated by n. dorsalis scapulae?

1. m. pectoralis major

2. m. levator scapulae 

3. m. latissimus dorsi

4. m. coracobrachialis

5. mm. rhomboidei 

Question 17
Correct

Mark 1.00 out of 1.00

To the branches of the brachial artery belong:

1. a. thoracodorsalis

2. a. collateralis ulnaris superior 

3. a. subscapularis

4. a. thoracoacromialis

5. a. collateralis ulnaris inferior 

6. a. profunda brachii 

https://ets.amu.edu.az/mod/quiz/review.php?attempt=68865&cmid=7722#question-98823-15 6/7
2021/4/20 Midterm exam: İnsan anatomiyası və tibbi terminologiya - 2 (EN) (MPF2): Attempt review

Question 18
Correct

Mark 1.00 out of 1.00

Find the accordance:

The external ear includes auricle and external acoustic meatus 

The inner ear consists of bony labyrinth and membranous labyrinth 

Question 19
Correct

Mark 1.00 out of 1.00

The external capsule is a layer of [[1]] matter that is situated between the claustrum and [[2]] nucleus.

[[1]] white 

[[2]] lentiform 

Question 20
Correct

Mark 1.00 out of 1.00

Find the ulnar nerve:

Answer: 7 

◄ Attendance

Keç ...

A. Abdullayev. 7. General overview of the peripheral nervous system. The classification, development, variation, and anomalies of the
cranial nerves. ►

https://ets.amu.edu.az/mod/quiz/review.php?attempt=68865&cmid=7722#question-98823-15 7/7
In which lobe of the cerebral hemisphere is the corpus amygdaloideum situated?

*A. temporal

B. insula

C. frontal

D. parietal

E. occipital

Find the lacrimal gland: 1


Question 4

Correct

Mark 1.00 out of 1.00

Flag question

Where is the gyrus parahippocampalis situated?

A between the lateral and superior temporal sulcus

B. in the posterior part of the gyrus temporalis inferior

*C. between the hippocampal and collateral sulcus

D. between the superior and inferior temporal sulcus

E. between the inferior temporal and

Question 5

Correct

Mark 1.00 out of 1.00

Flag question

In region of the lateral angles of the rhomboid fossa lays:

A. eminentia medialis
B. nucleus ruber

C. locus caeruleus

*D. area vestibularis

E. nucleus dorsalis nervi vagi

Question 6

Correct

Mark 1,00 out of 1.00

Flag question

In the superior triangle region of the rhomboid fossa is situated the nuclel of the

A. LIIV.XII cranial nerves

B. VIII, VIII, VI cranial nerves

C. VII,III,VI cranial nerves

D. X,XI,XII cranial nerves

*E. V, VI, VII, VIII cranial nerves

Question 7
Correct

Mark 1.00 out of 1.00

Flag quention

Which cranial nerve receives sound wave stimuli from the small bones of the middle ear, inner ear
and interpret this in the brain as hearing and also control human balance?

*A. vestibulocochlear nerve

Question 8

Correct

Mark 1.00 out of 1.00

Flag question

into how many parts does the facial nerve divide according to the topography

A. S

B. 6

C. 4

D.3

*E 2
Question 9

Correct

Mark 1.00 out of 1.00

Flag question

It does not belong to the branches of the ophthalmic

nerve:

L. N. infaorbitalis

II. N. lacrimalis

III. Rr. pterygopalatini

IV. N. nasociliaris

V. N. frontalis

VI. N. zygomaticus

AIV

BRIV: V

C. III; IV. VI

D. III: VI
*E. I;III;V

Question 10

Correct

Mark 1.00 out of 1,00

Flag question

How many layers can be distinguished in the myocardium of the ventricles?

B. 5

C.4

D. 2

*E 3

Question 11

Correct

Mark 1.00 out of 1,00

Flag question

Thoracic sorta locates between the

A. ThiV-ThX
B. Thill-ThVill

*D. ThiV-ThXII

E. THE-TAXII

Question 12

Correct

Mark 1.00 out of 1.00

Flag question

it does not belong to the parts of the internal carotid

artery:

A. cerebral part

B. petrous part

C. cervical part

Question 13

Mark 1.00 out of 1.00

Flag question
The branch of a profunda brachit

A a circumflexahumeriposterior

*B. a. collateralismedia

C. a. recurrens ulnaris

D. a. recurrens radialis

E. a. circumflexahumerianterior

The vein which is composed by the connecting occipital and posterior auricular veins.

A. superior venacava

8. brachiocephalic vein

C. internal jugular vein

D. anterior jugular vein

*E external jugular vein

Question 15

Correct

Mark 1.00 out of 1.00

Flag question
The visceral branches of the thoracic aorta are the

followings:

1. Rr. bronchiales

II. Rr. mediastinales

III. Superior phrenic arteries

IV. Posterior intercostalis arteries V. Rr. pericardiaci

A LILIV

*B. l,ll,V

C. ILIV

D. II,IV,V

Question 16

Correct

Mark 1:00 out of 1.00

Flag question

The anterior wall of the third ventricle is formed by:

*1. anterior commissure


2. pineal body

*3. terminal lamina

4. thalamus

*5. columns of the fornix

6. epithalamic commissure

Which muscles are innervated by n dorsalis scapulae?

*1. m. levator scapulae

2. m. coracobrachialis

*3. mm. rhomboidel

4. m. latissimus dorsi

5. m. pectoralis major

Question 18

Correct

Mark 1.00 out of 1,00

Flag question
Find the accordance:

Gyrus angularis embraces : posterior end of the superior tempor

Gyrus supramarginalis embraces : posterior branch of the lateral sulcus

Question 19

Correct

Mark 1.00 out of 1.00

Flag question

The ulnar artery departs from the [1] artery at the level of the [12]] of the ulna

[[1]] brachial

[[2]] coronoid process

The hemispheriae cerebrales consists of

DA vermit and the thinencephalon

B. the pallium pons and i

*C the pallium, the rhinencephalon and the basal ganglia

the minescepon the all and mengata


the basal ganglang mada binata

it does not belong to the branches of the cervical part of the vagus nerves

A superior laryngeal re

D.coedlaci ceescales subenotes

*E. Auricular branch

To the nucles of the medulla oblongata belong

2. Nucleus cuneatus

4. Nucleus gracilis
01023ÿ53ÿ01 6789 ÿ ÿÿ 8ÿÿ88ÿ 88ÿÿ0ÿ6ÿ67 !0

34567ÿ49:9ÿ94;6<47 =34567ÿ<45>4?67 =ÿ@59:5ÿ:5:AB76C:9DÿE4ÿA6FF6ÿAG?765B>BH6C:ÿIÿJÿKLMN=ÿO7P76


=ÿ36QAG?7ÿGR:7Sÿ@59:5ÿ:5:AB76C:9DÿE4ÿA6FF6ÿAG?765B>BH6C:ÿIÿJÿKLMNÿK3TUJN

VWXYZ[\]ÿ_
MBAÿCGAÿ:59`G?GQ
3:?aGQÿBPAÿB<ÿbcdd

gGA`GG5ÿA;Gÿ:5AG?6B?ÿh:?AÿB<ÿA;GÿhB59ÿ:5QÿA;Gÿ76QQ>GÿiG?GFG>>:?ÿhGQP5i>GÿG7G?HG9
jcA;Gÿ:FQPiG5Aÿ5G?EG
gcA;GÿBiP>B7BAB?ÿ5G?EG
kcA;GÿA?6HG765:>ÿ5G?EGc
lcA;Gÿ<:i6:>ÿ5G?EG
LcA;GÿA?Bi;>G:?ÿ5G?EG
k>G:?ÿ7Cÿi;B6iG

VWXYZ[\]ÿe
MBAÿCGAÿ:59`G?GQ
3:?aGQÿBPAÿB<ÿbcdd

mAÿ;:hhG59ÿ6<ÿA;Gÿ7BAB?ÿ:5:>CnG?ÿB<ÿ̀?6AAG5ÿ9hGGi;ÿ69ÿQ:7:HGQS
jc7BAB?ÿ:h;:96:
gc:H?:h;6:
kc9G59B?Cÿ:h;:96:
lc:h?:R6:
LcA:iA6>Gÿ:H5B96:
k>G:?ÿ7Cÿi;B6iG

VWXYZ[\]ÿf
MBAÿCGAÿ:59`G?GQ
3:?aGQÿBPAÿB<ÿbcdd

m5ÿ̀;6i;ÿ>BFGÿB<ÿA;GÿiG?GF?:>ÿ;G769h;G?Gÿ69ÿA;GÿiB?hP9ÿ:7CHQ:>B6QGP7ÿ96AP:AGQo
jc<?B5A:>
gcBii6h6A:>
kcAG7hB?:>
lch:?6GA:>
Lc659P>:
k>G:?ÿ7Cÿi;B6iG
p
" #33 $ %$9%$&3 93'%8&3 #$#"#( #)*+1,-./ 89)00001'% 8+2++22- 230
01023ÿ53ÿ01 6789 ÿ ÿÿ 8ÿÿ88ÿ 88ÿÿ0ÿ6ÿ67 !0
3456789:ÿ<
=>?ÿAB?ÿCDEFBGBH
ICGJBHÿ>K?ÿ>LÿMNOO

R?ÿH>BEÿD>?ÿSBT>DUÿ?>ÿBVW?XCTCYKEZ
[NXCSBDKTCGÿ?GWU>DB
\NXCSBDKTCGÿ]>YYWEEKGB
^NBVW?XCTCYW]ÿ]>YYWEEKGB
_NXAV>VXAEWE
`NBVWVXAEWE
^TBCGÿYAÿ]X>W]B

3456789:ÿP
=>?ÿAB?ÿCDEFBGBH
ICGJBHÿ>K?ÿ>LÿMNOO

aXBGBÿWEÿ?XBÿbCTTB]KTCÿ]BGBSBTTWÿT>]C?BHc
[N>Dÿ?XBÿWDLBGW>GÿEKGLC]Bÿ>Lÿ]BGBSBTTKY
\N>Dÿ?XBÿEKVBGW>GÿEKGLC]Bÿ>Lÿ]BGBSBTTKY
^N>Dÿ?XBÿCD?BGW>GÿCDHÿV>E?BGW>GÿS>GHBGEÿ>Lÿ]BGBSBTTKY
_N>Dÿ?XBÿV>E?BGW>GÿS>GHBGÿ>Lÿ]BGBSBTTKY
`N>Dÿ?XBÿCD?BGW>GÿS>GHBGÿ>Lÿ]BGBSBTTKY
^TBCGÿYAÿ]X>W]B

3456789:ÿQ
=>?ÿAB?ÿCDEFBGBH
ICGJBHÿ>K?ÿ>LÿMNOO

dXBÿT>FBGÿVCG?ÿ>Lÿ?XBÿGX>YS>WHÿL>EECÿWEÿ]>GGBEV>DHBHÿ?>Z
[NYBHKTTCÿ>ST>DUC?C
\N?BTBD]BVXCT>D
^NYBEBD]BVXCT>D
_NHWBD]BVXCT>D
`NYB?BD]BVXCT>D
^TBCGÿYAÿ]X>W]B

e
" #33 $ %$9%$&3 93'%8&3 #$#"#( #)*+1,-./ 89)00001'% 8+2++22- 030
01023ÿ53ÿ01 6789 ÿ ÿÿ 8ÿÿ88ÿ 88ÿÿ0ÿ6ÿ67 !0
3456789:ÿ<
=>?ÿAB?ÿCDEFBGBH
ICGJBHÿ>K?ÿ>LÿMNOO

R?ÿH>BEDS?ÿTBU>DVÿ?>ÿ?WBÿT>DAÿUCTAGXD?WY
ZN[CDCUXEÿEB\X[XG[KUCGXEÿ]>E?BGX>G
^N[>[WUBC
_N`BE?XTKUK\
aN[CDCUXEÿEB\X[XG[KUCGXEÿCD?BGX>G
bNK?GX[KUKE
_UBCGÿ\Aÿ[W>X[B

3456789:ÿP
=>?ÿAB?ÿCDEFBGBH
ICGJBHÿ>K?ÿ>LÿMNOO

RD?>ÿW>Fÿ\CDAÿ]CG?EÿH>BEÿ?WBÿLC[XCUÿDBG`BÿHX̀XHBÿC[[>GHXDVÿ?>ÿ?WBÿ?>]>VGC]WAÿY
ZNc
^Nd
_Ne
aNf
bNg
_UBCGÿ\Aÿ[W>X[B

3456789:ÿQ
=>?ÿAB?ÿCDEFBGBH
ICGJBHÿ>K?ÿ>LÿMNOO

R?ÿH>BEÿD>?ÿTBU>DVÿ?>ÿ?WBÿTGCD[WBEÿ>Lÿ?WBÿ[BG`X[CUÿ]CG?ÿ>Lÿ?WBÿ̀CVKEÿDBG`BY
ZNCKGX[KUCGÿTGCD[W
^NEK]BGX>GÿUCGADVBCUÿDBG`B
_N]WCGADVBCUÿTGCD[WBE
aNGGNÿ[CGHXC[Xÿ[BG`X[CUBEÿEK]BGX>GBE
bNGB[KGGBD?ÿUCGADVBCUÿDBG`B
_UBCGÿ\Aÿ[W>X[B

h
" #33 $ %$9%$&3 93'%8&3 #$#"#( #)*+1,-./ 89)00001'% 8+2++22- ,30
01023ÿ53ÿ01 6789 ÿ ÿÿ 8ÿÿ88ÿ 88ÿÿ0ÿ6ÿ67 !0
3456789:ÿ<=
>?@ÿBC@ÿDEFGCHCI
JDHKCIÿ?L@ÿ?MÿNOPP

RSTUSÿVDBCHFÿ?Mÿ@SCÿFCH?LFÿWCHTUDHITLXY
ZOXCITDF@TEDV[ÿU?F@DV
\OWDHTC@DV[ÿ]TFUCHDV
^OWDHTC@DV[ÿU?F@DV
_OITDWSHD`XDHTU[]TFUCHDV
aOU?F@DV[WDHTC@DV
^VCDHÿXBÿUS?TUC

3456789:ÿ<<
>?@ÿBC@ÿDEFGCHCI
JDHKCIÿ?L@ÿ?MÿNOPP

RSCHCÿTFÿV?UD@CIÿU?XX?EÿUDH?@TIÿDH@CHBÿbTMLHUD@T?EY
ZOFLWCHT?Hÿb?HICHÿ?Mÿ@SCÿ@SBH?TIÿUDH@TVD`C
\OTEMCHT?Hÿb?HICHÿ?Mÿ@SCÿ@SBH?TIÿUDH@TVD`C
^OW?F@CHT?Hÿb?HICHÿ?Mÿ@SCÿWDVD@C
_OcÿUCH]TUDVÿ]CH@CbHD
aOH??@ÿ?Mÿ@SCÿ@?E`LC
^VCDHÿXBÿUS?TUC

3456789:ÿ<Q
>?@ÿBC@ÿDEFGCHCI
JDHKCIÿ?L@ÿ?MÿNOPP

dSCÿbHDEUSCFÿ?Mÿ@SCÿbHDUST?UCWSDVTUÿ@HLEKeÿÿ
fOÿZOÿFLbUVD]TDÿICg@HDÿÿ
ffOÿZOÿUDH?@TFÿU?XXLETFÿFTETF@HDÿÿ
fffOÿZOÿUDH?@TFÿU?XXLETFÿICg@HDÿÿ
fcOÿZOÿFLbUVD]TDÿFTETF@HDÿÿ
cOÿZOÿ@SBH?TICDÿTXD
ZOfffhfc
\Ofhfff
^Ofchc
_Ofhff
aOffhfff
^VCDHÿXBÿUS?TUC

i
" #33 $ %$9%$&3 93'%8&3 #$#"#( #)*+1,-./ 89)00001'% 8+2++22- 530
01023ÿ53ÿ01 6789 ÿ ÿÿ 8ÿÿ88ÿ 88ÿÿ0ÿ6ÿ67 !0
3456789:ÿ<=
>?@ÿBC@ÿDEFGCHCI
JDHKCIÿ?L@ÿ?MÿNOPP

S@ÿTFÿE?@ÿ@UCÿVHDEWUÿ?Mÿ@UCÿHDITDXÿDH@CHBY
ZOHOÿWDH[DXTFÿ[DX\DHTF
]OHOÿWDH[DXTFÿI?HFDXTF
^ODOÿ[HTEWC[Fÿ[?XXTWTF
_O`OÿTE@CH?FFCDÿW?\\LETF
aODOÿHCWLHHCEFÿHDITDXTF
^XCDHÿ\BÿWU?TWC

3456789:ÿ<Q
>?@ÿBC@ÿDEFGCHCI
JDHKCIÿ?L@ÿ?MÿNOPP

bUCHCÿTFÿX?WD@CIÿ@UCÿTEMCHT?HÿFDcT@@DXÿFTELFd
ZOVC@GCCEÿ@UCÿM?HD\CEÿWDCWL\ÿDEIÿTE@CHEDXÿ?WWT[T@DXÿ[H?@LVCHDEWC
]OTEÿ@UCÿW?EMXLCEWCÿWTELL\
^OTEÿ@UCÿW?EECW@TEcÿ[XDWCÿ?Mÿ@UCÿMDXeÿWCHCVHTÿGT@Uÿ@UCÿ@CE@?HTL\ÿWCHCVCXXT
_OTEÿ@UCÿ@HDEFfCHFCÿFTELFÿcH??fCÿ?Mÿ@UCÿ?WWT[T@DXÿV?EC
aOTEÿ@UCÿTEMCHT?HÿW?EWDfCÿ\DHcTEÿ?Mÿ@UCÿMDXeÿWCHCVHT
^XCDHÿ\BÿWU?TWC

3456789:ÿ<R
>?@ÿBC@ÿDEFGCHCI
JDHKCIÿ?L@ÿ?MÿNOPP

gUCÿVHDWUTDXÿ[XCeLFÿTFÿM?H\CIÿVBÿ@UC
ZOM?LHÿ@HLEKF
]O@UHCCÿ@HLEKF
^O@G?ÿ@HLEKF
_OFTeÿ@HLEKF
aOMTfCÿ@HLEKF

h
" #33 $ %$9%$&3 93'%8&3 #$#"#( #)*+1,-./ 89)00001'% 8+2++22- -30
01023ÿ53ÿ01 6789 ÿ ÿÿ 8ÿÿ88ÿ 88ÿÿ0ÿ6ÿ67 !0
3456789:ÿ<=
>?@ÿBC@ÿDEFGCHCI
JDHKCIÿ?L@ÿ?MÿNOPP

T?ÿ@UCÿVHDEWUCFÿ?Mÿ@UCÿ?XU@UDYZ[WÿECH\CÿVCY?E]^
NOEOÿYDWH[ZDY[F
_OEOÿ[EMHD?HV[@DY[F
`OEOÿXC@H?FLFÿZDa?H
bOEOÿDLH[WLY?@CZX?HDY[F
cOEOÿMH?E@DY[F
dOEOÿEDF?W[Y[DH[F

3456789:ÿ<Q
>?@ÿBC@ÿDEFGCHCI
JDHKCIÿ?L@ÿ?MÿNOPP

T?ÿ@UCÿVHDEWUCFÿ?Mÿ@UCÿVHDWU[DYÿDH@CHBÿVCY?E]^
NODOÿXH?MLEIDÿVHDWU[[
_ODOÿW?YYD@CHDY[FÿLYEDH[FÿFLXCH[?H
`ODOÿFLVFWDXLYDH[F
bODOÿ@U?HDW?I?HFDY[F
cODOÿ@U?HDW?DWH?Z[DY[F
dODOÿW?YYD@CHDY[FÿLYEDH[Fÿ[EMCH[?H

3456789:ÿ<R
>?@ÿBC@ÿDEFGCHCI
JDHKCIÿ?L@ÿ?MÿNOPP

e[EIÿ@UCÿDWW?HIDEWC^
fBHLFÿDE]LYDH[FÿCZVHDWCF X?F@CH[?HÿCEIÿ?Mÿ@UCÿFLXCH[?Hÿ@CZX?HDYÿFLYWLF
fBHLFÿFLXHDZDH][EDY[FÿCZVHDWCF X?F@CH[?HÿVHDEWUÿ?Mÿ@UCÿYD@CHDYÿFLYWLF

3456789:ÿ<S
>?@ÿBC@ÿDEFGCHCI
JDHKCIÿ?L@ÿ?MÿNOPP

TUCÿ@H?WUYCDHÿECH\CÿCE@CHFÿ@UCÿ?HV[@ÿ@UH?L]Uÿ@UCÿggNhhÿDEIÿ[EECH\D@CFÿ@UCÿgg_hhO
ggNhh FLXCH[?Hÿ?HV[@DYÿM[FFLHC
gg_hh FLXCH[?Hÿ?VY[iLCÿZLFWYC

j
" #33 $ %$9%$&3 93'%8&3 #$#"#( #)*+1,-./ 89)00001'% 8+2++22- *30
01023ÿ53ÿ01 6789 ÿ ÿÿ 8ÿÿ88ÿ 88ÿÿ0ÿ6ÿ67 !0
3456789:ÿ<=
>?@ÿBC@ÿDEFGCHCI
JDHKCIÿ?L@ÿ?MÿNOPP

QREIÿ@SCÿLTEDHÿECHUCV

ÿ
WEFGCHV X

YÿW@@CEIDEZC
[C\ÿOOO
WOÿW]ILTTDBCUOÿXOÿ^CECHDTÿ?UCHURCGÿ?Mÿ@SCÿ_CHR_SCHDTÿECHU?LFÿFBF@C`OÿaSCÿZTDFFRMRZD@R?EbÿICUCT?_`CE@bÿUDHRD@R?EbÿDEIÿDE?`DTRCFÿ?Mÿ@SCÿ
ZHDERDTÿECHUCFOÿc

d
" #33 $ %$9%$&3 93'%8&3 #$#"#( #)*+1,-./ 89)00001'% 8+2++22- 030
Elektron Tadris Sistemi English(en)

uton 24 Define the ghobus patlidus


Net yet

Maadouaf
100
Nerves innervating superior oblique and lateral rectus muscles of eyeball

O1.Oculomotor nerve
O 2 Abducens nerve
O3.Trochlear nerve V
O4.Ophthalmic nerve
O5.Facal nerve

19
ict anoered
yet
The lesser (pulmon
D ory trunk

) lefl ventnck

) bor iutuniuen
Detemine compliance

Codile Sepn E) lor acCum


7. The
nuclel of which
Wesibule Segin um of the cer

locates af center of bory labyrintty aeteriorly corninects weti cochlea


Om 22tdvides by cest into anterio-eiptical and poštetiotpbetical recesse pestesiory it connets with 1iwe senicircutar analsch
5 eparated by scala fympani and scala vestibat, consists of pernyph
nitpostenorty it connects with eiibule, there s rourd widow near the apex
Market t
locafes at center o bomy latyrt there s cwial window.on fmedial wall
tEspiralbony canal the base lockS at exterrnal acoustic meatus

dazlay nerve leaves the skl by 1911 diides into and 13l enters to pten9palatioe

0) supeor

SE 9.The morpbo-functio

ebers
Question 24
Not yet answered

Marked out of 1,00

P Flag question

Determine falx cerebri:

Answer:
Antiser

Oet 25 Define the anterior hotn of lateral verticde


Nut i

Meted ot a

aueion
Facial netve

1. Innervates the anterior belly of digastric muscle

0 2. Innervates the muscles of facial expression

0 3. Forms the parotid plexus

4. Innervates the stylopharyngeal muscle

TD
5. Innervates the mylohyoid muscle

6. Divides into extracranial branches in facial canal

7. The greater petrosal nerve carries sympathetic fibers


9. Define tectum mesencephalicus:

Define tectum mesencephalicuS:

Answer:
080/2021 yar semesti Tmulx

CA https/jets.amaduaz/mod/qaiz atemgtph

Mad cut of
TRgnonston

Mailary nerve feaves the slaull by I11) divides into I(211 and, I13]l enters to pterygopalatine
ganglion

Seçin

two brancthes
IBS three branches
ganglionic branch
ganglionic, zygonatic lactimal branches
forarmen rotundum
foranen spinosum
ct yet foramen ovale
Marted out
t
Big qurston

Ulnar nerve ariges from the fl cord of the brachha pleus at the lievel of the IR23

31 Segi

Secin
nerve
Befee ie Bng ciluny

Ansvwer
13
12

11

10

Answer:
Which of the followings belong to the branches of the radial artery

1.a.recurrens radialis
02. a.profunda brachi
3. a.suprascapularis
D4. a.thoracoacromialis
a.metacarpalis dorsalis
I

5.
6.a.princeps policis
6
15
10

ans
Question 18
Not yet answered

Marked out of 1,00

P Flag question

Vascular layer of eyeball consists.of:

O1. Choroid
O2. Conjunctiva

3. Outer nuciear layer

O 4. Optic disc
O5. Ciliary bopy
O 6 Iris
O 7. Pigmented layer

Question 19
Not yet answered
These are belonged to old structures of brain by filogenesis:

D1. lsocortex
02. Cerebellum
3. Amygdaloid body

04.Claustrum

5. Brainstem

6. Caudate nucleus
D7.Lentiform nuclei
Branches exitingfrom parotid plexus:

1.Zygomatic branches

02. Communicating branch to glc sopharyngealnerve

3.Cervical branches
4. Buccal branches
plexus
Communicating branch to tympanic
O5.
6. Stylohyoid branch
Which of the followings belong to the branches of the ulnar artery?

1.a.recurrens ulnaris

2. a.interossea communis
3.palmaris profundus
4.a.suprascapularis
5. aprofunda brachi
6.a.thoracoacromialis
Question 17
Not yet answered

Marked out of 100

F Flag question

The structures which take part in formation of


posterior wall of ll ventricle:

O1. claustrum
2. habenular commissure
O3. epithalamic .commissure
O 4. velum medullare superius

5. posterior surface of pons

O 6. pineal body
These are belonged to old structures of cerebellum.

1.Flocculus
2. Dentate nucleus

3.Vermis
4. Emboliformis nucleus

5.Globosus nucleus
6. Fastigii nucleus

Vascular layer of eyeball consists of

1.Outer nuclear layer


2.Opticdisc

D3. Pigmented layer


4. Conjunctiva

5. Iris

6.Ciliary body
7.Choroid
18 The infraorbital nerve atter eierging ito the lace inrerviate
uit
yet
O1.Skin oft upper lip
d ovt
af 2.1ower eyelid
0
g 4vestio
3 Skin of exterhalhe
4SkiIn ol lower ip

5. Skin offorehend
6Uppereyetid
Answer

Define the auditory


center:

Answer
Question 24 Define the vestibular ganglion:
Not yet
answered
Marked out of
1,00

FFlag question

Answer
Show sinus rectus:

Answer:
I
Done ets.amu.edu.az AA C

Not yet answered

Marked out of 1,00

F Flag question

Define the a.gastroepiploica dextra:

Answer:
Question 224

Not yet answered

Marked out of 1,00

P Flag question

Determine falx cerebri:

11

Answer:
3020/0t jar
seei Fole

G itps://etsamueduaz/mod/qudiz/ attempt.ph

Uhe akkddoY analyat of spoken speedh s


Sepn

ion 20
kt yet and
Maknd out if too
Pag uenstm

Determine compliance

Lerwand vitreous body nucleus of the eyeball


Chay booy
it coordition near and far vision
t belongs to retina

21
ccoordinates
cleus of the eyebal
the amount of luminous fluc
coordimatles near and tar vision

eopmecomkne
coche e
ioule en
C https://ets.amu.edu.az/mod/quiz/attemptph.

6Globosus nucleus

eton 18
tot yet onvwered
Mackedd out of 1.00

f tag question

Nerves innervating superior oblique and lateral rectus muscles of eyebalt:

D1.Ocolomotor nerve
D2.Abducens nerve
3 Trochlear nerve
C3 4:Ophithalmic nerve

BShc nerve

19
36. These are belonged to old structures of cerebellum.
1. Dentate nucleus
2. Fiocys
3. Emboliformis nucleus

4. Globosus nucleus
5. Fastigii hucteus (maybe idk this depends on ur luck)
6. Vermis
Answer

5 Define the auditory


center:

tof

Bton

Answes
P gestion
131 Choose

Oeon 24 Define the vestibular ganigliont


Not jet
ansetred
Maried out of
100
rfig estion

Anet
Show sinus rectus:

Answer
I
Define nucleus of n.trochlearis:

Answer:
Not yet answered

Marked out of 100

Flagquestion

Define the a.gastroepiploica dextra:

Answer:

Testin son
ueston 20
Not yet answered
Marked out oft 1.00

FFag question

Determine compliance

Lens and vitreous body


nucleus of the eyeball
Ciliary body t coordition near and far vision
it belongs to retina,
Nucieus of the eyeball.
Queston 21 coordinates the amount of luminous lux.
Not yet answered itcoordinates near and far vision.
Marked out of 100

Rag question

Determine compliance

Cochle Segin
caviy
estibule Secin
9Type here to search O
ueston 20
Not yet answered
Marked out ot 1.00

FFag question

Determine compliance

Lens and vitreous body Seçin

Ciliary body Seçin.


it belongs to fibrous layer of eyebal
it belongs to retina
Nucleus of the eyeball.
Queston 21 coordinates the amount of luminous lux.
Not yet answered it coordinates near and far vision
Marked out of 1300

F RAg question

Determine compliance

Cochlea
Secin
caviy
estibule Secin
9Type here to search O
Itdoesn'tbelong to the visceral lymph nodes of the pelvis:
O A. nodi limhpatici pararectales

O B. nodi limhpatici parauterini

O C nodi limhpatici paravaginales


OD. nodi limhpatici obturatorii
OE. nodi limhpatici paravesiculares
These are belonged to old structures of brain by filogenesis:

D1. lsocortex
02. Cerebellum
3. Amygdaloid body

04.Claustrum

5. Brainstem

6. Caudate nucleus
D7.Lentiform nuclei
ject

e Month ate:

The visnal anlay ze aiwlitin spench


15 Situetee in -HSfran.talisinfeio
lobalas perie telis 10feriov 9yYsan jalayiS

the audi tor analayz aeot spuker peni


islecated n the vatial pot of
Superie tee e
heschl's yaSH) ha

lan
ophtalamic avterg oA)
O
A-oph thalamicoi the brnnch
15he fivst bvanch athe11leenaLcaye ti
SinuuS
at clidalfal te-culer louS
segmet
15 he branch 0f

1s
hy vocertiCa trank evnich
hclavia
truncus
Yetus to the brancheso eenca
cas
(of thjvocev vicalis k to tunus toovical
A branches inPenoy thgrald a
Supas capala ar iere
Ascen ding eical vta
tvansVevsalis te
1
k ak
tvavsveu eruical art eg
Subject:
a Year
Date

Codo cervieal trank


onjin (nis #e loranh o subclavian ar 1)
ranches(veters to the oancha)t
SLpxiof intercestel aieeep.cencnla A

belonged to old stvuctur at Wain 1 filogeness

brain Sean
celEhelluw
Jeattoiad nucle
hinecephalah
Branches fran patoid plexus
rtna choetea (z 287-300 cinclnturt

Subiect:

tnatomnchigk 4uesirns af dantistar

-he pex.te/ie all of tird ventricle


cendninsthe pestee cesS pone hod
h babenulow c se and he spenpineal

-lhe vasalaw lagad of the eye


Chardic
cluy ba

( petrasn aj
The rertev
fspatcial petresal hecve
th anch
f henedethntermedin
n. tyPanicM$is the branchof
glassophagngenl ne

vena cav Cornav Stnus


inPeriv and 5aPio
Vc opnd into he qkt

Pal men A V veinS ar orndfto th


VERDA
OASai

DAf
DAreit
Which of
the followings belong to the branche: o theradiaf artery
a.recurrens radialis
1.
2. aprofunda brachii
3.a.suprascapularis
4.a.thoracoacromialis
E 5. a.metacarpalis dorsalis
6.aprinceps polics

py
These are Delonged to old sinuctures of brain riogenesi
1. Which muscles are innervated by branches exiting from ansa cervicalis?

1. Stylohyoid

2. Omohyoid

3. Mylohyoid

4. Sternothyroid

5. Digastric

6. Sternohyoid

2. Which of the followings are located in midbrain?

I. Substantia grisea centralis

2. Medial nuclei of mamillary body

3. Paraventricular nuclei

4. Nucleus of tuber cinereum

5. Substantia nigra

6. Lateral nuclei of mamillary body

7. Nucleus ruber

3. The anterior spino-cerebellar tract passes by:

A. cerebral peduncle

B. middle cerebellar peduncle

C. superior cerebellar peduncle

D. capsula interna

E. inferior cerebellar peduncle


4. The terminating nuclei of which sensory nerves are not located in the brain
stem?

A. cochlear nerve

B. vestibular nerve

C. sensory part of the trigeminal nerve

D. olfactory nerve

E. optic nerve

5. The infraorbital nerve enters the orbit through the:

A. inferior orbital fissure

B. anterior ethmoid foramen

C. posterior ethmoid foramen

D. foramen ovale

E. superior orbital fissure

6. The retropharyngeal lymph nodes do not accept lymph from the:

A. root of the tongue

B. frontal sinuses

C. lower teeth

E. middle ear

F. nasal part of pharynx


7. It is not supplied by the a. vesicalis inferior:

A. prostate (in male )

B. seminal vesicles (in male )

C. fundus of urinary bladder

D. rectum

E. vagina (in female )

8. They are belonged to the dura mater derivatives:

l. Falx cerebri
ll. Velum medullare superior
Ill. Tuber cinereum
IV. Falx cerebelli
V. Diaphragm of Turkish saddle

A. Il; Ill; IV
B. l; Il; Ill
C. Il; IV; V
D. l; Ill; IV
E. I; IV; V

9. Define tectum mesencephalicus:

4
10. It is the subcortical nuclei of the optic nerve:

l. Corpus geniculatum mediale

ll. Corpus geniculatum laterale

Ill. Colliculi inferiores

IV. Colliculi superiores

V. Pulvinar thalami

A. II, IV, V

B. Il, Ill, IV

C. l, Ill, V

D. l, Il, Ill

E. l, IV, V

11. The nucleus of the auditory analyzer is situated in cerebral cortex in


the:

A. parietal lobe

B. frontal lobe

C. insula

D. temporal lobe

E. occipital lobe
12. It is the subcortical nuclei of the optic nerve:

l. Corpus geniculatum mediale

Il. Corpus geniculatum laterale

Ill. Colliculi inferiores

IV. Colliculi superiores

V. Pulvinar thalami

A. Il, IV, V

B. Il, Ill, IV

C. l, Ill, V

D. l, Il, Ill

E. I, IV, V
14. They belong to basal nuclei:

1. insula

2. internal capsule

3. corpus striatum

4. claustrum

5. external capsule

6. corpus amygdaloideum

7. marginal capsule

16. The descending part of the aorta is the:

A. bulbus aortae, arcus aortae

B. isthmus aortae,pars thoracica aortae

C. pars thoracica aortae, pars abdominalis aorta

D. bifurcatio aortae, pars ascendens

E. pars abdominalis aorta,bulbus aortae


17. Define the cranial nerve roots that emerge between the pyramid and
the olive:

A. XII

B. X

C. VII

D. IX

E. VII

18. Determine compliance:

Trochlear nerve Page 374

Trigeminal nerve Page 483

19. Define the hypoglossal nerve:

12
20. The thoracic part of the sympathetic trunk

A. consist of 12 or 14 ganglia

B. consist of 14 or 15 ganglia

C. consist of 11 or 13 ganglia

D. consist of 15 or 17 ganglia

E. consists of 10 or 12 ganglia

21. How the continuation of the musculocutaneous nerve is called in the

forearm ?

A. the medial cutaneous nerve of the forearm

B. the posterior cutaneous nerve of the forearm

C. the lateral cutaneous nerve of the forearm

D. posterior interosseal nerve

E. anterior interosseal nerve

22. Find the accordance:

superior cerebellar peduncle Page 856

inferior cerebellar peduncle Page 856


23. It refers to the branches of the cervical plexus [[1]], the branches of
the lumbar plexus [[2]] and the branches of the sacral plexus [[3]].

[[1]] Page 451 gl

[[2]] Page 723 gl

[[3]] Page 737 gl

24. Forms the anterior wall of the third ventricle [[1]], posterior wall [[2]],
lateral wall [[3]].

[[1]] lamina cinerea terminalis or page 116

[[2]] epithalamic, habenular commissures and pineal body

[[3]] medial surfaces of the 2 thalami

25. The sacral part of the sympathetic trunk consists of the

A. 3 to 4 of sacral ganglia

B. 2 to 5 sacral ganglia

C. 3 to 5 ganglia

D. 3 to 6 sacral ganglia

E. 4 to 6 ganglia
26. They are belonged to middle ear:

I. Auricula

ll. Tuba auditiva

Ill. Meatus acusticus externus osseus

IV. Meatus acusticus externus cartilagineus

V. Cavitas tympani

A. l; lll

B. Il; lll

C. Ill; lV

D. II; V

E. IV, I

27. Define the long ciliary nerve:

1
31. Superior vena cava is formed by the connection of the [[1]] at the level
of the junction of the right I rib and [[2]].

[[1]] right and left brachiocephalic veins Page 566

[[2]] sternum
35. It does not belong to the branches of the thyrocervical trunk:

1. Subscapular artery

2. Superior thyroid artery

3. Transverse cervical artery

4. Suprascapular artery

5. Ascending cervical artery

6. Inferior thyroid artery


37. Which nerves belong to the branches of the lumbar plexus?

1. n.femoralis

2. n.ilioinquinalis

3. n.pudendus

4. n.gluteus inferior

5. n.gluteus superior

6. n.iliohypogastricus

45. Sulcus intermedius posterior divides the posterior funiculus into two
parts:

1. Fasciculus cuneatus

2. Lateral pyramidal tract

3. Fasciculus gracilis

4. Anterior pyramidal tract

5. Dorsal spinocerebellar tract or Flechsig’s tract

6. Anterior spinocerebellar tract or Gower’s tract

7. Rubrospinal tract
46. It does not belong to the branches of the thyrocervical trunk:

1. Transverse cervical artery

2. suprascapular artery

3. superior thyroid artery

4. subscapular artery

5. inferior thyroid artery

6. Ascending cervical artery


48. Facial nerve: (good luck lol)

1. Forms the parotid plexus

2. Divides into extracranial branches in facial canal

3. Innervates the muscles of facial expression

4. The greater petrosal nerve carries sympathetic fibers

5. Innervates the stylopharyngeal muscle

6. Innervates the mylohyoid muscle

7. Innervates the anterior belly of digastric muscle

50. Branches exiting from parotid plexus:

1. Zygomatic branches

2. Communicating branch to glossopharyngeal nerve

3. Cervical branches

4. Stylohyoid branch

5. Buccal branches

6. Communicating branch to tympanic plexus

51. Define the a.gastroepiplocia dextra:

6
53. Determine compliance:

Vestibule Page 782

Cochlear cavity Page 783

54. Define corpus callosum:

1
55. Define the otic ganglion:

15

56. Determine the nucleus ambiguous:


57. Determine compliance:

the visual analyzer of written speech is situated in is situated in the angular gyrus
of the inferior parietal lobule. If it is damaged, the ability to read is lost (alexia) but vision is
preserved

the auditory analyzer of spoken speech is located in the posterior part of the
superior temporal gyrus Page 821

58. The structures which take part in formation of posterior wall of Ill
ventricle:

1. claustrum

2. habenular commissure

3. epithalamic commissure

4. velum medullare superius

5. posterior surface of pons

6. pineal body
61. Determine compliance:

The posterior wall of the third ventricle posterior wall is formed by the
epithalamic, habenular commissures and pineal body

The tela chorioidea of the third ventricle is triangle shaped fold and
consists of 2 layers of the pia mater Page 827

68. Inferior and superior vena cava, coronary sinus are opened into the
[[1]] and pulmonary veins into the [[2]].

[[1]] right atrium

[[2]] left atrium

69. Determine compliance.

Vestibulocochlear nerve Page 142

Facial nerves good luck

70. the body of the [[1]] lies in the spiral ganglion that contain the [[2]]

[[1]] first neuron

[[2]] bipolar cells


71. [[1]] is belonged to branches of head part of vagus nerve, [[2]] is
belonged to branches of cervical part of vagus nerve and, [[3]] - to the
branches of thoracic part of same nerve.

[[1]] meningeal branch and auricular branch Page 506

[[2]] Pharyngeal branche and superior laryngeal nerve and recurrent


laryngeal nerve and upper cardiac branches Page 507

[[3]] Thoracic cardiac branches and Tracheal branches and Bronchial


branches and Esophageal branches Page 510

74. Find the accordance:

belong to telencephalon The cerebral hemispheres (hemispheriae cerebri)


consist of the pallium, the rhinencephalon and the basal ganglia
(subcortical ganglia).Page 807

belong to diencephalon The diencephalon (Fig. 28) is divided into two


parts: dorsal (phylogenetically younger) part, the thalamencephalon and
ventral (phylogenetically older) part, thehypothalamus. The third ventricle
is a remnant of the primary diencephalic cavity Page 822

75. Determine compliance: Page 704

a.collateraiis ulnaris inferior

76. Centers which are located in the frontal lobe:

1. Auditory analyser of spoken speech.

2. The motor analyzer of speech articulation.

3. Motor analyser of written speech.

4. Auditory analyser.

5. Visual analyser of written speech.

……………………………………………………………………………………………………………
……………………………………………………………………………………………………………
The branches of trigeminal nerve belonged to the nasociliary nerve:
1. Short ciliary nerves
2. Long ciliary nerve 
3. Lacrimal nerve
4. Infratrochlear nerve 
5. Posterior ethmoid nerve 
6. Supratrochlear nerve

Refers to the branches of the ophtalmic nerve:


1. n.frontalis 
2. n.nasociliaris 
3. n.nasales externi
4. n.lacrimalis 
5. n.infraorbitalis
6. n.palpebrales inferiors

Determine compliance:
Each common iliac vein at the level of the sacroiliac joint comprises two veins: external and
internal iliac veins
The external iliac vein drains two veins: inferior epigastric vein and deep circumflex iliac vein

Determine compliance:
n.frontalis is the branch of: ophthalmic nerve
n.zygomaticus is the branch of: maxillary nerve

Determine compliance:
To vascular layer of eyeball: ciliary body and iris are belonged.
To fibrous layer of eyeball: sclera and cornea are belonged.

[[1]] refers to the branches of the truncus thyrocervicalis and [[2]] to


truncus costocervicalis.
[[1]] a.cervicalis superficialis
[[2]] a.intercostalis suprema

Define the anterior horn of lateral ventricle:

Answer: 17

Define the zygomatic nerve:


Answer: 11

Which of the followings belong to the branches of the trigeminal nerve?


1. mandibular nerve 
2. greater petrosal nerve
3. lesser petrosal nerve
4. chorda tympany
5. maxillary nerve 
6. ophthalmic nerve

These are belonged to basal nuclei:


1. Capsula interna
2. Capsula extrema
3. Amygdaloid body 
4. Capsula externa
5. Claustrum 
6. Striate body

Find the accordance:


n.tympanicus is the branch of: n.glossopharyngeus
n.petrosus major is the branch of: n.facialis

Determine compliance:
The descending genicular artery: supplies m. vastus medialis with blood and participates in the
formation of the arterial network of the knee joint
The middle genicular artery: perforates the articular bursa, and branches out in
cruciate ligaments

Maxillary nerve leaves the skull by [[1]], divides into [[2]] and, [[3]] enters
to pterygopalatine ganglion.
[[1]] foramen rotundum
[[2]] three branches
[[3]] ganglionic branch

Anterior superior alveolar nerve passes downward in the anterior wall of


the [[1]] sinus and innervates the [[2]].
[[1]] maxillary
[[2]] dentes canine and dentes incisores
Define the otic ganglion:

Answer: 13

Show sinus rectus:

Answer: 8

These are belonged to old structures of cerebellum


1. Flocculus
2. Dentate nucleus
3. Vermis
4. Emboliformis nucleus
5. Globosus nucleus
6. Fastigii nucleus

Vascular layer of eyeball consists of:


1.Outer nuclear layer
2.Optic disc
3. Pigmented layer
4. Conjunctiva
5. Iris
6. Cillary body
7. Choroid

the short branches of the sacral plexus belong:


1. n.ilioinguinalis
2. n.iliohypogastricus
3. n.genitofemoralis
4. n.gluteus superior
5. n.gluteus inferior
6. n.pudendus

Determine compliance:
Parietal lymph nodes of the pelvis: nody lymphatici iliaci interni, nodi limphatici obturatorii, nody
lymphatici gluteales, nodi lymphatici…
Visceral lymph nodes of the pelvis: nodi lymphatici pararectales, nodi lymphatici paravaginales, nodi
lymphaticl paravezicular

Determine compliance
paired visceral branches of the abdominal aorta: a. renalis, a.testicularis, a.suprarenalis media
unpaired visceral branches of the abdominal aorta: tr.coelacus, a mesenterica superior, a.mesenterica
inferior

Find the accordance:


the motor analyzer of speech articulation is situated in: posterior part of inferior frontal gyrus
the motor analyzer of written speech lies in: posterior part of middle frontal gyrus

Optic nerve has [[1]] fibers, oculomotor nerve has [[2]] fibers and,
trigeminal nerve [[3]] fibers
[[1]] sensory
[[2]] Parasympathetic preganglionic
[[3]] Mixed nerve Or both motor and sensory

Covers the heart from the front [[1]], from below [[2]], from the lateral sides
[[3]].
[[1]] sternum and ribs
[[2]] diaphragm
[[3]] lungs

Define the a.gastroepiploica dextra:

Answer:6

Nerves innervating superior oblique and lateral rectus muscles of eyeball:


1. Trochlear nerve
2. Facial nerve
3. Oculomotor nerve
4. Abducens nerve
5. Ophthalmic nerve

Skin of which regions are innervated by branches of infraorbital nerve?


1. Forehead
2. Upper lip
3. Auricle
4. Lower eyelid
5. Temple
6. Occiput
7. Cheek
These are belonged to old structures of brain by filogenesis:
1. Claustrum
2. Caudate nucleus
3. Brainstem
4. Amygdaloid body
5. Cerebellum
6. Isocortex
7. Lentiform nuclei

Determine compliance:
Lateral thoracic artery: supplies blood to the serratus anterior muscle
Internal thoracic artery: supplies blood to the diaphragm, bronchi and...

Determine compliance:
if the motor analyzer of speech articulation is damaged: motor aphasia appears.
if the motor analyzer of written speech is damaged agraphy appears.

Determine compliance:
Mandibular nerve: is mixed nerve; it enters into infratempora...
Ophthalmic nerve: is sensory nerve; it enters into orbit via fiss...

V.rectalis superior drains through the [[1]] into the portal vein system,
while w.rectales mediae empty into the system of the vena cava inferior
through the [[2]]
[[1]] inferior mesenteric vein
[[2]] internal iliac vein

Brachiocephalic vein is formed by the fusion of the internal jugular vein


with the [[1]] and collects venous blood from the [[2]].
[[1]] subclavian vein
[[2]] head, neck and upper extremities

Determine the lemniscus medialis.


Answer:5

Define the tectum mesencephali


Answer:1

Define the globus pallidus :


Answer:9

Define the vestibular ganglion :


Answer:7

Define nucleus of n.trochlearis :


Answer:14

Define the auditory center :


Answer:4
doesn't belong to the visceral lymph nodes of the pelvis :
O A. nodi limhpatici pararectales
O B. nodi limhpatici parauterini
O C. nodi limhpatici paravaginales
O D. nodi limhpatici obturatori.
O E nodi limhpatici paravesiculares

The infraorbital nerve after emerging into the face innervates


1. Skin of upper lip
2. Lower eyelid
3. Skin of external nose.
4. Skin of lower lip
5 Skin of forehead
6. Upper eyelid

Determine compliance :
to the branches of costocervical trunk belong : a.intercostalis suprema
to the branches of internal thoracic artery belong : a.pericardiacophreica
Define the tractus olfactorius :
Answer:1

Determine compliance :
The external capsule: It is composed of white matter and is located on the lateral side of the
lentiform nucleus
The caudate nucleus : occupies the outer and lower part of the corpus striatum, covers the
islet from the inside

The internal carotid artery branches in the petrous part [[1]] in the cerebral
part [[2]].
[[1]] rr.caroticotympanicae
[[2]] A.cerebri anterior
4
4/20/2021 Midterm exam: İnsan anatomiyası və tibbi terminologiya - 2 (EN) (MPF2): Attempt review

Dashboard / My courses / İnsan anatomiyası və tibbi terminologiya - 2 (EN) / General

/ Midterm exam: İnsan anatomiyası və tibbi terminologiya - 2 (EN) (MPF2)

Started on Tuesday, 20 April 2021, 12:30 PM


State Finished
Completed on Tuesday, 20 April 2021, 12:56 PM
Time taken 25 mins 47 secs
Grade 17 out of 20 (85%)

Question 1
Correct

Mark 1.00 out of 1.00

The length of the spinal cord in female is (cm) :

A. 35

B. 32

C. 50

D. 43 

E. 46

Question 2
Correct

Mark 1.00 out of 1.00

The basilar groove is belonged to:

A. midbrain

B. pons 

C. diencephalon

D. lamina terminalis

E. medulla oblongata

https://ets.amu.edu.az/mod/quiz/review.php?attempt=69006&cmid=7722 1/8
4/20/2021 Midterm exam: İnsan anatomiyası və tibbi terminologiya - 2 (EN) (MPF2): Attempt review

Question 3
Correct

Mark 1.00 out of 1.00

The floor of the central part of the lateral ventricle is formed by:
I. Body of caudate nucleus
II. Head of caudate nucleus
III. Thalamus
IV. Corpus fornicis
V. Stria terminalis

A. I; II; III

B. II; III; V

C. III; IV; V

D. I; III; V 

E. I; III; IV

Question 4
Correct

Mark 1.00 out of 1.00

Where is the gyrus parahippocampalis situated?

A. between the superior and inferior temporal sulcus

B. between the lateral and superior temporal sulcus

C. between the hippocampal and collateral sulcus 

D. in the posterior part of the gyrus temporalis inferior

E. between the inferior temporal and occipitotemporalis sulcus

Question 5
Correct

Mark 1.00 out of 1.00

In region of the lateral angles of the rhomboid fossa lays:

A. locus caeruleus

B. eminentia medialis

C. nucleus dorsalis nervi vagi

D. area vestibularis 

E. nucleus ruber

https://ets.amu.edu.az/mod/quiz/review.php?attempt=69006&cmid=7722 2/8
4/20/2021 Midterm exam: İnsan anatomiyası və tibbi terminologiya - 2 (EN) (MPF2): Attempt review

Question 6
Correct

Mark 1.00 out of 1.00

What extends from the superior to the inferior angle of the rhomboid fossa?

A. eminentia medialis

B. striae medullares

C. sulcus medianus 

D. colliculus facialis

E. sulcus limitans

Question 7
Correct

Mark 1.00 out of 1.00

The nucleus of the olfactory analyzer lies in the:

A. medial surface of the occipital lobe

B. superior parietal lobule

C. uncus of the parahippocampal gyrus 

D. middle temporal gyrus

E. superior temporal gyrus

Question 8
Correct

Mark 1.00 out of 1.00

It arises from the facial nerve just above the stylomastoid foramen:

A. lesser petrosal nerve

B. branch to the tympanic plexus

C. the nerve to the stapedius

D. chorda tympani 

E. greater petrosal nerve

https://ets.amu.edu.az/mod/quiz/review.php?attempt=69006&cmid=7722 3/8
4/20/2021 Midterm exam: İnsan anatomiyası və tibbi terminologiya - 2 (EN) (MPF2): Attempt review

Question 9
Correct

Mark 1.00 out of 1.00

It does not belong to the branches of the cervical part of the vagus nerve:

A. pharyngeal branches

B. superior laryngeal nerve

C. recurrent laryngeal nerve

D. rr. cardiaci cervicales superiores

E. auricular branch 

Question 10
Correct

Mark 1.00 out of 1.00

Which layers of the serous pericardium?

A. mediastinal, costal

B. costal,parietal

C. parietal, costal

D. diaphragmaric,visceral

E. parietal, visceral 

Question 11
Incorrect

Mark 0.00 out of 1.00

The first branch of the internal carotid artery is the following:

A. a. ophthalmica 

B. a. cerebri media

C. a. cerebri anterior

D. rr. caroticotympanici

E. a. communicans posterior

https://ets.amu.edu.az/mod/quiz/review.php?attempt=69006&cmid=7722 4/8
4/20/2021 Midterm exam: İnsan anatomiyası və tibbi terminologiya - 2 (EN) (MPF2): Attempt review

Question 12
Correct

Mark 1.00 out of 1.00

It does not belong the branches of the internal carotid artery:

A. a. cerebri posterior 

B. a. ophthalmica

C. a. cerebri anterior

D. a. cerebri media

E. a. communicans posterior

Question 13
Correct

Mark 1.00 out of 1.00

It is not the branch of the radial artery:

A. r. carpalis dorsalis

B. a. recurrens radialis

C. r. carpalis palmaris

D. а. interossea communis 

E. a. princeps pollicis

Question 14
Correct

Mark 1.00 out of 1.00

The vein which is composed by the connecting occipital and posterior auricular veins:

A. superior venаcava

B. anterior jugular vein

C. brachiocephalic vein

D. internal jugular vein

E. external jugular vein 

https://ets.amu.edu.az/mod/quiz/review.php?attempt=69006&cmid=7722 5/8
4/20/2021 Midterm exam: İnsan anatomiyası və tibbi terminologiya - 2 (EN) (MPF2): Attempt review

Question 15
Correct

Mark 1.00 out of 1.00

What is called the continuation of the right ascending lumbar vein?

A. azygos vein 

B. pericardiaco-phrenic veins

C. internal thoracic vein

D. accessory hemiazygos vein

E. hemiazygos vein

Question 16
Incorrect

Mark 0.00 out of 1.00

The anterior wall of the third ventricle is formed by:

1. pineal body 

2. thalamus

3. anterior commissure 

4. columns of the fornix

5. epithalamic commissure

6. terminal lamina 

Question 17
Incorrect

Mark 0.00 out of 1.00

Which muscles are innervated by n. dorsalis scapulae?

1. m. levator scapulae 

2. m. latissimus dorsi

3. m. coracobrachialis

4. m. pectoralis major 

5. mm. rhomboidei 

https://ets.amu.edu.az/mod/quiz/review.php?attempt=69006&cmid=7722 6/8
4/20/2021 Midterm exam: İnsan anatomiyası və tibbi terminologiya - 2 (EN) (MPF2): Attempt review

Question 18
Correct

Mark 1.00 out of 1.00

Find the accordance:

Gyrus angularis embraces posterior end of the superior temporal sulcus 

Gyrus supramarginalis embraces posterior branch of the lateral sulcus 

Question 19
Correct

Mark 1.00 out of 1.00

The trochlear nerve enters the orbit through the [[1]] and innervates the [[2]].

[[1]] superior orbital fissure 

[[2]] superior oblique muscle 

Question 20
Correct

Mark 1.00 out of 1.00

Find n. vagus:

Answer: 11 

◄ Attendance

Jump to...

A. Abdullayev. 7. General overview of the peripheral nervous system. The classification, development, variation, and anomalies of the
cranial nerves. ►

https://ets.amu.edu.az/mod/quiz/review.php?attempt=69006&cmid=7722 7/8
4/20/2021 Midterm exam: İnsan anatomiyası və tibbi terminologiya - 2 (EN) (MPF2): Attempt review

https://ets.amu.edu.az/mod/quiz/review.php?attempt=69006&cmid=7722 8/8
20/04/2021 Midterm exam: İnsan anatomiyası və tibbi terminologiya - 2 (EN) (MPF2): Attempt review

Mənim əsas səhifəm / Mənim fənlərim / İnsan anatomiyası və tibbi terminologiya - 2 (EN) / Ümumi

/ Midterm exam: İnsan anatomiyası və tibbi terminologiya - 2 (EN) (MPF2)

Testə başlandı Tuesday, 20 April 2021, 12:30 PM


State Finished
Sona çatdı Tuesday, 20 April 2021, 12:58 PM
Vaxt keçib 27 dəq. 37 san.
Qiymət 20 (68%) maksimumdan 14

Question 1
Correct

Mark 1.00 out of 1.00

The spinal cord is in a topoqaphic proportion are divided into:

A. cervical,lumbar-sacral parts

B. cervical,coccygeal and lumbar parts

C. cervical ,thoracic,sacral parts

D. cervical,thoracic,lumbar parts 

E. cervical,sacral parts

Question 2
Correct

Mark 1.00 out of 1.00

Which from the following gyruses does not belong to the temporal lobe?

A. supramarginal 

B. temporalis superior

C. parahyppocampalis

D. temporalis inferior

E. temporalis media

https://ets.amu.edu.az/mod/quiz/review.php?attempt=69027&cmid=7722 1/7
20/04/2021 Midterm exam: İnsan anatomiyası və tibbi terminologiya - 2 (EN) (MPF2): Attempt review

Question 3
Incorrect

Mark 0.00 out of 1.00

The upper wall or roof of the posterior horns of the lateral ventricle is formed by the lower surfaces of:

A. commissura posterior 

B. indusium griseum

C. forceps minor

D. commisura anterior

E. tapetum

Question 4
Correct

Mark 1.00 out of 1.00

The cerebral peduncles are located:

A. behind of cerebral aqueduct

B. under the tectum mesencephali 

C. laterally of cerebral aqueduct

D. medially of cerebral aqueduct

E. in front of optic chiasm

Question 5
Correct

Mark 1.00 out of 1.00

The roof of the fourth ventricle in front consists of:


I. Superior cerebellar peduncles
II. Middle cerebellar peduncles
III. Inferior cerebellar peduncles
IV. Velum medullare superius
V. Velum medullare inferius

A. I; IV 

B. I; II

C. II; III

D. III; IV

E. III; V

https://ets.amu.edu.az/mod/quiz/review.php?attempt=69027&cmid=7722 2/7
20/04/2021 Midterm exam: İnsan anatomiyası və tibbi terminologiya - 2 (EN) (MPF2): Attempt review

Question 6
Not answered

Marked out of 1.00

What extends from the superior to the inferior angle of the rhomboid fossa?

A. sulcus medianus

B. eminentia medialis

C. sulcus limitans

D. colliculus facialis

E. striae medullares

Question 7
Correct

Mark 1.00 out of 1.00

Which of the following are the parts of the tympanic membrane?


I. Pars mucosa
II. Pars muscularis
III. Pars tensa
IV. Pars flaccida
V. Pars membranacea

A. IV, V

B. II, IV

C. III, IV 

D. I, II

E. III, V

Question 8
Correct

Mark 1.00 out of 1.00

Which cranial nerve provides the human with the ability to smell a wide variety of odors?

A. vestibulocochlear nerve

B. olfactory nerve 

C. oculomotor nerve

D. optic nerve

E. intermedius nerve

https://ets.amu.edu.az/mod/quiz/review.php?attempt=69027&cmid=7722 3/7
20/04/2021 Midterm exam: İnsan anatomiyası və tibbi terminologiya - 2 (EN) (MPF2): Attempt review

Question 9
Correct

Mark 1.00 out of 1.00

Through which opening does the maxillary nerve pass out the cranial cavity?

A. spinosum

B. rotundum 

C. ovale

D. fissura orbitalis superior

E. lacerum

Question 10
Correct

Mark 1.00 out of 1.00

How is the called right atrio-ventricular valve?

A. atrioventricularis sinistra

B. seminularis

C. bicuspidalis

D. mitralis

E. tricuspidalis 

Question 11
Correct

Mark 1.00 out of 1.00

The ascending aorta passes to the aortic arch at the level of the:

A. left third sternocostal joint

B. left fourth sternocostal joint

C. right second sternocostal joint 

D. left second sternocostal joint

E. right third sternocostal joint

https://ets.amu.edu.az/mod/quiz/review.php?attempt=69027&cmid=7722 4/7
20/04/2021 Midterm exam: İnsan anatomiyası və tibbi terminologiya - 2 (EN) (MPF2): Attempt review

Question 12
Incorrect

Mark 0.00 out of 1.00

It doesn’t belong to the branches of thyrocervical trunk:

A. suprascapular artery

B. superficial cervical artery 

C. inferior thyroid artery

D. ascending cervical artery

E. subscapular artery

Question 13
Not answered

Marked out of 1.00

To which vein does v.cephalica vein open?

A. external jugular

B. anterior jugular

C. brachial

D. axillary

E. subclavian

Question 14
Correct

Mark 1.00 out of 1.00

At which level the internal jugular vein connects with the subclavian vein?

A. thyroid cartilage

B. I rib

C. III rib

D. sterno-clavicular joint 

E. II rib

https://ets.amu.edu.az/mod/quiz/review.php?attempt=69027&cmid=7722 5/7
20/04/2021 Midterm exam: İnsan anatomiyası və tibbi terminologiya - 2 (EN) (MPF2): Attempt review

Question 15
Incorrect

Mark 0.00 out of 1.00

The parietal branches of the thoracic aorta are the followings:


I. Rr. bronchiales
II. Rr. mediastinales
III. Superior phrenic arteries
IV. Posterior intercostalis arteries
V. Rr. pericardiaci

A. III,V

B. I,III

C. III,IV

D. I,II 

E. IV,V

Question 16
Correct

Mark 1.00 out of 1.00

The anterior wall of the third ventricle is formed by:

1. anterior commissure 

2. epithalamic commissure

3. terminal lamina 

4. pineal body

5. columns of the fornix 

6. thalamus

Question 17
Correct

Mark 1.00 out of 1.00

To the branches of the ophthalmic nerve belong:

1. n. infraorbitalis

2. n. petrosus major

3. n. nasociliaris 

4. n. lacrimalis 

5. n. auriculotemporalis

6. n. frontalis 

https://ets.amu.edu.az/mod/quiz/review.php?attempt=69027&cmid=7722 6/7
20/04/2021 Midterm exam: İnsan anatomiyası və tibbi terminologiya - 2 (EN) (MPF2): Attempt review

Question 18
Partially correct

Mark 0.50 out of 1.00

Find the accordance:

Gyrus supramarginalis embraces anterior end of the inferior temporal sulcus 

Gyrus angularis embraces posterior end of the superior temporal sulcus 

Question 19
Not answered

Marked out of 1.00

At the level of the lateral edge of the [[1]] rib, the axillary vein continues into the [[2]] vein.

[[1]] Seçin...

[[2]] Seçin...

Question 20
Correct

Mark 1.00 out of 1.00

Find the ulnar nerve:

Answer: 7 

◄ Attendance

Keç ...

A. Abdullayev. 7. General overview of the peripheral nervous system. The classification, development, variation, and anomalies of the
cranial nerves. ►

https://ets.amu.edu.az/mod/quiz/review.php?attempt=69027&cmid=7722 7/7
4/20/2021 Midterm exam: İnsan anatomiyası və tibbi terminologiya - 2 (EN) (MPF2): Attempt review

Dashboard / My courses / İnsan anatomiyası və tibbi terminologiya - 2 (EN) / General

/ Midterm exam: İnsan anatomiyası və tibbi terminologiya - 2 (EN) (MPF2)

Started on Tuesday, 20 April 2021, 12:30 PM


State Finished
Completed on Tuesday, 20 April 2021, 12:54 PM
Time taken 23 mins 52 secs
Grade 19 out of 20 (95%)

Question 1
Correct

Mark 1.00 out of 1.00

In the posterior funiculus of the spinal cord are located:

A. tractus tectospinalis, tractus vestibulospinalis

B. tractus rubrospinalis, tractus spinocerebellaris dorsalis

C. fasciculus gracilis, tractus tectospinalis

D. tractus vestibulospinalis, tractus spinocerebellaris ventralis

E. fasciculus cuneatus, fasciculus gracilis 

Question 2
Correct

Mark 1.00 out of 1.00

It happens when the motor analyzer of speech articulation is damaged:

A. sensory aphasia

B. agraphia

C. motor aphasia 

D. tactile agnosia

E. apraxia

https://ets.amu.edu.az/mod/quiz/review.php?attempt=68873&cmid=7722 1/8
4/20/2021 Midterm exam: İnsan anatomiyası və tibbi terminologiya - 2 (EN) (MPF2): Attempt review

Question 3
Correct

Mark 1.00 out of 1.00

The extremal capsule is the white matter separating:

A. globus pallidus lateralis and putamen

B. putamen and claustrum

C. claustrum from cortex of insula 

D. globi pallidus medialis et lateralis

E. globus pallidus medialis and nucleus caudatus

Question 4
Correct

Mark 1.00 out of 1.00

It does not belong to epithalamus:

A. epiphysis

B. habenular trigone

C. hypophysis 

D. epithalamic commissure

E. habenular commissure

Question 5
Correct

Mark 1.00 out of 1.00

Where is the vallecula cerebelli located?

A. on the superior surface of cerebellum

B. on the anterior border of cerebellum

C. on the anterior and posterior borders of cerebellum

D. on the inferior surface of cerebellum 

E. on the posterior border of cerebellum

https://ets.amu.edu.az/mod/quiz/review.php?attempt=68873&cmid=7722 2/8
4/20/2021 Midterm exam: İnsan anatomiyası və tibbi terminologiya - 2 (EN) (MPF2): Attempt review

Question 6
Correct

Mark 1.00 out of 1.00

The lower part of the rhomboid fossa is corresponded to:

A. diencephalon

B. mesencephalon

C. medulla oblongata 

D. telencephalon

E. metencephalon

Question 7
Correct

Mark 1.00 out of 1.00

They are belonged to middle ear:


I. Auricula
II. Tuba auditiva
III. Meatus acusticus externus osseus
IV. Meatus acusticus externus cartilagineus
V. Cavitas tympani

A. II;III

B. IV;V

C. III;IV

D. II;V 

E. I;III

Question 8
Correct

Mark 1.00 out of 1.00

The glossopharyngeal nerve descends through the upper part of the neck between the:

A. internal jugular vein and common carotid artery

B. internal jugular vein and external carotid artery

C. internal jugular vein and external jugular vein

D. internal carotid artery and external carotid artery

E. internal jugular vein and internal carotid artery 

https://ets.amu.edu.az/mod/quiz/review.php?attempt=68873&cmid=7722 3/8
4/20/2021 Midterm exam: İnsan anatomiyası və tibbi terminologiya - 2 (EN) (MPF2): Attempt review

Question 9
Correct

Mark 1.00 out of 1.00

It does not belong to the branches of the ophthalmic nerve:


I. N. infaorbitalis
II. N. lacrimalis
III. Rr. pterygopalatini
IV. N. nasociliaris
V. N. frontalis
VI. N. zygomaticus

A. I; III; V

B. II; IV; V

C. I; III; VI 

D. II; III; IV

E. III; IV; VI

Question 10
Correct

Mark 1.00 out of 1.00

Where is located the superior border of the heart?

A. line connecting upper edges of III ribs 

B. line connecting lower edges of the V ribs

C. line connecting upper edges of VII ribs

D. line connecting upper edges the IV ribs

E. line connecting lower edges of VI ribs

Question 11
Correct

Mark 1.00 out of 1.00

Thoracic aorta locates between the:

A. ThI-ThXII

B. ThVI-ThXII

C. ThIV-ThX

D. ThIV-ThXII 

E. ThIII-ThVIII

https://ets.amu.edu.az/mod/quiz/review.php?attempt=68873&cmid=7722 4/8
4/20/2021 Midterm exam: İnsan anatomiyası və tibbi terminologiya - 2 (EN) (MPF2): Attempt review

Question 12
Incorrect

Mark 0.00 out of 1.00

The vertebral artery (a. vertebralis), is the first branch of the

A. subclavian artery

B. superficial cervical artery

C. inferior thyroid artery

D. suprascapular artery

E. subclavian artery 

Question 13
Correct

Mark 1.00 out of 1.00

To which vein does v.cephalica vein open?

A. axillary 

B. anterior jugular

C. brachial

D. external jugular

E. subclavian

Question 14
Correct

Mark 1.00 out of 1.00

At which level does superior vena cava form?

A. I sternocostal synchondrosis 

B. III sternocostal joint

C. IV sternocostal joint

D. sternoclavicular joint

E. II sternocostal joint

https://ets.amu.edu.az/mod/quiz/review.php?attempt=68873&cmid=7722 5/8
4/20/2021 Midterm exam: İnsan anatomiyası və tibbi terminologiya - 2 (EN) (MPF2): Attempt review

Question 15
Correct

Mark 1.00 out of 1.00

This doesn’t belong to the cutaneous branches of the cervical plexus:

A. n. auricularis magnus

B. n. transversus colli

C. n. subclavius 

D. n. occipitalis minor

E. n. supraclavicularis

Question 16
Correct

Mark 1.00 out of 1.00

The anterior wall of the third ventricle is formed by:

1. thalamus

2. anterior commissure 

3. columns of the fornix 

4. pineal body

5. terminal lamina 

6. epithalamic commissure

Question 17
Correct

Mark 1.00 out of 1.00

To the nuclei of the medulla oblongata belong:

1. nucleus cuneatus 

2. nucleus olivaris 

3. nucleus ruber

4. nucleus caudatus

5. nucleus dentatus

6. nucleus gracilis 

https://ets.amu.edu.az/mod/quiz/review.php?attempt=68873&cmid=7722 6/8
4/20/2021 Midterm exam: İnsan anatomiyası və tibbi terminologiya - 2 (EN) (MPF2): Attempt review

Question 18
Correct

Mark 1.00 out of 1.00

Find the accordance:

Gyrus angularis embraces posterior end of the superior temporal sulcus 

Gyrus supramarginalis embraces posterior branch of the lateral sulcus 

Question 19
Correct

Mark 1.00 out of 1.00

The external capsule is a layer of [[1]] matter that is situated between the claustrum and [[2]] nucleus.

[[1]] white 

[[2]] lentiform 

Question 20
Correct

Mark 1.00 out of 1.00

Find the colliculus facialis:

Answer: 6 

◄ Attendance

Jump to...

A. Abdullayev. 7. General overview of the peripheral nervous system. The classification, development, variation, and anomalies of the
cranial nerves. ►

https://ets.amu.edu.az/mod/quiz/review.php?attempt=68873&cmid=7722 7/8
4/20/2021 Midterm exam: İnsan anatomiyası və tibbi terminologiya - 2 (EN) (MPF2): Attempt review

https://ets.amu.edu.az/mod/quiz/review.php?attempt=68873&cmid=7722 8/8
4/20/2021 Midterm exam: Human anatomy and medical terminology - 2 (EN) (MPF2): Attempt review

My main page / My subjects / Human Anatomy and Medical Terminology - 2 (EN) / General
/ Midterm exam: Human anatomy and medical terminology - 2 (EN) (MPF2)

The test began Tuesday, 20 April 2021, 12 30 PM


State Finished
It's over Tuesday, 20 April 2021, 12 57 PM
Time has passed 26 minutes 56 sec.
Price 20 ( 95 %) maximum 19
Question 1
Correct
Mark 1.00 out of 1.00

In the posterior funiculus of the spinal cord are located:


A. tectospinalis tract, vestibulospinalis tract
B. rubrospinalis tract, spinocerebellaris dorsal tract
C. fasciculus gracilis, tectospinalis tract
D. vestibulospinalis tract, ventricular spinocerebellaris tract
E.. fasciculus cuneatus, fasciculus gracilis 

Question 2
Correct
Mark 1.00 out of 1.00

Which of the following gyruses does not belong to the temporal lobe?
A. temporalis media
B. parahyppocampalis
C. inferior temporalis
D. superior temporalis
E.. supramarginal 

https://ets.amu.edu.az/mod/quiz/review.php?attempt=68704&cmid=7722 1/7
4/20/2021 Midterm exam: Human anatomy and medical terminology - 2 (EN) (MPF2): Attempt review

Question 3
Correct
Mark 1.00 out of 1.00

The anterior limb of the internal capsule is situated between:


A. putamen and claustrum
B. lentiform nucleus and caudate nucleus 

C. thalamus and lentiform nucleus


D. claustrum and cortex of insula
E.. globi pallidus medialis et lateralis

Question 4
Correct
Mark 1.00 out of 1.00

It does not belong to the epithalamus:


A. epithalamic commissure
B. epiphysis
C. hypophysis 

D. habenular commissure
E.. habenular trigone

Question 5
Correct
Mark 1.00 out of 1.00

The roof of the fourth ventricle in front consists of:


I. Superior cerebellar peduncles
II. Middle cerebellar peduncles
III. Inferior cerebellar peduncles
IV. Velum medullare superius
V. Velum medullare inferius
A. III; V
B. I; IV 

C. II; III
D. I; II
E.. III; IV

https://ets.amu.edu.az/mod/quiz/review.php?attempt=68704&cmid=7722 2/7
4/20/2021 Midterm exam: Human anatomy and medical terminology - 2 (EN) (MPF2): Attempt review

Question 6
Correct
Mark 1.00 out of 1.00

The lower part of the rhomboid fossa is corresponded to:


A. diencephalon
B. metencephalon
C. telencephalon
D. medulla oblongata 

E.. mesencephalon

Question 7
Correct
Mark 1.00 out of 1.00

Where does the optic nerve enter the cranial cavity from?
A. foramen zygomatico-orbitale
B. foramen ethmoidale posterior
C. canalis opticus 

D. foramen ovale
E.. foramen ethmoidae anterior

Question 8
Correct
Mark 1.00 out of 1.00

Which cranial nerve provides the human with the ability to smell a wide variety of odors?
A. intermedius nerve
B. optic nerve
C. oculomotor nerve
D. olfactory nerve 

E.. vestibulocochlear nerve

https://ets.amu.edu.az/mod/quiz/review.php?attempt=68704&cmid=7722 3/7
4/20/2021 Midterm exam: Human anatomy and medical terminology - 2 (EN) (MPF2): Attempt review

Question 9
Correct
Mark 1.00 out of 1.00

It does not belong to the branches of the ophthalmic nerve:


IN infaorbitalis
II. N. lacrimalis
III. Rr. pterygopalatini
IV. N. nasociliaris
VN frontalis
VI. N. zygomaticus
A. III; IV; VI
B. I; III; V
C. II; IV; V
D. I; III; VI 

E.. II; III; IV

Question 10
Correct
Mark 1.00 out of 1.00

Which layers of the serous pericardium?


A. diaphragmaric, visceral
B. mediastinal, costal
C. parietal, visceral 

D. parietal, costal
E.. costal, parietal

Question 11
Correct
Mark 1.00 out of 1.00

The ascending aorta passes to the aortic arch at the level of the:
A. left second sternocostal joint
B. left fourth sternocostal joint
C. left third sternocostal joint
D. right second sternocostal joint 

E.. right third sternocostal joint

https://ets.amu.edu.az/mod/quiz/review.php?attempt=68704&cmid=7722 4/7
4/20/2021 Midterm exam: Human anatomy and medical terminology - 2 (EN) (MPF2): Attempt review

Question 12
Incorrect
Mark 0.00 out of 1.00

The vertebral artery (a. Vertebralis), is the first branch of the


A. subclavian artery
B. superficial cervical artery
C. subclavian artery
D. inferior thyroid artery
E.. suprascapular artery

Question 13
Correct
Mark 1.00 out of 1.00

To which vein does v.basilica open?


A. brachial 

B. anterior jugular
C. subclavian
D. axillary
E.. external jugular

Question 14
Correct
Mark 1.00 out of 1.00

The vein which is composed by the connecting occipital and posterior auricular veins:
A. internal jugular vein
B. brachiocephalic vein
C. superior venacava
D. external jugular vein 

E.. anterior jugular vein

https://ets.amu.edu.az/mod/quiz/review.php?attempt=68704&cmid=7722 5/7
4/20/2021 Midterm exam: Human anatomy and medical terminology - 2 (EN) (MPF2): Attempt review

Question 15
Correct
Mark 1.00 out of 1.00

The azygos vein opens into the vena cava superior at the level of:
A. Th9
B. Th7
C. Th3
D. Th4
E.. Th5 

Question 16
Correct
Mark 1.00 out of 1.00

Which muscles are innervated by n. dorsal scapulae?


1. m. levator scapulae 

2. mm. rhomboidei 

3. m. pectoralis major
4. m. latissimus dorsi
5. m. coracobrachialis

Question 17
Correct
Mark 1.00 out of 1.00

To the nuclei of the medulla oblongata belong:


1. nucleus caudatus
2. nucleus olivaris 

3. nucleus gracilis 

4. nucleus ruber
5. nucleus cuneatus 

6. nucleus dentatus

https://ets.amu.edu.az/mod/quiz/review.php?attempt=68704&cmid=7722 6/7
4/20/2021 Midterm exam: Human anatomy and medical terminology - 2 (EN) (MPF2): Attempt review

Question 18
Correct
Mark 1.00 out of 1.00

Find the according:


The superior laryngeal nerve is the branch of vagus nerve 

The tympanic nerve is the branch of glossopharyngeal nerve 

Question 19
Correct
Mark 1.00 out of 1.00

At the level of the lateral edge of the [[1]] rib, the axillary vein continues into the [[2]] vein.
[[1]] first 

[[2]] subclavian 

Question 20
Correct
Mark 1.00 out of 1.00

Find the pons:

Answer: 11 

◀ Attendance
Go ...
A. Abdullayev. 7. General overview of the peripheral nervous system. The classification, development, variation, and anomalies of the
cranial nerves. ▶

https://ets.amu.edu.az/mod/quiz/review.php?attempt=68704&cmid=7722 7/7
4/20/2021 Midterm exam: İnsan anatomiyası və tibbi terminologiya - 2 (EN) (MPF2): Attempt review

Dashboard / My courses / İnsan anatomiyası və tibbi terminologiya - 2 (EN) / General

/ Midterm exam: İnsan anatomiyası və tibbi terminologiya - 2 (EN) (MPF2)

Started on Tuesday, 20 April 2021, 12:31 PM


State Finished
Completed on Tuesday, 20 April 2021, 12:59 PM
Time taken 28 mins 42 secs
Grade 17 out of 20 (83%)

Question 1
Incorrect

Mark 0.00 out of 1.00

On the midline of the anterior surface of the spinal cord is:

A. dorsal (posterior) median sulcus. 

B. ventrolateral grooves

C. dorsolateral grooves

D. fissura orbitalis superior

E. ventral median fissure

Question 2
Correct

Mark 1.00 out of 1.00

Temporal lobe is separated from the frontal and parietal lobes by the:

A. transversus occipital sulcus

B. lateral sulcus 

C. parieto-occipital sulcus

D. central sulcus

E. postcentral sulcus

https://ets.amu.edu.az/mod/quiz/review.php?attempt=69157&cmid=7722#question-99121-5 1/8
4/20/2021 Midterm exam: İnsan anatomiyası və tibbi terminologiya - 2 (EN) (MPF2): Attempt review

Question 3
Correct

Mark 1.00 out of 1.00

The extremal capsule is the white matter separating:

A. claustrum from cortex of insula 

B. globus pallidus lateralis and putamen

C. globi pallidus medialis et lateralis

D. globus pallidus medialis and nucleus caudatus

E. putamen and claustrum

Question 4
Correct

Mark 1.00 out of 1.00

Midbrain in Latin is called:

A. telencephalon

B. diencephalon

C. myelencephalon

D. mesencephalon 

E. metencephalon

Question 5
Incorrect

Mark 0.00 out of 1.00

The trigonum lemnisci is bounded medially by:

A. inferior cerebellar peduncle

B. brachium colliculi superioris

C. brachium colliculi inferioris 

D. superior cerebellar peduncle

E. cerebral peduncle

https://ets.amu.edu.az/mod/quiz/review.php?attempt=69157&cmid=7722#question-99121-5 2/8
4/20/2021 Midterm exam: İnsan anatomiyası və tibbi terminologiya - 2 (EN) (MPF2): Attempt review

Question 6
Correct

Mark 1.00 out of 1.00

The olive separated from the pyramid by the:

A. anterolateral sulcus 

B. posterior median sulcus

C. intermediate sulcus

D. posterior funiculus

E. posterolateral sulcus

Question 7
Correct

Mark 1.00 out of 1.00

Which cranial nerve receives sound wave stimuli from the small bones of the middle ear, inner ear and interpret this in the brain as hearing
and also control human balance?

A. vestibulocochlear nerve 

B. facial nerve

C. glossopharyngeal nerve

D. intermedius nerve

E. olfactory nerve

Question 8
Correct

Mark 1.00 out of 1.00

It arises from the facial nerve just above the stylomastoid foramen:

A. branch to the tympanic plexus

B. lesser petrosal nerve

C. the nerve to the stapedius

D. greater petrosal nerve

E. chorda tympani 

https://ets.amu.edu.az/mod/quiz/review.php?attempt=69157&cmid=7722#question-99121-5 3/8
4/20/2021 Midterm exam: İnsan anatomiyası və tibbi terminologiya - 2 (EN) (MPF2): Attempt review

Question 9
Correct

Mark 1.00 out of 1.00

The vagus nerve passes out of the skull through:

A. foramen juguluare 

B. foramen lacerum

C. foramen rotundum

D. foramen ovale

E. foramen spinosum

Question 10
Correct

Mark 1.00 out of 1.00

Which layers of the serous pericardium?

A. parietal, costal

B. mediastinal, costal

C. diaphragmaric,visceral

D. costal,parietal

E. parietal, visceral 

Question 11
Correct

Mark 1.00 out of 1.00

Where is located common carotid artery bifurcation?

A. superior border of the thyroid cartilage 

B. V cervical vertebra

C. inferior border of the thyroid cartilage

D. posterior border of the palate

E. root of the tongue

https://ets.amu.edu.az/mod/quiz/review.php?attempt=69157&cmid=7722#question-99121-5 4/8
4/20/2021 Midterm exam: İnsan anatomiyası və tibbi terminologiya - 2 (EN) (MPF2): Attempt review

Question 12
Correct

Mark 1.00 out of 1.00

The vertebral artery (a. vertebralis), is the first branch of the

A. subclavian artery

B. superficial cervical artery

C. inferior thyroid artery

D. suprascapular artery

E. subclavian artery 

Question 13
Correct

Mark 1.00 out of 1.00

To which vein does v.cephalica vein open?

A. anterior jugular

B. axillary 

C. external jugular

D. subclavian

E. brachial

Question 14
Correct

Mark 1.00 out of 1.00

The internal jugular vein is a continuation of:

A. inferior petrosal sinus

B. sigmoid sinus 

C. transverse sinus

D. superior petrosal sinus

E. superior sagittal sinus

https://ets.amu.edu.az/mod/quiz/review.php?attempt=69157&cmid=7722#question-99121-5 5/8
4/20/2021 Midterm exam: İnsan anatomiyası və tibbi terminologiya - 2 (EN) (MPF2): Attempt review

Question 15
Correct

Mark 1.00 out of 1.00

The brachial plexus is formed by the

A. two trunks

B. six trunks

C. three trunks 

D. five trunks

E. four trunks

Question 16
Partially correct

Mark 0.67 out of 1.00

The anterior wall of the third ventricle is formed by:

1. epithalamic commissure

2. anterior commissure

3. terminal lamina 

4. thalamus

5. pineal body

6. columns of the fornix 

Question 17
Incorrect

Mark 0.00 out of 1.00

To the nuclei of the medulla oblongata belong:

1. nucleus olivaris 

2. nucleus gracilis

3. nucleus caudatus

4. nucleus ruber 

5. nucleus cuneatus

6. nucleus dentatus

https://ets.amu.edu.az/mod/quiz/review.php?attempt=69157&cmid=7722#question-99121-5 6/8
4/20/2021 Midterm exam: İnsan anatomiyası və tibbi terminologiya - 2 (EN) (MPF2): Attempt review

Question 18
Correct

Mark 1.00 out of 1.00

Find the accordance:

The superior laryngeal nerve is the branch of vagus nerve 

The tympanic nerve is the branch of glossopharyngeal nerve 

Question 19
Correct

Mark 1.00 out of 1.00

At the level of the lateral edge of the [[1]] rib, the axillary vein continues into the [[2]] vein.

[[1]] first 

[[2]] subclavian 

Question 20
Correct

Mark 1.00 out of 1.00

Find n. vagus:

Answer: 11 

◄ Attendance

Jump to...

A. Abdullayev. 7. General overview of the peripheral nervous system. The classification, development, variation, and anomalies of the
cranial nerves. ►

https://ets.amu.edu.az/mod/quiz/review.php?attempt=69157&cmid=7722#question-99121-5 7/8
4/20/2021 Midterm exam: İnsan anatomiyası və tibbi terminologiya - 2 (EN) (MPF2): Attempt review

https://ets.amu.edu.az/mod/quiz/review.php?attempt=69157&cmid=7722#question-99121-5 8/8
4/20/2021 Midterm exam: İnsan anatomiyası və tibbi terminologiya - 2 (EN) (MPF2): Attempt review

Mənim əsas səhifəm / Mənim fənlərim / İnsan anatomiyası və tibbi terminologiya - 2 (EN) / Ümumi

/ Midterm exam: İnsan anatomiyası və tibbi terminologiya - 2 (EN) (MPF2)

Testə başlandı Tuesday, 20 April 2021, 12:30 PM


State Finished
Sona çatdı Tuesday, 20 April 2021, 12:56 PM
Vaxt keçib 26 dəq. 15 san.
Qiymət 20 (98%) maksimumdan 20

Question 1
Correct

Mark 1.00 out of 1.00

Vermis is belonged to the:

A. pons

B. midbrain

C. cerebellum 

D. medulla oblongata

E. caudate nucleus

Question 2
Correct

Mark 1.00 out of 1.00

They are situated on the inferior surface of the temporal lobe:


I. Sulcus occipitotemporalis
II. Gyrus temporalis medius
III. Gyri temporales transversi
IV. Sulcus collateralis

A. I; II

B. III; IV

C. II; III

D. II; IV

E. I; IV 

https://ets.amu.edu.az/mod/quiz/review.php?attempt=68726&cmid=7722#question-98698-17 1/7
4/20/2021 Midterm exam: İnsan anatomiyası və tibbi terminologiya - 2 (EN) (MPF2): Attempt review

Question 3
Correct

Mark 1.00 out of 1.00

The extremal capsule is the white matter separating:

A. claustrum from cortex of insula 

B. globi pallidus medialis et lateralis

C. putamen and claustrum

D. globus pallidus lateralis and putamen

E. globus pallidus medialis and nucleus caudatus

Question 4
Correct

Mark 1.00 out of 1.00

Where is the gyrus parahippocampalis situated?

A. in the posterior part of the gyrus temporalis inferior

B. between the hippocampal and collateral sulcus 

C. between the superior and inferior temporal sulcus

D. between the inferior temporal and occipitotemporalis sulcus

E. between the lateral and superior temporal sulcus

Question 5
Correct

Mark 1.00 out of 1.00

Where is the vallecula cerebelli located?

A. on the posterior border of cerebellum

B. on the superior surface of cerebellum

C. on the anterior border of cerebellum

D. on the inferior surface of cerebellum 

E. on the anterior and posterior borders of cerebellum

https://ets.amu.edu.az/mod/quiz/review.php?attempt=68726&cmid=7722#question-98698-17 2/7
4/20/2021 Midterm exam: İnsan anatomiyası və tibbi terminologiya - 2 (EN) (MPF2): Attempt review

Question 6
Correct

Mark 1.00 out of 1.00

What extends from the superior to the inferior angle of the rhomboid fossa?

A. sulcus medianus 

B. sulcus limitans

C. colliculus facialis

D. eminentia medialis

E. striae medullares

Question 7
Correct

Mark 1.00 out of 1.00

Where does the optic nerve enter the cranial cavity from?

A. foramen zygomatico-orbitale

B. foramen ovale

C. foramen ethmoidae anterior

D. foramen ethmoidale posterior

E. canalis opticus 

Question 8
Correct

Mark 1.00 out of 1.00

Which cranial nerve provides the human with the ability to smell a wide variety of odors?

A. optic nerve

B. olfactory nerve 

C. intermedius nerve

D. oculomotor nerve

E. vestibulocochlear nerve

https://ets.amu.edu.az/mod/quiz/review.php?attempt=68726&cmid=7722#question-98698-17 3/7
4/20/2021 Midterm exam: İnsan anatomiyası və tibbi terminologiya - 2 (EN) (MPF2): Attempt review

Question 9
Correct

Mark 1.00 out of 1.00

It does not belong to the branches of the cervical part of the vagus nerve:

A. rr. cardiaci cervicales superiores

B. pharyngeal branches

C. recurrent laryngeal nerve

D. auricular branch 

E. superior laryngeal nerve

Question 10
Correct

Mark 1.00 out of 1.00

How is the called right atrio-ventricular valve?

A. tricuspidalis 

B. atrioventricularis sinistra

C. mitralis

D. bicuspidalis

E. seminularis

Question 11
Correct

Mark 1.00 out of 1.00

The ascending aorta passes to the aortic arch at the level of the:

A. right third sternocostal joint

B. left fourth sternocostal joint

C. left third sternocostal joint

D. right second sternocostal joint 

E. left second sternocostal joint

https://ets.amu.edu.az/mod/quiz/review.php?attempt=68726&cmid=7722#question-98698-17 4/7
4/20/2021 Midterm exam: İnsan anatomiyası və tibbi terminologiya - 2 (EN) (MPF2): Attempt review

Question 12
Correct

Mark 1.00 out of 1.00

It doesn’t belong to the branches of thyrocervical trunk:

A. subscapular artery 

B. inferior thyroid artery

C. suprascapular artery

D. ascending cervical artery

E. superficial cervical artery

Question 13
Correct

Mark 1.00 out of 1.00

To which vein does v.basilica open?

A. subclavian

B. axillary

C. brachial 

D. anterior jugular

E. external jugular

Question 14
Correct

Mark 1.00 out of 1.00

It does not belong to the tributaries of the external jugular vein:

A. v. suprascapularis

B. v. canalis pterygoidei 

C. v. auricularis posterior

D. v. occipitalis

E. v. jugularis anterior

https://ets.amu.edu.az/mod/quiz/review.php?attempt=68726&cmid=7722#question-98698-17 5/7
4/20/2021 Midterm exam: İnsan anatomiyası və tibbi terminologiya - 2 (EN) (MPF2): Attempt review

Question 15
Correct

Mark 1.00 out of 1.00

The visceral branches of the thoracic aorta are the followings:


I. Rr. bronchiales
II. Rr. mediastinales
III. Superior phrenic arteries
IV. Posterior intercostalis arteries
V. Rr. pericardiaci

A. II,III,IV

B. I,II,IV

C. III,IV,V

D. I,II,V 

E. II,III,V

Question 16
Correct

Mark 1.00 out of 1.00

The anterior wall of the third ventricle is formed by:

1. terminal lamina 

2. anterior commissure 

3. columns of the fornix 

4. pineal body

5. thalamus

6. epithalamic commissure

Question 17
Partially correct

Mark 0.67 out of 1.00

To the nuclei of the medulla oblongata belong:

1. nucleus caudatus

2. nucleus olivaris 

3. nucleus dentatus

4. nucleus ruber

5. nucleus gracilis 

6. nucleus cuneatus

https://ets.amu.edu.az/mod/quiz/review.php?attempt=68726&cmid=7722#question-98698-17 6/7
4/20/2021 Midterm exam: İnsan anatomiyası və tibbi terminologiya - 2 (EN) (MPF2): Attempt review

Question 18
Correct

Mark 1.00 out of 1.00

Find the accordance:

A. interossea communis is the branch of ulnar artery 

A. thoracoacromialis is the branch of axillary artery 

Question 19
Correct

Mark 1.00 out of 1.00

At the level of the lateral edge of the [[1]] rib, the axillary vein continues into the [[2]] vein.

[[1]] first 

[[2]] subclavian 

Question 20
Correct

Mark 1.00 out of 1.00

Find the ulnar nerve:

Answer: 7 

◄ Attendance

Keç ...

A. Abdullayev. 7. General overview of the peripheral nervous system. The classification, development, variation, and anomalies of the cranial
nerves. ►

https://ets.amu.edu.az/mod/quiz/review.php?attempt=68726&cmid=7722#question-98698-17 7/7
01023ÿ53ÿ01 6789 ÿ ÿÿ 8ÿÿ88ÿ 88ÿÿ0ÿ6ÿ67 !0

34567ÿ49:9ÿ94;6<47 =34567ÿ<45>4?67 =ÿ@59:5ÿ:5:AB76C:9DÿE4ÿA6FF6ÿAG?765B>BH6C:ÿIÿJÿKLMN=ÿO7P76


=ÿ36QAG?7ÿGR:7Sÿ@59:5ÿ:5:AB76C:9DÿE4ÿA6FF6ÿAG?765B>BH6C:ÿIÿJÿKLMNÿK3TUJN

VWXYZ[\]ÿ_
MBAÿCGAÿ:59`G?GQ
3:?aGQÿBPAÿB<ÿbcdd

gGA`GG5ÿA;Gÿ:5AG?6B?ÿh:?AÿB<ÿA;GÿhB59ÿ:5QÿA;Gÿ76QQ>GÿiG?GFG>>:?ÿhGQP5i>GÿG7G?HG9
jcA;Gÿ:FQPiG5Aÿ5G?EG
gcA;GÿBiP>B7BAB?ÿ5G?EG
kcA;GÿA?6HG765:>ÿ5G?EGc
lcA;Gÿ<:i6:>ÿ5G?EG
LcA;GÿA?Bi;>G:?ÿ5G?EG
k>G:?ÿ7Cÿi;B6iG

VWXYZ[\]ÿe
MBAÿCGAÿ:59`G?GQ
3:?aGQÿBPAÿB<ÿbcdd

mAÿ;:hhG59ÿ6<ÿA;Gÿ7BAB?ÿ:5:>CnG?ÿB<ÿ̀?6AAG5ÿ9hGGi;ÿ69ÿQ:7:HGQS
jc7BAB?ÿ:h;:96:
gc:H?:h;6:
kc9G59B?Cÿ:h;:96:
lc:h?:R6:
LcA:iA6>Gÿ:H5B96:
k>G:?ÿ7Cÿi;B6iG

VWXYZ[\]ÿf
MBAÿCGAÿ:59`G?GQ
3:?aGQÿBPAÿB<ÿbcdd

m5ÿ̀;6i;ÿ>BFGÿB<ÿA;GÿiG?GF?:>ÿ;G769h;G?Gÿ69ÿA;GÿiB?hP9ÿ:7CHQ:>B6QGP7ÿ96AP:AGQo
jc<?B5A:>
gcBii6h6A:>
kcAG7hB?:>
lch:?6GA:>
Lc659P>:
k>G:?ÿ7Cÿi;B6iG
p
" #33 $ %$9%$&3 93'%8&3 #$#"#( #)*+1,-./ 89)00001'% 8+2++22- 230
01023ÿ53ÿ01 6789 ÿ ÿÿ 8ÿÿ88ÿ 88ÿÿ0ÿ6ÿ67 !0
3456789:ÿ<
=>?ÿAB?ÿCDEFBGBH
ICGJBHÿ>K?ÿ>LÿMNOO

R?ÿH>BEÿD>?ÿSBT>DUÿ?>ÿBVW?XCTCYKEZ
[NXCSBDKTCGÿ?GWU>DB
\NXCSBDKTCGÿ]>YYWEEKGB
^NBVW?XCTCYW]ÿ]>YYWEEKGB
_NXAV>VXAEWE
`NBVWVXAEWE
^TBCGÿYAÿ]X>W]B

3456789:ÿP
=>?ÿAB?ÿCDEFBGBH
ICGJBHÿ>K?ÿ>LÿMNOO

aXBGBÿWEÿ?XBÿbCTTB]KTCÿ]BGBSBTTWÿT>]C?BHc
[N>Dÿ?XBÿWDLBGW>GÿEKGLC]Bÿ>Lÿ]BGBSBTTKY
\N>Dÿ?XBÿEKVBGW>GÿEKGLC]Bÿ>Lÿ]BGBSBTTKY
^N>Dÿ?XBÿCD?BGW>GÿCDHÿV>E?BGW>GÿS>GHBGEÿ>Lÿ]BGBSBTTKY
_N>Dÿ?XBÿV>E?BGW>GÿS>GHBGÿ>Lÿ]BGBSBTTKY
`N>Dÿ?XBÿCD?BGW>GÿS>GHBGÿ>Lÿ]BGBSBTTKY
^TBCGÿYAÿ]X>W]B

3456789:ÿQ
=>?ÿAB?ÿCDEFBGBH
ICGJBHÿ>K?ÿ>LÿMNOO

dXBÿT>FBGÿVCG?ÿ>Lÿ?XBÿGX>YS>WHÿL>EECÿWEÿ]>GGBEV>DHBHÿ?>Z
[NYBHKTTCÿ>ST>DUC?C
\N?BTBD]BVXCT>D
^NYBEBD]BVXCT>D
_NHWBD]BVXCT>D
`NYB?BD]BVXCT>D
^TBCGÿYAÿ]X>W]B

e
" #33 $ %$9%$&3 93'%8&3 #$#"#( #)*+1,-./ 89)00001'% 8+2++22- 030
01023ÿ53ÿ01 6789 ÿ ÿÿ 8ÿÿ88ÿ 88ÿÿ0ÿ6ÿ67 !0
3456789:ÿ<
=>?ÿAB?ÿCDEFBGBH
ICGJBHÿ>K?ÿ>LÿMNOO

R?ÿH>BEDS?ÿTBU>DVÿ?>ÿ?WBÿT>DAÿUCTAGXD?WY
ZN[CDCUXEÿEB\X[XG[KUCGXEÿ]>E?BGX>G
^N[>[WUBC
_N`BE?XTKUK\
aN[CDCUXEÿEB\X[XG[KUCGXEÿCD?BGX>G
bNK?GX[KUKE
_UBCGÿ\Aÿ[W>X[B

3456789:ÿP
=>?ÿAB?ÿCDEFBGBH
ICGJBHÿ>K?ÿ>LÿMNOO

RD?>ÿW>Fÿ\CDAÿ]CG?EÿH>BEÿ?WBÿLC[XCUÿDBG`BÿHX̀XHBÿC[[>GHXDVÿ?>ÿ?WBÿ?>]>VGC]WAÿY
ZNc
^Nd
_Ne
aNf
bNg
_UBCGÿ\Aÿ[W>X[B

3456789:ÿQ
=>?ÿAB?ÿCDEFBGBH
ICGJBHÿ>K?ÿ>LÿMNOO

R?ÿH>BEÿD>?ÿTBU>DVÿ?>ÿ?WBÿTGCD[WBEÿ>Lÿ?WBÿ[BG`X[CUÿ]CG?ÿ>Lÿ?WBÿ̀CVKEÿDBG`BY
ZNCKGX[KUCGÿTGCD[W
^NEK]BGX>GÿUCGADVBCUÿDBG`B
_N]WCGADVBCUÿTGCD[WBE
aNGGNÿ[CGHXC[Xÿ[BG`X[CUBEÿEK]BGX>GBE
bNGB[KGGBD?ÿUCGADVBCUÿDBG`B
_UBCGÿ\Aÿ[W>X[B

h
" #33 $ %$9%$&3 93'%8&3 #$#"#( #)*+1,-./ 89)00001'% 8+2++22- ,30
01023ÿ53ÿ01 6789 ÿ ÿÿ 8ÿÿ88ÿ 88ÿÿ0ÿ6ÿ67 !0
3456789:ÿ<=
>?@ÿBC@ÿDEFGCHCI
JDHKCIÿ?L@ÿ?MÿNOPP

RSTUSÿVDBCHFÿ?Mÿ@SCÿFCH?LFÿWCHTUDHITLXY
ZOXCITDF@TEDV[ÿU?F@DV
\OWDHTC@DV[ÿ]TFUCHDV
^OWDHTC@DV[ÿU?F@DV
_OITDWSHD`XDHTU[]TFUCHDV
aOU?F@DV[WDHTC@DV
^VCDHÿXBÿUS?TUC

3456789:ÿ<<
>?@ÿBC@ÿDEFGCHCI
JDHKCIÿ?L@ÿ?MÿNOPP

RSCHCÿTFÿV?UD@CIÿU?XX?EÿUDH?@TIÿDH@CHBÿbTMLHUD@T?EY
ZOFLWCHT?Hÿb?HICHÿ?Mÿ@SCÿ@SBH?TIÿUDH@TVD`C
\OTEMCHT?Hÿb?HICHÿ?Mÿ@SCÿ@SBH?TIÿUDH@TVD`C
^OW?F@CHT?Hÿb?HICHÿ?Mÿ@SCÿWDVD@C
_OcÿUCH]TUDVÿ]CH@CbHD
aOH??@ÿ?Mÿ@SCÿ@?E`LC
^VCDHÿXBÿUS?TUC

3456789:ÿ<Q
>?@ÿBC@ÿDEFGCHCI
JDHKCIÿ?L@ÿ?MÿNOPP

dSCÿbHDEUSCFÿ?Mÿ@SCÿbHDUST?UCWSDVTUÿ@HLEKeÿÿ
fOÿZOÿFLbUVD]TDÿICg@HDÿÿ
ffOÿZOÿUDH?@TFÿU?XXLETFÿFTETF@HDÿÿ
fffOÿZOÿUDH?@TFÿU?XXLETFÿICg@HDÿÿ
fcOÿZOÿFLbUVD]TDÿFTETF@HDÿÿ
cOÿZOÿ@SBH?TICDÿTXD
ZOfffhfc
\Ofhfff
^Ofchc
_Ofhff
aOffhfff
^VCDHÿXBÿUS?TUC

i
" #33 $ %$9%$&3 93'%8&3 #$#"#( #)*+1,-./ 89)00001'% 8+2++22- 530
01023ÿ53ÿ01 6789 ÿ ÿÿ 8ÿÿ88ÿ 88ÿÿ0ÿ6ÿ67 !0
3456789:ÿ<=
>?@ÿBC@ÿDEFGCHCI
JDHKCIÿ?L@ÿ?MÿNOPP

S@ÿTFÿE?@ÿ@UCÿVHDEWUÿ?Mÿ@UCÿHDITDXÿDH@CHBY
ZOHOÿWDH[DXTFÿ[DX\DHTF
]OHOÿWDH[DXTFÿI?HFDXTF
^ODOÿ[HTEWC[Fÿ[?XXTWTF
_O`OÿTE@CH?FFCDÿW?\\LETF
aODOÿHCWLHHCEFÿHDITDXTF
^XCDHÿ\BÿWU?TWC

3456789:ÿ<Q
>?@ÿBC@ÿDEFGCHCI
JDHKCIÿ?L@ÿ?MÿNOPP

bUCHCÿTFÿX?WD@CIÿ@UCÿTEMCHT?HÿFDcT@@DXÿFTELFd
ZOVC@GCCEÿ@UCÿM?HD\CEÿWDCWL\ÿDEIÿTE@CHEDXÿ?WWT[T@DXÿ[H?@LVCHDEWC
]OTEÿ@UCÿW?EMXLCEWCÿWTELL\
^OTEÿ@UCÿW?EECW@TEcÿ[XDWCÿ?Mÿ@UCÿMDXeÿWCHCVHTÿGT@Uÿ@UCÿ@CE@?HTL\ÿWCHCVCXXT
_OTEÿ@UCÿ@HDEFfCHFCÿFTELFÿcH??fCÿ?Mÿ@UCÿ?WWT[T@DXÿV?EC
aOTEÿ@UCÿTEMCHT?HÿW?EWDfCÿ\DHcTEÿ?Mÿ@UCÿMDXeÿWCHCVHT
^XCDHÿ\BÿWU?TWC

3456789:ÿ<R
>?@ÿBC@ÿDEFGCHCI
JDHKCIÿ?L@ÿ?MÿNOPP

gUCÿVHDWUTDXÿ[XCeLFÿTFÿM?H\CIÿVBÿ@UC
ZOM?LHÿ@HLEKF
]O@UHCCÿ@HLEKF
^O@G?ÿ@HLEKF
_OFTeÿ@HLEKF
aOMTfCÿ@HLEKF

h
" #33 $ %$9%$&3 93'%8&3 #$#"#( #)*+1,-./ 89)00001'% 8+2++22- -30
01023ÿ53ÿ01 6789 ÿ ÿÿ 8ÿÿ88ÿ 88ÿÿ0ÿ6ÿ67 !0
3456789:ÿ<=
>?@ÿBC@ÿDEFGCHCI
JDHKCIÿ?L@ÿ?MÿNOPP

T?ÿ@UCÿVHDEWUCFÿ?Mÿ@UCÿ?XU@UDYZ[WÿECH\CÿVCY?E]^
NOEOÿYDWH[ZDY[F
_OEOÿ[EMHD?HV[@DY[F
`OEOÿXC@H?FLFÿZDa?H
bOEOÿDLH[WLY?@CZX?HDY[F
cOEOÿMH?E@DY[F
dOEOÿEDF?W[Y[DH[F

3456789:ÿ<Q
>?@ÿBC@ÿDEFGCHCI
JDHKCIÿ?L@ÿ?MÿNOPP

T?ÿ@UCÿVHDEWUCFÿ?Mÿ@UCÿVHDWU[DYÿDH@CHBÿVCY?E]^
NODOÿXH?MLEIDÿVHDWU[[
_ODOÿW?YYD@CHDY[FÿLYEDH[FÿFLXCH[?H
`ODOÿFLVFWDXLYDH[F
bODOÿ@U?HDW?I?HFDY[F
cODOÿ@U?HDW?DWH?Z[DY[F
dODOÿW?YYD@CHDY[FÿLYEDH[Fÿ[EMCH[?H

3456789:ÿ<R
>?@ÿBC@ÿDEFGCHCI
JDHKCIÿ?L@ÿ?MÿNOPP

e[EIÿ@UCÿDWW?HIDEWC^
fBHLFÿDE]LYDH[FÿCZVHDWCF X?F@CH[?HÿCEIÿ?Mÿ@UCÿFLXCH[?Hÿ@CZX?HDYÿFLYWLF
fBHLFÿFLXHDZDH][EDY[FÿCZVHDWCF X?F@CH[?HÿVHDEWUÿ?Mÿ@UCÿYD@CHDYÿFLYWLF

3456789:ÿ<S
>?@ÿBC@ÿDEFGCHCI
JDHKCIÿ?L@ÿ?MÿNOPP

TUCÿ@H?WUYCDHÿECH\CÿCE@CHFÿ@UCÿ?HV[@ÿ@UH?L]Uÿ@UCÿggNhhÿDEIÿ[EECH\D@CFÿ@UCÿgg_hhO
ggNhh FLXCH[?Hÿ?HV[@DYÿM[FFLHC
gg_hh FLXCH[?Hÿ?VY[iLCÿZLFWYC

j
" #33 $ %$9%$&3 93'%8&3 #$#"#( #)*+1,-./ 89)00001'% 8+2++22- *30
01023ÿ53ÿ01 6789 ÿ ÿÿ 8ÿÿ88ÿ 88ÿÿ0ÿ6ÿ67 !0
3456789:ÿ<=
>?@ÿBC@ÿDEFGCHCI
JDHKCIÿ?L@ÿ?MÿNOPP

QREIÿ@SCÿLTEDHÿECHUCV

ÿ
WEFGCHV X

YÿW@@CEIDEZC
[C\ÿOOO
WOÿW]ILTTDBCUOÿXOÿ^CECHDTÿ?UCHURCGÿ?Mÿ@SCÿ_CHR_SCHDTÿECHU?LFÿFBF@C`OÿaSCÿZTDFFRMRZD@R?EbÿICUCT?_`CE@bÿUDHRD@R?EbÿDEIÿDE?`DTRCFÿ?Mÿ@SCÿ
ZHDERDTÿECHUCFOÿc

d
" #33 $ %$9%$&3 93'%8&3 #$#"#( #)*+1,-./ 89)00001'% 8+2++22- 030
2021/4/20 Midterm exam: İnsan anatomiyası və tibbi terminologiya - 2 (EN) (MPF2): Attempt review

Mənim əsas səhifəm / Mənim fənlərim / İnsan anatomiyası və tibbi terminologiya - 2 (EN) / Ümumi

/ Midterm exam: İnsan anatomiyası və tibbi terminologiya - 2 (EN) (MPF2)

Testə başlandı Tuesday, 20 April 2021, 12:30 PM


State Finished
Sona çatdı Tuesday, 20 April 2021, 12:59 PM
Vaxt keçib 29 dəq. 14 san.
Qiymət 20 (80%) maksimumdan 16

Question 1
Correct

Mark 1.00 out of 1.00

On the midline of the anterior surface of the spinal cord is:

A. fissura orbitalis superior

B. ventrolateral grooves

C. dorsal (posterior) median sulcus.

D. dorsolateral grooves

E. ventral median fissure 

Question 2
Incorrect

Mark 0.00 out of 1.00

It happens if the motor analyzer of written speech is damaged:

A. apraxia

B. motor aphasia 

C. tactile agnosia

D. sensory aphasia

E. agraphia

https://ets.amu.edu.az/mod/quiz/review.php?attempt=68865&cmid=7722#question-98823-15 1/7
2021/4/20 Midterm exam: İnsan anatomiyası və tibbi terminologiya - 2 (EN) (MPF2): Attempt review

Question 3
Correct

Mark 1.00 out of 1.00

The floor of the central part of the lateral ventricle is formed by:
I. Body of caudate nucleus
II. Head of caudate nucleus
III. Thalamus
IV. Corpus fornicis
V. Stria terminalis

A. I; III; IV

B. II; III; V

C. I; II; III

D. I; III; V 

E. III; IV; V

Question 4
Correct

Mark 1.00 out of 1.00

Midbrain in Latin is called:

A. metencephalon

B. telencephalon

C. mesencephalon 

D. myelencephalon

E. diencephalon

Question 5
Incorrect

Mark 0.00 out of 1.00

The trigonum lemnisci is bounded medially by:

A. cerebral peduncle

B. brachium colliculi superioris

C. brachium colliculi inferioris 

D. superior cerebellar peduncle

E. inferior cerebellar peduncle

https://ets.amu.edu.az/mod/quiz/review.php?attempt=68865&cmid=7722#question-98823-15 2/7
2021/4/20 Midterm exam: İnsan anatomiyası və tibbi terminologiya - 2 (EN) (MPF2): Attempt review

Question 6
Incorrect

Mark 0.00 out of 1.00

In the superior triangle region of the rhomboid fossa is situated the nuclei of the

A. VIII,VII,III,VI cranial nerves 

B. V,II,III,VI cranial nerves

C. X,XI,XII cranial nerves

D. V, VI, VII , VIII cranial nerves

E. I,II.V,XII cranial nerves

Question 7
Correct

Mark 1.00 out of 1.00

The oculomotor nerve emerges from cranial cavity through:

A. foramen ethmoidale anterior

B. canalis opticus

C. fissura orbitalis inferior

D. foramen rotundum

E. fissura orbitalis superior 

Question 8
Correct

Mark 1.00 out of 1.00

The glossopharyngeal nerve leaves the skull through:

A. canalis hypoglossus

B. foramen ovale

C. foramen lacerum

D. foramen rotundum

E. foramen jugulare 

https://ets.amu.edu.az/mod/quiz/review.php?attempt=68865&cmid=7722#question-98823-15 3/7
2021/4/20 Midterm exam: İnsan anatomiyası və tibbi terminologiya - 2 (EN) (MPF2): Attempt review

Question 9
Correct

Mark 1.00 out of 1.00

Which muscles are innervated by the nervus accesorius?


I. M. sternocleidomastoideus
II. M. trapezius
III. M. digastricus
IV. M. latissimus dorsi
V. M. buccinator

A. III,V

B. I,II 

C. II,III

D. III,IV

E. IV,V

Question 10
Correct

Mark 1.00 out of 1.00

Which layers of the serous pericardium?

A. costal,parietal

B. mediastinal, costal

C. parietal, visceral 

D. parietal, costal

E. diaphragmaric,visceral

Question 11
Correct

Mark 1.00 out of 1.00

Where is located common carotid artery bifurcation?

A. superior border of the thyroid cartilage 

B. root of the tongue

C. inferior border of the thyroid cartilage

D. posterior border of the palate

E. V cervical vertebra

https://ets.amu.edu.az/mod/quiz/review.php?attempt=68865&cmid=7722#question-98823-15 4/7
2021/4/20 Midterm exam: İnsan anatomiyası və tibbi terminologiya - 2 (EN) (MPF2): Attempt review

Question 12
Correct

Mark 1.00 out of 1.00

The vertebral artery (a. vertebralis), is the first branch of the

A. suprascapular artery

B. superficial cervical artery

C. subclavian artery 

D. inferior thyroid artery

E. subclavian artery

Question 13
Correct

Mark 1.00 out of 1.00

It is not the branch of the ulnar artery:

A. а. carpalis dorsalis

B. а. carpalis palmaris

C. а. interossea communis

D. а. recurrens radialis 

E. а. recurrens ulnaris

Question 14
Correct

Mark 1.00 out of 1.00

The vein which is composed by the connecting occipital and posterior auricular veins:

A. internal jugular vein

B. superior venаcava

C. anterior jugular vein

D. brachiocephalic vein

E. external jugular vein 

https://ets.amu.edu.az/mod/quiz/review.php?attempt=68865&cmid=7722#question-98823-15 5/7
2021/4/20 Midterm exam: İnsan anatomiyası və tibbi terminologiya - 2 (EN) (MPF2): Attempt review

Question 15
Incorrect

Mark 0.00 out of 1.00

The visceral branches of the thoracic aorta are the followings:


I. Rr. bronchiales
II. Rr. mediastinales
III. Superior phrenic arteries
IV. Posterior intercostalis arteries
V. Rr. pericardiaci

A. II,III,V

B. I,II,V

C. I,II,IV

D. III,IV,V

E. II,III,IV 

Question 16
Correct

Mark 1.00 out of 1.00

Which muscles are innervated by n. dorsalis scapulae?

1. m. pectoralis major

2. m. levator scapulae 

3. m. latissimus dorsi

4. m. coracobrachialis

5. mm. rhomboidei 

Question 17
Correct

Mark 1.00 out of 1.00

To the branches of the brachial artery belong:

1. a. thoracodorsalis

2. a. collateralis ulnaris superior 

3. a. subscapularis

4. a. thoracoacromialis

5. a. collateralis ulnaris inferior 

6. a. profunda brachii 

https://ets.amu.edu.az/mod/quiz/review.php?attempt=68865&cmid=7722#question-98823-15 6/7
2021/4/20 Midterm exam: İnsan anatomiyası və tibbi terminologiya - 2 (EN) (MPF2): Attempt review

Question 18
Correct

Mark 1.00 out of 1.00

Find the accordance:

The external ear includes auricle and external acoustic meatus 

The inner ear consists of bony labyrinth and membranous labyrinth 

Question 19
Correct

Mark 1.00 out of 1.00

The external capsule is a layer of [[1]] matter that is situated between the claustrum and [[2]] nucleus.

[[1]] white 

[[2]] lentiform 

Question 20
Correct

Mark 1.00 out of 1.00

Find the ulnar nerve:

Answer: 7 

◄ Attendance

Keç ...

A. Abdullayev. 7. General overview of the peripheral nervous system. The classification, development, variation, and anomalies of the
cranial nerves. ►

https://ets.amu.edu.az/mod/quiz/review.php?attempt=68865&cmid=7722#question-98823-15 7/7
4/20/2021 Midterm exam: İnsan anatomiyası və tibbi terminologiya - 2 (EN) (MPF2): Attempt review

Mənim əsas səhifəm / Mənim fənlərim / İnsan anatomiyası və tibbi terminologiya - 2 (EN) / Ümumi

/ Midterm exam: İnsan anatomiyası və tibbi terminologiya - 2 (EN) (MPF2)

Testə başlandı Tuesday, 20 April 2021, 12:30 PM


State Finished
Sona çatdı Tuesday, 20 April 2021, 12:57 PM
Vaxt keçib 27 dəq. 46 san.
Qiymət 20 (80%) maksimumdan 16

Question 1
Correct

Mark 1.00 out of 1.00

How many lumbar segments does the spinal cord have?

A. 5 

B. 12

C. 8

D. 6

E. 4

Question 2
Incorrect

Mark 0.00 out of 1.00

It happens when the motor analyzer of speech articulation is damaged:

A. apraxia 

B. agraphia

C. tactile agnosia

D. sensory aphasia

E. motor aphasia

https://ets.amu.edu.az/mod/quiz/review.php?attempt=68702&cmid=7722#question-98660-17 1/8
4/20/2021 Midterm exam: İnsan anatomiyası və tibbi terminologiya - 2 (EN) (MPF2): Attempt review

Question 3
Incorrect

Mark 0.00 out of 1.00

The floor of the central part of the lateral ventricle is formed by:
I. Body of caudate nucleus
II. Head of caudate nucleus
III. Thalamus
IV. Corpus fornicis
V. Stria terminalis

A. I; III; V

B. I; II; III 

C. I; III; IV

D. III; IV; V

E. II; III; V

Question 4
Correct

Mark 1.00 out of 1.00

Where is the gyrus parahippocampalis situated?

A. in the posterior part of the gyrus temporalis inferior

B. between the hippocampal and collateral sulcus 

C. between the inferior temporal and occipitotemporalis sulcus

D. between the lateral and superior temporal sulcus

E. between the superior and inferior temporal sulcus

Question 5
Correct

Mark 1.00 out of 1.00

It doesn’t belong to the isthmus rhombencephali:

A. pedunculi cerebellaris superiores and trigonum lemnisci

B. velum medullare inferius 

C. velum medullare superius

D. trigonum lemnisci

E. pedunculi cerebellaris superiores

https://ets.amu.edu.az/mod/quiz/review.php?attempt=68702&cmid=7722#question-98660-17 2/8
4/20/2021 Midterm exam: İnsan anatomiyası və tibbi terminologiya - 2 (EN) (MPF2): Attempt review

Question 6
Correct

Mark 1.00 out of 1.00

In the superior triangle region of the rhomboid fossa is situated the nuclei of the

A. I,II.V,XII cranial nerves

B. V, VI, VII , VIII cranial nerves 

C. V,II,III,VI cranial nerves

D. VIII,VII,III,VI cranial nerves

E. X,XI,XII cranial nerves

Question 7
Correct

Mark 1.00 out of 1.00

Which of the following are the parts of the tympanic membrane?


I. Pars mucosa
II. Pars muscularis
III. Pars tensa
IV. Pars flaccida
V. Pars membranacea

A. IV, V

B. III, V

C. II, IV

D. I, II

E. III, IV 

Question 8
Correct

Mark 1.00 out of 1.00

The glossopharyngeal nerve descends through the upper part of the neck between the:

A. internal carotid artery and external carotid artery

B. internal jugular vein and external carotid artery

C. internal jugular vein and common carotid artery

D. internal jugular vein and external jugular vein

E. internal jugular vein and internal carotid artery 

https://ets.amu.edu.az/mod/quiz/review.php?attempt=68702&cmid=7722#question-98660-17 3/8
4/20/2021 Midterm exam: İnsan anatomiyası və tibbi terminologiya - 2 (EN) (MPF2): Attempt review

Question 9
Correct

Mark 1.00 out of 1.00

The anterior ethmoid nerve enters the cranial cavity through:

A. optic canal

B. anterior ethmoid foramen 

C. olfactory foramina

D. medial side of the fissure orbitalis superior

E. lateral side of the fissure orbitalis inferior

Question 10
Correct

Mark 1.00 out of 1.00

Where is located the superior border of the heart?

A. line connecting upper edges of VII ribs

B. line connecting upper edges the IV ribs

C. line connecting lower edges of the V ribs

D. line connecting lower edges of VI ribs

E. line connecting upper edges of III ribs 

Question 11
Correct

Mark 1.00 out of 1.00

At which level does the external carotid artery divide into two terminating branches?

A. coronoid process of mandible

B. angle of mandible

C. thyroid cartilage

D. neck of mandible 

E. hyoid bone

https://ets.amu.edu.az/mod/quiz/review.php?attempt=68702&cmid=7722#question-98660-17 4/8
4/20/2021 Midterm exam: İnsan anatomiyası və tibbi terminologiya - 2 (EN) (MPF2): Attempt review

Question 12
Incorrect

Mark 0.00 out of 1.00

It doesn’t belong to the branches of thyrocervical trunk:

A. ascending cervical artery

B. subscapular artery

C. superficial cervical artery

D. suprascapular artery 

E. inferior thyroid artery

Question 13
Correct

Mark 1.00 out of 1.00

It is not the branch of the ulnar artery:

A. а. carpalis palmaris

B. а. recurrens ulnaris

C. а. carpalis dorsalis

D. а. interossea communis

E. а. recurrens radialis 

Question 14
Correct

Mark 1.00 out of 1.00

Where is located the inferior sagittal sinus?

A. between the foramen caecum and internal occipital protuberance

B. in the inferior concave margin of the falx cerebri 

C. in the transverse sinus groove of the occipital bone

D. in the connecting place of the falx cerebri with the tentorium cerebelli

E. in the confluence cinuum

https://ets.amu.edu.az/mod/quiz/review.php?attempt=68702&cmid=7722#question-98660-17 5/8
4/20/2021 Midterm exam: İnsan anatomiyası və tibbi terminologiya - 2 (EN) (MPF2): Attempt review

Question 15
Correct

Mark 1.00 out of 1.00

What is called the continuation of the right ascending lumbar vein?

A. accessory hemiazygos vein

B. azygos vein 

C. pericardiaco-phrenic veins

D. internal thoracic vein

E. hemiazygos vein

Question 16
Correct

Mark 1.00 out of 1.00

To the branches of the ophthalmic nerve belong:

1. n. auriculotemporalis

2. n. nasociliaris 

3. n. frontalis 

4. n. petrosus major

5. n. lacrimalis 

6. n. infraorbitalis

Question 17
Incorrect

Mark 0.00 out of 1.00

The anterior wall of the third ventricle is formed by:

1. columns of the fornix

2. epithalamic commissure

3. pineal body

4. thalamus 

5. anterior commissure 

6. terminal lamina 

https://ets.amu.edu.az/mod/quiz/review.php?attempt=68702&cmid=7722#question-98660-17 6/8
4/20/2021 Midterm exam: İnsan anatomiyası və tibbi terminologiya - 2 (EN) (MPF2): Attempt review

Question 18
Correct

Mark 1.00 out of 1.00

Find the accordance:

The superior laryngeal nerve is the branch of vagus nerve 

The tympanic nerve is the branch of glossopharyngeal nerve 

Question 19
Correct

Mark 1.00 out of 1.00

The ulnar artery departs from the [[1]] artery at the level of the [[2]] of the ulna.

[[1]] brachial 

[[2]] coronoid process 

Question 20
Correct

Mark 1.00 out of 1.00

Find the colliculus facialis:

Answer: 6 

◄ Attendance

Keç ...

A. Abdullayev. 7. General overview of the peripheral nervous system. The classification, development, variation, and anomalies of the
cranial nerves. ►

https://ets.amu.edu.az/mod/quiz/review.php?attempt=68702&cmid=7722#question-98660-17 7/8
4/20/2021 Midterm exam: İnsan anatomiyası və tibbi terminologiya - 2 (EN) (MPF2): Attempt review

https://ets.amu.edu.az/mod/quiz/review.php?attempt=68702&cmid=7722#question-98660-17 8/8
Mǝnim ǝsas sǝhifǝm / Mǝnim fǝnlǝrim / İnsan anatomiyası vǝ tibbi terminologiya - 2 (EN) / Ümumi
/ Midterm exam: İnsan anatomiyası vǝ tibbi terminologiya - 2 (EN) (MPF2)

Testǝ başlandı Tuesday, 20 April 2021, 12N30 PM


State Finished
Sona çatdı Tuesday, 20 April 2021, 12N57 PM
Vaxt keçib 26 dǝq. 4 san.
Qiymǝt 20 (95%) maksimumdan 19

Question 1 In the medial border of the cerebral peduncle emerges


Correct

Mark 1.00 out A. the trochlear nerve


of 1.00
B. the abducent nerve

C. tthe facial nerve

D. the oculomotor nerve !

E. the trigeminal nerve

Question 2 It happens when the motor analyzer of speech articulation is damaged:


Correct

Mark 1.00 out A. agraphia


of 1.00
B. tactile agnosia

C. sensory aphasia

D. motor aphasia !

E. apraxia

Question 3 The extremal capsule is the white matter separating:


Correct

Mark 1.00 out A. globus pallidus medialis and nucleus caudatus


of 1.00
B. claustrum from cortex of insula !

C. globi pallidus medialis et lateralis

D. globus pallidus lateralis and putamen

E. putamen and claustrum

Question 4 Around the cerebral aqueduct is placed:


Correct

Mark 1.00 out A. substantia grisea centralis !


of 1.00
B. substantia perforata anterior

C. substantia perforata posterior et substantia grisea centralis

D. substantia perforata posterior

E. substantia perforata anterior et substantia grisea centralis


Question 5 The roof of the fourth ventricle in front consists of:
Correct I. Superior cerebellar peduncles
Mark 1.00 out II. Middle cerebellar peduncles
of 1.00 III. Inferior cerebellar peduncles
IV. Velum medullare superius
V. Velum medullare inferius

A. III; V

B. I; II

C. III; IV

D. I; IV !

E. II; III

Question 6 The olive separated from the pyramid by the:


Correct

Mark 1.00 out A. intermediate sulcus


of 1.00
B. anterolateral sulcus !

C. posterior funiculus

D. posterior median sulcus

E. posterolateral sulcus

Question 7 Which cranial nerve receives sound wave stimuli from the small bones of the middle ear, inner ear and
Correct interpret this in the brain as hearing and also control human balance?
Mark 1.00 out
of 1.00 A. olfactory nerve

B. glossopharyngeal nerve

C. intermedius nerve

D. vestibulocochlear nerve !

E. facial nerve

Question 8 It arises from the facial nerve just above the stylomastoid foramen:
Correct

Mark 1.00 out A. the nerve to the stapedius


of 1.00
B. branch to the tympanic plexus

C. greater petrosal nerve

D. lesser petrosal nerve

E. chorda tympani !

Question 9 The vagus nerve passes out of the skull through:


Correct

Mark 1.00 out A. foramen lacerum


of 1.00
B. foramen ovale

C. foramen juguluare !

D. foramen spinosum
D. foramen spinosum

E. foramen rotundum

Question 10 Which layers of the serous pericardium?


Correct

Mark 1.00 out A. mediastinal, costal


of 1.00
B. costal,parietal

C. parietal, costal

D. diaphragmaric,visceral

E. parietal, visceral !

Question 11 Thoracic aorta locates between the:


Correct

Mark 1.00 out A. ThI-ThXII


of 1.00
B. ThIII-ThVIII

C. ThIV-ThX

D. ThVI-ThXII

E. ThIV-ThXII !

Question 12 The branches of the brachiocephalic trunk:


Correct I. A. subclavia dextra
Mark 1.00 out II. A. carotis communis sinistra
of 1.00 III. A. carotis communis dextra
IV. A. subclavia sinistra
V. A. thyroidea ima

A. I;III !

B. III;IV

C. IV;V

D. II;III

E. I;II

Question 13 To which vein does v.basilica open?


Correct

Mark 1.00 out A. axillary


of 1.00
B. external jugular

C. brachial !

D. anterior jugular

E. subclavian

Question 14 It does not belong to the tributaries of the external jugular vein:
Correct

Mark 1.00 out A. v. auricularis posterior


of 1.00
B. v. canalis pterygoidei !
C. v. occipitalis

D. v. suprascapularis

E. v. jugularis anterior

Question 15 What is called the continuation of the left ascending lumbar vein?
Correct

Mark 1.00 out A. internal thoracic vein


of 1.00
B. pericardiaco-phrenic veins

C. accessory hemiazygos vein

D. hemiazygos vein !

E. azygos vein

Question 16 The anterior wall of the third ventricle is formed by:


Correct

Mark 1.00 out 1. epithalamic commissure


of 1.00
2. terminal lamina !

3. anterior commissure !

4. columns of the fornix !

5. pineal body

6. thalamus

Question 17 To the nuclei of the medulla oblongata belong:


Correct

Mark 1.00 out 1. nucleus cuneatus !


of 1.00
2. nucleus ruber

3. nucleus gracilis !

4. nucleus caudatus

5. nucleus dentatus

6. nucleus olivaris !

Question 18 Find the accordance:


Correct

Mark 1.00 out


A. thoracoacromialis is the branch of axillary artery !
of 1.00

A. interossea communis is the branch of ulnar artery !

Question 19 Middle superior alveolar nerve passes in the lateral wall of the [[1]] and innervates the upper [[2]] teeth, gums
Correct and the cheek.
Mark 1.00 out
of 1.00
[[1]] maxillary sinus !

[[2]] premolar !
Question 20 Find the ulnar nerve:
Incorrect

Mark 0.00 out


of 1.00

Answer: 9 "

A. Abdullayev. 7. General overview


of the peripheral nervous system.
◀ Attendance Keç ... The classification, development,
variation, and anomalies of the
cranial nerves. ▶︎
4/21/2021 Midterm exam: İnsan anatomiyası və tibbi terminologiya - 2 (EN) (MPF2): Attempt review

Dashboard / My courses / İnsan anatomiyası və tibbi terminologiya - 2 (EN) / General

/ Midterm exam: İnsan anatomiyası və tibbi terminologiya - 2 (EN) (MPF2)

Started on Tuesday, 20 April 2021, 12:30 PM


State Finished
Completed on Tuesday, 20 April 2021, 12:59 PM
Time taken 29 mins 22 secs
Grade 19 out of 20 (95%)

Question 1
Incorrect

Mark 0.00 out of 1.00

In the posterior funiculus of the spinal cord are located:

A. tractus rubrospinalis, tractus spinocerebellaris dorsalis

B. tractus vestibulospinalis, tractus spinocerebellaris ventralis

C. tractus tectospinalis, tractus vestibulospinalis

D. fasciculus gracilis, tractus tectospinalis 

E. fasciculus cuneatus, fasciculus gracilis

Question 2
Correct

Mark 1.00 out of 1.00

Which from the following gyruses does not belong to the temporal lobe?

A. temporalis inferior

B. temporalis superior

C. temporalis media

D. supramarginal 

E. parahyppocampalis

https://ets.amu.edu.az/mod/quiz/review.php?attempt=68698&cmid=7722 1/8
4/21/2021 Midterm exam: İnsan anatomiyası və tibbi terminologiya - 2 (EN) (MPF2): Attempt review

Question 3
Correct

Mark 1.00 out of 1.00

Posterior end of the thalamus is called:

A. fissura chorioidea

B. tuberculum anterius

C. lamina affixa

D. stria terminalis

E. pulvinar 

Question 4
Correct

Mark 1.00 out of 1.00

It does not belong to epithalamus:

A. hypophysis 

B. epithalamic commissure

C. habenular trigone

D. habenular commissure

E. epiphysis

Question 5
Correct

Mark 1.00 out of 1.00

The dorsal part of metencephalon is:

A. cerebellum 

B. corpus mamillaris

C. pedunculus cerebri

D. pons

E. tuber cinereum

https://ets.amu.edu.az/mod/quiz/review.php?attempt=68698&cmid=7722 2/8
4/21/2021 Midterm exam: İnsan anatomiyası və tibbi terminologiya - 2 (EN) (MPF2): Attempt review

Question 6
Correct

Mark 1.00 out of 1.00

In medulla oblongata is not located:

A. center of vomiting

B. center of swallowing

C. circulatory center

D. respiratory center

E. highest center of vegetative nervous system 

Question 7
Correct

Mark 1.00 out of 1.00

The oculomotor nerve emerges from cranial cavity through:

A. fissura orbitalis superior 

B. canalis opticus

C. foramen ethmoidale anterior

D. foramen rotundum

E. fissura orbitalis inferior

Question 8
Correct

Mark 1.00 out of 1.00

The glossopharyngeal nerve leaves the skull through:

A. canalis hypoglossus

B. foramen jugulare 

C. foramen lacerum

D. foramen rotundum

E. foramen ovale

https://ets.amu.edu.az/mod/quiz/review.php?attempt=68698&cmid=7722 3/8
4/21/2021 Midterm exam: İnsan anatomiyası və tibbi terminologiya - 2 (EN) (MPF2): Attempt review

Question 9
Correct

Mark 1.00 out of 1.00

It does not belong to the branches of the ophthalmic nerve:


I. N. infaorbitalis
II. N. lacrimalis
III. Rr. pterygopalatini
IV. N. nasociliaris
V. N. frontalis
VI. N. zygomaticus

A. III; IV; VI

B. I; III; V

C. I; III; VI 

D. II; III; IV

E. II; IV; V

Question 10
Correct

Mark 1.00 out of 1.00

How many layers can be distinguished in the myocardium of the ventricles?

A. 2

B. 3 

C. 4

D. 1

E. 5

Question 11
Correct

Mark 1.00 out of 1.00

Thoracic aorta locates between the:

A. ThI-ThXII

B. ThIV-ThXII 

C. ThVI-ThXII

D. ThIII-ThVIII

E. ThIV-ThX

https://ets.amu.edu.az/mod/quiz/review.php?attempt=68698&cmid=7722 4/8
4/21/2021 Midterm exam: İnsan anatomiyası və tibbi terminologiya - 2 (EN) (MPF2): Attempt review

Question 12
Correct

Mark 1.00 out of 1.00

It does not compose the thyrocervical trunk:

A. inferior thyroid artery

B. superficial cervical artery

C. suprascapular artery

D. vertebral artery 

E. ascending cervical artery

Question 13
Correct

Mark 1.00 out of 1.00

To which vein does v.cephalica vein open?

A. subclavian

B. axillary 

C. anterior jugular

D. external jugular

E. brachial

Question 14
Correct

Mark 1.00 out of 1.00

The vein which is composed by the connecting occipital and posterior auricular veins:

A. internal jugular vein

B. brachiocephalic vein

C. anterior jugular vein

D. superior venаcava

E. external jugular vein 

https://ets.amu.edu.az/mod/quiz/review.php?attempt=68698&cmid=7722 5/8
4/21/2021 Midterm exam: İnsan anatomiyası və tibbi terminologiya - 2 (EN) (MPF2): Attempt review

Question 15
Correct

Mark 1.00 out of 1.00

Median nerve originates from the

A. axillary nerve

B. musculocutaneous nerve

C. medial and lateral cords of the brachial plexus 

D. radial nerve

E. ulnar nerve

Question 16
Correct

Mark 1.00 out of 1.00

Which muscles are innervated by n. dorsalis scapulae?

1. m. coracobrachialis

2. m. pectoralis major

3. m. latissimus dorsi

4. m. levator scapulae 

5. mm. rhomboidei 

Question 17
Correct

Mark 1.00 out of 1.00

To the branches of the ophthalmic nerve belong:

1. n. petrosus major

2. n. nasociliaris 

3. n. auriculotemporalis

4. n. lacrimalis 

5. n. infraorbitalis

6. n. frontalis 

https://ets.amu.edu.az/mod/quiz/review.php?attempt=68698&cmid=7722 6/8
4/21/2021 Midterm exam: İnsan anatomiyası və tibbi terminologiya - 2 (EN) (MPF2): Attempt review

Question 18
Correct

Mark 1.00 out of 1.00

Find the accordance:

A. interossea communis is the branch of ulnar artery 

A. thoracoacromialis is the branch of axillary artery 

Question 19
Correct

Mark 1.00 out of 1.00

The ulnar artery departs from the [[1]] artery at the level of the [[2]] of the ulna.

[[1]] brachial 

[[2]] coronoid process 

Question 20
Correct

Mark 1.00 out of 1.00

Find n. vagus:

Answer: 11 

◄ Attendance

Jump to...

A. Abdullayev. 7. General overview of the peripheral nervous system. The classification, development, variation, and anomalies of the
cranial nerves. ►

https://ets.amu.edu.az/mod/quiz/review.php?attempt=68698&cmid=7722 7/8
4/21/2021 Midterm exam: İnsan anatomiyası və tibbi terminologiya - 2 (EN) (MPF2): Attempt review

https://ets.amu.edu.az/mod/quiz/review.php?attempt=68698&cmid=7722 8/8
4/21/2021 Midterm exam: İnsan anatomiyası və tibbi terminologiya - 2 (EN) (MPF2): Attempt review

Dashboard / My courses / İnsan anatomiyası və tibbi terminologiya - 2 (EN) / General

/ Midterm exam: İnsan anatomiyası və tibbi terminologiya - 2 (EN) (MPF2)

Started on Tuesday, 20 April 2021, 12:30 PM


State Finished
Completed on Tuesday, 20 April 2021, 12:59 PM
Time taken 29 mins 22 secs
Grade 19 out of 20 (95%)

Question 1
Incorrect

Mark 0.00 out of 1.00

In the posterior funiculus of the spinal cord are located:

A. tractus rubrospinalis, tractus spinocerebellaris dorsalis

B. tractus vestibulospinalis, tractus spinocerebellaris ventralis

C. tractus tectospinalis, tractus vestibulospinalis

D. fasciculus gracilis, tractus tectospinalis 

E. fasciculus cuneatus, fasciculus gracilis

Question 2
Correct

Mark 1.00 out of 1.00

Which from the following gyruses does not belong to the temporal lobe?

A. temporalis inferior

B. temporalis superior

C. temporalis media

D. supramarginal 

E. parahyppocampalis

https://ets.amu.edu.az/mod/quiz/review.php?attempt=68698&cmid=7722 1/8
4/21/2021 Midterm exam: İnsan anatomiyası və tibbi terminologiya - 2 (EN) (MPF2): Attempt review

Question 3
Correct

Mark 1.00 out of 1.00

Posterior end of the thalamus is called:

A. fissura chorioidea

B. tuberculum anterius

C. lamina affixa

D. stria terminalis

E. pulvinar 

Question 4
Correct

Mark 1.00 out of 1.00

It does not belong to epithalamus:

A. hypophysis 

B. epithalamic commissure

C. habenular trigone

D. habenular commissure

E. epiphysis

Question 5
Correct

Mark 1.00 out of 1.00

The dorsal part of metencephalon is:

A. cerebellum 

B. corpus mamillaris

C. pedunculus cerebri

D. pons

E. tuber cinereum

https://ets.amu.edu.az/mod/quiz/review.php?attempt=68698&cmid=7722 2/8
4/21/2021 Midterm exam: İnsan anatomiyası və tibbi terminologiya - 2 (EN) (MPF2): Attempt review

Question 6
Correct

Mark 1.00 out of 1.00

In medulla oblongata is not located:

A. center of vomiting

B. center of swallowing

C. circulatory center

D. respiratory center

E. highest center of vegetative nervous system 

Question 7
Correct

Mark 1.00 out of 1.00

The oculomotor nerve emerges from cranial cavity through:

A. fissura orbitalis superior 

B. canalis opticus

C. foramen ethmoidale anterior

D. foramen rotundum

E. fissura orbitalis inferior

Question 8
Correct

Mark 1.00 out of 1.00

The glossopharyngeal nerve leaves the skull through:

A. canalis hypoglossus

B. foramen jugulare 

C. foramen lacerum

D. foramen rotundum

E. foramen ovale

https://ets.amu.edu.az/mod/quiz/review.php?attempt=68698&cmid=7722 3/8
4/21/2021 Midterm exam: İnsan anatomiyası və tibbi terminologiya - 2 (EN) (MPF2): Attempt review

Question 9
Correct

Mark 1.00 out of 1.00

It does not belong to the branches of the ophthalmic nerve:


I. N. infaorbitalis
II. N. lacrimalis
III. Rr. pterygopalatini
IV. N. nasociliaris
V. N. frontalis
VI. N. zygomaticus

A. III; IV; VI

B. I; III; V

C. I; III; VI 

D. II; III; IV

E. II; IV; V

Question 10
Correct

Mark 1.00 out of 1.00

How many layers can be distinguished in the myocardium of the ventricles?

A. 2

B. 3 

C. 4

D. 1

E. 5

Question 11
Correct

Mark 1.00 out of 1.00

Thoracic aorta locates between the:

A. ThI-ThXII

B. ThIV-ThXII 

C. ThVI-ThXII

D. ThIII-ThVIII

E. ThIV-ThX

https://ets.amu.edu.az/mod/quiz/review.php?attempt=68698&cmid=7722 4/8
4/21/2021 Midterm exam: İnsan anatomiyası və tibbi terminologiya - 2 (EN) (MPF2): Attempt review

Question 12
Correct

Mark 1.00 out of 1.00

It does not compose the thyrocervical trunk:

A. inferior thyroid artery

B. superficial cervical artery

C. suprascapular artery

D. vertebral artery 

E. ascending cervical artery

Question 13
Correct

Mark 1.00 out of 1.00

To which vein does v.cephalica vein open?

A. subclavian

B. axillary 

C. anterior jugular

D. external jugular

E. brachial

Question 14
Correct

Mark 1.00 out of 1.00

The vein which is composed by the connecting occipital and posterior auricular veins:

A. internal jugular vein

B. brachiocephalic vein

C. anterior jugular vein

D. superior venаcava

E. external jugular vein 

https://ets.amu.edu.az/mod/quiz/review.php?attempt=68698&cmid=7722 5/8
4/21/2021 Midterm exam: İnsan anatomiyası və tibbi terminologiya - 2 (EN) (MPF2): Attempt review

Question 15
Correct

Mark 1.00 out of 1.00

Median nerve originates from the

A. axillary nerve

B. musculocutaneous nerve

C. medial and lateral cords of the brachial plexus 

D. radial nerve

E. ulnar nerve

Question 16
Correct

Mark 1.00 out of 1.00

Which muscles are innervated by n. dorsalis scapulae?

1. m. coracobrachialis

2. m. pectoralis major

3. m. latissimus dorsi

4. m. levator scapulae 

5. mm. rhomboidei 

Question 17
Correct

Mark 1.00 out of 1.00

To the branches of the ophthalmic nerve belong:

1. n. petrosus major

2. n. nasociliaris 

3. n. auriculotemporalis

4. n. lacrimalis 

5. n. infraorbitalis

6. n. frontalis 

https://ets.amu.edu.az/mod/quiz/review.php?attempt=68698&cmid=7722 6/8
4/21/2021 Midterm exam: İnsan anatomiyası və tibbi terminologiya - 2 (EN) (MPF2): Attempt review

Question 18
Correct

Mark 1.00 out of 1.00

Find the accordance:

A. interossea communis is the branch of ulnar artery 

A. thoracoacromialis is the branch of axillary artery 

Question 19
Correct

Mark 1.00 out of 1.00

The ulnar artery departs from the [[1]] artery at the level of the [[2]] of the ulna.

[[1]] brachial 

[[2]] coronoid process 

Question 20
Correct

Mark 1.00 out of 1.00

Find n. vagus:

Answer: 11 

◄ Attendance

Jump to...

A. Abdullayev. 7. General overview of the peripheral nervous system. The classification, development, variation, and anomalies of the
cranial nerves. ►

https://ets.amu.edu.az/mod/quiz/review.php?attempt=68698&cmid=7722 7/8
4/21/2021 Midterm exam: İnsan anatomiyası və tibbi terminologiya - 2 (EN) (MPF2): Attempt review

https://ets.amu.edu.az/mod/quiz/review.php?attempt=68698&cmid=7722 8/8

You might also like